The Practice of Neurocritical Care

765 Pages • 209,014 Words • PDF • 13.5 MB
Uploaded at 2021-09-24 14:46

This document was submitted by our user and they confirm that they have the consent to share it. Assuming that you are writer or own the copyright of this document, report to us by using this DMCA report button.


www.medicalebookpdf.com

The Practice of Neurocritical Care

www.medicalebookpdf.com

by the Neurocritical Care Society

Editors: J. Claude Hemphill III, Alejandro A. Rabinstein & Owen B. Samuels

www.medicalebookpdf.com

This book is dedicated to the patients we serve. Copyright © 2015 by Neurocritical Care Society All rights reserved. No part of this publication may be reproduced, stored in a retrieval system, or transmitted, in any form or by any means, electronic, mechanical, photocopying, recording, or otherwise, without the written prior permission of the author. ISBN: 978-0-9909917-5-5 Cover Design by Jenni Wheeler Typeset by Mary K. Ross

www.medicalebookpdf.com

Table of Contents Foreword List of Contributors 1. Acute Ischemic Stroke Nerissa Ko and Wade Smith 2. Intracerebral Hemorrhage Thorsten Steiner and Claude Hemphill 3. Subarachnoid Hemorrhage Adam Webb and Owen Samuels 4. Seizures and Epilepsy Jan Claassen, Ira Chang, and Thomas Bleck 5. Acute Neuromuscular Disorders Christopher Kramer, Eelco Wijdicks, and Alejandro Rabinstein 6. Neuroinfectious Diseases Eric Rosenthal and Bart Nathan 7. Traumatic Brain Injury Joshua Levine and Monisha Kumar 8. Traumatic Spinal Cord Injury William Coplin 9. Non-Neurological Trauma, Burns, and Thermal Injury Deborah Stein, Christos Lazaridis, and Geoffrey Ling 10. Multimodality Neuromonitoring Mauro Oddo 11. Perioperative Neurosurgical Critical Care Chris Zacko and Peter Le Roux 12. Metabolic Encephalopathies and Delirium Panayiotis Varelas and Carmelo Graffagnino 13. Hypoxic-Ischemic Encephalopathy in Adults Max Mulder and Romer Geocadin

www.medicalebookpdf.com

14. Prognostic Assessment in Neurocritical Care Gary L. Bernardini 15. Clinical Evaluation of Coma and Brain Death David Greer 16. Sedation and Analgesia in Neurocritical Care John Lewin, Haley Gibbs, and Marek Mirski 17. Nutrition and Metabolism Neeraj Badjatia 18. Respiratory Support of the Neurocritically Ill: Airway, Mechanical Ventilation, and Management of Respiratory Diseases Julian Bösel and David Seder 19. Cardiovascular Monitoring and Complications Jesse Corry and Andrew Naidech 20. Endocrine Disorders in Neurocritical Care Nancy Edwards and Kiwon Lee 21. Pediatric Neurocritical Care Jose Pineda and Mark Wainwright 22. Pharmacology in the Neurointensive Care Unit Shaun Rowe and Theresa MurphyHuman 23. Neurocritical Care Nursing Special Considerations Jennifer Robinson 24. Quality of Care Metrics in Neurocritical Care Michelle Van Demark 25. Administrative and Management Principles and Techniques Wendy Wright

www.medicalebookpdf.com

FOREWORD A principal mission of the Neurocritical Care Society (NCS) is to promote “Quality Patient Care by identifying and implementing best medical practices for acute neurological disorders that are consistent with current scientific knowledge, and that promote compassionate care and respect for patientcentered values.” Ever since 2007, NCS has held a session at its annual meeting focusing on practical education regarding current neurocritical care and general critical care topics. This began principally as a board review course to prepare physicians who were taking the UCNS Neurocritical Care certification examination. However, we quickly realized that most of the attendees were actually not taking the certification test. Instead they were fellows, nurses, pharmacists, and practicing physicians who were looking for an update regarding best practices in neurocritical care. Two things quickly became clear: there is a strong need for practical education in clinical neurocritical care and NCS is in a unique position to provide this service. It is out of the evolution of those NCS annual meeting courses that this textbook arises. As neurocritical care has grown, there has been an expansion in the number of textbooks and educational offerings related to neurocritical care topics. Numerous texts have been written, many by NCS members, and published by commercial publishers. So what are we doing here? Well The Practice of Neurocritical Care, by the Neurocritical Care Society aims to be a little different. We have brought together topics presented at the 2011 and 2013 NCS annual meetings, updated their content, and taken on the publishing role ourselves, as the Neurocritical Care Society. Each of the 25 topics in this text starts with a clinical case, includes practical clinical information to be used at the bedside, and finishes with a set of questions (and answers). This text can certainly be used to prepare for the neurocritical care certification examination, or for the neurocritical care portion of the boards for neurology, neurosurgery, or other critical care specialties. However, we believe it is valuable to any practitioner interested in a current update on Neurocritical Care from experts in the field. This book also represents the Neurocritical Care Society's first effort at publishing. We did this for several reasons. First, by keeping the creation of the monograph “in house” we have been able to turn around the material into a widely available offering in about one-third the time of a typical publishing house textbook. Also importantly, proceeds from the sale of this book are

www.medicalebookpdf.com

brought back to NCS to use for research program funding, rather than becoming income for a commercial publisher. We see this effort as leveraging education for the scientific advancement of the field. Production of this text has been a labor of love. We are grateful to the hard work of the authors, the direction of the NCS Publications Committee in getting this project to fruition, and the support of the NCS executive office for helping make it happen. This product is certainly not as slick as many other commercial print textbooks available in the market. You will probably notice that the formatting may not be perfect and you may even find some errors in grammar or spelling along the way. Don't hesitate to let us know. That's ok, because our main goal is to provide quality and reliable content, distribute it widely, and help advance neurocritical care education worldwide while learning how to do so independently of for profit commercial entities. We hope you and your patients benefit from this textbook. And we welcome your feedback as we hope that this will be the first of many similar educational offerings to come from the Neurocritical Care Society. Claude Hemphill, Alejandro Rabinstein, and Owen Samuels on behalf of the Neurocritical Care Society January 2015

www.medicalebookpdf.com

LIST OF CONTRIBUTORS Neeraj Badjatia, MD, MSc Associate Professor of Neurology Chief of Neurocritical Care Program in Trauma University of Maryland School of Medicine Baltimore, MD, USA Gary L. Bernardini, MD, PhD Professor and Edith M. Hellman Endowed Chair in Cerebrovascular Disease Director, Stroke and Neurocritical Care Albany Medical Center Albany, NY, USA Thomas P Bleck, MD Professor of Neurological Sciences, Neurosurgery, Anesthesiology, and Medicine Rush Medical College Chicago, IL, USA Julian Bösel, MD Director of Neurocritical Care Department of Neurology University of Heidelberg

www.medicalebookpdf.com

Heidelberg, Germany Ira Chang, MD Chair, Department of Medicine Swedish Medical Center Medical Director, Neurocritical Care Exempla Lutheran Medical Center Colorado Neurological Institute Denver, CO, USA Jan Claassen, MD, PhD Assistant Professor of Neurology and Neurosurgery Head of Neurocritical Care and Medical Director of the Neurological Intensive Care Unit Columbia University College of Physicians & Surgeons New York, NY, USA William M. Coplin, MD, FCCM, FNCS Neurocritical Care & Neurosciences Medical Director Centura Health Denver, CO, USA Jesse James Corry, MD Medical Director-Acute Inpatient Neurology Marshfield Clinic Marshfield, WI, USA Nancy J. Edwards, MD Assistant Professor

www.medicalebookpdf.com

Departments of Neurology and Neurosurgery University of Texas Health Science Center at Houston Houston, TX, USA Romergryko G. Geocadin, MD, FNCS Division of Neurosciences Critical Care Johns Hopkins University School of Medicine Baltimore, MD, USA Haley G. Gibbs, PharmD Clinical Pharmacy Specialist, Neurocritical Care Johns Hopkins Hospital Baltimore, MD, USA Carmelo Graffagnino, MD, FRCPC Professor of Neurology Division of Neurocritical Care Department of Neurology Duke University Medical Center Durham, NC, USA David M. Greer, MD, MA, FCCM, FAHA, FNCS Professor of Neurology Yale University School of Medicine New Haven, CT, USA J. Claude Hemphill III, MD, MAS, FNCS Professor of Neurology and Neurological Surgery

www.medicalebookpdf.com

University of California, San Francisco Chief of Neurology San Francisco General Hospital San Francisco, CA, USA Theresa Human PharmD, BCPS, FNCS Barnes Jewish Hospital Washington University in St. Louis St. Louis, MO, USA Nerissa U. Ko, MD, MAS Professor of Neurology Neurovascular and Neurocritical Care Service Department of Neurology University of California, San Francisco San Francisco, CA, USA Christopher L. Kramer, MD Mayo Clinic, Rochester Rochester, MN, USA Monisha A. Kumar, MD Assistant Professor Division of Neurocritical Care Director, Neurocritical Care Fellowship Program Departments of Neurology, Neurosurgery and Anesthesiology & Critical Care University of Pennsylvania

www.medicalebookpdf.com

Philadelphia, PA, USA Christos Lazaridis, MD Assistant Professor Neurocritical Care, Intensive Care Medicine, and Vascular Neurology Division of Neurocritical Care and Vascular Neurology Department of Neurology Baylor College of Medicine Houston, TX, USA Peter Le Roux, MD, FACS Brain and Spine Center Lankenau Medical Center Lankenau Institute of Medical Research Philadelphia, PA, USA Kiwon Lee, MD, FACP, FAHA, FCCM Associate Professor and Vice Chair, Neurosurgery and Neurology Head of Neurocritical Care Director of Neuroscience and Neurotrauma Intensive Care Unit The University of Texas Health Science Center at Houston Mischer Neuroscience Institute Houston, TX, USA Joshua M. Levine, MD, FANA Chief, Division of Neurocritical Care, Department of Neurology Co-Director, NeuroIntensive Care Unit

www.medicalebookpdf.com

Associate Professor, Departments of Neurology, Neurosurgery, and Anesthesiology and Critical Care Perelman School of Medicine at the University of Pennsylvania Philadelphia, PA, USA John J. Lewin III, PharmD, MBA, FASHP, FCCM, FNCS Division Director, Critical Care & Surgery Pharmacy Associate Professor Anesthesiology & Critical Care Medicine The Johns Hopkins Hospital and Johns Hopkins University School of Medicine Baltimore, MD, USA Geoffrey Ling, MD, PhD Professor of Neurology Uniformed Services University of the Health Sciences Bethesda, MD, USA Marek A. Mirski, MD, PhD Thomas & Dorothy Tung Professor & Vice-Chair Departments of Neurology, Neurosurgery & Anesthesiology/Critical Care Medicine Johns Hopkins University School of Medicine Baltimore, MD, USA Maximilian Mulder, MD Department of Medicine Neurocritical Care, Medical Intensive Care, Cardiac and Cardiothoracic Intensive Care Units

www.medicalebookpdf.com

Abbott Northwestern Hospital Minneapolis, MN, USA Andrew M. Naidech, MD, MSPH, FANA Associate Professor of Neurology, Neurological Surgery, Anesthesiology, and Medical Social Sciences Medical Director, Neuro/Spine ICU Northwestern Medicine Chicago, IL, USA Barnett R. Nathan, MS, MD Associate Professor of Neurology and Internal Medicine Division of Neurocritical Care University of Virginia School of Medicine Charlottesville, VA, USA Mauro Oddo, MD Head, Neuroscience Critical Care Research Group Attending Physician Department of Critical Care Medicine CHUV-Lausanne University Hospital Faculty of Biology and Medicine University of Lausanne Lausanne, Switzerland Jose A. Pineda, MD, MSc Associate Professor

www.medicalebookpdf.com

Departments of Pediatrics and Neurology Division of Critical Care Medicine Washington University School of Medicine Saint Louis, MO, USA Alejandro A. Rabinstein, MD, FNCS Professor of Neurology Director, Neuroscience ICU Mayo Clinic, Rochester Rochester, MN, USA Jennifer D. Robinson, APRN Neuroscience Nurse Practitioner Yale New Haven Hospital New Haven, CT, USA Eric S. Rosenthal, MD Associate Director, Neurosciences Intensive Care Unit Medical Director, Critical Care Neurology Service Department of Neurology Massachusetts General Hospital Boston, MA, USA Anthony Shaun Rowe, PharmD, BCPS Assistant Professor of Clinical Pharmacy Department of Clinical Pharmacy The University of Tennessee College of Pharmacy

www.medicalebookpdf.com

Knoxville, TN, USA Owen B. Samuels, MD Associate Professor of Neurology & Neurosurgery Director, Neuroscience Critical Care Emory University School of Medicine Atlanta, GA, USA David B. Seder, MD, FCCP, FCCM Director of Neurocritical Care Maine Medical Center Portland, ME, USA Wade Smith, MD, PhD Daryl R. Gress Professor of Neurocritical Care and Stroke Vice Chair, Department of Neurology University of California, San Francisco San Francisco, CA, USA Deborah M. Stein, MD, MPH Associate Professor of Surgery and Chief of Trauma R Adams Cowley Shock Trauma Center University of Maryland School of Medicine Baltimore, MD, USA Thorsten Steiner, MD, MME Department of Neurology Klinikum Frankfurt Höchst and Heidelberg University Hospital

www.medicalebookpdf.com

Frankfurt, Germany Michelle Van Demark, MSN, RN, ANP-BC, CNRN, CCNS Neurocritical Care Nurse Practitioner Sanford USD Medical Center Sioux Falls, SD, USA Panayiotis N. Varelas, MD, PhD Professor of Neurology Wayne State University Departments of Neurology & Neurosurgery Henry Ford Hospital Detroit, MI, USA Mark S. Wainwright, MD, PhD Founders' Board Chair in Neurology Ruth D. & Ken M. Davee Pediatric Neurocritical Care Program Northwestern University Feinberg School of Medicine Chicago, IL, USA Adam Webb, MD Assistant Professor of Neurology and Neurosurgery Neuroscience Critical Care Emory University School of Medicine Medical Director, Neuroscience ICU Marcus Stroke and Neuroscience Center Grady Memorial Hospital

www.medicalebookpdf.com

Atlanta, GA, USA Eelco Wijdicks, MD, PhD Professor of Neurology Mayo Clinic Rochester, MN, USA Wendy L. Wright, MD, FCCM, FNCS Chief of Neurology and Medical Director of the Neuroscience ICU Emory University Hospital Midtown Atlanta, GA, USA J. Christopher Zacko, MS, MD, FAANS Assistant Professor of Neurosurgery Director of Neurotrauma and Neurocritical Care Co-Director of Penn State Spinal Cord Injury Center Head of Penn State Neurologic Sports Injury Program Penn State Hershey Medical Center Hershey, PA, USA

www.medicalebookpdf.com

Chapter 1

ACUTE ISCHEMIC STROKE Nerissa Ko and Wade Smith CLINICAL CASE A 62 year-old woman with hypertension and hyperlipidemia was last seen normal after going to bed around midnight. Her husband awoke when he heard a loud sound in the bathroom at 3:00 am. She was unable to speak but was nodding appropriately. She could not move her right side. Her husband called 911 and paramedics brought her to the ED where a code stroke was activated. Her initial blood pressure was 190/110 mmHg. On examination, she had a right homonymous hemianopia, leftward gaze preference, and a dense right hemiparesis. She was unable to speak, but nodded appropriately to simple questions. Her NIHSS was 20. A CT scan of the head was obtained, and did not show any evidence of hemorrhage. There was a hyperdensity noted in the proximal left middle cerebral artery, with blurring of her insular ribbon on that side. Given that it was 4 hours since she was last seen normal, her husband was consented for IV t-PA within the extended time window of 4.5 hours. Per her husband, she was taking baby aspirin as her only antithrombotic agent. Her Dstick glucose was 110 mg/dl. Her blood pressure was lowered to 80, any anticoagulant use, NIHSS >25, and history of stroke and diabetes mellitus (DM) (see Table 1-1) [11]. Imaging The standard of care is to perform CT or MRI scan of the head prior to t-PA administration. As discussed above, advanced imaging techniques should not be performed if they delay the administration of t-PA. Interpretation of the CT findings should be available within 45 minutes of ED arrival by personnel with specific training. The most important finding is the exclusion of blood products. In addition, most centers use the presence of infarction (hypodensity) within more than 1/3 of the middle cerebral artery territory as an exclusion, given the higher likelihood of hemorrhagic transformation. IV t-PA should be given in the setting of early ischemic changes other than frank hypodensity on CT [7]. t-PA Infusion Table 1-2 lists important steps in IV t-PA administration. Blood pressure must be below 185/110 mmHg and maintained throughout the bolus and 1 hour infusion. It is recommended that the blood pressure continue to be watched closely and maintained below 180/105 mmHg for at least the first 24 hours following t-PA administration. Use of labetolol and nicardipine are considered standard medications to reduce blood pressure to safe targets.

www.medicalebookpdf.com

Management of t-PA complications If the patient’s exam declines during t-PA administration stop the infusion immediately, request the emergency administration of cryoprecipitate, before performing another non-contrast CT scan. Be prepared to support with blood products and additional cryoprecipitate containing Factor VIII. If intracranial hemorrhage is found, neurosurgical evacuation should be considered based on location. Lack of neurosurgical services for hematoma evacuation should not prevent primary stroke centers from administering t-PA, but they should have ready access to neurosurgical services on patient transfer. Additionally, one should follow blood count closely especially if there is any increased risk of bleeding (recent femoral puncture, etc.) The serum half-life of t-PA is short (4-10 minutes) so cessation of the infusion is the quickest method to prevent further bleeding exacerbation, but because t-PA lyses open vessels, bleeding needs to be controlled by standard measures including blood product support. In addition to bleeding complications, other rare events to monitor after t-PA administration include allergic reactions such as anaphylaxis and angioedema. Orolingual angioedema is typically mild and self-limited, but treatment with ranitidine, diphenhydramine and methylprednisolone can be considered to prevent more severe reactions. Endovascular Stroke Therapy Endovascular therapy of AIS is directed at opening large intracranial vessels via catheter infusion of thrombolytic drugs or use of mechanical devices. Reviews of these techniques can be found elsewhere but are briefly reviewed here [13].

www.medicalebookpdf.com

Use of plasminogen activators delivered intraarterially (IA) has been proven effective in the PROACT-I and II trials for acute MCA occlusion treated within the first 6 hours, using a randomized single blind trial design [14]. However, tPA and urokinase are not approved for IA use by the FDA. IA t-PA is used offlabel at many comprehensive stroke centers for large vessel occlusion in patients with AIS within 6 hours of symptom onset based on the PROACT trial data. Mechanical techniques alone or in combination with t-PA have been shown effective at opening the basilar artery, carotid terminus, and middle cerebral arteries [15-17]. In these trials, recanalization correlated with improved clinical outcome and decreased mortality. Six randomized prospective trials with blinded outcome completed in late 2014 have shown that stent retrievers used in anterior circulation ischemia within 6-8 hours improve outcomes and one trial reduced mortality. Stent retrievers are generally preferred to coil retrievers because of their ability to obtain faster and better recanalization rates [18,19]. For patients who are not candidates for thrombolytic therapy based on bleeding risk, mechanical embolectomy remains a viable option in AIS. Similar to t-PA, earlier intervention would increase the likelihood of clinical benefit. Emergent angioplasty and stenting could be considered in selected cases, but with limited data. Many centers administer IV t-PA for eligible AIS patients and then, based on the CT angiogram, MR angiogram, severity of stroke (NIHSS >9 typically) or presence of a hyperdense artery sign on non-contrast CT imaging, decide to take the patient to the angiography suite and attempt to open the vessel if still blocked. This is considered “bridging therapy”. The interventionalist has two main options if they find a persistent large vessel occlusion: (1) infuse a thrombolytic drug (typically t-PA) intra-arterially, and (2) perform mechanical thrombectomy embolectomy using dedicated endovascular devices. The results of the IMS-III trial, which randomized patients to IV t-PA alone versus bridging with additional endovascular techniques, showed no benefit for additional endovascular therapy after IV t-PA [20]. However, the recently completed MR CLEAN trial demonstrated that intra-arterial (IA) treatment administered within 6 hours after onset of anterior circulation stroke due to a documented large vessel occlusion improves functional outcome and does not increase disability [21]. Notably this trial utilized newer generation embolectomy devices and 89% of enrolled subjects had also been treated with intravenous t-PA as well; additionally all patients had a CTA prior to enrollment confirming a large vessel occlusion. Two other recently published clinical trials (ESCAPE and EXTENDIA) confirmed this benefit of endovascular therapy, especially in patients with salvageable brain tissue identified on initial CT imaging [22,23]. These findings

www.medicalebookpdf.com

strongly suggest that the use of endovascular treatment within 6 hours of ischemic stroke onset is beneficial and that eligible patients should receive IV-IA “bridging therapy”. Also, imaging techniques such as CT or MR angiography, and probably perfusion imaging, should be used as part of the initial stroke evaluation to identify appropriate patients. Basilar artery occlusion Basilar artery occlusion is typically caused by embolic occlusion, and also by atherothrombosis. In general, symptomatic occlusion of the basilar artery is associated with a poor prognosis. Signs and symptoms of basilar artery occlusion are variable including alternating hemiparesis, diplopia, stupor and coma. Otherwise unexplained coma in a patient with a normal CT scan should prompt consideration of basilar occlusion, especially if there is pupil asymmetry or asymmetry in the oculocephalic reflex. (Sometimes the basilar clot appears on CT as the “hyperdense basilar artery” sign but this is insensitive; CTA and MRA should be definitive). In addition, the observation of brief myoclonic jerks concomitant with coma suggests basilar occlusion (and may be misdiagnosed as status epilepticus). Treatment of basilar occlusion includes use of IV t-PA, IA tPA or mechanical embolectomy, angioplasty or stenting. There is likely a reduction in mortality if one recanalizes the basilar artery with endovascular methods although this has only been the subject of one small randomized trial (many vascular neurologists feel uncomfortable randomizing such patients to medical therapy alone) [24]. Newer data suggest a benefit of thrombolysis independent of time to treatment especially in the absence of extensive baseline ischemia on initial imaging [25]. Medical management Cardiac monitoring Atrial fibrillation and other potentially serious cardiac arrhythmias are more common after AIS. Continuous cardiac monitoring for at least 24 hours is recommended. Electrocardiography should be considered part of the evaluation of any AIS patient. Additional cardiac evaluations may be considered in patients with suspected cardiac source for their stroke. Temperature Hyperthermia worsens cerebral ischemia in animal models and is associated with increased mortality in observational studies in humans. As discussed above, fever suppression seems prudent yet no randomized data currently exists to support more aggressive means to suppress fever beyond simple antipyretics. Acetaminophen is preferred because it has less risk of gastric ulcer formation. Use of surface or endovascular cooling methods remains investigational. Glucose

www.medicalebookpdf.com

Hypoglycemia exacerbates cerebral infarction and should be treated with glucose administration and reduction in insulin doses. Hyperglycemia should be prevented as well. Insulin infusion for tight control of serum glucose has been shown to decrease all-cause mortality in ICU populations, but can be complicated by hypoglycemia with too aggressive a target glucose. Neurointensivists often recommend tight control with insulin infusion for AIS, but the actual target glucose value and intensity of insulin treatment is unknown. More directed research on AIS patients has failed to show a superior strategy of insulin infusions compared to insulin sliding scales. The exact threshold at which one should treat with insulin is debated. The results of the ongoing SHINE trial will provide specific evidence on the optimal glucose management strategy (NCT01369069). Guideline recommendations suggest keeping serum glucose between 140-180 mg/dL with insulin (infusion or sliding scale) and avoiding hypoglycemia and hypokalemia by concomitant glucose and potassium infusions. DVT prophylaxis AIS patients admitted to NICUs typically are bed-bound and immobile and at high risk of deep venous thrombosis (DVT) and pulmonary embolus. DVT prophylaxis should include subcutaneous anticoagulant administration and/or pneumatic compression devices (unless DVT is already present clinically). Use of sub-cutaneous anticoagulants for prophylaxis is effective in reduction of DVT and is superior to aspirin or external compression devices alone. Recent data shows superiority of low-molecular weight heparin (enoxaparin 40 mg daily) over sub-cutaneous heparin (UFH 5000 IU twice daily) in preventing DVT. Subcutaneous heparin has the advantage of having an antidote (protamine) and is cheaper, but has increased risk of heparin-associated thrombosis compared to LMWH. When to start heparinoids during hospitalization is unclear. Recent trials showed intermittent pneumatic compression but not graduated compression stockings reduced the risk of DVTs. Early mobilization of patients after AIS is recommended [26,27]. DVT should be treated with anticoagulant doses of heparin but this raises the risk of hemorrhagic transformation of AIS. No clear guidelines exist. Some neurointensivists chose aspirin and placement of an inferior vena cava filter. Newer IVC filters may be removed at a later date, preventing the need for lifelong warfarin therapy. Other neurointensivists treat with systemic heparin at anticoagulant doses and convert to warfarin based on clinical judgment about the risk of hemorrhagic transformation. In general, the chance of intracranial hemorrhage decreases the longer the time after the ischemic infarct. Nutrition

www.medicalebookpdf.com

Assessment of the patient’s swallowing ability is essential and should be documented in the patient’s record on admission. Physicians or nurses can clinically clear a patient’s ability to swallow by observing the patient sip water and document any cough. Presence of the gag reflex is not a reliable indicator of swallowing function. Speech therapists may be consulted to document swallowing function as well since they use more advanced techniques to document safety. If swallow ability has not been assessed, the patient should not be fed by mouth. However, early nutrition is important; thus, standing orders that make all stroke patients NPO for the first 24 hours should be discouraged. Energy demand increases during AIS and nutritional support is important for clinical recovery. Nasogastric feeding can be instituted within 24 hours for patients who are unable to swallow and have not received t-PA. If t-PA has been administered, placement of the nasogastric tube should be delayed. Conversion to a percutaneous feeding tube (PEG) may be necessary for patients who do not recover the ability to swallow safely; this route of feeding is both more comfortable and cosmetically desirable, but is invasive and carries risk. Limb restraint and/or diligent bandaging of the PEG site are important in the first few days following placement to avoid self-removal and peritonitis. Nutritional assessment should occur in all stroke patients. Special conditions Induced hypertension/hypervolemia Induced hypertension is used by some neurointensivists as a method to enhance collateral brain perfusion. Limited data on safety exists, and no data on efficacy beyond case series is available to form a recommendation; however, it makes sense in some clinical settings (based on brain perfusion imaging and vascular imaging). Arterial vasodilators such as pentoxifylline have not been shown to be effective, nor has hemodilution. A large trial of 25% albumin did not show benefit compared to saline at 90 days after AIS, and the trial was stopped due to futility [28]. Brain Edema Posterior fossa strokes may produce sufficient cerebellar edema to cause acute hydrocephalus and/or brainstem distortion. This typically results in coma and sudden respiratory failure. Urgent surgical intervention is warranted to decompress the posterior fossa or mitigate prevention and treatment of herniation and brainstem compression. Hydrocephalus should be treated with an external ventricular drain (EVD). An EVD alone may enhance upward herniation so should be considered a temporizing maneuver depending on the anatomy of the stroke. Occlusion of the carotid terminus (intracranial internal carotid artery

www.medicalebookpdf.com

bifurcation into the MCA and the ACA), or occlusion of the MCA M1 branch can cause enough infarction that the subsequent cytotoxic brain edema produces subfalcine and transtentorial herniation. Risk factors are absence of collateral flow (pial-pial arterial collaterals), large area of infarction on initial brain imaging, and younger age (likely because there is less brain atrophy). Cytotoxic edema from ischemic infarction increases the amount of brain water and produces mass effect. This can be imaged with CT or MRI and shows the infarcted hemisphere crossing over the midline with hypodense (CT) or hyperintense (MRI T2 or DWI) signal. Recent guideline recommended strategies to mitigate edema from massive hemispheric infarction include osmotherapy (mannitol, 3% saline, 23.5% saline), hyperventilation (with the caveat that this might enhance ischemia by causing vasoconstriction and decrease in blood flow), hypothermia and hemicraniectomy [29]. Avoidance of hyponatremia from cerebral salt wasting is critical as this complication can occur rapidly (over 12 hours) and marked osmotic shifts can occur that may enhance herniation risk; frequent serum sodium monitoring is important to detect this and intervene early. Use of intravenous and oral salt and maintenance of central venous volume (rather than fluid restriction) is common practice. Use of glucocorticoids to either reduce brain swelling or improve clinical outcome is both ineffective and harmful because of risks of secondary infection and hyperglycemia and should not be administered in AIS patients. The highest level of evidence supports hemicraniectomy as the treatment that both reduces mortality and leads to reasonably good outcomes. Hemicraniectomy for hemispheric infarction with uncal or central herniation is effective in decreasing mortality and improving clinical outcome in survivors. This is based on three randomized controlled trials and in combination- although the total number of randomized cases (N=93 in aggregate) is small- shows a reduction in mortally from 78% to 29% [30]. This is the only stroke therapy that has been shown to reduce mortality. In survivors, outcomes were improved: mRS 3 (43% vs. 21%). This is true regardless of the side of the hemisphere involved; there was no difference in modified Rankin scores between left and right hemisphere patients in these studies; the decision to withhold hemicraniectomy in left hemisphere stroke is discouraged. Note that only patients aged 18 - 60 years were included in these trials. More recent trial results showed a benefit in survival without severe disability in older patients >60 years, although functional outcomes were not as good as younger patients [31]. When to perform hemicraniectomy during the course of massive hemispheric infarction is unclear; enrollment was within 48 hours of stroke onset in these trials. Some physicians recommend performing hemicraniectomy even before signs of

www.medicalebookpdf.com

midline shift or decrease in mental status occur and base their decision on perfusion data or the size of the initial infarct and vascular anatomy (CTA, MRA or conventional angiography). Perhaps many more physicians wait until there is evidence of marked hemispheric swelling, then treat with osmotherapy and only if that fails (with failure defined as clear signs of herniation) go on to perform hemicraniectomy. There are case series of hypothermia for treatment of malignant hemispheric swelling showing amelioration of herniation but return of swelling on cessation of hypothermia. There are other case series and case reports showing that hypothermia can reverse increases in intracranial pressure. Whether hypothermia should be a penultimate step to hemicraniectomy is controversial and further trials are recommended. End-of-life care Unfortunately, despite advances in the acute management of patients with AIS, patients do not always have a satisfactory level of recovery. The transition to end-of-life care involves complex decision-making to align the goals of care with the patient and family wishes guided by informed prognostication of the anticipated outcomes. Providing appropriate palliative care and consultation is an important aspect of neurocritical care of patients with AIS [32]. SECONDARY STROKE PREVENTION This topic is the subject of multiple reviews found elsewhere [33] but a few issues are germane to neurointensivists. Aspirin 325 mg should be given within 24-48 hours to most AIS patients unless a contraindication exists; it should not be administered as adjunctive therapy for patients treated with thrombolytics until 24 hours passes. There is no current evidence to support other intravenous antiplatelet agents or acute clopidogrel loading until further data is available. Much of the controversy surrounds use of anticoagulants. In general, there is no role for anticoagulants in routine stroke treatment except for a few important conditions. Atrial Fibrillation Patients with atrial fibrillation benefit from dose adjusted warfarin to a target INR of 2-3 unless warfarin is contraindicated. There is a growing literature that bridging patients with heparin or LMWH is unnecessary and is associated with an increased bleeding risk. Simply starting oral warfarin once the acute stroke period is over (3-7 days typically) is likely sufficient; stopping aspirin once the warfarin is therapeutic is recommended. Rate control and warfarin sodium has proven superior to rhythm control and conversion to aspirin long term, so the

www.medicalebookpdf.com

utility of acute ICU cardioversion is unclear (unless the patient is considered unstable). Newer target-specific oral anticoagulants have shown efficacy similar to warfarin but with less bleeding complications. Initiation on these newer anticoagulants after acute stroke will depend on risk of hemorrhage, as there is currently no specific reversal for these agents. Carotid Disease Any patient with anterior circulation cerebral ischemia should have an assessment of the ipsilateral carotid. More and more, centers are combining CTA of the neck with the initial brain imaging during stroke so the state of the carotid artery will be typically known on ICU admission. Depending on the amount of injured brain, it is becoming more common to perform carotid endarterectomy (CEA) or carotid stenting during the stroke hospitalization, as the recurrent stroke rate for carotid disease is high. Use of heparin compared with aspirin to “keep the carotid open” is debated, but discouraged. Use of clopidogrel has been associated with excessive neck bleeding following CEA so its routine use before a decision about how the carotid artery will be recanalized may be a problematic strategy. STROKE CENTERS A systems approach to stroke treatment requires a coordinated system, codified extensively under the concept of primary stroke centers [34] and comprehensive stroke centers [35]. In brief, primary stroke centers have an acute stroke team, usually headed by a neurologist that responds to the emergency arrival of stroke patients and assesses them for eligibility for acute treatment. In addition, stroke centers follow established protocols for all stroke patients and report performance measures to the Joint Commission or other state health departments that credential primary stroke centers and monitor quality of stroke care. The development and certification of comprehensive stroke centers is currently being formally defined; comprehensive stroke centers include all the features of a primary stroke center but also have endovascular stroke therapies and likely fellowship training programs in vascular neurology, neurocritical care and endovascular surgery. To further support hospitals without access to adequate imaging interpretation and stroke expertise onsite, use of approved teleradiology systems and telestroke consultation can be useful in increasing the timeliness and use of intravenous t-PA [1]. REFERENCES 1. Jauch EC, Saver JL, Adams HP, Jr., et al. Guidelines for the early

www.medicalebookpdf.com

management of patients with acute ischemic stroke: a guideline for healthcare professionals from the American Heart Association/American Stroke Association. Stroke; a journal of cerebral circulation. 2013;44:870947. 2. Smith WS, Hemphill JC, Johnston SC. Cerebrovascular Diseases , Chapter 446. In Kasper D, Fauci A, Hauser S, Longo D, Jameson J, Loscalzo J. Harrison's Principles of Internal Medicine, 19th ed, McGraw-Hill; 2015. 3. Go AS, Mozaffarian D, Roger VL, et al. Heart disease and stroke statistics-2013 update: a report from the American Heart Association. Circulation.2013;127:e6-e245. 4. Astrup J, Symon L, Branston NM, Lassen NA. Cortical evoked potential and extracellular K+ and H+ at critical levels of brain ischemia. Stroke. 1977;8:51-7. 5. Easton JD, Saver JL, Albers GW, et al. Definition and evaluation of transient ischemic attack: a scientific statement for healthcare professionals from the American Heart Association/American Stroke Association Stroke Council; Council on Cardiovascular Surgery and Anesthesia; Council on Cardiovascular Radiology and Intervention; Council on Cardiovascular Nursing; and the Interdisciplinary Council on Peripheral Vascular Disease. The American Academy of Neurology affirms the value of this statement as an educational tool for neurologists. Stroke. 2009;40:2276-93. 6. Sacco RL, Kasner SE, Broderick JP, et al. An updated definition of stroke for the 21st century: a statement for healthcare professionals from the american heart association/american stroke association. Stroke. 44:2064-89. 7. Latchaw RE, Alberts MJ, Lev MH, et al. Recommendations for imaging of acute ischemic stroke: a scientific statement from the American Heart Association. Stroke. 2009;40:3646-78. 8. Kidwell CS, Jahan R, Gornbein J, et al. A trial of imaging selection and endovascular treatment for ischemic stroke. N Engl J Med. 2013;368:91423. 9. He J, Zhang Y, Xu T, et al. Effects of immediate blood pressure reduction on death and major disability in patients with acute ischemic stroke: the CATIS randomized clinical trial. JAMA. 2014;311:479-89. 10. Tissue plasminogen activator for acute ischemic stroke. The National Institute of Neurological Disorders and Stroke rt-PA Stroke Study Group. N

www.medicalebookpdf.com

Engl J Med. 1995;333:1581-7. 11. Hacke W, Kaste M, Bluhmki E, et al. Thrombolysis with alteplase 3 to 4.5 hours after acute ischemic stroke. N Engl J Med. 2008;359:1317-29. 12. Lees KR, Bluhmki E, von Kummer R, et al. Time to treatment with intravenous alteplase and outcome in stroke: an updated pooled analysis of ECASS, ATLANTIS, NINDS, and EPITHET trials. Lancet.375:1695-703. 13. Smith WS. Technology Insight: recanalization with drugs and devices during acute ischemic stroke. Nat Clin Pract Neurol. 2007;3:45-53. 14. Furlan A, Higashida R, Wechsler L, et al. Intra-arterial prourokinase for acute ischemic stroke. The PROACT II study: a randomized controlled trial. Prolyse in Acute Cerebral Thromboembolism. JAMA. 1999;282:200311. 15. The penumbra pivotal stroke trial: safety and effectiveness of a new generation of mechanical devices for clot removal in intracranial large vessel occlusive disease. Stroke. 2009;40:2761-8. 16. Smith WS, Sung G, Saver J, et al. Mechanical thrombectomy for acute ischemic stroke: final results of the Multi MERCI trial. Stroke. 2008;39:1205-12. 17. Smith WS, Sung G, Starkman S, et al. Safety and efficacy of mechanical embolectomy in acute ischemic stroke: results of the MERCI trial. Stroke. 2005;36:1432-8. 18. Nogueira RG, Lutsep HL, Gupta R, et al. Trevo versus Merci retrievers for thrombectomy revascularisation of large vessel occlusions in acute ischaemic stroke (TREVO 2): a randomised trial. Lancet. 2012;380:123140. 19. Saver JL, Jahan R, Levy EI, et al. Solitaire flow restoration device versus the Merci Retriever in patients with acute ischaemic stroke (SWIFT): a randomised, parallel-group, non-inferiority trial. Lancet. 2012;380:1241-9. 20. 20. Broderick JP, Palesch YY, Demchuk AM, et al. Endovascular therapy after intravenous t-PA ver-sus t-PA alone for stroke. N Engl J Med. 2013;368:893-903. 21. Berkhemer OA, Fransen PS, Beumer D, et al. A randomized trial of intraarterial treatment for acute ischemic stroke. N Engl J Med.

www.medicalebookpdf.com

2015;372:11-20. 22. Campbell BC, Mitchell PJ, Kleinig TJ, et al. Endovascular therapy for ischemic stroke with perfusion-imaging selection. N Engl J Med. 2015;372:1009-1018. 23. Goyal M, Demchuk AM, Menon BK, et al. Randomized assessment of rapid endovascular treatment of ischemic stroke. N Engl J Med. 2015;372:1019-1030. 24. Macleod MR, Davis SM, Mitchell PJ, et al. Results of a multicentre, randomised controlled trial of intra-arterial urokinase in the treatment of acute posterior circulation ischaemic stroke. Cerebrovascular diseases. 2005;20:12-7. 25. Strbian D, Sairanen T, Silvennoinen H, Salonen O, Kaste M, Lindsberg PJ. Thrombolysis of basilar artery occlusion: Impact of baseline ischemia and time. Ann Neurol. 2013;73:688-94. 26. Sherman DG, Albers GW, Bladin C, et al. The efficacy and safety of enoxaparin versus unfractionated heparin for the prevention of venous thromboembolism after acute ischaemic stroke (PREVAIL Study): an openlabel randomised comparison. Lancet. 2007;369:1347-55. 27. Dennis M, Sandercock P, Reid J, Graham C, Forbes J, Murray G. Effectiveness of intermittent pneumatic compression in reduction of risk of deep vein thrombosis in patients who have had a stroke (CLOTS 3): a multicentre randomised controlled trial. Lancet. 2013;382:516-24. 28. Ginsberg MD, Palesch YY, Hill MD, et al. High-dose albumin treatment for acute ischaemic stroke (ALIAS) Part 2: a randomised, double-blind, phase 3, placebo-controlled trial. The Lancet Neurology. 2013;12:1049-58. 29. Wijdicks EF, Sheth KN, Carter BS, et al. Recommendations for the management of cerebral and cerebellar infarction with swelling: a statement for healthcare professionals from the American Heart Association/American Stroke Association. Stroke; a journal of cerebral circulation. 2014;45:1222-38. 30. Vahedi K, Hofmeijer J, Juettler E, et al. Early decompressive surgery in malignant infarction of the middle cerebral artery: a pooled analysis of three randomised controlled trials. The Lancet Neurology. 2007;6:215-22. 31. Juttler E, Unterberg A, Woitzik J, et al. Hemicraniectomy in older patients

www.medicalebookpdf.com

with extensive middle-cerebral-artery stroke. N Engl J Med. 2014;370:1091-100. 32. Holloway RG, Arnold RM, Creutzfeldt CJ, et al. Palliative and end-of-life care in stroke: a statement for healthcare professionals from the American Heart Association/American Stroke Association. Stroke; a journal of cerebral circulation. 2014;45:1887-916. 33. Kernan WN, Ovbiagele B, Black HR, et al. Guidelines for the prevention of stroke in patients with stroke and transient ischemic attack: a guideline for healthcare professionals from the American Heart Association/American Stroke Association. Stroke; a journal of cerebral circulation. 2014;45:2160236. 34. Schwamm LH, Pancioli A, Acker JE, 3rd, et al. Recommendations for the establishment of stroke systems of care: recommendations from the American Stroke Association's Task Force on the Development of Stroke Systems. Stroke. 2005;36:690-703. 35. Leifer D, Bravata DM, Connors JJ, 3rd, et al. Metrics for measuring quality of care in comprehensive stroke centers: detailed follow-up to Brain Attack Coalition comprehensive stroke center recommendations: a statement for healthcare professionals from the American Heart Association/American Stroke Association. Stroke.42:849-77.

www.medicalebookpdf.com

ACUTE ISCHEMIC STROKE QUESTIONS 1. Controlled trials of acute ischemic stroke have found ALL of the following EXCEPT? a. Administration of IV t-PA improves neurological outcome at 3 months b. Aspirin administration within 24 hours of stroke onset lowers secondary stroke risk c. Administration of IV streptokinase is associated with increased risk of intracranial hemorrhage and should not be administered. d. Administration of IV heparin prevents early recurrent stroke e. Intraarterial administration of pro-urokinase improves neurological outcomes in patients with middle cerebral artery occlusions when treated under 6 hours 2. All of the following should be considered for a patient with massive hemispheric infarction with early signs of uncal herniation, EXCEPT? a. Hemicraniectomy only if it is a right hemispheric infarction b. Mannitol c. Hyperventilation d. Frequent serum sodium checks e. Hypertonic saline infusion 3. Please choose the FALSE statement. a. It is now recommended to lower systemic blood pressure below 220/120 mmHg even if no end-organ ischemia is occurring b. Glucocorticoids have no role in the routine treatment of cytotoxic brain edema following acute ischemic stroke c. IV t-PA administration for acute ischemic stroke lowers stroke mortality d. Hemicraniectomy in pooled analysis lowers stroke mortality by approximately 50%

www.medicalebookpdf.com

e. Suboccipital craniotomy for cerebellar decompression of cerebellar infarction has not undergone testing in randomized trials 4. Use of thrombolytic therapy with IV t-PA is indicated in which of the following cases, choose the most appropriate scenario? a. An 85-year old man was found by his caregiver with left-sided weakness one hour after feeding him dinner and brought to the ED within 30 minutes. Head CT showed no blood. His serum glucose was 410 mg/dl. b. An 85-year old man with diabetes and prior stroke on warfarin for atrial fibrillation. He arrived to the ED at 4 hours after onset. Head CT showed no blood, INR 1.5, glucose 100 mg/dl. c. An 85-year old man was brought to the ED in the morning with rightsided weakness. He was last seen normal the evening prior. His head CT showed hypodensity in 2/3 of the left MCA territory. d. An 85-year old man was witnessed to fall out of his chair at the dinner table. He was brought to ED within 30 minutes, and noted to have right-sided weakness. Head CT did not show any blood, serum glucose was 180 mg/dl. e. An 85-year old man was brought to ED with left-sided weakness. He was last seen normal 2 hours ago by family. Head CT showed no blood, glucose on arrival was 150 mg/ dl. His blood pressure was 200/110 mmHg. 5. All of the following statements regarding acute cerebral ischemia are true EXCEPT? a. In focal cerebral ischemia, both necrotic and apoptotic cellular death occurs. b. Salvage of the core infarct is the goal of acute revascularization therapies. c. Hyperglycemia is associated with increased stroke mortality. d. Hyperthermia can increase infarct volume in animal models of ischemia. e. Collateral flow contributes to the ischemic penumbra in focal

www.medicalebookpdf.com

ischemia. 6. Prior to IV t-PA administration, your patient required 2 doses of IV labetalol to get his blood pressure below 185/110mmHg. What is the next step in his blood pressure management? a. No further treatment needed unless blood pressure exceeds 220/120 mmHg. b. No further treatment needed as long as blood pressure remains below 185/110 mmHg. c. Treatment is needed to maintain blood pressure below 180/105 mmHg for 24 hours. d. Treatment is needed to maintain blood pressure below 180/105 mmHg for one hour during the t-PA infusion. 7. In this first 24 hours after acute ischemic stroke not treated with thrombolytics, all of the following are true EXCEPT? a. All patients should remain NPO because of the risk for aspiration. b. Blood glucose should be monitored and maintained between 140-180 mg/dl. c. Pneumatic compression devices should be part of the DVT prophylaxis regimen. d. Fever should be treated with antipyretics. e. Nutritional support should be provided via nasogastric tube in patients unable to swallow safely. 8. The following statements regarding endovascular stroke therapy are true EXCEPT? a. The PROACT trials showed intra-arterial infusion of thrombolytic drugs was superior to medical management for MCA occlusion within 6 hours of onset. b. Intra-arterial use of t-PA is not FDA approved and is used off-label. c. Several devices are approved for embolectomy and cleared for use in AIS.

www.medicalebookpdf.com

d. Current trial data do not support the use of endovascular treatments after IV t-PA. e. Stent retrievers have shown increased recanalization rates compared to coil retrievers. 9. Which of the following statements about antithrombotic therapy after AIS is TRUE? a. Acute anticoagulation with IV heparin is indicated prior to initiation of warfarin for stroke prevention in atrial fibrillation. b. Current trial evidence supports use of IV antiplatelet agents when IV tPA is not given. c. Aspirin 325 mg is recommended within 24 hours (or 48 hours if t-PA is given). d. Loading with clopidogrel is currently recommended after acute stroke. e. Use of thrombin inhibitors is currently recommended after acute stroke. 10. The following statements about imaging in AIS are true EXCEPT? a. The standard of care is to perform CT or MRI scan of the head prior to t-PA administration to exclude intracranial bleeding. b. Hypodensity on CT of greater than 1/3 of the MCA territory is associated with a higher likelihood of hemorrhagic transformation with t-PA administration. c. Vessel imaging should be obtained prior to treatment with IV t-PA. d. MRI diffusion-weighted imaging is more sensitive and specific for acute infarct than CT.

www.medicalebookpdf.com

ACUTE ISCHEMIC STROKE ANSWERS 1. The correct answer is D. IV heparin has not been found to be effective in preventing early recurrent stroke, in preventing neurological worsening, or improving outcomes in AIS; this is based on several large subcutaneous heparin trials (IST and CST). All other answers are correct and are based on published randomized trials. 2. The correct answer is A. Hemicraniectomy was shown to be effective regardless of the side of the brain. These data from three randomized trials found no difference in outcome using the modified Rankin scale at 3 months based on side of the stroke. All other answers are common considerations/treatment that may be employed to reverse/reduce uncal herniation at least temporarily. 3. The correct answer is C. t-PA improves neurological outcome using several standard scales of neurological impairment and disability; however, mortality between those receiving t-PA and those not receiving t-PA is nearly identical. One should not consent patients that t-PA will save their life. All other answers are correct; answer A is new in recent guidelines, and is undergoing randomized study currently. 4. The correct answer is D. IV t-PA is indicated within 3 hours of symptom onset. Brain imaging excluding hemorrhage is required. The only laboratory value required is serum glucose excluding values 400 mg/dl. Blood pressure needs to be 80. 5. The correct answer is B. Salvage of the ischemic penumbra is the target of acute revascularization therapies. The size of the penumbra is often dependent on the degree of collateral flow. Both cytotoxic and apoptotic pathways occur, with infarct volume increased by hyperglycemia and hyperthermia. 6. The correct answer is C. Elevated blood pressure has been associated with increased hemorrhagic complications of t-PA administration. Blood pressure needs to be maintained at 75% of this largest slice and two 25-75% of this largest slice), then C is 4 cm. Total hematoma volume is ABC/2 (5 x 3 x 4)/2 = 30 cc

TREATMENT

www.medicalebookpdf.com

Blood Pressure Elevated blood pressure (BP) is extremely common in the setting of acute ICH. Blood pressure management in this setting remains controversial because of concerns over balancing the competing interests of limiting hematoma expansion or rebleeding while avoiding the theoretical risk of secondary ischemic brain injury by hypoperfusing peri-hematoma brain parenchyma. Studies have conflicted over whether elevated blood pressure predisposes to hematoma expansion after acute ICH [33 ,34]. However, recent studies have suggested that peri-hematoma ischemia is unlikely to be a major contributor to ICH-related brain injury in most cases [16,17]. Until recently there has been very limited data to support specific blood pressure goal recommendations and the last American Heart Association/American Stroke Association guidelines for the management of ICH continued to recommend individualized blood pressure goals based upon individual patient characteristics such as presumed etiology of hemorrhage (hypertension versus underlying vascular anomaly), history of chronic hypertension and baseline blood pressure, and known or suspected major vessel arterial stenosis where a significant decline in blood pressure could cause secondary organ damage [1]. These guidelines suggested the following potential approaches: [1] if systolic blood pressure (SBP) is >200 mmHg or mean arterial pressure (MAP) >150 mmHg then consider aggressive BP reduction with a continuous intravenous infusion and frequent monitoring of BP and neurologic examination; [2] if SBP is >180 mmHg or MAP is >130 mmHg and there is evidence of or suspicion of elevated intracranial pressure (ICP), then consider monitoring ICP and reducing BP using intermittent or continuous intravenous medications to keep the cerebral perfusion pressure (CPP) between 60 and 80 mmHg; [3] if SBP is >180 or MAP is >130 and there is not evidence of or suspicion of elevated ICP, then consider a modest reduction of BP (e.g. MAP < 110 mmHg or target BP < 160/90 mmHg) using continuous or intermittent IV medications to control BP with frequent monitoring of BP and neurologic examination. There is now more evidence from prospective trials that aggressive BPlowering is safe for ICH patients: INTERACT-1 was a multi-center randomized prospective trial (n=400) which demonstrated that intensive lowering of SBP to goal < 140 mmHg as opposed to SBP to goal < 180 mmHg decreased the absolute risk of significant hematoma growth (defined as ≥ 33% of baseline hematoma volume) by 8% without increasing the rate of adverse events [35] (see Table 2-3). INTERACT-2 used the same treatment arms but looked at clinical outcome in 2794 patients with acute ICH [36]. Patients were treated if blood

www.medicalebookpdf.com

pressure was between 150 and 220 mmHg and treatment could be intiated within 6 hours of onset. There was no significant difference in the primary endpoint of death and disability, but there was a significant difference when the ordinal analysis of the modified Rankin Score was considered. Still, there was no significant difference in the reduction hematoma expansion. Given these overall results, it is probably safe and reasonable to consider early rapid reduction of systolic blood pressure to below 140 mmHg, especially if the presenting systolic blood pressure is between 150 and 220 mmHg as in the INTERACT trials. Blood pressure variability may also play a role [37]. The Antihypertensive Treatment of Acute Cerebral Hemorrhage (ATACH) study evaluated the tolerability and safety of targeting 3 different BP goals (SBP 170-200, SBP 140-170, SBP 110-140) using an intravenous nicardipine infusion and found that patients tolerated acute lowering of SBP to the three tiers without significant differences in neurologic deterioration between the three tiers [38]. The follow-up phase III clinical trial, ATACH-2, is currently ongoing, with BP needing to be lowered within 4.5 hours and assessing the two SBP targets of 140 and 180 mmHg [39]. While the choice of BP lowering agent should be individualized based on factors such as heart rate and medical comorbidities (e.g. renal or heart failure), our usual preference is to use agents that preferentially affect cardiac output or are arterial vasodilators such as bolus doses of intravenous labetalol or continuous intravenous infusion of nicardipine. We try to avoid medications, which might cause significant venodilation such as hydralazine or nitroprusside. The substances used in INTERACT ranged from the alpha-adrenergic antagonist ( e.g. Urapidil, in about 1/3 of intensive blood pressure treatment) to calciumchannel blockers (as nicardipine or nimodipine) to combined alpha- and betablockers (as labetolol), diuretics, nitroprusside, hydralazine and others [40].

www.medicalebookpdf.com

Coagulopathy ICH is more frequent in patients treated with anticoagulants and fibrinolytics, and the risk of warfarin-related ICH increases with increasing INR. Warfarin-related ICH is associated with an even higher rate of mortality than ICH in the absence of coagulopathy and ongoing bleeding in warfarin-related ICH continues for a more prolonged duration [41]. The main problem with warfarin-related ICH is hematoma expansion. But it occurs at an even higher rate compared with non-coagulopathic ICH (up to 50% of patients) and over an even longer period of up to 60 hours, while with non-coagulopathic spontaneous ICH the majority (30% of patients) suffers hematoma expansion within 4 hours after symptom onset [41]. The obvious goal is to urgently reverse the coagulopathy as soon as possible. While this has historically been done using vitamin K and FFP, it is now recognized that this approach is suboptimal and often leads to excessively slow correction or failure to correct the coagulopathy entirely [42]. Current guidelines [43] recommend the use of vitamin K 5-10 milligrams usually administered intravenously by slow push and concurrent treatment with a more rapidly acting reversal agent as it usually takes hours after vitamin K administration for

www.medicalebookpdf.com

reversal of warfarin-induced coagulopathy. Full warfarin correction usually necessitates the administration of large volumes of FFP and the logistics surrounding cross-matching, thawing, and infusion rates makes this generally a slower option for correction. Consequently, recent interest has turned to the use of concentrated factor preparations such as prothrombin complex concentrate (PCC). Prothrombin complex concentrate administration generally reverses an elevated INR more rapidly than FFP [44] and consequently may be more advantageous in limiting hematoma growth due to ongoing warfarin-related coagulopathy. However, in a retrospective study comparing PCC and FFP, there was no difference in hematoma growth between FFP and PCC in patients whose INR was corrected within 2 hours [28]. This strongly suggests that it is timing of coagulopathy reversal, not a specific agent, that makes the difference. The International normalised ratio normalisation in patients with coumarin-related intracranial haemorrhages (INCH)-trial is currently randomizing patients with ICH related to oral vitamin-K antagonists to treatment with FFP or PCC initiated within 3 hours after onset of the bleeding [45]. Various current guidelines for warfarin-reversal in the setting of life-threatening hemorrhage now emphasis the use of a rapid reversal agent such as PCC or recombinant Factor VIIa in addition to Vitamin K [43,46,47]. Limited data exist on ICH related to unfractionated heparin. In these cases the application of protamine sulfate should be considered depending on the time from cessation of heparin (as a rule of thumb 1 mg of protamine should be given for every 100 units of heparin received by the patient within the last 4 hours, with a maximum protamine dose of 50 mg). There are only case reports on hemostatic treatment in instances of ICH related to intravenous t-PA [48]. Intracranial bleeding rates with novel oral anticoagulants (NOAC: apixaban, dabigatran, rivaroxaban) are significantly lower than with warfarin. Still hemorrhage occurrence from NOACs may become more frequent as these agents become prescribed more frequently, ostensibly because of the better risk-benefit profile compared with warfarin. Whether and how often hematoma expansion (HE) occurs in association with NOAC-ICH is unclear, though mortality rates from a retrospective analysis of bleeding rates from the RELY-trial suggest that HE does occur [49]. Thus far there is no specific reversal agent for any of the available NOACs, though there are some substances being tested in phase II trials [50,51]. Until specific agents are available factor concentrates might be considered [52]. PCC may be considered for apixaban or rivaroxaban and recombinant Factor VIIa might be considered for dabigatran, with activated charcoal or dialysis a consideration for all of these as well. However, optimal treatment of NOAC-related ICH remains unclarified and a point of great

www.medicalebookpdf.com

speculation. Hemostatic Agents The recognition that hematoma expansion worsens outcome and is common even in the absence of coagulopathy has generated significant interest in the potential use of hemostatic agents to limit hematoma growth. Developed as an agent for the treatment of a subset of hemophiliac patients, recombinant Factor VIIa (rFVIIa) has now been investigated in a wide range of bleeding disorders in patients with normal coagulation, including ICH [53]. In a phase IIa trial, 399 acute ICH patients who had initial CT diagnosis within 3 hours of symptom onset received either placebo or one of three doses of recombinant Factor VIIa (40, 80, or 160 μg/kg) within one hour of CT scan. Overall, patients who received rFVIIa had less hematoma expansion and this translated to a lower risk of mortality and improved functional outcome, despite a small increase in thrombotic events such as myocardial infarction [54]. Given these encouraging results, a larger phase III trial including 821 patients was conducted with essentially the same inclusion criteria, but comparing placebo and two doses of rFVIIa (20 and 80 μg/kg). In this pivotal phase III trial, hematoma expansion was once again significantly reduced by treatment with rFVIIa. However, there was no statistically significant change in the proportion of patients who died or were severely disabled [55]. Post-hoc analysis suggested that the subset of the study population who were < 70 years old and who had baseline ICH volumes < 60 mL, intraventricular hemorrhage volume < 5 mL, and time from onset-totreatment < 2.5 hours may have clinically benefited from being administered the drug [56]. However, this should be considered as an exploratory analysis for identifying a target population for an additional clinical trial. At present hemostatic therapy cannot be recommended as routine treatment for ICH patients without coagulopathy.

www.medicalebookpdf.com

Figure 2-3. Right thalamic ICH with positive spot sign on CT angiography. A) Non-contrast CT. B) Spot-sign is seen in center of hematoma (red arrow).

Antiplatelet agents and ICH There are conflicting reports as to the role of prior antiplatelet therapy on hematoma expansion and outcome for patients presenting with ICH [1,57,58]. Consequently, there is wide heterogeneity in clinical practice ranging from practitioners who advocate platelet transfusion in patients with ICH while taking antiplatelet agents such as aspirin or clopidogrel, to those who advocate the use of laboratory tests for platelet function, to those who choose not to treat. Evaluation of the placebo group from a neuroprotective ICH study did not find an association between antiplatelet use and hematoma expansion or outcome [59]. In contrast, recently published work on antiplatelet use and platelet function has suggested that the results of platelet activity assays (but not merely the history of aspirin usage) correlated with occurrence of IVH, a greater ICH Score, hematoma growth, and worse outcomes in ICH [60,61]. Given the widespread use of antiplatelet agents, further clarification of the impact of antiplatelet use and platelet dysfunction on ICH occurrence, growth, and outcome is an important future direction. Intensive Care Management Intracranial Pressure Patients with moderate or large ICH or intraventricular hemorrhage often have increased intracranial pressure (ICP) or hydrocephalus that warrants

www.medicalebookpdf.com

consideration of treatment. The AHA/ASA guidelines advocate a graded stepwise approach with initial routine use of less invasive measures prior to instituting more invasive measures. These less invasive measures include elevation of the head of the bed to 30 degrees, maintenance of the neck in a neutral position to facilitate jugular venous drainage, and adequate analgesia and sedation. Prophylactic mannitol use prior to development of intracranial hypertension in the setting of ICH has not been shown to be beneficial [62]. More invasive measures include CSF drainage via an extraventricular drain (EVD) placed directly into the ventricles. An EVD allows continuous measurement of intracranial pressure as well as drainage of CSF to treat elevated ICP, but does carry a small risk of hemorrhage or infection. Osmotic agents such as mannitol and hyper-tonic saline may be used to decrease ICP, but overuse of mannitol may cause hypovolemia, renal failure, and cerebral vasoconstriction. Neuromuscular blockade may also be considered in patients with refractory elevated ICP but is likely associated with an increased risk of infection and critical illness neuromuscular disease. While hyperventilation may rapidly reduce elevated ICP by causing cerebral arterial vasoconstriction, this effect is generally transient (few hours) and reduces cerebral blood flow which might potentially engender secondary brain injury. Thus, hyperventilation is usually used as a temporizing measure in preparation for other more definitive medical or surgical treatments. Finally, barbiturate coma may be considered in patients that have failed other therapies but is associated with a significant risk of hypotension and requires continuous electroencephalographic monitoring to titrate effective dosing. Induced hypothermia to 32 to 34 degrees Celsius may also be attempted for a brief period, but is associated with a high rate of complications. The use of barbiturate coma and induced hypothermia have not been systematically investigated in ICH and are presently considered salvage second-tier therapies. Hemicraniectomy was retrospectively studied in 12 consecutive patients with 11 patients surviving, half of them with a modified Rankin Score between 0 and 3 [63]. Still, none of these treatments have been proven beneficial in prospective trials on acute ICH. Moreover, the question on how to treat elevated ICP also brings forth the question on what ICP values to treat. Ziai and co-workers performed an analysis of ICP-recordings in patients with IVH and ICH of less than 30 ml who required an EVD (n=100) [64]. Ninety percent of ICP readings were below 20mmHg, and about 2% above 30 mmHg. The percentage of readings above 30mmHg was an independent predictor of mortality (p 9). There are a number of case series which report that patients with spontaneous cerebellar hemorrhage who present with large cerebellar hematomas (> 3 cm in diameter) or with compression of the brain stem or hydrocephalus may still have a favorable outcome with surgical intervention. However, there has not been a prospective randomized trial of surgery for cerebellar ICH analogous to STICH. Even so, cerebellar ICH is generally considered as a potentially surgical lesion by most neurologists and neurosurgeons, especially in patients with obstructive hydrocephalus or clinical deterioration. The 2015 AHA/ASA ICH management guidelines recommend surgical removal of the hematoma as soon as possible in patients with cerebellar hemorrhage who are deteriorating neurologically or who have brain stem compression and/or hydrocephalus from ventricular obstruction [1]. A number of minimally invasive surgical alternatives to open craniotomy have also been considered and studied in small case series or pilot clinical trials. These techniques include: simple aspiration of the hematoma; mechanical aspiration with a screw and suction technique, instillation of a thrombolytic such as urokinase or recombinant tissue plasminogen activator into the hematoma with aspiration of contents, and endoscopic aspiration of the hematoma with lavage of the hematoma cavity and photocoagulation of oozing vessels. An NIH sponsored phase III multicenter trial is currently underway comparing catheter directed t-PA treatment for hematoma evacuation versus conventional medical management for patients presenting with ICH

www.medicalebookpdf.com

(http://clinicaltrials.gov/ct2/show/NCT01827046). This trial is based on the results of the minimally invasive surgery and t-PA in ICH evacuation phase II clinical trial (MISTIE II) that found surgery plus t-PA as generally safe and associated with a significant reduction in perihematomal edema [78]. Another phase III trial (CLEAR III) is evaluating the effectiveness of catheter-directed tPA for the treatment of intraventricular hemorrhage (http://clinicaltrials.gov/ct2/show/NCT00784134). REFERENCES 1. Hemphill JC, 3rd, Greenberg SM, Anderson CS, et al. Guidelines for the management of spontaneous intracerebral hemorrhage: a guideline for healthcare professionals from the American Heart Association/American Stroke Association. Stroke 2015;46:2032-60. 2. Qureshi AI, Tuhrim S, Broderick JP, Batjer HH, Hondo H, Hanley DF. Spontaneous intracerebral hemorrhage. New England Journal of Medicine. 2001;344:1450-60. 3. van Asch CJ, Luitse MJ, Rinkel GJ, van der Tweel I, Algra A, Klijn CJ. Incidence, case fatality, and functional outcome of intracerebral haemorrhage over time, according to age, sex, and ethnic origin: a systematic review and meta-analysis. Lancet Neurol. 2010;9:167-76. 4. Taylor TN, Davis PH, Torner JC, Holmes J, Meyer JW, Jacobson MF. Lifetime cost of stroke in the United States. Stroke. 1996;27:1459-66. 5. Becker KJ, Baxter AB, Cohen WA, et al. Withdrawal of support in intracerebral hemorrhage may lead to self-fulfilling prophecies. Neurology. 2001;56:766-72. 6. Hemphill JC, 3rd, Newman J, Zhao S, Johnston SC. Hospital usage of early do-not-resuscitate orders and outcome after intracerebral hemorrhage. Stroke. 2004;35:1130-4. 7. Zahuranec DB, Brown DL, Lisabeth LD, et al. Early care limitations independently predict mortality after intracerebral hemorrhage. Neurology. 2007;68:1651-7. 8. O'Donnell HC, Rosand J, Knudsen KA, et al. Apolipoprotein E genotype and the risk of recurrent lobar intracerebral hemorrhage. New England Journal of Medicine. 2000;342:240-5.

www.medicalebookpdf.com

9. Dierksen GA, Skehan ME, Khan MA, et al. Spatial relation between microbleeds and amyloid deposits in amyloid angiopathy. Annals of Neurology. 2010;68:545-8. 10. Al-Shahi Salman R, Labovitz DL, Stapf C. Spontaneous intracerebral haemorrhage. Bmj. 2009;339:b2586. 11. Gebel JM, Jr., Jauch EC, Brott TG, et al. Relative edema volume is a predictor of outcome in patients with hyperacute spontaneous intracerebral hemorrhage. Stroke. 2002;33:2636-41. 12. Florczak-Rzepka M, Grond-Ginsbach C, Montaner J, Steiner T. Matrix Metalloproteinases in human spontaneous intracerebral hemorrhage – an update. Cerebrovascular Diseases. 2012;34:249–62. 13. Aronowski J, Hall CE. New horizons for primary intracerebral hemorrhage treatment: experience from preclinical studies. Neurol Res. 2005;27:268-79. 14. Wagner KR. Modeling intracerebral hemorrhage: glutamate, nuclear factorkappa B signaling and cytokines. Stroke. 2007;38:753-8. 15. Xi G, Keep RF, Hoff JT. Mechanisms of brain injury after intracerebral haemorrhage. Lancet Neurol. 2006;5:53-63. 16. Qureshi AI, Wilson DA, Hanley DF, Traystman RJ. No evidence for an ischemic penumbra in massive experimental intracerebral hemorrhage. Neurology. 1999;52:266-72. 17. Zazulia AR, Diringer MN, Videen TO, et al. Hypoperfusion without ischemia surrounding acute intracerebral hemorrhage. Journal of Cerebral Blood Flow and Metabolism. 2001;21:804-10. 18. Butcher KS, Jeerakathil T, Hill M, et al. The intracerebral hemorrhage acutely decreasing arterial pressure trial. Stroke. 2013;44:620-6. 19. Gould B, McCourt R, Asdaghi N, et al. Autoregulation of Cerebral Blood Flow is Preserved in Primary Intracerebral Hemorrhage. Stroke. 2013;44:1726-8. 20. Brott T, Broderick J, Kothari R, et al. Early hemorrhage growth in patients with intracerebral hemorrhage. Stroke. 1997;28:1-5. 21. Davis SM, Broderick J, Hennerici M, et al. Hematoma growth is a determinant of mortality and poor outcome after intracerebral hemorrhage. Neurology. 2006;66:1175-81.

www.medicalebookpdf.com

22. Weir CJ, Murray GD, Adams FG, Muir KW, Grosset DG, Lees KR. Poor accuracy of stroke scoring systems for differential clinical diagnosis of intracranial haemorrhage and infarction. Lancet. 1994;344:999-1002. 23. Broderick JP, Brott TG, Duldner JE, Tomsick T, Huster G. Volume of intracerebral hemorrhage. A powerful and easy-to-use predictor of 30-day mortality. Stroke. 1993;24:987-93. 24. Hemphill JC, 3rd, Bonovich DC, Besmertis L, Manley GT, Johnston SC. The ICH score: a simple, reliable grading scale for intracerebral hemorrhage. Stroke. 2001;32:891-7. 25. Tuhrim S, Horowitz DR, Sacher M, Godbold JH. Validation and comparison of models predicting survival following intracerebral hemorrhage. Crit Care Med. 1995;23:950-4. 26. Hemphill JC, 3rd, Farrant M, Neill TA, Jr. Prospective validation of the ICH Score for 12-month functional outcome. Neurology. 2009;73:1088-94. 27. Kothari RU, Brott T, Broderick JP, et al. The ABCs of measuring intracerebral hemorrhage volumes. Stroke. 1996;27:1304-5. 28. Huttner HB, Schellinger PD, Hartmann M, et al. Hematoma growth and outcome in treated neurocritical care patients with intracerebral hemorrhage related to oral anticoagulant therapy: comparison of acute treatment strategies using vitamin K, fresh frozen plasma, and pro-thrombin complex concentrates. Stroke. 2006;37:1465-70. 29. Kidwell CS, Chalela JA, Saver JL, et al. Comparison of MRI and CT for detection of acute intracerebral hemorrhage. Jama. 2004;292:1823-30. 30. Becker KJ, Baxter AB, Bybee HM, Tirschwell DL, Abouelsaad T, Cohen WA. Extravasation of radiographic contrast is an independent predictor of death in primary intracerebral hemorrhage. Stroke. 1999;30:2025-32. 31. Goldstein JN, Fazen LE, Snider R, et al. Contrast extravasation on CT angiography predicts hematoma expansion in intracerebral hemorrhage. Neurology. 2007;68:889-94. 32. Demchuk AM, Dowlatshahi D, Rodriguez-Luna D, et al. Prediction of haematoma growth and outcome in patients with intracerebral haemorrhage using the CT-angiography spot sign (PREDICT): a prospective observational study. Lancet Neurol. 2012;11:307-14. 33. Jauch EC, Lindsell CJ, Adeoye O, et al. Lack of evidence for an association

www.medicalebookpdf.com

between hemodynamic variables and hematoma growth in spontaneous intracerebral hemorrhage. Stroke. 2006;37:2061-5. 34. Ohwaki K, Yano E, Nagashima H, Hirata M, Nakagomi T, Tamura A. Blood pressure management in acute intracerebral hemorrhage: relationship between elevated blood pressure and hematoma enlargement. Stroke. 2004;35:1364-7. 35. Anderson CS, Huang Y, Wang JG, et al. Intensive blood pressure reduction in acute cerebral haemorrhage trial (INTERACT): a randomised pilot trial. Lancet Neurol. 2008;7:391-9. 36. Anderson CS, Heeley E, Huang Y, et al. Rapid Blood-Pressure Lowering in Patients with Acute Intracerebral Hemorrhage. New England Journal of Medicine. 2013. 37. Manning L, Hirakawa Y, Arima H, et al. Blood pressure variability and outcome after acute intra-cerebral haemorrhage: a post-hoc analysis of INTERACT2, a randomised controlled trial. Lancet Neurol. 2014;13:36473. 38. Anti hypertensive treatment of acute cerebral hemorrhage. Crit Care Med. 2010;38:637-48. 39. Qureshi AI, Palesch YY. Anti hypertensive Treatment of Acute Cerebral Hemorrhage (ATACH) II: design, methods, and rationale. Neurocritical care. 2011;15:559-76. 40. Anderson CS, Heeley E, Huang Y, et al. Supplemental Appendix: Rapid Blood-Pressure Lowering in Patients with Acute Intracerebral Hemorrhage. New England Journal of Medicine. 2013. 41. Flibotte JJ, Hagan N, O'Donnell J, Greenberg SM, Rosand J. Warfarin, hematoma expansion, and outcome of intracerebral hemorrhage. Neurology. 2004;63:1059-64. 42. Goldstein JN, Thomas SH, Frontiero V, et al. Timing of fresh frozen plasma administration and rapid correction of coagulopathy in warfarin-related intracerebral hemorrhage. Stroke. 2006;37:151-5. 43. Ansell J, Hirsh J, Poller L, Bussey H, Jacobson A, Hylek E. The pharmacology and management of the vitamin K antagonists: the Seventh ACCP Conference on Antithrombotic and Thrombolytic Therapy. Chest. 2004;126:204S-33S.

www.medicalebookpdf.com

44. Makris M, Greaves M, Phillips WS, Kitchen S, Rosendaal FR, Preston EF. Emergency oral anticoagulant reversal: the relative efficacy of infusions of fresh frozen plasma and clotting factor concentrate on correction of the coagulopathy. Thromb Haemost. 1997;77:477-80. 45. Steiner T, Freiberger A, Griebe M, et al. International normalised ratio normalisation in patients with coumarin-related intracranial haemorrhages the INCH trial: a randomised controlled multicentre trial to compare safety and preliminary efficacy of fresh frozen plasma and pro-thrombin complex - study design and protocol. International Journal of Stroke. 2011;6:271-7. 46. Baker RI, Coughlin PB, Gallus AS, Harper PL, Salem HH, Wood EM. Warfarin reversal: consensus guidelines, on behalf of the Australasian Society of Thrombosis and Haemostasis. Med J Aust. 2004;181:492-7. 47. Hanley JP. Warfarin reversal. Journal of clinical Pathology. 2004;57:1132-9. 48. French KF, White J, Hoesch RE. Treatment of intracerebral hemorrhage with tranexamic acid after thrombolysis with tissue plasminogen activator. Neurocritical care. 2012;17:107-11. 49. Hart RG, Diener H-C, Yang S, et al. Intracranial hemorrhage in atial fibrillation patients during anticoagulation with warfarin or dabigatran The RE-LY trial. Stroke. 2012;43:1511-7. 50. Dolgin E. Antidotes edge closer to reversing effects of new blood thinners. Nat Med. 2013;19:251. 51. Bakhru S, Laulicht B, Lee C, et al. Small Molecule Antidote for Anticoagulants. In: Thrombosis and Hemostasis Summit of North America (THSNA). Chicago; 2012. 52. Dizdarevic K, Hamdan A, Omerhodzic I, Kominlija-Smajic E. Modified Lund concept versus cerebral perfusion pressure-targeted therapy: a randomised controlled study in patients with secondary brain ischaemia. Clin Neurol Neurosurg. 2012;114:142-8. 53. Mayer SA, Brun NC, Broderick J, et al. Safety and feasibility of recombinant factor VIIa for acute intracerebral hemorrhage. Stroke. 2005;36:74-9. 54. Mayer SA, Brun NC, Begtrup K, et al. Recombinant activated factor VII for acute intracerebral hemorrhage. The New England journal of medicine. 2005;352:777-85.

www.medicalebookpdf.com

55. Mayer SA, Brun NC, Begtrup K, et al. Efficacy and safety of recombinant activated factor VII for acute intracerebral hemorrhage. New England Journal of Medicine. 2008;358:2127-37. 56. Mayer SA, Davis SM, Skolnick BE, et al. Can a subset of intracerebral hemorrhage patients benefit from hemostatic therapy with recombinant activated factor VII? Stroke. 2009;40:833-40. 57. Foerch C, Sitzer M, Steinmetz H, Neumann-Haefelin T. Pretreatment with antiplatelet agents is not independently associated with unfavorable outcome in intracerebral hemorrhage. Stroke. 2006;37:2165-7. 58. Toyoda K, Okada Y, Minematsu K, et al. Antiplatelet therapy contributes to acute deterioration of intracerebral hemorrhage. Neurology. 2005;65:10004. 59. Sansing LH, Messe SR, Cucchiara BL, Cohen SN, Lyden PD, Kasner SE. Prior antiplatelet use does not affect hemorrhage growth or outcome after ICH. Neurology. 2009;72:1397-402. 60. Naidech AM, Jovanovic B, Liebling S, et al. Reduced platelet activity is associated with early clot growth and worse 3-month outcome after intracerebral hemorrhage. Stroke. 2009;40:2398-401. 61. Naidech AM, Bernstein RA, Levasseur K, et al. Platelet activity and outcome after intracerebral hemorrhage. Annals of Neurology. 2009;65:352-6. 62. Misra UK, Kalita J, Ranjan P, Mandal SK. Mannitol in intracerebral hemorrhage: a randomized controlled study. Journal of the neurological sciences. 2005;234:41-5. 63. Murthy JM, Chowdary GV, Murthy TV, Bhasha PS, Naryanan TJ. Decompressive craniectomy with clot evacuation in large hemispheric hypertensive intracerebral hemorrhage. Neurocritical care. 2005;2:258-62. 64. Ziai WC, Melnychuk E, Thompson CB, Awad I, Lane K, Hanley DF. Occurrence and impact of intracranial pressure elevation during treatment of severe intraventricular hemorrhage. Critical Care Medicine. 2012;40:1601-8. 65. Stein DM, Hu PF, Brenner M, et al. Brief episodes of intracranial hypertension and cerebral hypoperfusion are associated with poor functional outcome after severe traumatic brain injury. Journal of Trauma.

www.medicalebookpdf.com

2011;71:364-73; discussion 73-4. 66. Santos E, Diedler J, Sykora M, et al. Low-frequency sampling for PRx calculation does not reduce prognostication and produces similar CPPopt in intracerebral haemorrhage patients. Acta Neurochirurgica (Wien). 2011;153:2189-95. 67. Eide PK, Bentsen G, Sorteberg AG, Marthinsen PB, Stubhaug A, Sorteberg W. A randomized and blinded single-center trial comparing the effect of intracranial pressure and intracranial pressure wave amplitude-guided intensive care management on early clinical state and 12-month outcome in patients with aneurysmal subarachnoid hemorrhage. Neurosurgery. 2011;69:110515. 68. Van den Berghe G, Schoonheydt K, Becx P, Bruyninckx F, Wouters PJ. Insulin therapy protects the central and peripheral nervous system of intensive care patients. Neurology. 2005;64:134853. 69. Oddo M, Schmidt JM, Carrera E, et al. Impact of tight glycemic control on cerebral glucose metabolism after severe brain injury: a microdialysis study. Critical Care Medicine. 2008;36:32338. 70. Vespa P, McArthur DL, Stein N, et al. Tight glycemic control increases metabolic distress in traumatic brain injury: a randomized controlled within-subjects trial. Critical Care Medicine. 2012;40:1923-9. 71. Lacut K, Bressollette L, Le Gal G, et al. Prevention of venous thrombosis in patients with acute intracerebral hemorrhage. Neurology. 2005;65:865-9. 72. Boeer A, Voth E, Henze T, Prange HW. Early heparin therapy in patients with spontaneous intra-cerebral haemorrhage. J Neurol Neurosurg Psychiatry. 1991;54:466-7. 73. Messe SR, Sansing LH, Cucchiara BL, Herman ST, Lyden PD, Kasner SE. Prophylactic antiepileptic drug use is associated with poor outcome following ICH. Neurocritical care. 2009;11:38-44. 74. Gregson BA, Mendelow AD. International variations in surgical practice for spontaneous intra-cerebral hemorrhage. Stroke. 2003;34:2593-7. 75. Mendelow AD, Gregson BA, Fernandes HM, et al. Early surgery versus initial conservative treatment in patients with spontaneous supratentorial intracerebral haematomas in the International Surgical Trial in Intracerebral Haemorrhage (STICH): a randomised trial. Lancet. 2005;365:387-97.

www.medicalebookpdf.com

76. Mendelow AD, Gregson BA, Rowan EN, Murray GD, Gholkar A, Mitchell PM. Early surgery versus initial conservative treatment in patients with spontaneous supratentorial lobar intracerebral haematomas (STICH II): a randomised trial. Lancet. 2013. 77. Gregson BA, Broderick JP, Auer LM, et al. Individual patient data subgroup meta-analysis of surgery for spontaneous supratentorial intracerebral hemorrhage. Stroke. 2012;43:1496-504. 78. Mould WA, Carhuapoma JR, Muschelli J, et al. Minimally invasive surgery plus recombinant tissue-type plasminogen activator for intracerebral hemorrhage evacuation decreases perihematomal edema. Stroke. 2013;44:627-34.

www.medicalebookpdf.com

INTRACEREBRAL HEMORRHAGE QUESTIONS 1. Which of the following is NOT a well recognized independent predictor of acute ICH outcome? a. GCS score b. Advanced patient age c. Warfarin use with elevated INR at ICH onset d. Prehospital systolic blood pressure e. Intraventricular hemorrhage 2. Based on existing RCTs, lowering blood pressure in acute ICH below 140 mmHg within one hour: a. ... leads to a substantial clinical benefit for patients b. ... is associated with perilesional ischemia c. ... is probably safe d. ...can only be achieved with labetolol 3. Which of the following has been tested in a phase III randomized for acute ICH? a. Maintaining ICP < 20 mmHg b. Factor VIIa administration within 4 hours of onset c. Acute warfarin reversal d. Neuroprotection with a free-radical scavenger e. Use of the iron chelating agent deferoxamine 4. The “spot sign” is: a. A clinical sign seen on fundoscopy and indicating chronic hypertension. b. Another name for round hematomas in the cerebellum

www.medicalebookpdf.com

c. Frequently seen on non-contrast head CT d. Associated with mortality at 3 moths e. Reversed with factor VIIa treatment 5. Probable mechanisms of secondary brain injury after ICH include all of the following EXCEPT: a. Fever b. Seizures c. Hematoma expansion d. Iron toxicity e. Perihematoma ischemia 6. The STICH II trial: a. Has shown that early hematoma evacuation in lobar hemorrhage without IVH does improve outcome b. Has shown that early hematoma evacuation in basal ganglia hemorrhage > 60 cc in patients under age 50 is effective c. included patients with lobar ICH and no IVH when there was uncertainty about whether to apply hematoma evacuation or conservative treatment strategies d. did reveal that hematoma evacuation reduced perihematomal edema e. looked at modified Rankin Score as primary endpoint 7. ICH related to oral vitamin-K-antagonists (e.g. warfarin): a. Is best treated with recombinant Factor VIIa b. Is best treated with prothrombin complex concentrate and Vitamin K c. Should be treated with platelet transfusions if due to aspirin or clopidogrel d. Should be reversed within 24 hours after onset e. Is associated with prolonged hematoma expansion and worsened

www.medicalebookpdf.com

outcome 8. Prophylaxis against deep vein thrombosis after ICH: a. Is rarely needed because DVT is uncommon b. Can be safely done with low molecular weight heparin within the first day c. Should be initiated immediately using intermittent compression devices on the legs d. Should be avoided if an external ventricular drain is in place e. Is contraindicated if a “spot sign” is present 9. Anticonvulsant prophylaxis after ICH: a. Should be initiated using phenytoin in most patients for a 7 day course b. Should not be used unless continuous EEG is available c. May be associated with worsened long-term functional outcome d. Decreases the risk of subsequent seizures e. Should be done using levetiracetam because of its more favorable safety profile 10. Intracranial pressure monitoring after ICH: a. Has been shown to improve outcome b. Should only be done in conjunction with brain tissue oxygen monitoring c. Is strongly recommended for comatose salvageable patients d. Should only be done as part of a randomized clinical trial e. Allows CPP to be assessed and targeted

www.medicalebookpdf.com

INTRACEREBRAL HEMORRHAGE ANSWERS 1. The correct answer is D. While acutely elevated blood pressure has been associated with worsened outcome in some series, different studies have conflicted regarding the importance of this and prehospital blood pressure has not been identified as an independent predictor. 2. The correct answer is C. INTERACT-2 demonstrated that acute lowering of systolic blood pressure below 140 mmHg within 1 hour does not increase mortality in these patients. Other RCT (INTERACT-1 and ATACH-1) do support this finding. A clinical benefit of this treatment has still not been proven. 3. The correct answer is B. The only phase III trials completed to date for acute ICH are for recombinant Factor VIIa (FAST) and supratentorial hematoma evacuation (STICH). The free radical trapping agent NXY-059 was studied in a phase II trial (CHANT) and a small clinical trial of defuroxamine is being initiated. Despite recommended treatment guidelines, warfarin-reversal has not been systematically studied. 4. The correct answer is D. The “spot sign” is a radiographic finding which may be seen on CT scan after administration of intravenous contrast. It is thought to represent extravasation of contrast material from the intravascular to the extravascular space. The PREDICT trial could not prove that it is associated with risk of hematoma expansion, but did show that it was significantly linked to mortality. 5. The correct answer is E. Precise mechanisms of secondary brain injury after intracerebral hemorrhage are still being elucidated. Peri-hematoma ischemia has long been considered as a concern given findings of low cerebral blood flow. However, animal studies and human neuro-imaging studies have not found true ischemia (as an impaired CMRO2) as a common peri-he-matoma finding. The lack of significant peri-hematoma ischemia has allayed some concerns regarding safety of acute blood pressure lowering, which is currently being evaluated in at least two phase III clinical trials. 6. The correct answer is C. STICH II could not demonstrate a clinical benefit of patient with lobar ICH and no ICH. The interpretation of the trial is difficult since 20 percent of patients who were initially randomized to conservative treatment were switched to surgery.

www.medicalebookpdf.com

7. The correct answer is E. While coagulopathy, especially due to warfarin, worsens outcome and prolongs bleeding after ICH, the optimal reversal regimen has not been clearly defined. Guidelines recommend urgent reversal and 24 hours is much too long (preferred goal 2 hours or less). The utility of platelet transfusions remains speculative. 8. The correct answer is C. DVT is common after ICH, occurring in 5-15% of patients. Intermittent compression stockings should be initiated immediately after hospital admission. While guidelines and practice are leading to earlier use of heparin and heparinoids for DVT prophylaxis, the timing and contraindications remove a point of debate. 9. The correct answer is C. Two studies have found that phenytoin prophylaxis is associated with worsened functional outcome and the 2015 AHA/ASA guidelines recommend against its routine use. The safety of alternative anticonvulsant medications has not been systematically studied. Continuous EEG monitoring may help in the detection of subclinical seizures, but is a different issue than anticonvulsant prophylaxis in the absence of seizures. 10. The correct answer is E. Because of only limited data in ICH patients, ICP monitoring is only a level IIb recommendation. However, if a CPP target is sought, then ICP monitoring is necessary to provide this information. Other advanced neuromonitoring tools such as brain tissue oxygen monitoring or cerebral microdialysis are available, but information about their utility in ICH is sparse.

www.medicalebookpdf.com

Chapter 3

SUBARACHNOID HEMORRHAGE Adam Webb and Owen Samuels CLINICAL CASE A 65 year old woman with a past medical history significant for hypertension and smoking presents to a small rural emergency department with sudden onset of what she describes as “the worst headache of my life” while at work. Soon after arrival, she is noted to be vomiting and becomes increasingly somnolent and difficult to arouse. She is intubated for airway protection. A non-contrast head CT is obtained and shown below.

Figure 3-1. Non-contrast head CT scan showing subarachnoid hemorrhage and hydrocephalus

While obtaining a CBC with platelet count, chemistry panel, troponin, toxicology screen and 12-lead EKG, the patient's systolic blood pressure is reduced to below 160 mmHg with a nicardipine infusion and she is given a bolus of 5 grams of aminocaproic acid in preparation for helicopter transfer to the

www.medicalebookpdf.com

nearest comprehensive stroke center. Once she arrives, an external ventricular catheter is emergently placed and reveals an opening pressure of 40 cm H20. The patient is maintained on an aminocaproic acid infusion. A CT angiogram is obtained revealing an aneurysm. A catheter cerebral angiogram is obtained. Both studies are shown below.

Figure 3-2. CT angiogram (left panel) and catheter angiogram (right panel) showing aneurysm

OVERVIEW AND EPIDEMIOLOGY The acute treatment of aneurysmal SAH (aSAH) requires an understanding and appreciation of both the neurological implications and the multitude of systemic issues that can occur. The literature reports that one in eight patients die before reaching the hospital. The case fatality rate has been reported to range between 26 and 50%, however this has been declining over the past 40 years [1, 47]. Of survivors, nearly half will have long term functional impairment [2]. aSAH represents 85% of all non-traumatic SAH [3]. aSAH accounts for only 3% of all strokes and 4% of all stroke mortality but disproportionately accounts for 27% of all stroke-related years of potential life lost before the age of 65 [4, 5]. The incidence of aSAH ranges from 2-22 per 100,000 patient years with the highest incidence seen in Finland and Japan [48, 49]. The incidence is higher among women [6, 50]. Risk factors include hypertension, smoking, family history, and cocaine use. Aneurysm characteristics that contribute to a higher risk of rupture are larger size (>7 mm) and location in the posterior circulation and on the posterior communicating artery [47]. PATHOPHYSIOLOGY

www.medicalebookpdf.com

Intracranial Aneurysms Intracranial saccular aneurysms are acquired lesions of the cerebral arterial circulation. They predominantly develop at the branching points around the circle of Willis. The prevalence in autopsy series is between 1 and 6%, though the majority of these represent small asymptomatic aneurysms. While most arise spontaneously, the development of intracranial aneurysms is influenced by both genetic and environmental factors. Familial clustering of aneurysms has been well described with studies demonstrating that first-degree relatives of patients with SAH are more than 3 times more likely to suffer from SAH when compared to the general population. Additionally, many connective tissue disorders have been associated with intracranial aneurysms including Ehlers-Danlos syndrome, Neurofibromatosis Type I, Marfan's syndrome and most convincingly autosomal dominant polycystic kidney disease [7]. Perimesencephalic Subarachnoid Hemorrhage Perimesencephalic hemorrhages are a radiographically and clinically distinct category of non-aneurysmal subarachnoid hemorrhage. Perimesencephalic hemorrhages account for approximately 10% of all SAH and two thirds of those with a negative angiogram. The extravasated blood is predominantly located ventral to the brainstem in the perimesencephalic and prepontine cisterns. With the exception of a small amount of blood in the posterior horns of the lateral ventricles, intraventricular hemorrhage is rare. Perimesencephalic hemorrhages typically occur in patients over 50 years old. Patients typically have less severe neurological symptoms and rarely develop seizures or focal deficits. These patients are important to distinguish from those with aneurysmal rupture because rebleeding and delayed cerebral ischemia related to vasospasm is unusual [3]. Other Causes of Subarachnoid Hemorrhage There are several additional causes of SAH that need to be discussed because the diagnostic evaluation and management may differ from that of aneurysmal SAH. Head trauma is the most common cause of non-aneurysmal subarachnoid hemorrhage. Usually the clinical history defines trauma as a cause, though there can be some confusion as to whether a patient lost consciousness from a SAH causing the trauma or as a result of a primary traumatic event. Often, traumatic SAH can be distinguished radiographically with a thin layer of blood in the subarachnoid space around the cerebral convexities or within the sylvian fissure. Although possible, it is less common to see blood in the basal cisterns or surrounding the circle of Willis as a result of trauma. This pattern should raise suspicion for aSAH. Several other causes make up the remaining 5% of SAH. Intradural arterial dissection, especially of the vertebral artery can cause SAH. Suspicion should be high in the setting SAH with early ischemic stroke (i.e.

www.medicalebookpdf.com

Posterior inferior cerebellar artery distribution). Rupture of a cerebral arteriovenous malformation (AVM) can extravasate blood into the subarachnoid space. Saccular aneurysms may form on the feeding vessels or within the AVM itself. Other less common causes include dural AV fistulas, septic or mycotic aneurysms, pituitary apoplexy, moya moya disease, and cocaine or stimulant abuse [3], cerebral vasculitis, Call-Fleming Syndrome, and posterior reversible encephalopathy syndrome (PRES). CLINICAL FEATURES Symptoms The classical description of symptoms of aSAH is the sudden onset of severe headache. The patient may describe this as the “worst headache of my life.” Based on history alone the diagnosis can be difficult because the headache is actually described as developing instantaneously in only 50% of patients [8]. Of those patients prospectively screened for acute severe headache only 6-17% were demonstrated to have SAH [9, 10]. Additional symptoms include seizure at the onset of hemorrhage [6%], transient loss of consciousness [26%] and vomiting preceding the onset of the headache [69%] [9]. Controversy remains over the incidence of a transient “sentinel headache” preceding the onset of the hemorrhage with an incidence quoted from 5% all the way up to 43%. [11, 53]. Physical Exam Findings The most common findings on neurological exam are depressed level of consciousness or a confusional state. The physical and neurological exam of a patient with SAH may be normal. Specific findings on exam may point towards the location of the aneurysm. For instance, a pupil involving cranial nerve III palsy in an otherwise healthy individual should raise the suspicion for the presence of a posterior communicating artery aneurysm. Cranial nerve VI palsy may be a sign of elevated intracranial pressure. Findings on funduscopic exam can include subhyaloid hemorrhage or papilledema. DIAGNOSIS Head CT Non-contrasted head CT is the diagnostic modality of choice for the initial evaluation of suspected SAH. It is quick and widely available. It is highly sensitive for the detection of blood in the subarachnoid space, though this is dependent on the timing relative to the onset of symptoms. The sensitivity approaches 100% within the first 6 hours of onset of hemorrhage and then diminishes to 85% at 5 days and 50% after 1 week [12, 13, 14, 50]. False

www.medicalebookpdf.com

negative results are more common in patients with normal neurological exams and with anemic patients (hematocrit 65 years of age. Traumatic subarachnoid hemorrhage (tSAH) results from tearing of small pial vessels. Blood is typically located over the cerebral convexities and is often confined to a few sulci or fissures. However, tSAH may distribute diffusely over the cortical surface, into the basal cisterns, or into the ventricles. Isolated intraventricular hemorrhage due to trauma is unusual. Diffuse axonal injury (DAI) is commonly observed after rapid acceleration and deceleration of the head, typically in the lateral plane. DAI is both a form of primary and a form of secondary injury. There may be immediate mechanical damage (primary injury) to structural elements of axons, such as microtubules, at the time of the trauma. This typically occurs in severe TBI that involves tearing of tissue. It has become evident, however, that axonal swelling, and ultimately axotomy, is the consequence of cellular cascades that are initiated by the trauma (secondary injury). This form of secondary injury may occur in severe, moderate, and even mild TBI (concussion). In severe cases, DAI is multifocal and bilateral. Regions commonly affected include the junction between cortex and white matter, white matter structures close to the midline (corpus callosum, internal capsule), the brainstem, cerebellum, and the corona radiata. Histopathologically, DAI is characterized by the presence of axonal swellings that are the result of accumulated material due to interrupted axonal transport. These swellings may have a periodic arrangement along the length of the axon (“axonal varicosities”), or there may be a single point of swelling (“axonal bulb” or “retraction ball”) that likely reflects axonal disconnection. Some have proposed use of the term, “traumatic axonal injury (TAI)” or “diffuse traumatic axonal injury (dTAI)” to describe theses pathological and pathophysiological changes [9]. Penetrating TBI While contusions, epidural hematomas, subdural hematomas, and subarachnoid hemorrhage are commonly observed in penetrating TBI, the hallmark of penetrating TBI is the cerebral laceration (Figure 7-3e). In penetrating TBI, the nature of primary injury is largely dictated by the ballistic properties of the projectile (e.g. bullet) and any secondary projectiles (e.g. bullet fragments, bone fragments). As a missile penetrates the brain, it tears the parenchyma, leaving a track with necrosis and hemorrhage (laceration). In the wake of the projectile, tissue is compressed, collapses and re-expands in a

www.medicalebookpdf.com

repeating wave-like pattern that further injures tissue. The degree of tissue injury is dependent on the kinetic energy transferred from the missile to the tissue. Since kinetic injury = 1/2(mass)(velocity)2, higher velocity projectiles cause more tissue injury than lower velocity projectiles. Projectile paths that cross the hemispheres, violate the ventricles, or that involve the brainstem have a poor prognosis and are most frequently fatal. Blast TBI Cerebral blast injury occurs when acoustic, electromagnetic, light, and thermal energy (blast wave) that emanates from an explosion are transferred to the brain directly through the cranium, and indirectly through oscillating pressures in fluid containing structures, such as blood vessels. While much remains to be elucidated about the pathophysiology of blast TBI, some important distinguishing features have been observed. Diffuse axonal injury occurs in a dose-dependent fashion that likely differs from the DAI observed with closedhead injury. Malignant cerebral edema may occur rapidly (within an hour) as opposed to the more slowly developing edema seen in blunt TBI (hours to days). Cerebral vasospasm may occur in up to 50% of moderate to severe blast TBI and may last as long as one month. Lastly, patients with blast TBI frequently have concomitant injury to the eyes and to the auditory and vestibular systems [10]. SECONDARY CEREBRAL INJURY Secondary injury involves a host of cellular and molecular cascades that promote cell death, and that exacerbate cerebral edema and ischemia. While these processes may begin immediately, they often last for hours to days or longer. Studies of secondary injury are largely in experimental models and in humans with blunt TBI. Mechanisms of secondary injury include: neuronal depolarization, disturbance of ionic homeostasis, glutamate excitotoxicity, generation of nitric oxide and oxygen free radicals, lipid peroxidation, bloodbrain barrier disruption, secondary hemorrhage, ischemia, cerebral edema, intracranial hypertension, mitochondrial dysfunction, axonal disruption, inflammation, and apoptotic and necrotic cell death. Cerebral ischemia, intracranial hypertension, systemic hypotension, hypoxia, fever, hypocapnia, and hypoglycemia have all been shown to independently worsen survival after blunt TBI [11]. Coagulopathy occurs in roughly 1/3 of patients with severe TBI and may exacerbate ischemic brain injury through microvascular thrombosis and embolism. It is likely that the coagulopathy of TBI is a distinct entity from the coagulopathy of systemic trauma [12,13].

www.medicalebookpdf.com

Figure 7-3. Typical radiological appearance of the various primary injuries in TBI: a) non-contrast axial CT demonstrating right > left frontal lobe contusions with hemorrhage; b) non-contrast axial CT demonstrating left convexity epidural hematoma; c) non-contrast axial CT demonstrating left convexity subdural hematoma with mass-effect, effacement of the left lateral ventricle, and left to right midline shift; d) non-contrast axial CT demonstrating traumatic subarachnoid hemorrhage; e) non-contrast axial CT demonstrating trans-hemispheric laceration from bullet with hemorrhage, bullet fragments, and bone fragments in the track; f) axial gradient echo MRI sequence demonstrating punctate foci of hemorrhage (black spots) consistent with diffuse axonal injury.

Clinical Features The clinical features of TBI are dictated by baseline patient characteristics (e.g. pre-existing brain injury), type of traumatic injury (e.g. contusion vs. extraaxial hematoma), severity of the injury, and location of the lesion. The following discussion addresses the clinical features typically observed in moderate to severe blunt TBI. Parenchymal contusions are the most commonly observed mass lesion in patients with TBI. Contusions may be unilateral or bilateral, and may be

www.medicalebookpdf.com

ipsilateral to the site of impact (coup) or contralateral (contra-coup). Clinical features reflect dysfunction in the affected brain regions, frequently the orbitofrontal and inferior temporal lobes. Patients may deteriorate within hours of presentation due to expansion of contusions. On non-contrast computed tomography (CT), contusions appear as hypodense regions without macroscopic hemorrhage, or as mixed-high density lesions if gross hemorrhage is present (Figure 7-3a). Epidural hematomas may present with focal findings based on the side of injury. They may expand rapidly and lead to depressed level of consciousness when they exert mass effect sufficient to cause herniation and brainstem compression. The classic clinical description of EDH is the “lucid interval,” in which the patient is initially unconscious, wakes up without obvious deficit, and subsequently deteriorates. This may be seen in approximately 50% of patients with EDH. On non-contrast head CT, epidural hematomas appear as lens-shaped hyperdense extra-axial collections that do not cross skull suture lines (Figure 73b). Subdural hematomas, as with epidural hematomas, produce clinical symptoms from local compression of cortical and subcortical structures, and when large, from herniation and brainstem compression. Subdural hematomas are most often unilateral but may be bilateral in 15% of cases. Subdural hematomas may enlarge over time and cause clinical deterioration. A minority of patients may have a lucid interval. On non-contrast head CT, subdural hematomas appear as hyperdense crescent-shaped extra-axial collections that may cross skull suture lines (Figure 7-3c). Subarachnoid hemorrhage (SAH) may produce clinical symptoms by precipitating acute hydrocephalus, although this is uncommon. Small volume of SAH is associated with an increased mortality, and large volumes may increase the odds of death by a factor of 2. Intraventricular hemorrhages are relatively uncommon, but are associated with significant morbidity and mortality and may be associated with increased intracranial pressure. CT imaging may demonstrate hyperdense collections in the cerebral sulci, fissues, ventricular system, or basal cisterns (Figure 7-3d). Diffuse axonal injury is rarely fatal but is associated with increased odds of a poor functional recovery. Classically, patients with DAI have a depressed level of arousal that is out of proportion to the burden of injury observed on CT scan. Since DAI involves microscopic injury, it cannot be observed directly on neuroimaging studies; rather, indirect evidence of DAI (associated, macroscopic injury) is sought. CT imaging may reveal small punctate foci of hemorrhage but is frequently unremarkable. Magnetic resonance imaging (MRI) is considerably

www.medicalebookpdf.com

more sensitive and may display abnormalities on diffusion weighted, gradientecho, and diffusion tensor sequences (Figure 7-3f). Diffuse cerebral swelling typically occurs hours to days after the insult but may occur within the first hour, particularly in blast TBI. Signs and symptoms are that of intracranial hypertension and the herniation syndromes. These include agitation, bradycardia, hypertension, progressive decrease in level of arousal culminating in coma, abnormalities of the pupillary light reflex, loss of other brainstem reflexes, abnormal breathing patterns, and abnormal motor posturing. CT imaging reveals sulcal effacement, loss of differentiation between gray and white matter, compression of the ventricles, and effacement of the basal cisterns. Vascular injury may include arterial dissection, formation of arterial pseudoaneurysms, formation of arteriovenous fistulae, and arterial or venous perforation. The actual incidence of vascular damage is unknown, and is likely under-reported since vascular imaging is usually performed only when injury is suspected. Blunt injuries to the extracranial carotid and vertebral arteries, although likely rare (0.10.5%), may present with late-onset ischemic strokes. The internal carotid artery stretches over the lateral masses of the third and fourth cervical vertebrae, perhaps increasing susceptibility to intimal tearing, dissection, pseudoaneurysm formation, and thrombosis. Vertebral artery injury may occur in patients with concomitant cervical spine trauma, although no specific cervical vertebral fracture pattern has a higher association with blunt vertebral artery injury. DIAGNOSIS The diagnosis of TBI is usually made by the history provided by the patient, by bystanders, or by emergency medical personnel. When the history is unavailable, the diagnosis is typically made by physical examination in conjunction with neuroimaging studies. On physical examination, superficial evidence of trauma is sought, such as abrasions, lacerations, and soft tissue swelling of the head. The presence of entrance and exit wounds should be assessed (penetrating TBI). Sings of a basilar skull fracture may be present, including retroauricular ecchymosis (Battle's sign), periorbital ecchymosis (Raccoon's eyes), hemotympanum, and CSF otorrhea or rhinorrhea. A focused neurological assessment is made to determine the severity of the injury. The Glasgow Coma Scale (GCS) score should be used to assess, categorize and to communicate severity of injury (see Table 14-1). Accordingly, severe TBI is defined by a GCS of 3-8, moderate TBI by a GCS of 9–12, and mild TBI by a GCS of 13-15. The GCS score may be

www.medicalebookpdf.com

determined quickly, has good inter-rater reliability, has prognostic value, and is widely used. Recently, it has been suggested that patient age affects the relationship between GCS and severity of TBI, with younger patients (18-64years-old) having worse GCS scores than older patients (> 64-years-old) for a given TBI severity [14]. The GCS has other limitations, particularly for use in patients who are intubated or aphasic. These limitations are addressed in other scales, such as the Full Outline of UnResponsiveness (FOUR) score, however use of the GCS is presently more widely adopted. The neurological examination should also include assessment of spinal cord and peripheral nerve function. Brain, spinal cord, and nerve injuries may co-exist. A thorough systemic examination should seek to determine the presence and extent of non-nervous system injuries. Neuroimaging studies aid in diagnosing the particular types of primary injury present. Together with the clinical examination, imaging can help guide decisions about subsequent therapy. The radiological characteristics of primary injury types are discussed above. While some patients with mild TBI may not warrant imaging, nearly all patients with moderate or severe TBI do. Imaging should be performed in all patients with declining level of consciousness, prolonged loss of consciousness, persistent alteration in consciousness, focal neurological signs, seizures, penetrating injury, signs of depressed or basilar skull fracture, confusion, or agitation. CT is the initial imaging modality of choice in the acute setting because it is widely available, may be performed rapidly, and is highly sensitive for acute blood. MRI is more sensitive than CT for soft tissue pathology but is less widely available and may pose logistic challenges and patient safety issues related to transportation and monitoring. Neuroimaging studies may also be used to categorize TBI, particularly for research purposes. Two classification schemes, the Marshall [15] and Rotterdam scores [16], are most commonly used (Table 7-1). When applied to CT scans in moderate-severe TBI, the Marshall score, an ordinal numbering scale with 6 categories, aids in predicting risk of intracranial hypertension and outcome in adults. The Marshall classification is widely used and pragmatic, but has many recognized and accepted limitations, including difficulties in classifying patients with multiple injury types and standardization of certain features of the CT scan. The Rotterdam score is a more standardized CT-based classification system, which uses combinations of findings to predict outcome. TREATMENT Treatment may be divided by phase of TBI: pre-hospital, emergency department,

www.medicalebookpdf.com

and subsequent, which may include both surgical and intensive care unit (ICU) treatment. The following recommendations are based on those of the Brain Trauma Foundation for adults with blunt TBI [1]. Separate guidelines exist for infants, children, and adolescents [17]. I. PRE-HOSPITAL TREATMENT Minimization of secondary cerebral injury begins in the pre-hospital phase, where the primary goals of therapy are avoidance and treatment of hypotension and hypoxia, both of which are associated with worse clinical outcomes. Management strategies that address these issues have been associated with improved outcome. Correction of hypotension is accomplished through intravenous fluid resuscitation with isotonic crystalloid. Hypertonic saline resuscitation has not demonstrated benefit and resuscitation with albumin may be associated with harm [18]. Endotracheal intubation in the field is generally considered for patients with a GCS of < 8, however, evidence of benefit over bag-mask ventilation is mixed. Endotracheal intubation should only be performed by paramedical or first repsonder personnel with expertise. Care should be taken to stabilize the cervical spine and the patient should be rapidly transported to a trauma center. II. EMERGENCY DEPARTMENT TREATMENT Initial treatment in the emergency department should proceed according to Advanced Trauma Life Support (ATLS) guidelines. These include maintenance of adequate oxygenation (PaO2 > 60 mmHg) and blood pressure (systolic blood pressure > 90 mmHg). Vital signs are monitored and therapy is adjusted to maintain cardiopulmonary homeostasis. Neurological assessment includes an initial and then serial determinations of GCS score. Signs of intracranial hypertension, such as decreased pupillary responsiveness to light, hypertension with bradycardia, posturing, or respiratory abnormalities, should prompt empiric treatment with head of bed elevation, hyperventilation, and an osmolar agent (mannitol or hypertonic saline). The patient is assessed for systemic trauma. Laboratory assessment includes a complete blood count, electrolytes, glucose, coagulation profile, blood alcohol level, and urine toxicology screen. Coagulation abnormalities should be rapidly corrected. Imaging, including a non-contrast head CT, is performed to help define the extent of injury and to guide subsequent management.

www.medicalebookpdf.com

III. SURGICAL MANAGEMENT For mass lesions, indications for surgical evacuation are assessed on clinical and radiological findings. Table 7-2 summarizes recommendations for surgical intervention [19-23]. For diffuse TBI, decompressive craniectomy for the treatment of refractory ICP in patients with diffuse TBI is can be performed. In the DECRA trial, 155 patients with severe diffuse non-penetrating traumatic brain injury and refractory intracranial hypertension were assigned to bifrontal-temporoparietal decompressive craniectomy with durotomy or standard care [24]. Despite a significantly lower mean ICP, functional outcome was worse in the craniectomy group. A major criticism of the study was a significant difference in patients with

www.medicalebookpdf.com

unreactive pupils on admission in the surgical group. A post hoc analysis that adjusted for pupil reactivity at baseline, found no difference in functional outcome between groups. The authors proposed that expansion of the swollen brain outside the skull may cause axonal stretch leading to neural injury or may impair cerebral blood flow or metabolism overcoming any beneficial effect of lowering ICP. This is an area of ongoing study [25]. It remains unclear whether unilateral craniectomy and craniectomy for focal TBI improve outcome.

www.medicalebookpdf.com

www.medicalebookpdf.com

IV. MEDICAL (INTENSIVE CARE UNIT) MANAGEMENT Medical management of the patient with severe TBI typically occurs in an intensive care unit where the focus is on minimization of secondary cerebral injury and on prevention of systemic complications. A. Blood pressure and oxygenation

www.medicalebookpdf.com

The Brain Trauma Foundation recommends that blood pressure be monitored and that hypotension (systolic blood pressure < 90 mmHg) be avoided. The threshold value of 90mmHg to define hypotension was determined by statistical analysis rather than physiological data. Substantial evidence suggests that considerable secondary brain injury occurs from hypotension, although the precise threshold is often unclear in an individual TBI patient. Both pre-hospital and in-hospital hypotension are associated with worse outcome after severe TBI. A single episode of hypotension, defined as SBP 40 years; posturing; systolic blood pressure < 90 mmHg. Typically, ICP is monitored with a ventriculostomy or an

www.medicalebookpdf.com

intraparenchymal probe. While invasive ICP monitoring has been standard of care in the United States, it has not been shown to improve outcome. In 2012, a multicenter randomized trial of 324 patients with TBI conducted in Ecuador and Bolivia found that therapy targeted to maintain ICP < 20 mmHg with the use of an invasive monitor was not superior to therapy based on clinical examination [29]. Whether these results are generalizable to TBI populations in developed countries is unclear. Initial therapeutic measures in the ICU are largely preventative and include head of bed elevation, maintenance of the neck in a neutral position, avoidance of neck constriction (e.g. loosening endotracheal tube ties), prevention of hypercarbia, and adequate treatment of pain, agitation, fever, and seizures. When ICP remains > 20 mmHg, a series of tiered therapies are employed. CSF drainage: CSF drainage through a ventriculostomy should be considered. The optimal method of drainage (continuous vs. intermittent) has not been established. Osmotherapy: If CSF diversion is unsuccessful, or if a ventriculostomy is not present, then osmotic agents, typically mannitol or hypertonic saline, can be administered. While both can be effective, there are insufficient data to suggest superiority of one agent over the other. The optimal concentration and mode of administration (bolus vs. continuous infusion) of hypertonic saline is unknown. Mannitol (usually 20%) should be administered as a bolus, typically 0.25 – 1 gm/kg, however, the optimal dose and concentration of mannitol are unknown. When mannitol is used, great care should be taken to avoid intravascular volume depletion and hypotension, which are deleterious to the patient with severe TBI. One preventative strategy is to replace urinary losses on a cc per cc basis for the first few hours after drug administration. Surgery: Should intracranial hypertension persist despite administration of osmotic agents, then decompressive craniectomy may be considered. Craniectomy, either unilateral or bilateral, is the most effective way to lower ICP. As mentioned above, the impact of decompressive surgery on outcome is unclear. Metabolic therapy: The goal of metabolic therapy is to suppress cerebral metabolic rate (CMRO2). A reduction in CMRO2 leads to a reduction in cerebral blood flow (CBF) which lowers cerebral blood volume and hence ICP. Furthermore, a reduction of CMRO2 in the face of decreased fuel delivery, might preserve brain tissue. Reduction of CMRO2 may be accomplished by induction of pharmacological coma or hypothermia. Classically, pharmacologic coma has been achieved with barbiturates, however, it is unclear whether the risks associated with high-dose barbiturates

www.medicalebookpdf.com

(e.g. immune suppression, hypotension, poikilothermia, gastroparesis, decreased mucocilliary clearance) are outweighed by any cerebral benefit. In clinical practice, multiple sedatives infusions are used, including opiates, benzodiazepines, and propofol. There is insufficient data to guide choice of sedative and decisions must be made based on patient characteristics and sideeffect profiles. When pharmacologic coma is employed, the agent should be titrated to an ICP < 20 mmHg, an isoelectric EEG, or deleterious side effects – whichever occurs first. Hypothermia may also be used to lower CMRO2 and to reduce ICP. Numerous studies have addressed the role of mild to moderate hypothermia (3234 °C) in TBI. Most single-center studies suggest that induced hypothermia is associated with improved outcome. However, 2 large randomized multicenter studies in adults with severe TBI (National Acute Brain Injury Study: Hypothermia I and II) failed to show benefit [30, 31], and a randomized study of hypothermia in children with TBI suggested harm [32]. While mild to moderate hypothermia has not been shown to improve outcome, the preponderance of literature suggests it is effective in lowering ICP. Laparotomy: Perhaps as a last resort, decompressive laparotomy (or thoracotomy) should be considered to treat refractory intracranial hypertension. Both intra-abdominal and intrathoracic hypertension may contribute to raised intracranial pressure, presumably through transmission of pressure from those cavities to the spinal subarachnoid space (and hence the cranial subarachnoid space) through the vertebral veins. In a series of 17 patients, all with refractory intracranial hypertension and none with abdominal compartment syndrome, Joseph et al. reported a fall in ICP in all patients after laparotomy [33]. Of these, eleven patients maintained a lower ICP and survived. Further study is needed to define the optimal role of laparotomy and its impact on functional outcome. Hyperventilation: Hyperventilation results in blood and CSF alkalosis, which leads to cerebral vasoconstriction, reduced cerebral blood volume and therefore a lower ICP. Sustained and vigorous hyperventilation may result in cerebral ischemia and is therefore not recommended as a routine therapy. However, in emergency situations (e.g. acute herniation), hyperventilation may be used transiently as a bridge to more definitive therapy (e.g. surgery, osmotic agent). Some suggest that jugular bulb oximetry allows for safer titration of hyperventilation insofar as it may detect cerebral hypoxia. C. Cerebral perfusion pressure (CPP) If cerebral autoregulation is disturbed after TBI, then cerebral perfusion pressure may affect cerebral blood flow. Therefore, an attempt is made to target CPP within a range that minimizes cerebral ischemia. It is common practice to

www.medicalebookpdf.com

maintain CPP > 60 mmHg, although this may not be optimal in all patients. The Brain Trauma Foundation currently recommends maintaining CPP between 50 and 70 mmHg. Elevating CPP above 70 mmHg with intravenous fluids and vasopressors should be avoided because of the risk of lung injury. A randomized controlled trial of CPP-targeted therapy versus ICP-targeted therapy was performed. In the CPP group, CPP was maintained at >70mmHg; in the ICP group, CPP was maintained at >50mmHg and ICP 60mmHg. Although lowering CPP below a critical threshold appears deleterious, raising it does not appear to be advantageous. Optimization of CPP in the normotensive patient should begin with lowering ICP. Although CPP is an integral physiological parameter in modern intensive care of the TBI patient, there is considerable variability in how it is derived. A survey study suggests that placement of the arterial line transducer (from which MAP is derived for CPP calculations) varies both across institutions and among the 11 studies cited by the Brain Trauma Foundation for their CPP recommendations [36]. While some zero the transducer at the level of the heart (phlebostatic axis), others zero it at the head. If the patient is flat, there is no difference. However, when head of bed is upright, MAP measured at the right atrium is higher than that measured at the level of the tragus. Therefore, transducing blood pressure with an arterial line zeroed at the phlebostatic axis will result in an overestimate of actual CPP. This is particularly problematic in patients who are nursed with head of bed elevation to >30 degrees for ICP control, as the discrepancy between CPP measured at the phlebostatic axis versus the tragus could be as high as 20mm Hg. This lack of uniformity in clinical practice and in the published literature is problematic and potentially clinically significant. D. Seizure prophylaxis BTF guidelines recommend the use of anticonvulsant medication (phenytoin) for one week following TBI and recommend against longer durations of prophylactic therapy. Many centers use alternative agents, such as leviteracitam. Seizures occur in 10% - 30% of patients with TBI. Theoretically, seizures may worsen outcome by increasing CMRO2 and ICP, thereby increasing the likelihood of cerebral ischemia. In comatose patients, up to 25% may have nonconvulsive seizures. Prophylactic anticonvulsant medications reduce the incidence of early post-traumatic seizures but do not lessen the odds of

www.medicalebookpdf.com

developing post-traumatic epilepsy. The impact of prophylactic anticonvulsant medications on outcome and their comparative efficacies is unknown. E. Other general critical care strategies The Brain Trauma Foundation guidelines address select areas of general critical care of the TBI patient including, infection prophylaxis, deep vein thrombosis (DVT) prophylaxis, nutrition, and steroid administration. Recommendations are as follows: Periprocedural antibiotics for intubation and early tracheostomy are recommended Graduated compression stockings or intermittent pneumatic compression stockings should be used until patients are ambulatory. Low molecular weight heparin or low dose unfractionated heparin should be used but may increase the risk for expansion of intracranial hemorrhage. No recommendations are made regarding the timing, dose, or duration of pharmacological prophylaxis. Full caloric needs should be administered by day 7 post-injury Steroids should not be used to improve outcome or reduce ICP. Steroids are associated with increased mortality and are contra-indicated. This was demonstrated in the CRASH trial, a large (10,008 adults), international, multicenter placebo-controlled trial of methylprednisolone after head injury. The group that was treated with steroids had an increased odds of death (relative risk of 1.18), regardless of injury severity [37]. Fever is strongly, independently, and consistently associated with worse clinical outcomes across a variety of severe brain injuries. While in experimental models there is a clear causal relationship, in humans it remains unclear whether fever exacerbates or is merely a marker of brain injury. Nonetheless it is common practice to treat fever with antipyretic medications, ice packs, surface cooling devices, or intravascular cooling devices. The impact of fever control on outcome has yet to be determined. Similarly, hyperglycemia is associated with worse clinical outcomes after severe TBI. However, the brain is an obligate glucose consumer and hypoglycemia is also injurious. Avoidance of both hyper-and hypoglycemia is therefore recommended. Coagulopathy is frequent in patients with TBI due to the use of anticoagulant and antiplatelet medications, traumatic brain injury itself, or due to multisystem trauma. Efforts should be taken to rapidly correct coagulopathy, however the

www.medicalebookpdf.com

optimal means by which to do so are ill defined. F. Multimodality Neuromonitoring While data are currently insufficient to define the optimal role of advanced neuromonitoring tools, the Brain Trauma Foundation specifically addresses brain oxygenation, and offers a level III recommendation for use of jugular venous oxygen saturation (SjvO2) and brain tissue oxygen tension (PbtO2) monitoring. They recommend maintenance of SjvO2 > 50% and PbtO2 > 15 mmHg. It is increasingly recognized that traditional goals of cerebral resuscitation – ICP, CPP, and the clinical examination are distant surrogates for cerebral perfusion that do not account for dynamic changes in cerebral autoregulation, tissue metabolic rate, cellular fuel utilization, and microcirculatory dysfunction, all of which impact tissue metabolic health. Although standard, it seems intuitively obvious that a uniform approach of maintaining ICP < 20mmHg and CPP >60mmHg is overly simplistic. This approach, based on statistical averages across large populations, addresses neither significant baseline differences in patient physiology nor the complex, dynamic, and variable pathophysiological changes that ensue following severe brain injury. It is evident that neuronal injury may occur despite apparent physiological homeostasis (normal SBP, PaO2, ICP, CPP). A more tailored therapeutic strategy that responds to multiple simultaneously measured and more relevant physiological variables is logically appealing but has not been subjected to rigorous scientific scrutiny. The emergence of technology that allows for continuous real-time bedside monitoring of cerebral physiology might facilitate assessment of therapeutic efficacy and provide more relevant physiological endpoints for resuscitation. Combining these monitors in a multimodal approach may allow goal-directed cerebral resuscitation that emphasizes the individual patient's unique neurological and systemic physiology. This approach must ultimately be compared to algorithms that target more traditional physiological variables. A variety of monitors are now available that permit bedside assessment of advanced physiology in real-time or near real-time. CBF may be measured quantitatively in small regions of brain tissue with thermal diffusion flowmetry probes. Whole brain CBF may be trended in a non-quantitative way with continuous EEG through the use of software that provides a measure of the ratio of fast waves to slow waves. Cerebral oxygenation may be measured regionally (PbtO2) with the use of a Clark-type electrode or with probes that use fluorescence quenching, or non-invasively with near-infrared spectroscopy. Whole brain oxygenation may be measured with jugular bulb oximetry (SjvO2). Cerebral biochemistry, including markers of neuronal ischemia and injury (lactate, pyruvate, glycerol, glutamate, glucose), may be measured regionally

www.medicalebookpdf.com

with cerebral microdialysis. These monitors alone or in combination are not expected to help patients; rather, it is hoped that therapeutic responses to information provided by these tools will improve outcome. Ongoing research aims to understand better the information provided by these tools and the optimal therapeutic responses. PROGNOSIS Outcomes from TBI span the spectrum from death and vegetative state to full recovery. While many factors predict poor outcome in large populations (e.g. GCS, age, etc.), these should not be used for prognostication in individual patients. The IMPACT (International Mission on Prognosis and Analysis of Clinical Trials) and the CRASH (Corticosteroid Randomization after Significant Head Injury) scores are externally validated models derived from large datasets that aid in prediction of 6-month outcome after TBI. However, functional recovery may continue for at least 18-months following severe injury, and these scores are of minimal utility for predicting ultimate individual patient outcome. As a general rule, traumatic coma has a better prognosis than coma from hypoxia-ischemia, and coma from blunt trauma has a better prognosis than coma from penetrating TBI. Much work remains to be done to better define accurate predictors of outcome. A promising line of investigation involves the use of advanced MRI imaging (functional and diffusion tensor sequences) to improve prognostic accuracy. To date, no medications have proved useful in improving outcome. There have been over 200 failed neuroprotective drug trials. It is unlikely that a single drug will prove efficacious as the pathways involved in secondary injury are complex and redundant. Perhaps the best hope for neuroprotection lies in combinations, or “bundles,” of therapies that target multiple pathways. REFERENCES 1. Brain Trauma Foundation, American Association of Neurological Surgeons, Congress of Neurological Surgeons, et al. Guidelines for the management of severe traumatic brain injury. Introduction. J Neurotrauma 2007; 24 Suppl 1:S1-S106. 2. Stein SC, Georgof P, Sudha M, et al. Relationship of aggressive monitoring and treatment to improved outcomes in severe traumatic brain injury. Journal of Neurosurgery 2010;112(5):1105-1112. 3. Coronado VGm Xu L, Basavaraju SV, et al. Surveillance for traumatic

www.medicalebookpdf.com

brain injury-related Deaths – United States, 1997-2007. Centers for Disease Control and Prevention, Morbidity and Mortality Weekly Report 2011; 60(5):1-32. 4. Kraus JF, McArthur DL. Epidemiologic aspects of brain injury. Neurol Clin 1996; 14:435. 5. Feigin VL, Theadom A, Barker-Collo S, et al. Incidence of traumatic brain injury in New Zealand: a population- based study. Lancet Neurol 2013; 12:53. 6. Liao CC, Chiu WT, Yeh CC, et al. Risk and outcomes for traumatic brain injury in patients with mental disorders. J Neurol Neurosurg Psychiatry 2012; 83:1186. 7. http://www.cdc.gov/traumaticbraininjury/data/dist_hosp.html 8. Ling GSF, Ecklund JM. Traumatic brain injury in modern war. Curr Opin Anesthesiol 2011;24:124-130. 9. Johnson VE, Stewart W, Smith DH. Axonal pathology in traumatic brain injury. Experimental Neurology 2013;246:35-45. 10. Magnuson J, Leonessa F, Ling G. Neuropathology of explosive blast traumatic brain injury. Current Neurol and Neurosci Rep 2012;12(5):570579. 11. McHugh GS, Engel DC, Butcher I, et al. Prognostic value of secondary insults in traumatic brain injury: results from the IMPACT study. J Neurotrauma 2007; 24:287. 12. Stein SC, Smith DH. Coagulopathy in traumatic brain injury. Neurocrit Care. 2004;1(4):47988. 13. Harhangi BS, Kompanje EJ, Leebeek FW, Maas AI. Coagulation disorders after traumatic brain injury. Acta Neurochir (Wien). 2008 Feb;150(2):16575; discussion 75. 14. Salottolo K, Levy AS, Slone DS, et al. The effect of age on Glasgow Coma Scale score in patients with Traumatic Brain Injury. JAMA Surgery 2014; published online ahead of print on June 4, 2014. 15. Marshall LF, Marshall SB, Klauber MR, et al. The diagnosis of head injury requires a classification based on computed axial tomography. J Neurotrauma 1992; 9 Suppl 1:S287.

www.medicalebookpdf.com

16. Maas Al, Hukkelhoven CW, Marshall LF, Steyerberg EW. Prediction of outcome in traumatic brain injury with computed tomographic characteristics: a comparison between the computed tomographic classification and combinations of computed tomographic predictors. Neurosurgery 2005; 57:1173. 17. Kochanek PM, Carney N, Adelson PD. Guidelines for the acute medical management of severe traumatic brain injury in infants, children, and adolescents – Second Edition. Pediatric Critical Care Medicine 2012; 13: S1-S2. 18. SAFE Study Investigators, Australian and New Zealand Intensive Care Society Clinical Trials Group, Australian Red Cross Blood Service, et al. Saline or albumin for fluid resuscitation in patients with traumatic brain injury. N Engl J Med 2007; 357:874. 19. Bullock MR, Chesnut R, Ghajar J, et al. Surgical management of acute epidural hematomas. Neurosurgery 2006; 58:S7. 20. Bullock MR, Chesnut R, Ghajar J, et al. Surgical management of acute subdural hematomas. Neurosurgery 2006; 58:S16. 21. Bullock MR, Chesnut R, Ghajar J, et al. Surgical management of posterior fossa mass lesions. Neurosurgery 2006; 58:S47. 22. Bullock MR, Chesnut R, Ghajar J, et al. Surgical management of traumatic parenchymal lesions. Neurosurgery 2006; 58:S25. 23. Bullock MR, Chesnut R, Ghajar J, et al. Surgical management of depressed cranial fractures. Neurosurgery 2006; 58:S56. 24. Cooper DJ, Rosenfeld JV, Murray L, et al. Decompressive craniectomy in diffuse traumatic brain injury. N Engl J Med 2011; 364:1493. 25. Hutchinson PJ, Corteen E, Czosnyka M, et al. Decompressive craniectomy in traumatic brain injury: the randomized multicenter RESCUEicp study ( www.RESCUEicp.com ). Acta Neurochir Suppl 2006; 96:17. 26. Chestnut RM, Marshall LF, Klauber RM, et al. The role of secondary brain injury in determining outcome from severe head injury. J Trauma 1993;34:216-222. 27. Marmarou A, Anderson RL, Ward JD, et al. Impact of ICP instability and hypotension on outcome in patients with severe head trauma. J Neurosurg 1991;75:159-166.

www.medicalebookpdf.com

28. Stochetti N, Furlan A, Volta F. Hypoxemia and arterial hypotension at the accident scene in head injury. J Trauma 1996;40:764-767. 29. Chestnut RM, Temkin N, Carney N, et al. A trial of intracranial-pressure monitoring in traumatic brain injury. New Engl J Med 2012; 367(26): 24712481. 30. Clifton GL, Miller ER, Choi SC, et al. Lack of effect of induction of hypothermia after acute brain injury. N Engl J Med 2001; 344:556. 31. Clifton GL, Valadka A, Zygun D, et al. Very early hypothermia induction in patients with severe brain injury (the National Acute Brain Injury Study: Hypothermia II): a randomised trial. Lancet Neurol 2011; 10:131. 32. Hutchison JS, Ward RE, Lacroix J, et al. Hypothermia therapy after traumatic brain injury in children. N Engl J Med 2008;358(23):2447-56. 33. Joseph DK, Dutton RP, Aarabi B, Scalea TM. Decompressive laparotomy to treat intractable intracranial hypertension after traumatic brain injury. J Trauma. 2004;57(4):687-93. 34. Robertson CS, Valadka AB, Hannay HJ, et al. Prevention of secondary ischemic insults after severe head injury. Crit Care Med 1999; 27:2086. 35. Contant CF, Valadka AB, Gopinath SP, et al. Adult respiratory distress syndrome: a complication of induced hypertension after severe head injury. J Neurosurg 2001; 95:560. 36. Kosty JA, Le Roux PD, Levine J, Park S, Kumar MA, Frangos S, MaloneyWilensky E, Kofke WA: A Comparison of Clinical and Research Practices in Measuring Cerebral PerfusionPressure (CPP): A Literature Review and Practitioner Survey. Anesth & Analg 2013; 117(3):694-698. 37. Edwards P, Arango M, Balica L, et al. Final results of MRC CRASH, a randomised placebo-controlled trial of intravenous corticosteroid in adults with head injury-outcomes at 6 months. Lancet 2005; 365:1957.

www.medicalebookpdf.com

TRAUMATIC BRAIN INJURY QUESTIONS 1. All of the following are examples of secondary injury except: a. Free radical formation b. Inflammation c. Contusions d. Hypotension 2. Seizure prophylaxis with an anticonvulsant medication is recommended for how long after a severe traumatic brain insult? a. never b. 3 days c. 5 days d. 7 days e. 2 weeks f. indefinitely 3. The energy imparted to brain tissue from a projectile is most strongly dependent on the projectile's: a. Mass b. Shape c. Velocity d. Material 4. The incidence, nature or time course of which of the following distinguish blast TBI from blunt TBI: a. Cerebral vasospasm b. Malignant cerebral edema c. Diffuse axonal injury

www.medicalebookpdf.com

d. All of the above e. None of the above 5. The presence of which of the following distinguishes penetrating traumatic brain injury from blunt traumatic brain injury? a. Contusions b. Epidural hematoma c. Subdural hematoma d. Brain laceration e. Subarachnoid hemorrhage 6. The currently accepted threshold for treatment of intracranial hypertension is intracranial pressure: a. > 12 mmHg b. > 15 mmHg c. > 20 mmHg d. > 25 mmHg e. > 30 mmHg 7. Use of vasopressors and intravenous fluids to maintain cerebral perfusion pressure > 70 mmHg is associated with: a. Better clinical outcomes b. Increased incidence of lung injury c. Fewer cerebral infarctions and less ischemia d. Increased diffuse cerebral edema 8. The relationship between cerebral blood flow and cerebral perfusion pressure becomes more linear: a. In all patients with severe TBI b. When cerebral vascular autoregulation is impaired

www.medicalebookpdf.com

c. When cerebral oxygen demand exceeds cerebral metabolic rate d. When intracranial pressure is low 9. The DECRA study demonstrated that a. Unilateral craniectomy for focal TBI improved outcome b. Bifrontal craniectomy for diffuse TBI reduced ICP c. Bifrontal craniectomy for diffuse TBI improved outcome d. Unilateral craniectomy for focal TBI reduced ICP 10. Corticosteroids: a. Are useful as a primary therapy for diffuse traumatic cerebral edema b. Are useful as an adjunctive therapy for diffuse traumatic cerebral edema c. Should not be used to treat cerebral edema from TBI d. Should be used only to treat focal cerebral edema from traumatic injuries

www.medicalebookpdf.com

TRAUMATIC BRAIN INJURY ANSWERS 1. The correct answer is C. Primary injury is injury that occurs immediately at the time of the trauma and is typically caused by mechanical forces. Contusion, or bruising of the brain, is a form of injury caused by acceleration/deceleration. Secondary (delayed) injury may begin at the time of the traumatic insult or may begin in the subsequent hours to days. Secondary injury involves a host of cellular, biochemical, and organ-level pathological cascades, including free radical formation, inflammation, and hypotension that exacerbate brain damage. The central goal of TBI management is minimization of secondary injury. 2. The correct answer is D. Anticonvulsants decrease the risk of early posttraumatic seizures but do not impact the likelihood of developing posttraumatic epilepsy. Brain Trauma Foundation guidelines therefore recommend prophylactic treatment with an anti-convulsant medication for 7 days post-injury and no longer. 3. The correct answer is C. While a projectile's shape, angle of penetration, and the material influence the type of injury, kinetic energy is a product of its mass and the square of its velocity. Therefore velocity is the primary determinant of the energy transferred to brain tissue. 4. The correct answer is D. Diffuse axonal injury in blast TBI occurs in a dose-dependent fashion that likely differs from the DAI observed with closed-head injury. Malignant cerebral edema may occur rapidly (within an hour) as opposed to the more slowly developing edema seen in blunt TBI (hours to days). Cerebral vasospasm may occur in up to 50% of moderate to severe blast TBI and may last as long as one month. Additionally, patients with blast TBI frequently have concomitant blast injury to the eyes and to the auditory and vestibular systems. 5. The correct answer is D. While epidural, subdural, and subarachnoid hemorrhages may occur in both blunt and penetrating TBI, cerebral lacerations, or tearing of tissue, are the hallmark of penetrating TBI. Contusions typically result for acceleration/deceleration. 6. The correct answer is C. The Brain Trauma Foundation recommends initiation of therapy once ICP exceeds 20 mmHg. This is based on observational studies that established a correlation between ICP > 20 mmHg and poor outcome. There is a lack of convincing evidence that

www.medicalebookpdf.com

therapy guided by invasive ICP monitoring is superior to therapy guided by clinical (and radiological) examinations. 7. The correct answer is B. While initial studies suggested that using volume expansion and vasopressors to maintain CPP> 70 mmHg improved outcome, subsequent studies suggested that this approach does not improve outcome and is associated with increased risk of extracerebral injury, including acute respiratory distress syndrome. Brain Trauma Foundation guidelines therefore recommend a CPP target of 60 mmHg and avoidance of CPP < 50 mmHg and CPP > 70 mmHg. Optimization of CPP in a normotensive patient should start with efforts to lower ICP. 8. The correct answer is B. Normally, cerebral blood flow (CBF) is maintained constant across a wide range of cerebral perfusion pressures (CPP). This is accomplished by modulation of vascular diameter. As CPP increases, vascular diameter decreases to maintain constant cerebral blood flow. This is termed cerebrovascular autoregulation. In patients with TBI, autoregulation may be abnormal due to vasoplegia and the relationship between CPP and CBF becomes more linear. 9. The correct answer is B. The DECRA (DEcompressive CRAniectomy) trial randomized patients with severe diffuse blunt traumatic brain injury and refractory intracranial hypertension to bifrontal-temporoparietal decompressive craniectomy with durotomy or standard care. The surgical group had a significantly lower mean ICP and worse functional outcomes. 10. The correct answer is C. Multiple studies have examined the effects of corticosteroids on outcome after TBI. Most recently, the CRASH (Corticosteroid Randomization After Significant Head injury) study, a large international randomized placebo-controlled study of early administration of methylprednisolone, found that steroid administration was associated with increased risk of death. Steroids are therefore contraindicated for the treatment of cerebral edema due to TBI.

www.medicalebookpdf.com

Chapter 8

TRAUMATIC SPINAL CORD INJURY William Coplin CLINICAL CASE A 73-year old man slipped while practicing Tai Chi in the park. At the scene, he was awake, never having lost consciousness, and complained of pain in his neck. He had no movement or sensation in his arms or legs. Blood pressure (BP) was 148 mm Hg/palpable and heart rate was 92 beats per minute (bpm). In the Emergency Department, he was awake and breathing comfortably with a BP of 130/75 mmHg. Within an hour, his BP had fallen to 100/60 mmHg, while his heart rate remained relatively steady at 82 bpm. His neurological examination demonstrated that he could shrug his right shoulder, and that he barely had antigravity strength in his left biceps; he had minimal sensation on the lateral aspect of his forearms. In the spinal cord levels below these findings, he was insensate and flaccid. He underwent CT and MRI imaging of the cervical spinal cord.

Figure 8-1. Computed tomography of the cervical spine demonstrated degenerative joint disease and ossification of the posterior longitudinal ligament (A). Magnetic resonance imaging showed increased T2 signal in the mid- cervical spinal cord (B).

www.medicalebookpdf.com

His American Spinal Injury Association (ASIA) scale grade was B with a C5 injury level (motor incomplete but less than anti-gravity; some sensation). He received a 24-hour infusion of methylprednisolone, in accordance with prevailing management guidelines at the time. He underwent posterior surgical decompression via C4 and C5 laminectomies in the seventh hour after injury. His post-operative care included monitoring with subclavian central venous and radial arterial catheters, hemodynamic augmentation with two liters of 0.9% NaCl and a phenylephrine infusion titrated to a mean arterial pressure (MAP) of 85-90 mm Hg, venous thromboembolism prophylaxis with sequential compression devices on his legs and subcutaneous unfractionated heparin through the first day and low molecular weight heparin starting 24 hours after surgery. He was liberated from the ventilator the morning after surgery, and, with externally applied abdominal pressure (with a pillow) in synchrony with his coughing efforts, he was safely extubated that morning. He started eating with assistance later that day. He received a bowel regimen with scheduled senna tablets twice daily and a rectal laxative suppository every other day as needed. Foley catheter was removed the day after surgery and he received intermittent bladder catheterization as needed. His nurses repositioned him every two hours, and his skin was kept clean and dry. Temperature > 37.5oC was controlled with an external water-circulating gel pad device after he developed mild fevers to 37.9oC; all cultures across the first 72 hours were unremarkable, and a firstgeneration cephalosporin was discontinued 24 hours after surgery. Physical Medicine & Rehabilitation and physical and occupational therapy consultations were obtained the day after admission, and he was assisted to a chair the second day after injury with 2/5 strength below his initial spinal injury level, competent sacral innervation with near-normal bowel and bladder function, and return of most sensation below his initial level of injury (ASIA C). He was discharged from the ICU on post-injury day 5 to rehabilitation OVERVIEW/EPIDEMIOLOGY Traumatic spinal cord injury (SCI) has an incidence of about 12,000 per year in the United States, according to a 2009 report from the National Spinal Cord Injury Statistics Center and from the SCI Model Systems project of the National Institute of Disability and Rehabilitation Research (NIDRR, part of the US Department of Education). The majority of these injuries affect the cervical cord, resulting in incomplete tetraparesis in about 30% and complete tetraparesis in about 20% of all patients with SCI. About 25% of patients with thoracic SCI are left completely paraparetic, and 18.5% are incompletely paraparetic. The

www.medicalebookpdf.com

average age at injury has increased from 29 to 40 years, in line with the ageing of the general population. Eight in 10 of those injured are male; this figure has remained unchanged for well over 30 years. The most common mechanisms of injury are motor vehicle accidents (42%), falls, assaults (including penetrating injuries such as gunshot wounds), and sports-related injuries. Far more injuries occur during the summer months and around holidays. Survival from Spinal Cord Injury Mean survival for patients after SCI has steadily increased over the years from 52.8 months in 1955 [1], 110.5 months by 1976, 126 months by 1982, and now upwards of 12 years after injury. The largest gains have been in survival during the first two years after SCI [2]. The leading causes of death after SCI are cardiovascular diseases, infections, and suicide [3]. Nearly 88% of those surviving from the time of injury through hospital discharge return to private non-institutional domiciles. PATHOPHYSIOLOGY Primary non-penetrating injury occurs primarily as a result of disc herniation, fracture, and/or sub-luxation compressing upon the cord in the bony spinal canal. Complete transection of the cord (often from missile or other penetrating injuries) is rare. The series of concomitant events include hemorrhage into the cord, release of excitatory amino acids, accumulation of endogenous opiates, lipid hydrolysis, free radical release, ischemia and reperfusion injury. Inflammation, free radicals, excitotoxicity [4], and vascular disruption and ischemia ultimately lead to necrotic and apoptotic cell death within the cord. These various components have been the putative targets, along with edema formation, for various therapeutic trials to prevent secondary cord injury [5]. Pressure injury from compromise of the spinal canal also contributes to impedance of blood flow through the single anterior spinal artery and paired dorsal spinal arteries; this can lead to further oligemia and ischemia. Other causes of secondary injury to the cord, in addition to hypoperfusion, inflammation, and edema, include fever and apoptosis.. CLINICAL FEATURES Airway Issues in SCI As with any post-traumatic assessment and resuscitation, first maintaining or secondarily securing the airway is a primary concern. Care must be taken to prevent further spinal cord damage from manipulating an unstable cervical spine. Nasotracheal intubation, with proper topical mucosal anesthesia, can be

www.medicalebookpdf.com

preferred in some of these cases. Fiberoptic assistance reduces the need for neck movement and should be used when available. Short-acting sedation with a nonmuscle relaxing agent such as etomidate can be useful when these forms of awake intubation are not practically possible. A short-acting non-depolarizing agent such as rocuronium is preferred if muscle relaxation is truly needed; using a depolarizing neuromuscular blocking agent is not advisable due to the potential for excessive muscular potassium release in a paralyzed patient who may also have sustained muscular crush injury. There are no randomized studies comparing these various techniques. Manual in-line stabilization is recommended with the recognition that it may not always protect against cervical spine motion [6]. Additionally, traction may be dangerous in patients with ligamentous injury, so care must be taken not to distract or extend the neck when using in-line stabilization. Table 8-1 describes indications for intubation in traumatic spinal cord injury. Ventilatory Dysfunction in SCI As the phrenic nerve originates from the third through fifth cervical spinal cord level, there is necessarily ventilatory paralysis with complete injuries above this level; if these patients are to survive, they require immediate ventilatory support. Patients with injuries to the C3-5 area of the spinal cord may maintain varying degrees of ventilatory function. These patients usually require ventilatory support at least initially, as they will have substantial reductions in airflow and vital capacity (VC) [7]. Similarly, patients with lower cervical and high thoracic SCI may also have reduced VC early after injury albeit to a lesser degree. Paralysis of the intercostal and other non-diaphragmatic ventilatory musculature whose innervation derives from below the C5 spinal level may result from an initial period of spinal shock or from permanent injury. Loss of strength, tone, and proprioception in these muscles compounds weaknessinduced ventilatory dysfunction by changing the shape of the thoracic cavity, furthering ventilatory inefficiency. Similar weakness and loss of abdominal tone can hamper the effectiveness of coughing to clear airway secretions. Patients may become relatively hypoxemic when supine, as the abdominal contents may press on the lower portions of the lungs, interfering with normal basal expansion. This may produce mismatch, as the blood flow, particularly in the setting of the reduced vascular tone of neurogenic shock, tends to follow gravity into these lower relatively less-ventilated lung regions. Once spasticity develops, often by the middle of the second week after injury, paradoxical abdominal ventilatory movements decrease, the thoracic cavity resumes a more normal shape and capacity, and cough can improve [8]. With assisted pulmonary toilet and strengthening of the cervical accessory musculature of breathing, up to 80% of

www.medicalebookpdf.com

those with C3-5 lesions can be liberated from mechanical ventilation. Those with functioning diaphragms (lesions below C5) infrequently need long–term mechanical ventilatory support.

Cardiovascular Disturbances in SCI The main changes in cardiac performance after SCI are related to sympathetic denervation. Cardiac innervation derives from levels above T5; consequently injury below this level rarely produce any cardiac manifestations. Electrographic changes include diffuse ST segment and T-wave depression. Also seen are large upright T-waves with ST elevation, QT interval prolongation and prominent U-waves. Bradycardia, from unopposed vagal innervation, may occur episodically with vagal stimulation such as may occur during oral suctioning. A manifestation of neurogenic shock, these cardiac changes generally peak about post-injury day 4 and usually resolve within 2-6 weeks after injury [9]. Spinal Shock and Neurogenic Shock Spinal shock is the temporary loss of spinal cord neural function that may

www.medicalebookpdf.com

occur in the setting of acute SCI. As a result of this phenomenon, the initial neurological manifestations of SCI may be magnified. This effect may last between several hours and up to a few days. Neurogenic shock is a form of distributive shock that may lead to hypoperfusion (including the spinal cord) from associated bradycardia (due to unopposed vagal tone) and hypotension (from loss of sympathetically-mediated vascular tone and decreased systemic vascular resistance). This is usually seen with lesions at T4 (the lowest takeoff of cardiac sympathetic innervation) and above. The presence and severity of spinal shock depend on the spinal level affected, being commonly seen in cervical injuries and variably seen in thoracic injuries. It is uncommon with injuries in the region of the conus medullaris. Varying degrees of neurogenic shock may last for weeks. Treatment includes maintenance of euvolemia and administration of α sympathetic agonists such as phenylephrine, ephedrine, or midodrine (the latter two following the acute phase of injury) if needed. Fludrocortisone produces fluid retention, but does not improve the clinical syndrome of neurogenic shock. Associated Cervical Spinal Column Injuries The spectrum of neck and thoracic injuries associated with SCI include multiple pathologies. Fractures can be stable or unstable. Those involving posterior elements alone (e.g. spinous processes, lamina) tend to be stable. Those involving facets, pedicles, and/or vertebral bodies can be unstable by allowing abnormal movement of the spine with the potential for entrapment of the cord or nerve roots. Axial loading with about 30o of cervical flexion can lead to fracturedislocation [10 57]. Compression with flexion tends to yield fracture of the vertebral body. This can lead to “teardrop” fractures where pieces of bone may be retropulsed into the spinal canal. These fractures and those with vertical compression leading to a “burst” fracture of the vertebral body can lead to angulation (> 11o) and compression of the spinal canal. Subluxation (especially > 2-3 mm antero- or retrolisthesis) and/or dislocation can occur with such anterior spinal column or pedicle fractures or when the facets perch or “jump” over each other and “lock” in place. Traumatic disc herniation and/or ligamentous injuries (such as in the initial case presented in this chapter) can also compromise the canal and render the spine unstable. Vascular injuries such as dissection or pseudoaneurysm formation may occur as a result of blunt force to the cervical carotid artery, jugular vein, or vertebral artery. Dissections of the vertebral arteries typically occur at their V2 segments and are caused by entrapment at the edges of the transverse processes of cervical vertebrae 2 through 6,. Associated stretch injuries to nerve roots and/or brachial plexus may be difficult to distinguish clinically from acute SCI. However, nerve root or

www.medicalebookpdf.com

brachial plexus injuries tend to be unilateral and affect a single limb. DIAGNOSIS Imaging Neck Injuries and “Clearing” the Cervical Spine This chapter will discuss three of the many cervical spine radiographic imaging guidelines. The National Emergency X-Radiography Utilization Study (NEXUS) criteria were designed to attempt to identify those at low risk for cervical fracture/subluxation/dislocation [11]. The patient should be without posterior midline cervical tenderness. Problematically, the absence of midline cervical tenderness does not rule out cervical pathology. The patient should not be intoxicated, should have normal mental status, and no painful injuries that might distract attention from the examiner; these criteria are to allow valid neurological examination. Finally, the NEXUS low-risk patient should not have any focal neurological deficits. The patient meeting these five NEXUS criteria is said to be at low risk of cervical injury and may not need roentgenographic or other imaging of the neck or the spinal cord. These may be coupled with the Canadian Cervical-Spine Guidelines, which are intended to identify patients who should receive imaging [12,13]. According to these often-used guidelines, a patient should receive imaging if they meet ANY of the following criteria: midline cervical tenderness, age > 65 years, “dangerous mechanism” of injury, neurological symptoms, or when the patient needs to remain supine, had the immediate onset of neck pain, or is unable to fully rotate the neck. More recently, the 2013 guidelines suggest an approach based on level of consciousness, ability to undergo valid examination (as for the NEXUS criteria), and presence or absence of neurological symptoms or other injuries [14]. Radiographic evaluation of the cervical spine is not recommended for the awake asymptomatic patient (e.g. someone without neck pain or tenderness, who has a normal neurological examination, has no other injury that may detract from accurate evaluation, and is able to complete full active functional range of motion). For such patients, any cervical immobilization may be removed, and no spine imaging is recommended. For the awake patient with any of the symptoms just mentioned, computed tomography (CT) of the cervical spine is the preferred imaging modality. Plain x-ray film radiographs are no longer recommended if CT scanning is available. The traditional three plain-film views (anteroposterior, lateral, and odontoid +/- “swimmer's” view) are recommended only when CT is not available and are to be supplemented with CT, when available, for any suspicious or poorly seen areas. Plain tomograms were often previously used to obtain odontoid views; however, the equipment to even perform this study has

www.medicalebookpdf.com

become obsolete and is increasingly difficult to find. If the imaging is normal, the neck should be immobilized (i.e., properly-fitting hard cervical collar in neutral position) until the patient is without symptoms. Alternatively, the collar may be removed after adequate dynamic flexion and extension radiographs or spine MRI are proven to be normal. If the patient is obtunded or cannot cooperate with the physical examination (e.g., painful other injury distracting from adequate neurological examination, intoxicated, or otherwise with abnormal mental status) the same initial CT imaging paradigm is recommended. The collar is removed once the patient can be properly interviewed and examined or spine MRI shows no injury. In these patients without a reliable physical examination, flexion/extension films appear to be of marginal benefit and are not recommended to clear the cervical spine. ASIA Impairment Scale The American Spinal Injury Association (ASIA) has published a standardized ordinal impairment scale for communicating the functional severity of SCI. The scale and useful bedside diagrams and charts for dermatomes and functional muscle groups by spinal level are available online at www.asiaspinalinjury.org.

The scale predicts the chances of regaining functional status [15]. About 25% of injuries present with complete deficits (ASIA A); of these, nearly 90% will not regain function, but as many as 15% will improve including 3% who regain functionally meaningful motor function. About 15% of patients present with complete motor deficits by with some preserved sensory function (ASIA B); of these, 54% will regain motor function. About 40% of patients present with incomplete motor loss (ASIA C-D); of these, 86% will regain the ability to ambulate. These odds may now be improved (see section on Early Surgery

www.medicalebookpdf.com

below). SCI Syndromes Various spinal cord syndromes can be produced by traumatic injury. These include: – Transverse myelopathy – Central cord syndrome –so-called “man in a barrel” presentation in which the arms are more affected than the legs. This syndrome most frequently occurs among older persons with cervical spondylosis; however, it also may occur in younger individuals. It is the most common form of incomplete SCI, representing about 9% of SCI. Prognosis is generally good for at least some functional recovery; however, patients > 70 years old, those with complicating pre-existing comorbidities, and those with severe injuries may not recover as well. – Anterior spinal artery syndrome – motor and anterolateral sensory modalities affected with preservation of proprioception; associated with hyperflexion injuries or cord contusion from extruded disc material or retropulsed bone –Brown-Sequard syndrome – lateral hemi-cord, ipsilateral motor, contralateral sensory; most commonly seen with penetrating SCI –Posterior cord syndrome – proprioception affected >> motor –Transient quadriparesis – may represent a form of spinal shock TREATMENT Management of patients with acute traumatic SCI, particularly those with cervical injuries, is recommended to take place in a setting with cardiopulmonary monitoring. The three major areas of therapy include surgery, pharmacological therapy (e.g. corticosteroids), and critical care issues, the latter of which include, but are not limited to airway management and ventilation, hemodynamics, venous thromboembolism prophylaxis, prevention and treatment of systemic complications, and fever control. Consensus guidelines for managing acute cervical spine and spinal cord injuries were updated in 2013 [16]. Airway Management As described previously, the key point is to ensure oxygenation and ventilation while avoiding additional iatrogenic injury to the spinal cord. It is important to avoid hyperextension, rotation, or other movement of the neck during intubation. Awake, fiberoptic intubation is usually preferred; in-line stabilization without traction is an alternative when a fiberoptic laryngoscope or

www.medicalebookpdf.com

bronchoscope is unavailable. A spinal level above C5 will almost necessarily mean at least some degree of diaphragmatic compromise. Those with a C5-C8 level often, but not always, will require some degree of acute mechanical ventilatory support. In patients with ventilatory failure due to cervical spinal cord injury, intubation is preferred over non-invasive ventilation because recovery of adequate ventilatory function is uncertain and slow when it occurs.. Hemodynamics Neurogenic shock can occur with any lesion above the lowest takeoff for the cardiac sympathetic afferents (T1-T4). Some degree of post-traumatic functional sympathectomy is nearly ubiquitous in cervical SCI. This may render the heart unable to increase cardiac output by raising its rate or stroke volume and reduces the tone of the peripheral arteries, thus leading to hypotension. The resulting effect may be hypoperfusion of smaller downstream arterioles and capillaries in the watershed regions of the spinal cord, which in turn may induce secondary vascular injury to the cord. Although usual treatment of shock aims to maintain a systolic BP above 90 mmHg, this target may be insufficient in patients with SCI In these patients, the current recommendation is to maintain a mean arterial pressure (MAP) 8590 mmHg for the first seven days after SCI [17]. This duration is loosely based on the time when it is presumed that the spinal cord damage has become completed. Of note, this recommendation is only supported by class III evidence [18]. A recent study suggested that spinal perfusion pressure may be measured guide hemodynamic augmentation with vasopressors; this strategy may improve evoked potentials and even neurological deficits in some patients [19]. Care must be taken regarding venous pooling of administered IV fluids, as sympathetic and muscular tone is decreased. As such, after initial intravenous saline resuscitation (as per the recommendations for initial resuscitation of the injured patient in Advanced Trauma Life Support), exogenous administration of α agonists, such as phenylephrine, should be considered. Euvolemia (as directed by standard endpoints such as urine output or lactate clearance should be the usual goals of initial fluid resuscitation. If some β agonism is also needed or cardiac output falls due to increased afterload from pure α agonism, norepinephrine could be considered. Dopamine has also been used, but may be associated with tachyarrhythmias [20]. Vasopressin infusion or cardiac inotropic agents are rarely used in isolated spinal cord injury given the availability of other pressor agents. Very frequent BP monitoring is needed and this is usually best accomplished through continuous BP monitoring with an indwelling arterial catheter. The unopposed parasympathetic tone in the setting of severe spinal cord injury may lead to reflex bradycardia, particularly with vagally stimulating

www.medicalebookpdf.com

maneuvers such as oral or endotracheal suctioning; pretreatment or rescue treatment with atropine (0.3 mg IV) may be useful in symptomatic situations related to this phenomenon. Venous Thromboembolism Prophylaxis Patients with paraparesis or tetraparesis after SCI represent a group with very high risk of venous thromboembolism (VTE), with unprotected patients running upwards of a 40% risk of developing VTE during the acute hospitalization. Current guidelines recommend the use of prophylaxis for any patient with motor deficits, especially severe deficits preventing ambulation. Low molecular weight heparins (LMWH), rotating beds, or a combination are recommended [21]. Alternatively, low dose subcutaneous heparin (e.g., 5000 units every 8 hours) in combination with pneumatic compression stocking devices can be used. Prophylaxis should begin within 72 hours of injury. The timing to commence VTE chemoprophylaxis is dependent upon several factors. There is a dearth of studies evaluating the safety of beginning prophylactic anticoagulation in the first two days after the injury. Arguably, this is because of discomfort in conducting randomized trials of heparin or LMWH within this very early time window. There are observational data that beginning chemoprophylaxis after the first day appears safe with regard to hemorrhage risk. One possible exception is the administration of LMWH within 24 hours of spinal surgery (e.g. decompression, fixation) or other procedures (e.g. lumbar puncture or drains) that may violate the dura. There is a warning that LMWH may increase the chances of local bleeding in such procedures within this time frame; low-dose subcutaneous heparin combined with pneumatic compression devices represent a safe and effective alternative in these instances. In the past, some used oral anticoagulation alone (e.g. warfarin) in fixed low doses; however, this practice is not currently recommended. Also not recommended as routine prophylaxis are inferior vena cava filters when anticoagulation is not contraindicated [22]. Filters do not prevent lower body deep venous thrombosis, and smaller emboli can still travel through or from the filter to the lungs via the right heart. Patients may develop inferior vena cava obstruction from clot suspended in the filter preventing forward blood flow and long-term use of filters increases the risk of deep vein thrombosis. Such filters are recommended for select patients who either fail anticoagulants or are not candidates for LMWH (e.g. ongoing bleeding). The duration of chemoprophylaxis depends on the motor recovery of the patient. Pharmacotherapy for SCI Several drugs have failed to improve neurological function after SCI in human trials. These include naloxone, GM-1 ganglioside, and 21-aminosteroids.

www.medicalebookpdf.com

Perhaps the most studied, storied, and controversial issue in SCI care is that of corticosteroids. There have been several randomized trials evaluating the administration of high-dose corticosteroids for acute SCI; the most “famous” and controversial of these were the National Acute Spinal Cord Injury Study (NASCIS) II and III which compared various doses of corticosteroids, with naloxone, the 21-aminosteroid tirilizad, or placebo [23,24]. Time to administration appeared to be key for any effect of corticosteroids seen in these studies, with a maximum time window of 8 hours identified for clinical effect. Dosing regimens were derived from weight-based animal studies: 30 mg/kg IV over the first hour and 5.4 mg/kg IV over the following 2347 hours. The methodology of the NASCIS studies has been criticized because of issues regarding the time criteria, analysis of injury severity and its relation to treatment efficacy, use of unilateral motor measurements as opposed to more solid functional outcome measures and insufficient standardization of concurrent treatments. Initial interpretation of benefit from the intervention was subsequently questioned. Moreover, concerns over a higher risk of steroidrelated complications, such as infection or poor wound healing, in clinical practice progressively led many trauma centers to abandon the use of high dose corticosteroids in SCI. In fact, while the 2002 SCI Guidelines had left methylprednisolone infusion as an option [18], the most recent 2013 Guidelines do not recommend its use [16]. Early Surgery Emergency reduction of unstable fractures, dislocations, or subluxations should be undertaken, particularly in the patient with any neurological symptoms. For closed reduction, one must be cautious not to overly distract the injured area with traction lest ligamentous injury or laxity allow further distracting injury. Unstable fractures, subluxations, or dislocations are most commonly treated with reduction and internal fixation. The Surgical Timing in Acute Spinal Cord Injury Study (STASCIS) was completed and recently published [25]. In brief, STASCIS suggested benefit from early decompressive surgery (< 24 hours after injury versus later) for symptomatic cervical SCI, when decompression of the canal was otherwise indicated (e.g. traumatic disc rupture, fracture, subluxation, etc). With this approach, the odds of improving at least two grades on the ASIA scale (e.g. ASIA B to D or E) were more than doubled (OR 2.83, 95% CI 1.10-7.28, P = 0.03). Complications were similar between the two study groups. Therapeutic Hypothermia Laboratory investigations and animal studies suggest that hypothermia may be beneficial in acute SCI. However, clinical experience remains very limited.

www.medicalebookpdf.com

This intervention captured attention from the public and the media after the case of a professional football player whose recovery was widely credited to therapeutic hypothermia. After an on field injury, he reportedly had a complete spinal cord syndrome (ASIA A) below the clavicles. Of note, he received methylprednisolone infusion in the ambulance as well as IV chilled saline and ice packs to the groin. In the ED, he was hemodynamically stable with a temperature of 36.6oC. His C3-4 facet dislocation was operatively reduced about three hours after injury. He subsequently underwent mild hypothermia therapy to 33oC for several days. Initial recovery of strength was noted about 15 hours post-injury and he has subsequently regained the ability to ambulate [26]. This case report demonstrates the challenge of assessing the effectiveness of a single intervention when multiple rational but unproven treatments are administered concurrently. However, enthusiasm over the potential of hypothermia has led to proposals for clinical trials to test the hypothesis that initial cooling to 33oC may improve clinical outcome after traumatic SCI. Although animal studies have been mostly very promising, hypothermia for SCI is considered experimental at this time [27]. Other Recovery Strategies Under Investigation Other strategies for the treatment of spinal cord injury focus on recovery after injury. Preclinical and clinical studies are investigating a range of potential options including stem cells, axonal bridging across scar tissue, advanced rehabilitation strategies such as microarray electrical stimulation, and novel biomedical assistive devices. REFERENCES 1. Dietrick RB, Russi S. Tabulation and review of autopsy findings in fiftyfive paraplegics. Journal of the American Medical Association. 1958;166:41-4. 2. Strauss DJ, Devivo MJ, Paculdo DR, Shavelle RM. Trends in life expectancy after spinal cord injury. Archives of physical medicine and rehabilitation. 2006;87:1079-85. 3. Le CT, Price M. Survival from spinal cord injury. Journal of chronic diseases. 1982;35:487-92. 4. Faden AI, Simon RP. A potential role for excitotoxins in the pathophysiology of spinal cord injury. Annals of neurology. 1988;23:623-6. 5. Sekhon LH, Fehlings MG. Epidemiology, demographics, and

www.medicalebookpdf.com

pathophysiology of acute spinal cord injury. Spine. 2001;26:S2-12. 6. Lennarson PJ, Smith D, Todd MM, et al. Segmental cervical spine motion during orotracheal intubation of the intact and injured spine with and without external stabilization. Journal of neurosurgery. 2000;92:201-6. 7. Brown R, DiMarco AF, Hoit JD, Garshick E. Respiratory dysfunction and management in spinal cord injury. Respiratory care. 2006;51:85368;discussion 69-70. 8. Goldmann AL, George J. Postural hypoxemia in quadriplegic patients. Neurology. 1976;26:815. 9. Lehmann KG, Lane JG, Piepmeier JM, Batsford WP. Cardiovascular abnormalities accompanying acute spinal cord injury in humans: incidence, time course and severity. Journal of the American College of Cardiology. 1987;10:46-52. 10. Torg JS, Vegso JJ, O'Neill MJ, al e. The epidemiologic, pathologic, biomechanical, and cinematographic analysis of football-induced cervical spine trauma. Am J Sports Med. 1990;18:507. 11. Hoffman JR, Wolfson AB, Todd K, Mower WR. Selective cervical spine radiography in blunt trauma: methodology of the National Emergency XRadiography Utilization Study (NEXUS). Annals of emergency medicine. 1998;32:461-9. 12. Stiell IG, Wells GA, Vandemheen KL, et al. The Canadian C-spine rule for radiography in alert and stable trauma patients. JAMA : the journal of the American Medical Association. 2001;286:1841-8. 13. Stiell IG, Clement CM, McKnight RD, et al. The Canadian C-spine rule versus the NEXUS low-risk criteria in patients with trauma. The New England journal of medicine. 2003;349:2510-8. 14. Ryken TC, Hadley MN, Walters BC, et al. Radographic Assessment. Neurosurgery. 2013;72:5472. 15. Stevens RD, Bhardwaj A, Kirsch JR, al e. Critical care and perioperative management in traumatic spinal cord injury. J Neurosurg Anesthesiol. 2003;15:215-29. 16. Hadley MN, Walters BC, Arabi B, et al. Guidelines for the Management of Acute Cervical Spine and Spinal Cord Injuries. Neurosurgery. 2013;72:1259.

www.medicalebookpdf.com

17. Casha S, Christie S. A Systematic Review of Intensive Cardiopulmonary Management after Spinal Cord Injury. J Neurotrauma. 2011;28:1479-14-95. 18. Hadley MN, Walters BC, Grabb PA, et al. Guidelines for the Management of Acute Cervical Spine and Spinal Cord Injuries. Clin Neurosurg. 2002;49:407-98. 19. Werndle MC, Saadoun S, Phang I, et al. Monitoring of spinal cord perfusion pressure in acute spinal cord injury: initial findings of the injured spinal cord pressure evaluation study*. Critical care medicine. 2014;42:646-55. 20. Inoue T, Manley GT, Patel N, Whetstone WD. Medical and surgical management after spinal cord injury: vasopressor usage, early surgerys, and complications. J Neurotrauma. 2014;31:284-91. 21. Dhall SS, Hadley MN, Arabi B, et al. Deep Venous Thrombosis and Thromboembolism in Patients With Cervical Spinal Cord Injuries. Neurosurgery. 2013;72:244-54. 22. Dhall SS, Hadley MN, Aarabi B, et al. Deep venous thrombosis and thromboembolism in patients with cervical spinal cord injuries. Neurosurgery. 2013;72 Suppl 2:244-54. 23. Bracken MB, Shepard MJ, Collins WF, et al. A randomized, controlled trial of methylprednisolone or naloxone in the treatment of acute spinal-cord injury. Results of the Second National Acute Spinal Cord Injury Study. The New England journal of medicine. 1990;322:1405-11. 24. Bracken MB, Shepard MJ, Holford TR, et al. Administration of methylprednisolone for 24 or 48 hours or tirilazad mesylate for 48 hours in the treatment of acute spinal cord injury. Results of the Third National Acute Spinal Cord Injury Randomized Controlled Trial. National Acute Spinal Cord Injury Study. JAMA : the journal of the American Medical Association. 1997;277:1597604. 25. Fehlings MG, Vaccaro A, Wilson JR, et al. Early versus Delayed Decompression for Traumatic Cervical Spinal Cord Injury: Results of the Surgical Timing in Acute Spinal Cord Injury Study (STASCIS). PLoS ONE. 2012;7:e32037. 26. Cappuccino A, Bisson LJ, Carpenter B, Marzo J, Dietrich WD, Cappuccino H. The Use of Systemic Hypothermia for the Treatment of an Acute Cervical Spinal Cord Injury in a Professional Football Player. Spine.

www.medicalebookpdf.com

2010;35:E57-E62. 27. Kwon BK, Mann C, Sohn HM, et al. Hypothermia for spinal cord injury. Spine J. 2008;8:859–74. 28. Poonnoose PM, Ravichandran G, McClelland MR. Missed and mismanaged injuries of the spinal cord. The Journal of trauma. 2002;53:314-20.

www.medicalebookpdf.com

TRAUMATIC SPINAL CORD INJURY QUESTIONS 1. Some have suggested worse outcomes for patients with SCI who receive endotracheal intubation. What is a speculated cause? a. Traction during in-line stabilization distracting unstable fractures or subluxations b. Episodic hyperventilation or hypoventilation c. Delay in surgical decompression due to time taken for intubation d. latrogenic pneumothorax 2. Spinal cord injury above T5 may result in what type of hemodynamic shock? a. Cardiogenic b. Spinal c. Obstructive d. Neurogenic 3. Resuscitation after SCI includes all of the following concepts EXCEPT: a. Patients should receive fluids liberally until blood pressure normalizes b. Infusion of an alpha agonist may be necessary soon after initial volume resuscitation c. Follow usual ATLS resuscitation parameters regarding plasma and red blood cell infusion d. Saline infusion should be used to target euvolemia e. Resuscitation may continue to a MAP of 85-90 mmHg 4. Which one of the following is NOT an acceptable associated condition when clearing the cervical spine? a. Alert and oriented b. Normal neurological examination

www.medicalebookpdf.com

c. Absence of neck pain d. Extremity fracture e. No alcohol on board 5. Central cervical SCI has the following characteristics: a. Motor deficits usually affect the legs more than the arms b. Absence of increased T2 MRI signal is a poor predictor of recovery c. Most patients recover some neurological function below the level of the injury d. It occurs commonly in persons less than 50 years of age 6. The most common organ injured in transabdominal gunshot wounds to the spine is: a. Small bowel b. Colon c. Liver d. Abdominal vascular structures 7. A 41-year-old man is brought in after a car accident. He was wearing a seat belt. He is said to be weak in all extremities, and his blood pressure is unstable with an unwavering heart rate. He has jumped cervical facets. He is taken to the OR for an emergency open reduction and internal fixation. Which of the following statements is true? a. Reduction of his subluxation should be performed as soon as possible b. Surgical stabilization of the cervical spine should be deferred until after 24 hours c. There is little need to evaluate for cervical vascular injuries d. Detailed neurological examination can be deferred until after reducing his facets e. A hard cervical collar alone is sufficient until the time of surgical reduction

www.medicalebookpdf.com

8. In the 2013 Guidelines for the Management of Acute Cervical Spine and Spinal Cord Injuries [16], all are correct EXCEPT: a. Augmenting BP to an MAP of 85-90 mm Hg is considered an option b. Prophylactic hypothermia is considered experimental c. The majority of patients are managed non-operatively d. Steroids are considered a necessary part of medical management 9. Which of the following may be missed on initial cursory evaluation: a. Fracture-subluxation on CT scan of the cervical spine b. Spinal cord injury c. Associated carotid-vertebral injuries d. Ligamentous injuries e. All of the above 10. Spinal shock is characterized by the following parameter: a. Permanent neurological deficit below the level of radiographic SCI b. Unstable blood pressure c. Recovery of initially non-functioning spinal levels below the level of apparent functional loss d. Bradycardia with vagal stimulation

www.medicalebookpdf.com

TRAUMATIC SPINAL CORD INJURY ANSWERS 1. The correct answer is A. Endotracheal intubation has been associated with worse outcomes in patients with SCI. Inadvertent traction on or hyperextension of an unstable spine during attempted in-line stabilization have been incriminated as causes for the observed worse outcomes. Only highly skilled personnel should perform intubation by direct laryngoscopy in patients with possible cervical spine injury, using in-line traction without distracting traction. 2. The correct answer is D. Cervical and upper thoracic spinal cord injury may result in neurogenic shock, a distributive type of hemodynamic shock. There is loss of sympathetic tone resulting in vasomotor paralysis and unopposed vagal tone. 3. The correct answer is A. During initial resuscitation after SCI, saline infusion should be limited, as excessive fluids, in the face of loss of vasomotor tone, may lead to venous pooling. Completion of resuscitation, especially in the face of neurogenic shock may well require infusion of alpha agonists after restoration of euvolemia. A MAP of 85-90 mmHg is the target option, in theory, to facilitate perfusion of the centripetal smaller arterial branches of the spinal cord. 4. The correct answer is D. Extremity fracture can be a source of severe pain and distress for the patient, potentially distracting him from examination. The patient must meet all the other listed criteria and not have any suspected brain injury. 5. The correct answer is C. Prognosis for traumatic central cord injury varies, but most people have some recovery of neurological function. Evaluation of abnormal signal on MRI images can help predict the likelihood that neurological recovery may occur. The syndrome is seen most commonly in persons over age 50. The syndrome produces the classic “man in a barrel”, with the arms being affected usually more so than are the legs. 6. The correct answer is A. The small bowel occupies the most space and thus is most liable to injury after transabdominal gunshot wounds to the spine. Colon, liver and vascular structures are less common in descending order.

www.medicalebookpdf.com

7. The correct answer is A. The patient has neurogenic shock, indicating at least some degree of either cervical SCI or extramedullary sympathetic dysfunction. This is an unstable spine dislocation that may be associated with ligamentous injury or facet fracture. It should be reduced as soon as possible with roentgenographic guidance to avoid over-distraction, should the longitudinal or other ligaments be incompetent. The STASCIS study suggests that stabilization of the spine within the first 24 hours after injury may well improve outcome versus waiting beyond that time period. Cervical carotid-vertebral injuries may occur in association with such severe forceful “whiplash” mechanism of injury. A cervical collar alone, without reduction of the facets, is unacceptable. Complete neurological examination should be undertaken both before and after reduction to ensure the procedure has not caused further injury. 8. The correct answer is D. The majority of cervical spine injuries are managed non-operatively. Guidelines now recommend against giving methylprednisolone infusions. 9. The correct answer is E. About 5% of cervical fractures are missed, and about 2/3 of these patients have further spinal-cord damage as a result. About 30% of cases of delayed diagnosis of cervical spine injury develop permanent neurological deficits. Ligamentous injuries may be missed where the facets may have perched or jumped, and then have fallen back into place after the distracting injury. In the absence of suspicion and adequate neurological examination, associated carotid-vertebral injury or SCI may be overlooked [28]. 10. The correct answer is C. Spinal shock is characterized by return of function to spinal levels below the permanent injury. Answers B and D are features of neurogenic shock.

www.medicalebookpdf.com

Chapter 9

NON-NEUROLOGICAL TRAUMA, BURNS, AND THERMAL INJURY Deborah Stein, Christos Lazaridis, and Geoffrey Ling CLINICAL CASE A 21 year old male presents to the Trauma Center after a motor vehicle collision. He is complaining of shortness of breath and abdominal and pelvic pain. Initial vital signs: blood pressure (BP) 82/45 mmHg, heart rate (HR) 130 beats per minute, respiratory rate (RR) 38 breaths per minute, oxygen saturation (SpO2) 82%. He is noted to have crepitus on the left chest, absent breath sounds on the left and tracheal deviation to the right. A needle decompression is rapidly performed with a “gush of air” noted. Subsequently, vital signs are BP 92/50 mmHg, HR 110 per minute, RR 25 per minute, SpO2 98%. The patient complains of increasing pain and appears pale and diaphoretic. He begins to become lethargic. The patient is intubated for airway protection and a thoracostomy tube is placed on the left side with return of a small amount of blood. Chest radiograph reveals a pulmonary contusion on the left and multiple rib fractures. His mediastinum is abnormal. Pelvic radiograph reveals a lateral compression pelvic fracture with no displacement of the sacroiliac joints noted (Figure 9-1).

www.medicalebookpdf.com

Figure 9-1. Lateral compression Type I fracture

His FAST exam is positive for hemoperitoneum. Blood products are hung immediately after 2 large bore peripheral venous catheters are inserted and blood is sent to the laboratory, for testing including a type and crossmatch. The blood bank is notified of the need for a possible massive transfusion event. Repeat BP is 80/42 mmHg , HR 120 per minute, SpO2 100% on 50% FIO2. The patient is taken emergently to the operating room (OR). In the operating room, a shattered spleen is noted without other injuries. A splenectomy is performed and the patient's hemodynamics normalize after 4 units of packed red blood cells and 4 units of fresh frozen plasma. A computed tomography (CT) scan demonstrates a traumatic aortic rupture with pseudoaneurysm (Figure 9-2), left pulmonary contusions without significant hemo- or pneumothorax, and a lateral compression type 1 pelvic fracture with small associated pelvic hematoma and no active contrast extravasation.

www.medicalebookpdf.com

Figure 9-2. CT of traumatic aortic rupture (arrow)

He is immediately placed on an esmolol drip due to his aortic injury to decrease shearing forces on the aortic wall and is transferred to the ICU. Enteral nutrition via a naso-jejunal tube is begun and he is started on prophylactic low molecular weight heparin. On post-operative day 1, he is taken to the OR for a thoracic endovascular aortic repair (TEVAR). His chest tube is removed on hospital day (HD) 4 and he is extubated on HD 5. The patient is transferred to an acute rehabilitation facility on HD 7. OVERVIEW Trauma is the leading cause of death for children and young adults and is a major public health issue. The initial management of the trauma patient follows a prescribed set of steps – the primary, secondary, and tertiary survey – in order to assure that life threatening injuries are immediately prioritized and addressed. Hemorrhagic shock, being the leading cause of preventable death and the most common cause of death in the first few hours following injury, requires significant resources and expertise to manage it effectively. After initial stabilization, trauma patients remain at risk for development of severe

www.medicalebookpdf.com

complications in the ICU. Acute and ICU management of burns and thermal injuries are also important for any intensivist to understand as they not only occur in isolation, but also in association with traumatic injury. TRAUMA EPIDEMIOLOGY Traumatic injury is the leading cause of death in the United States for all people aged 1-44 and accounts more than 5.8 million deaths each year worldwide [1]. Neurological injury is the leading cause of death following trauma. Hemorrhage is the second most common cause of death and the leading cause of preventable death. In U.S. Trauma Centers, motor vehicle crashes are the leading cause of injury, but in the young and old, falls represent the most common mechanism of injury. Injuries are classified in one of 2 ways – by mechanism or intent. Injuries can be described as accidental (motor vehicle crashes, falls, pedestrian stuck incidents) or non-accidental (assault, abuse, violence), but the most useful general classification is by mechanism of injury, namely blunt (vehicular incidents or falls), penetrating (gunshot or stab wounds), or crush injuries. INITIAL ASSESSMENT The general approach to multisystem trauma involves a proscribed set of sequential assessments where injuries are systematically identified and treated. As outlined in the American College of Surgeon's Advance Trauma Life Support (ATLS) Course, the initial evaluation of any patient is dictated by the “primary survey” which identifies immediately life-threatening injuries that require emergent intervention, the “secondary survey” during which a more detailed clinical examination is combined with imaging, and a “tertiary survey” which is typically often performed after hospital admission and focuses on missed injuries and identification of early complications [2]. Primary Survey The principle of the primary survey is a rapid and systematic search for acutely life-threatening injuries requiring prompt treatment. It should be completed in a few minutes and follows the ABCDEs of resuscitation: Airway: Look, listen (stridor/wheeze), feel Breathing: Look (symmetry, paradoxical movement), listen, feel (subcutaneous emphysema) Circulation: Hemodynamics (pulse rate and blood pressure), peripheral

www.medicalebookpdf.com

perfusion, signs of external or internal bleeding Disability: Glasgow Coma Score (GCS), pupillary response, other cranial nerve function, and major motor or sensory deficits Exposure: Undress the patient completely and prevent hypothermia with application of warm blankets. Airway Potential loss of the airway should be suspected in all seriously injured patients. It does not always happen immediately and it may progress in a subacute manner; therefore, a high index of suspicion should be maintained. Stridor and labored breathing are important signs of impending airway compromise. An altered level of consciousness due to head injury is the most common cause of airway obstruction, but obstruction from blood and bone fragments also frequently occurs, particularly in the setting of major facial fractures. Immediate action should be taken in the form of clearance of the airway of blood or vomitus and a jaw thrust with insertion of an oro- or nasopharyngeal airway. If needed, endotracheal intubation should be performed utilizing a rapid sequence intubation, as all these patients are at risk of aspiration. In-line stabilization of the cervical spine is a mandatory precaution. Breathing Supplemental oxygen should always be administered to injured patients. Pulse oximetry should be utilized as in all critically ill patients, but with the recognition that it can be inaccurate in patients with poor peripheral perfusion. A drop in SpO2 is a late sign of respiratory collapse. Important clinical signs and symptoms of respiratory compromise include: tachypnea, use of accessory muscles, unwillingness to lie down, and absent or diminished breath sounds. During the primary survey, there are a few imminently life-threatening injuries to the lungs and thorax that need to be rapidly identified and treated. Tension Pneumothorax (tPTX): May occur as a result of either blunt or penetrating trauma. It results from a one-way valve effect mechanism, in which air is progressively trapped in the pleural space raising intrathoracic pressure. As a result, the trachea, mediastinum, diaphragm, and lungs are compressed. Most importantly, venous return becomes compromised from mediastinal shift resulting in hypotension and death if left untreated. Classic clinical features include a shock state, tachypnea, absence of breath sounds and hyper-resonance on the affected side, tracheal deviation, and jugular venous distention from mediastinal distortion. tPTX is life-compromising and should be treated immediately by needle decompression, based on clinical features and a high level of clinical suspicion. Decompression is

www.medicalebookpdf.com

performed via a large-bore needle inserted into the second intercostal space (just above the third rib) in the mid-clavicular line. Chest tube insertion should then immediately follow. It is not uncommon for needles, especially ones placed on the pre-hospital environment, to not enter the thoracic compartment and as a result to not effectively decompress the pleural space [3]. Signs of persistent shock can be an indication of an incompletely decompressed tPTX. A common cause of a tPTX is progression of a simple pneumothorax when the patient is placed on positive pressure ventilation. Open Pneumothorax (oPTX): Although relatively rare, an oPTX occurs when there is a significant defect in the chest wall. The classic teaching is that if the diameter of this defect is more than two-thirds that of the trachea or greater, air is drawn preferentially through it and not via the trachea. An attempt to seal the opening can be made by the use of a 3/4 occlusive dressing which through a flutter valve mechanism allows escape of air from the chest and prevents further entry. A chest tube is then inserted on a separate site. Circulation During the primary survey, a rapid assessment of circulation includes several components: evaluation of hemodynamics, placement of venous access, and examination for and rapid treatment of active hemorrhage. Shock is the pathophysiological condition of inadequate tissue perfusion and oxygenation. In trauma, this can result from compromised circulating volume and hemorrhage, inadequate myocardial function (e.g. cardiac contusion), and sympathetic failure (e.g. spinal cord injury). Hemorrhage is by far the most frequent cause of shock following trauma and should always be assumed until definitively ruled out. The first and most important step is recognition of shock. Early clinical signs typically include tachycardia and cutaneous vasoconstriction. The skin should be examined for pallor, coolness and capillary refill. A relatively normal systolic BP (SBP) may be seen early in shock states as a result of intact compensatory mechanisms, making early clinical signs critical in the prompt recognition of a shock state. An early sign of shock that may occur prior to hypotension is narrowing of pulse pressure, a normal compensatory response. In order to maintain perfusion of critical organs (brain, heart, lung), circulating blood is shunted from the periphery via an increase in sympathetic tone causing peripheral vasoconstriction. This causes an increase in diastolic blood pressure (DBP) and thus narrowing of pulse pressure. Hemorrhagic shock is classically stratified into 4 classes. Class I is when a

www.medicalebookpdf.com

patient has lost less than 15% of his estimated blood volume ( 80 mmHg. In the right panel, a strong positive linear correlation is seen between the two variables (Pearson's R oxygen pressure correlation coefficient, ORx, is 0.74), indicating impairment in autoregulation and a high risk for secondary delayed cerebral ischemia.

www.medicalebookpdf.com

PbtO 2 monitoring may be useful to guide the management of CPP over time, particularly in comatose brain-injured patients at high risk for secondary cerebral ischemia and in whom clinical examination may not be reliable. Assessing the response or reactivity of PbtO2 to increases in CPP or mean arterial pressure (MAP) allows tailoring of an individual CPP threshold to avoid secondary cerebral ischemia, both in patients with poor grade SAH [13] and severe TBI [14], where optimal CPP can be found in up to 70% of patients. Also, use of the moving linear correlation coefficient between PbtO2 and CPP (known as the oxygen pressure reactivity index or ORx) allows identification of patients at particularly high risk of developing delayed cerebral ischemia and infarction after SAH [15]. As illustrated by the case presented here, this can facilitate individualized targeted therapy, such as hemodynamic augmentation, in realtime at the bedside [16]. It is important to note that although MAP and CPP may influence PbtO2, other physiologic variables can also affect PbtO2. These include the PaO2 [17], PaCO2 [18] and the systemic hemoglobin concentration [19,20]. As with ICP therapy, a stepwise management approach is also used for PbtO2 augmentation and this may involve increasing the MAP, adjusting respiratory rate or minute ventilation, and blood transfusions [21]. Given the large number of studies showing its safety and clinical utility, PbtO2 monitoring is a reasonable part of “routine” advanced multimodality neuromonitoring. Commercially available devices exist. It should be noted that significant differences in measured PbtO2 values can be observed when comparing the two main devices for routine monitoring (Licox®, Integra Neurosciences and Neurovent®, Raumedic), so these systems may not be used interchangeably. Although PbtO2 may often be considered as a qualitative surrogate for CBF, it is influenced by other parameters and does not provide a direct measurement of CBF [11]. Recent advances in technology allow for the direct measure of regional CBF (rCBF) via a Thermal Diffusion Probe (TDP Hemedex® Cambridge, Massachusetts) that can be inserted into the brain parenchyma. This technique is still not widely used clinically and therefore will not be discussed in detail here. A recent review on multimodality neuromonitoring provides more details [1]. Electroencephalography (EEG) is also emerging as a non-invasive tool to detect cerebral ischemia. The EEG has long been used to study the electrical activity of the cerebral cortex. In this standard approach, EEG shows brain electrical activity in the form of a line with repetitive wave-like activity. The classical indication of EEG is the detection of seizures and the prognostication of coma. A recent extensive review of the utility of EEG in the ICU has provides

www.medicalebookpdf.com

more details [22]. This review summarizes current recommendations of EEG for the purpose of detecting non-convulsive seizures in patients with an acute brain injury. All patients with protracted unexplained coma should undergo urgent EEG to rule out non-convulsive seizures. In the last decade, a more advanced form of EEG has been developed, known as quantitative EEG (qEEG), in which the raw EEG signal is converted to digital form using compressed spectral array. Using the analysis of the variability in α and δ power, it is possible to use qEEG for the prediction of delayed cerebral ischemia [23]. EEG-derived indices such as the alpha power or the alpha/delta ratio can be used to detect delayed cerebral ischemia in poor grade SAH and severe stroke patients. CLINICAL CASE 3 Monitoring of cerebral energy metabolism and supply in brain-injured patients. A 24 year old woman was admitted to the ICU because of severe TBI after a fall from a horse. She had a post-resuscitation GCS of 6 and bilaterally reactive pupils. The admission brain CT scan showed a small epidural hematoma and two small frontal contusions. Monitoring with ICP, PbtO2 and cerebral microdialysis (CMD) was initiated with probe placement in the right frontal lobe. Figure 10-4 shows the brain CT scan at 12 hours after admission. At this time, ICP and PbtO2 are within normal ranges, but CMD showed reduced brain glucose < 1 mmol/L (0.7 mmol/L). The arterial blood glucose concentration was 5.6 mmol/L (101 mg/dl) and the relationship between brain and blood glucose is shown in Figure 10-5. Enteral nutrition was rapidly instituted together with a slow infusion of hypertonic (10%) glucose. The blood glucose target was set at 7-8 mmol/L (126144 mg/dl) to avoid CMD glucose < 0.8 mmol/L; insulin infusion was withheld. At 48 hours, the brain glucose level progressively increased to > 1 mmol/L and the blood glucose target was set at > 6 mmol/L (108 mg/dl). Sedation was withdrawn and the patient developed elevated ICP (20-25 mmHg) with no decrease in PbtO2 (stable at 30 mmHg). Brain CT scan was repeated and showed no lesion progression and absence of cerebral edema. In this case, the elevated ICP was a consequence of agitation; given the absence of decreased PbtO2 and pathological brain CT scan signs, no aggressive treatment of elevated ICP was initiated. Progressive weaning of sedation coupled with clonidine and haloperidol to treat post-TBI agitation was undertaken and the patient was eventually extubated on day 5.

www.medicalebookpdf.com

Figure 10-4. Brain CT scan of a patient with severe TBI, showing the location of multimodal neuromonitoring (ICP, PbtO2, cerebral microdialysis) in the right frontal lobe.

This patient's case illustrates how multimodal neuromonitoring of cerebral metabolism can provide insight into the adequacy of energy supply after severe brain injury. This approach allowed intervention to avoid pathologically decreased cerebral glucose but also helped to avoid unnecessary intervention to treat a transiently elevated ICP that did not have negative impact on the cerebral metabolism. Cerebral microdialysis involves the insertion of a specialized catheter tipped with a semi-permeable dialysis membrane, usually with a 20 kDa pore size, in the brain parenchyma. The CMD catheter is constantly perfused with a cerebrospinal fluid-like solution, thereby allowing regular (usually every 60 min) sampling of patient's brain extracellular fluid into microvials and bedside analysis using an accompanying microanalyzer [24]. CMD technology allows near real time monitoring of dynamic changes in a patient's neurochemistry. The most commonly used CMD neurochemical markers for the management of secondary cerebral damage are glucose, the lactate/pyruvate ratio (LPR), and glutamate. Thresholds of abnormalities are CMD glucose < 0.7-1 mmol/L and LPR > 35-40. Increased glycolysis and glucose utilization are frequently observed in

www.medicalebookpdf.com

patients with severe acute brain injuries [25], potentially leading to reduced availability of the brain's main energy substrate, glucose [26]. Combined monitoring of CMD and blood glucose is particularly helpful for the management of insulin infusions in neurocritical care and allows individualization of optimal glucose targets [27,28]. Increase in LPR and glutamate have been used as a warning sign of a shift from aerobic to anaerobic metabolism and delayed cerebral ischemia in patients with poor grade SAH [29,30]. The CMD technology can be used in combination with PbtO2 monitoring for the detection of delayed ischemia and to target blood pressure and transfusion requirements in patients with SAH [20,31,32]. In patients with TBI, elevated LPR was associated with poor neurological recovery in a large cohort study [33].

Figure 10-5. Continuous monitoring of arterial blood and extracellular brain (using cerebral microdialysis) glucose at the bedside. An adequate supply of the main energy substrate (glucose) is essential for the injured brain. Cerebral microdialysis (CMD) monitoring can help to optimize glucose control and insulin therapy in patients with acute brain injury, thereby avoiding unwanted critical reductions of cerebral glucose. Here, infusion of 10% IV glucose allowed normalization of CMD glucose (ICP and PbtO2 remained in normal ranges) and prevention of sustained neuroglucopenia and cerebral metabolic distress.

Although these studies show considerable advances in terms of feasibility, implementation of CMD still requires significant time, effort, and cost. CMD

www.medicalebookpdf.com

catheters can be inserted together with ICP and PbtO2 probes using a triplelumen bolt. This option has some advantages, including reduced risk of catheter displacement. Alternatively, using the tunneling technique, the CMD catheter can be placed in selected areas, which may be of particular value when therapy is aimed to attenuating secondary insults around tissue at risk. Catheter displacement or injury to the catheter membrane, however, is more frequent. DOES MONITORING IMPROVE OUTCOME? By definition, a monitoring technique is not a therapeutic intervention and without effective interventions the natural course of an illness cannot be modified. Consequently, to realize a benefit from monitoring, appropriate therapy must derive from the information acquired from the monitoring itself. Stein [6] reviewed four decades of clinical trials and case series in which patients were treated for severe closed TBI. Aggressive ICP monitoring and treatment of patients with severe TBI was associated with a statistically significant improvement in outcome. However, Chesnut and colleagues in a recent randomized multicenter trial showed that ICP monitoring did not change the outcome of patients with severe TBI [7]. Although PbtO2-directed therapy in some historical-control single-center studies has been associated with better outcome [34], the issue remains controversial and no randomized studies have been confirmed this association so far. As demonstrated by past trials performed in the general ICU setting, monitoring per se may be insufficient to change patient prognosis substantially [35]. The same is likely to apply to brain multimodality monitoring. Monitoring does not mean effective therapy. Rather, monitoring, if appropriately interpreted and assisted by clinical experience, may help ICU physicians to provide adequate interventions and timely therapy. Recent clinical investigations by several independent groups show feasibility and utility of brain multimodality monitoring. The appropriate interpretation of brain physiological variables and the worldwide implementation of standardized management protocols driven by multimodal monitoring might offer optimal individualized therapy to brain-injured patients and may further improve overall prognosis and quality of care. REFERENCES 1. Oddo M, Villa F, Citerio G. Brain multimodality monitoring: an update. Curr Opin Crit Care. 2012;18:111-8.

www.medicalebookpdf.com

2. Bouzat P, Sala N, Payen JF, Oddo M. Beyond intracranial pressure: optimization of cerebral blood flow, oxygen, and substrate delivery after traumatic brain injury. Ann Intensive Care. 2013;3:23. 3. Bratton SL, Chestnut RM, Ghajar J, et al. Guidelines for the management of severe traumatic brain injury. VI. Indications for intracranial pressure monitoring. J Neurotrauma. 2007;24 Suppl 1:S37-44. 4. Bratton SL, Chestnut RM, Ghajar J, et al. Guidelines for the management of severe traumatic brain injury. X. Brain oxygen monitoring and thresholds. J Neurotrauma. 2007;24 Suppl 1:S6570. 5. Czosnyka M, Pickard JD. Monitoring and interpretation of intracranial pressure. J Neurol Neurosurg Psychiatry. 2004;75:813-21. 6. Stein SC, Georgoff P, Meghan S, Mirza KL, El Falaky OM. Relationship of aggressive monitoring and treatment to improved outcomes in severe traumatic brain injury. J Neurosurg. 2010;112:1105-12. 7. Chesnut RM, Temkin N, Carney N, et al. A trial of intracranial-pressure monitoring in traumatic brain injury. N Engl J Med. 2012;367:2471-81. 8. Li LM, Timofeev I, Czosnyka M, Hutchinson PJ. Review article: the surgical approach to the management of increased intracranial pressure after traumatic brain injury. Anesth Analg. 2010;111:736-48. 9. Schreckinger M, Marion DW. Contemporary management of traumatic intracranial hypertension: is there a role for therapeutic hypothermia? Neurocritical care. 2009;11:427-36. 10. Treggiari MM, Schutz N, Yanez ND, Romand JA. Role of intracranial pressure values and patterns in predicting outcome in traumatic brain injury: a systematic review. Neurocritical care. 2007;6:104-12. 11. Rosenthal G, Hemphill JC, 3rd, Sorani M, et al. Brain tissue oxygen tension is more indicative of oxygen diffusion than oxygen delivery and metabolism in patients with traumatic brain injury. Crit Care Med. 2008;36:1917-24. 12. Oddo M, Levine JM, Mackenzie L, et al. Brain hypoxia is associated with short-term outcome after severe traumatic brain injury independent of intracranial hypertension and low cerebral perfusion pressure. Neurosurgery. 2011. 13. Jaeger M, Schuhmann MU, Soehle M, Nagel C, Meixensberger J.

www.medicalebookpdf.com

Continuous monitoring of cerebrovascular autoregulation after subarachnoid hemorrhage by brain tissue oxygen pressure reactivity and its relation to delayed cerebral infarction. Stroke. 2007;38:981-6. 14. Jaeger M, Dengl M, Meixensberger J, Schuhmann MU. Effects of cerebrovascular pressure reactivity-guided optimization of cerebral perfusion pressure on brain tissue oxygenation after traumatic brain injury. Critical care medicine. 2010;38:1343-7. 15. Jaeger M, Soehle M, Schuhmann MU, Meixensberger J. Clinical significance of impaired cerebrovascu-lar autoregulation after severe aneurysmal subarachnoid hemorrhage. Stroke. 2012;43:2097-101. 16. Muench E, Horn P, Bauhuf C, et al. Effects of hypervolemia and hypertension on regional cerebral blood flow, intracranial pressure, and brain tissue oxygenation after subarachnoid hemorrhage. Crit Care Med. 2007;35:1844-51; quiz 52. 17. Oddo M, Nduom E, Frangos S, et al. Acute lung injury is an independent risk factor for brain hypoxia after severe traumatic brain injury. Neurosurgery. 2010;67:338-44. 18. Rangel-Castilla L, Lara LR, Gopinath S, Swank PR, Valadka A, Robertson C. Cerebral hemodynamic effects of acute hyperoxia and hyperventilation after severe traumatic brain injury. J Neurotrauma. 2010;27:1853-63. 19. Oddo M, Levine JM, Kumar M, et al. Anemia and brain oxygen after severe traumatic brain injury. Intensive Care Med. 2012;38:1497-504. 20. Oddo M, Milby A, Chen I, et al. Hemoglobin Concentration and Cerebral Metabolism in Patients With Aneurysmal Subarachnoid Hemorrhage. Stroke. 2009. 21. Bohman LE, Heuer GG, Macyszyn L, et al. Medical management of compromised brain oxygen in patients with severe traumatic brain injury. Neurocritical care. 2011;14:361-9. 22. Claassen J, Taccone FS, Horn P, Holtkamp M, Stocchetti N, Oddo M. Recommendations on the use of EEG monitoring in critically ill patients: consensus statement from the neurointensive care section of the ESICM. Intensive Care Med. 2013;39:1337-51. 23. Foreman B, Claassen J. Quantitative EEG for the detection of brain ischemia. Crit Care. 2012;16:216.

www.medicalebookpdf.com

24. Hillered L, Vespa PM, Hovda DA. Translational neurochemical research in acute human brain injury: the current status and potential future for cerebral microdialysis. J Neurotrauma. 2005;22:3-41. 25. Glenn TC, Kelly DF, Boscardin WJ, et al. Energy dysfunction as a predictor of outcome after moderate or severe head injury: indices of oxygen, glucose, and lactate metabolism. J Cereb Blood Flow Metab. 2003;23:123950. 26. Vespa PM, McArthur D, O'Phelan K, et al. Persistently low extracellular glucose correlates with poor outcome 6 months after human traumatic brain injury despite a lack of increased lactate: a microdialysis study. J Cereb Blood Flow Metab. 2003;23:865-77. 27. Oddo M, Schmidt JM, Carrera E, et al. Impact of tight glycemic control on cerebral glucose metabolism after severe brain injury: a microdialysis study. Crit Care Med. 2008;36:3233-8. 28. Vespa P, McArthur DL, Stein N, et al. Tight glycemic control increases metabolic distress in traumatic brain injury: a randomized controlled within-subjects trial. Crit Care Med. 2012;40:1923-9. 29. Sarrafzadeh A, Haux D, Sakowitz O, et al. Acute focal neurological deficits in aneurysmal sub-arachnoid hemorrhage: relation of clinical course, CT findings, and metabolite abnormalities monitored with bedside microdialysis. Stroke. 2003;34:1382-8. 30. Sarrafzadeh AS, Haux D, Ludemann L, et al. Cerebral ischemia in aneurysmal subarachnoid hemorrhage: a correlative microdialysis-PET study. Stroke. 2004;35:638-43. 31. Ko SB, Choi HA, Parikh G, et al. Multimodality monitoring for cerebral perfusion pressure optimization in comatose patients with intracerebral hemorrhage. Stroke. 2011;42:3087-92. 32. Schmidt JM, Ko SB, Helbok R, et al. Cerebral perfusion pressure thresholds for brain tissue hypoxia and metabolic crisis after poor-grade subarachnoid hemorrhage. Stroke. 2011;42:1351-6. 33. Timofeev I, Carpenter KL, Nortje J, et al. Cerebral extracellular chemistry and outcome following traumatic brain injury: a microdialysis study of 223 patients. Brain. 2011;134:484-94. 34. Nangunoori R, Maloney-Wilensky E, Stiefel M, et al. Brain tissue oxygen-

www.medicalebookpdf.com

based therapy and outcome after severe traumatic brain injury: a systematic literature review. Neurocritical care. 2012;17:131-8. 35. Ospina-Tascon GA, Cordioli RL, Vincent JL. What type of monitoring has been shown to improve outcomes in acutely ill patients? Intensive Care Med. 2008;34:800-20.

www.medicalebookpdf.com

MULTIMODALITY NEUROMONITORING QUESTIONS 1. The recent BEST TRIP randomized controlled trial from Chesnut and colleagues compared clinical examination plus repeated brain CT scan with or without ICP monitoring in patients with severe TBI (New England Journal of Medicine, Dec 2012). Which one of the following sentence about this study is true? a. ICP monitoring was effective in improving outcome b. ICP monitoring was associated with a worse outcome c. ICP monitoring was associated with reduced number of days in the ICU d. ICP monitoring was effective in guiding therapy of intracranial hypertension e. ICP monitoring had no benefit 2. The ICP curve of a young adult patient at day 2 following severe TBI is depicted, showing elevated ICP up to 30 mmHg. Which of the following sentences is wrong?

a. The ICP threshold alone should be used to guide ICP therapy b. P2 > P1 is a sign of poor intracranial compliance c. P1 > P2 is normally seen and is a sign of normal brain compliance d. The ICP curve is useful to diagnose brain compliance and identify patients at higher risk of intracranial hypertension that may benefit form aggressive ICP therapy e. P2 reflects cerebral venous return 3. What are the main physiologic determinants of brain tissue oxygen tension

www.medicalebookpdf.com

(PbtO2)? (More than one answer may apply). a. PaO2 b. Temperature c. MAP d. ICP e. CPP 4. What is the potential clinical utility of PbtO2 monitoring? (More than one answer may apply). a. Management of CPP b. Management of elevated ICP c. Management of mechanical ventilation d. Management of blood transfusion e. Management of sedation 5. Which one is not an indication for EEG monitoring? a. Detection of cerebral ischemia b. Detection of non-convulsive seizures c. Detection of elevated ICP d. Prognostication of coma e. Management of barbiturate coma 6. Regarding ICP monitoring, which of the following sentences is true? a. Non-invasive tools (optic sound ultrasound, transcranial Doppler ultrasound) can be used to monitor ICP b. ICP monitoring improves outcome in patients with severe TBI c. ICP monitoring reduces the length of ICU stay d. Based on randomized controlled trials, ICP monitoring is effective in guiding the treatment of intracranial hypertension

www.medicalebookpdf.com

e. ICP monitoring should be abandoned 7. What is the oxygen pressure reactivity index (ORx)? a. On-line linear correlation coefficient between ICP and PbtO2 b. On-line linear correlation coefficient between ICP and MAP c. An index that quantifies the response of PbtO2 to CPP d. An index that quantifies the response of PbtO2 to FiO2 e. On-line linear correlation coefficient between CPP and PbtO2 8. The alpha/delta ratio on the EEG can be used to detect: a. The depth of sedation b. Cellular hypoxia c. Burst-suppression d. Delayed cerebral ischemia e. Elevated ICP 9. Based on the 2007 Brain Trauma Foundation guidelines, which of the following is not an indication for ICP monitoring? a. Moderate TBI b. Severe TBI with an abnormal brain CT scan c. Severe TBI with polytrauma and ARDS d. Severe TBI with compressed basal cisterns on brain CT scan 10. What sentence about multimodal neuromonitoring is true ? a. It has been shown to improve outcome in randomized controlled trials b. It has been shown to improve outcome in retrospective studies c. It is still a research tool d. Its feasibility and safety has not been demonstrated yet e. The appropriate interpretation of brain physiological variables and the

www.medicalebookpdf.com

use of standardized management protocols driven by multimodal neuromonitoring might offer optimal individualized therapy to braininjured patients

www.medicalebookpdf.com

MULTIMODALITY NEUROMONITORING ANSWERS 1. The correct answer is D. Compared to the control group (repeated clinical examination + brain CT without ICP monitoring) the intervention group (clinical examination + CT with ICP monitoring) had comparable outcome (lack of efficacy in improving outcome) and spent more days in the ICU. However the trial showed that ICP monitoring reduced by 50% the number of treatments for ICP per patient and reduced the number of ICU days during which patients received brain-specific treatments. Therefore, ICP monitoring was effective in guiding therapy of intra-cranial hypertension. 2. The correct answer is A. Both the ICP threshold (generally > 20 mmHg) and the shape of the ICP waveform are important in determining optimal treatment of intracranial hypertension. Increased P2 (equal to or above P1) suggests poor cerebral compliance and thus should prompt aggressive ICP therapy. On the other hand, ICP values at 20-25 mmHg with an ICP waveform showing good compliance may not necessarily need treatment escalation. 3. The correct answers are A, B, C, D, E. PbtO2 can be expressed by the formula: CBF x (PaO2-PvO2). PaO2 is a major determinant of PbtO2, as well as all variables that may influence CBF (e.g. MAP, ICP, CPP). PbtO2 has to be adapted to temperature, ideally brain temperature. 4. The correct answers are A, B, C, D, E. Several independent single-center clinical studies have shown that PbtO2 monitoring might help in the management of all previously listed interventions. 5. The correct answer is C. Clinical studies have shown a potential utility of EEG monitoring for the detection of delayed cerebral ischemia after SAH, to detect non-convulsive seizures, to improve coma prognostication and to titrate barbiturate coma. 6. The correct answer is D. Non-invasive tools can only estimate ICP, but invasive ICP monitoring remains the only way to measure ICP. In the Chesnut trial (NEJM 2012), compared to the control group (repeated clinical examination + brain CT without ICP monitoring) the intervention group (clinical examination + CT with ICP monitoring) had comparable outcome (lack of efficacy in improving outcome) and spent more days in the ICU. However the trial showed that ICP monitoring reduced by 50% the number of treatments for ICP per patient and reduced the number of ICU

www.medicalebookpdf.com

days during which patients received brain-specific treatments. Therefore, ICP monitoring was effective in guiding therapy of intracranial hypertension. 7. The correct answer is E. The oxygen pressure reactivity index measures the linear correlation between PbtO2 and allows the assessment of cerebral autoregulation and of optimal CPP. 8. The correct answer is D. The alpha-delta ratio has been shown in some studies to predict ischemia in severe SAH patients. 9. The correct answer is A. ICP monitoring is not recommended in patients with moderate TBI. 10. The correct answer is E. Multimodal neuromonitoring might help ICU clinicians with the management of secondary cerebral damage. Incorporation of multimodal neuromonitoring into standardized algorithms is essential.

www.medicalebookpdf.com

Chapter 11

PERIOPERATIVE NEUROSURGICAL CRITICAL CARE Chris Zacko and Peter Le Roux CLINICAL CASE A 73 year old male with poorly controlled diabetes, coronary artery disease with cardiac stents, atrial fibrillation (non-therapeutic on warfarin), bilateral carotid stenosis, hypertension, hyperlipidemia, hypothyroidism and morbid obesity has just returned to your ICU after a decompressive craniectomy for malignant middle cerebral artery infarction. He is on a ventilator and muscle relaxation was not reversed by the anesthesia team. His serum sodium level is 132 mmol/L, blood pressure is 90/45 mmHg, glucose is 245 mg/dL, and he lost 750 cc of blood during surgery. What post-operative challenges do you face when balancing this patient's medical co-morbidities, fresh surgical wounds, and need for cerebral perfusion? OVERVIEW All patients who undergo neurosurgical procedures, even a well-performed operation, are in a potentially unstable cardiopulmonary state and at risk for secondary neuronal injury. Depending on the specific operation performed these patients can also have fresh surgical incisions, delicate vascular anastomoses, friable resection beds, brittle patency of newly open vessels, and tenuous hemostasis. All of these factors leave these patients especially vulnerable to post-operative complications. The object of postoperative neurosurgical care is to resuscitate, stabilize, prevent/minimize secondary neuronal damage, and optimize functional brain recovery. Therefore, the basic goals of postoperative neurosurgical care are: Provide smooth emergence from anesthesia Optimize post-operative hemodynamic, volume, and electrolyte status, Optimize airway and respiratory status, Treat coagulopathic states and hemostatic disorders, Optimize management of post-operative complications,

www.medicalebookpdf.com

Implement reliable and appropriate systemic and neuromonitoring tools Establish consistent and reproducible neurological examination methods These goals depend on many variables and important questions to ask and answer include: What was the status (medical and neurological) of the patient before surgery? What neurological disease is being treated? What other neurological disorders does the patient have? What the position of the patient during surgery? What procedure was performed (procedure specific and expected complications)? What happened during surgery, e.g. blood loss, vascular injury? What anesthetic technique was used? To manage a postoperative neurosurgical patient the neurointensivist requires knowledge of how the central nervous system (CNS) reacts to stress and anesthesia as well as the potential complications associated with each specific procedure. This chapter will focus on the following topics: 1. Who goes to and who stays in the Neurocritical Care Unit (NCCU) 2. The effects of anesthetic agents on the CNS and in neurosurgical patients 3. Basic complications after neurosurgical procedures 4. Emergence from anesthesia in neurosurgical patients 5. Extubation in neurosurgical patients 6. Post-operative pain 7. Postoperative nausea and vomiting 8. Basic post-operative neurosurgical care 9. Postoperative monitoring WHO GOES TO AND WHO STAYS IN THE NCCU?

www.medicalebookpdf.com

Postoperative neurosurgical cases account for numerous NCCU admissions. Traditionally, patients who had craniotomies and other invasive neurosurgical procedures were nursed postoperatively and overnight in the NCCU as a precaution. This practice is changing somewhat as most NCCUs also have welldeveloped “step-down” or intermediate care units that can often manage routine uncomplicated cases. Among those patients admitted to the NCCU primarily for precautionary/observational purposes, very few “patient days” are created. It is estimated that only ~15% actually require/receive active treatment. Furthermore, when a patient stay in the NCCU is < 24 hours, ~50% require no interventions beyond post-anesthetic care and frequent neurologic exams. Lastly, two-thirds of these patients require no further interventions of any kind after the first 4 hrs. The decision to admit a given patient to the NCCU can be subjective and surgeon specific but is often based upon: age, co-morbidities, pre-operative condition, intra-operative details (difficult hemostasis, unexpected cerebral edema), and need for post-operative respiratory and hemodynamic support. That said, a patient's risk for prolonged ICU stay (>1 day) can be anticipated by: a) preoperative radiologic findings (e.g. tumor location, mass effect), b) significant intraoperative blood loss, c) substantial intra-operative fluid requirements, and d) the decision to keep the patient intubated at the end of surgery [1]. EFFECT OF ANESTHETIC AGENTS ON THE CNS AND IN NEUROSURGICAL PATIENTS An important aspect of postoperative neurosurgical care is to distinguish residual effects of anesthetic agents (e.g. drowsiness or confusion) from signs that indicate intracranial pathology. While it often is believed that patients with neurological disease are prone to anesthetic effects, this is not universally true – particularly in those patients who are fully awake preoperatively. Confusion or dementia can undoubtedly be exacerbated by anesthetic agents. That said, some note it is a bit more unusual (but not unheard of) for focal deficits to be aggravated by anesthesia. Excessive benzodiazepine use can be an exception and in some situations may seem to unmask deficits. As a general rule, however, any progressive or fluctuating deterioration should be assumed to be a complication from the operative procedure rather than an anesthetic effect. The effects of anesthetic agents are complex and can depend on the agent used. For example, increased reflexes and extensor plantar responses may be observed in 50-60% of patients who receive enflurane or ethrane, in 40%). Being aware of some common complications and associated management strategies is fundamental to the practice of neurocritical care [33]. Craniotomy Complications can be general or specific to the type of surgery. The most common complications are cerebral edema, seizures, vascular injury, and postoperative hemorrhage.

www.medicalebookpdf.com

General complications after craniotomy: a. Agitation and discomfort are common and now commonly treated quite successfully with dexmedetomidine. b. Cerebral Infarction: Can be due to arterial or venous injury. Venous occlusion and infarct can occur when a bleeding vein must be coagulated or when massive cerebral edema leads to a compressive occlusion of venous outflow. One should also be attentive to possible venous injuries after meningioma surgery located near venous sinuses (tentorial, parasagittal, convexity, and parafalcine). Arterial infarct can occur with either traumatic laceration or sacrifice of an artery for hemostasis. This can occur in traumatic brain injury (TBI), tumor surgery with en passant vessels, and epilepsy surgery (i.e. anterior choroidal artery in temporal lobectomy). c. Seizures: particularly after penetrating TBI, epilepsy surgery, subdural empyema, and glial resection near motor cortex, but may occur in any patient post-operatively d. Pneumocephalus: air can be retained after craniotomy and act very much like mass lesions. Symptoms include lethargy, confusion, nausea/vomiting, and headache. Diagnosis is easily made with head CT. Once suspected one should actively investigate for the presence of tension pneumocephalus and cerebrospinal fluid (CSF) fistula as this will dictate management. Tension pneumocephalus can be urgent and is surgically evacuated. If there is a CSF leak, the leak should be managed in typical fashion before the pneumocephalus is addressed. Simple pneumocephalus can typically be managed expectedly as the air normally absorbs with time. Some advocate for the use of non-rebreathing mask with 100% oxygen for 24-48 hours. Brain sagging may be encountered as a related phenomenon – often seen after intraoperative over-drainage of CSF (e.g. during aneurysm surgery). The clinical triad consists of pneumocephalus, midbrain crowding, and neurological symptom, such as decreased mental status improving with reverse Trendelenburg e. Postoperative hematomas: Approximately 2% of patients who undergo a cranial procedure will develop a postoperative hematoma (PICH) with 0.8% of patients developing a hemorrhage that requires surgical evacuation. The most common PICH presentations include: 60% present with a decreased level of consciousness (as a result PICH

www.medicalebookpdf.com

should be considered in all patients who do not recover or improve in the expected manner after surgery). 33% of patients develop focal neurological deficits 90% will have elevated intracranial pressure (ICP) when ICP is being monitored. By contrast in the absence of a PICH, ICP is elevated in only 10% of post-operative patients [7]. In most patients (50%) clinical deterioration associated with a postoperative hematoma occurs within 6 hours of surgery [7,8]. However, ~20% of PICH may develop after 24 hours. Those patients at particular risk for delayed hematomas are those who underwent posterior fossa surgery or emergency craniotomy. Once should consider longer periods of ICU observation in such cases. Risk factors for a PICH, particularly one that requires surgery include: meningioma surgery; intraoperative or immediate (12 hour) postoperative hypertension [9], intraoperative blood loss >500ml, age >70 years, hypoxia, coughing and hiccoughs, and laboratory signs of coagulopathy (high prothrombin time, low fibrinogen and platelet counts). Remote hemorrhages from the surgical site can also be problematic. Etiologies/risk factors include reperfusion hemorrhages, releasing the tamponade effect of a contralateral hemorrhage with debulking of a mass lesion, CSF drainage/hyperosmolar therapy causing shift of parenchyma (especially in cerebellum), and coagulopathic states (including patients with history of alcohol abuse). Reoperation: Reoperation is necessary in some patients. Removal of various types of hematomas is the most common surgical procedure at reoperation. Outcome is favorable in only about half the patients indicating the importance of prevention. Factors associated with poor outcome include: histological type of the tumor, clinical state at admission, lower GCS score before urgent reoperation, time interval between primary surgery and urgent reoperation, and patient age [10]. Specific complications after craniotomy: Glioma: cerebral edema is more common after partial resection than with gross total resection.

www.medicalebookpdf.com

Epilepsy: hemiplegia can be seen if the anterior choroidal artery was injured. Word finding difficulties can be seen with injury to the left temporal lobe. Lastly, aseptic meningitis after depth electrode placement is a concern [11]. Pituitary and Transphenoidal Surgery: Complications to be aware of are hyponatremia and diabetes insipidus (DI), neuroendocrine disorders from panhypopituitarism (adrenal insufficiency, central hypothyroidism), CSF leaks, sinonasal injuries, and alterations in visual function (acuity, fields, ocular movement). Posterior Fossa Surgery: air embolism is a classical, yet uncommon, complication of surgery in the seated position. It is typically diagnosed and managed in the operation room by flooding the field with irrigation, applying bone wax to cut bone surfaces, lowering the head of bed, placing the patient in left lateral decubitus position (if possible), aspirating air from the left atrium via central venous catheter, and achieving hemostasis as soon as possible. Another complication associated with posterior fossa surgery is accelerated hypertension. Any unexpected or refractory hypertension should warrant careful examination and a low threshold for imaging looking for post-operative hemorrhage. Other complications include obstructive hydrocephalus (if 4th ventricle compressed), upward herniation (from over-drainage through an external ventricular drain [EVD] when the 4th ventricle is compressed), cranial nerve injuries, and CSF leak and/or pseudomeningocele due to dependency of dural opening. Craniotomy for securing ruptured aneurysm: the risk, diagnosis, and management of delayed cerebral ischemia from vasospasm is covered in another chapter in this book (see chapter dedicated to Subarachnoid Hemorrhage). Arteriovenous Malformation (AVM): these cases generally have especially tenuous and brittle hemostasis and very strict blood pressure control is crucial. In addition, liberal use of sedation to prevent coughing and straining against the ventilator may be critically important in the first few days after surgery. Seizures are also known to occur after AVM surgery as these patients often have pre-operative seizures. Transient mutism can occur with bilateral retraction of the cingulate gyrus or division of the corpus callosum [12]. Carotid Endarterectomy

www.medicalebookpdf.com

The need for ICU management after carotid endarterectomy (CEA) is highly variable and is dependent on surgeon preference, anesthetic/surgical technique (local vs. general), pre-operative clinical presentation and the patient's comorbidities. Medical complications to be mindful of are: a) cardiac arrhythmias, b) myocardial ischemia (cardiac complications may be more common if the procedure is done under general anesthesia), c) respiratory compromise (either from soft-tissue swelling after dissection or post-operative hematoma), d) seizures, e) hypotension or hypertension (deranged sensitivity of carotid sinus baroreceptor reflex or injury to the Hering nerve, a branch of cranial nerve IX), and f) bradycardia (also from carotid baroreceptor injury). Surgical and neurologic complications include: a. Stroke: This complication can be: a) embolic from the endarterectomized surface, b) hemorrhagic from reperfusion injury (see below), or c) occlusive from re-stenosis of the ICA (most common cause of major stroke). If the stroke is noted directly after surgery, the patient is typically taken immediately back to surgery for exploration without further imaging. If the deficit is delayed, a workup is initiated and management options vary depending on the findings. Options include surgery, anticoagulation, augmentation of cerebral perfusion via blood pressure elevation, and observation with generous fluid administration and serial examinations/imaging. b. Post-operative hematoma: sources can be from venous bleeding in the operative bed or from disruption of arterial suture line. The latter is an emergency situation. If suspected, the surgeon and a dedicated team to manage the airway should immediately be called. The patient is assessed for pulsatile swelling, tracheal deviation and respiratory distress. If there is any hint of respiratory compromise this is managed by OPENING THE WOUND AND THEN INTUBATION. If not already in the operating room, the patient can then go to the operating room for definitive repair. c. Cranial nerve injury: cited by some as the most common complication after CEA with an incidence of 8-10% [13]. Nerves at risk are the a) hypoglossal (tongue deviation, chewing swallowing deficits); b) vagus (hoarseness, diminished cough); c) glossopharyngeal nerve (dysphagia, nasal regurgitation, hypertension if Hering nerve damaged); d) spinal accessory nerve (drooping shoulder), e) recurrent laryngeal (unilateral vocal cord paralysis), great auricular nerve, and mandibular branch of the facial nerve (asymmetry of upper lip). These injuries are generally self-limited and

www.medicalebookpdf.com

recover in time. d. Hoarseness: more likely from laryngeal edema than injury to recurrent laryngeal nerve e. Reperfusion Injury: a relatively rare occurrence that is possibly more prevalent after re-opening of a high-grade stenosis in hypertensive patients – especially if there is contra-lateral carotid occlusion. This abrupt reestablishment of flow into an area that is postulated to have impaired autoregulation can lead to edema and hemorrhage (from microhemorrhages and to large hematomas). Symptoms include altered mental status, ipsilateral eye pain or headache [14]. This complication can also cause seizures. Endovascular Interventions This is an area of tremendous advancement in recent years for the management of stroke and various vascular anomalies. Many of the complications are inherent to the disorder being treated and are discussed elsewhere in this book. The complications specific to endovascular procedures will be discussed below and are grouped into vascular access site and neurologic complications. Vascular access site complications Due to the thrombogenic effect of the catheters used in these procedures, as well as the thromboembolic potential inherent to the conditions being treated, anti-coagulation is used much more readily in endovascular cases than in open procedures. This can predispose the patient to hemorrhagic complications. Complications seen at the groin puncture site include arteriovenous fistulas, local bleeding, pseudoaneurysm, and local nerve injury. The latter can be emergent and require immediate attention. Any hypotension in a patient having undergone an endovascular procedure should warrant careful inspection for signs of pseudoaneurysm. This includes checking distal pulses, assessing for pallor in the extremity, and feeling for a palpable pulsatile mass at the access site. Pseudoaneurysm can lead to distal ischemia in the leg and be limb-threatening. Pseudoaneurysms can be well visualized with ultrasound done at the bedside. Confirmation of a pseudoaneurysm triggers application of prolonged pressure to the site and consultation of a vascular surgeon. Most severe and actively bleeding cases demand surgical intervention, but less serious cases can be treated with local thrombin injection. Neurologic Complications Focal cerebral edema can occur after coiling of aneurysms and embolization

www.medicalebookpdf.com

of AVMs. This is typically managed with dexamethasone. Intracranial hemorrhage can be seen after these procedures and management is discussed elsewhere in this chapter. Common sources of ICH include intra-operative rupture of an aneurysm, perforation of a vessel, and reperfusion hemorrhage. More unique to endovascular procedures are possible thromboembolic and thrombo-occlusive complications. The catheters used for these procedures can physically disrupt atherosclerotic plaques causing emboli, create a dissection, or generate thrombus due to their inherent thrombogenicity. Other sources of emboli include microthrombotic shower after mechanical thrombectomy and glue emboli after embolization. These emboli are often not noted until the postoperative period. Once symptoms are suspected, urgent MRI/MRA is advised with concomitant initiation of generous fluid administration. After diagnostic imaging is complete, treatment options include additional endovascular therapy, anticoagulation, and/or hypertensive therapy with vasopressors. Other complications of these procedures include arterial dissection, acute thrombus and perforation. These complications are almost always noted during the procedure and management options consist of blood pressure control, anticoagulation, and generous fluid administration (depending on the problem encountered). For acute thrombus noted during the procedure, treatment options include intra-arterial thrombolysis, mechanical thrombectomy, and administration of abciximab. Abciximab is a potent glycoprotein IIb/IIIA inhibitor that has been found to be quite effective in managing thromboocclusive events during endovascular procedures. Due to the potent anti-platelet activity of abciximab it is necessary to carefully monitor for bleeding complications. Other complications It is important to recognize what may happen during surgery to best manage the patient after surgery. Complications depend in part on position or the procedure. Some specific examples include: 1. Ocular: Periorbital and/or conjunctival edema, as well as chemosis, tend to occur more often in the prone position, during pterional approaches, or with orbitozygomatic craniotomies. Posterior ischemic optic neuropathy or central retinal artery occlusion also may occur (particularly with longer procedures). A third nerve palsy or blindness may result from posterior communicating artery or carotid ophthalmic artery surgery. 2. Use of a lumbar drain may cause intracranial hypotension or remote hemorrhage distant to the surgical site

www.medicalebookpdf.com

3. Anterior cervical surgery: Soft tissue swelling can cause airway obstruction or swallowing abnormalities EMERGENCE FROM ANESTHESIA IN NEUROSURGICAL PATIENTS Recovery from anesthesia and surgery is a period of intense stress for patients. The effects can be systemic or CNS-specific. Systemic effects There are several physiological responses as a patient emerges from anesthesia, including: an increase in oxygen consumption (VO2), sympathetic activation with catecholamine release, increases in blood pressure and/or heart rate, alterations in arterial blood gases, and hyperglycemia. Shivering, pain, and regaining awareness are additional stress factors during recovery from anesthesia. 1) Shivering occurs in approximately 40% of patients recovering from general anesthesia with a body temperature of 20% blood pressure increase is considered a reasonable threshold for treatment, then 40-90% of patients require antihypertensive therapy during emergence. Analgesics, and particularly narcotics, reduce the sympathetic and catecholamine response to pain and extubation. Patients with PICH are 3.6 times more likely to be hypertensive than their matched controls. In particular there is a very strong association between intracranial hemorrhage and patients being normotensive intra-operatively but hypertensive postoperatively [9]. Hypertension after posterior fossa surgery should raise suspicions for possible post-operative hemorrhage. Postoperative blood pressure generally is managed to maintain systolic pressures in the range of 120–150 mmHg. Cerebral effects Stressful events, including surgery and emergence from anesthesia, can alter cerebral blood flow (CBF) and cerebral metabolic rate for oxygen (CMRO2).

www.medicalebookpdf.com

Sympathetic stimulation acting through beta-adreno-receptors may play a role in these effects. 1) CBF: Transcranial Doppler (TCD) studies suggest that CBF velocities increase significantly during emergence from anesthesia. The maximum increase is at extubation (as much as 60% over preoperative value) and return to normal in about 60 minutes [16]. The CBF increase is independent of anesthetic technique, PaCO2, or arterial pressure. This increase in CBF can cause cerebral edema, hemorrhage, and postoperative confusion. Changes in CBF are associated with deleterious effects on oxygen consumption (VO2) so prevention of agitation, shivering and coughing is important. 2) ICP: Up to 20% of patients who undergo intracranial surgery may develop increased ICP and when this occurs half will develop clinical deterioration in large part from edema or hemorrhage [17]. There is limited data on the specific effects of emergence and extubation on ICP. Endotracheal suctioning has been shown to increase ICP [18]. Similarly, extubation can increase ICP – particularly when associated with coughing. The ICP increase usually lasts 2 or 3 minutes, but is longer when intracranial compliance is reduced. 3) Hyperemia and normal perfusion pressure breakthrough: The cerebral arteriovenous oxygen content difference (AVDO2) often is depressed immediately after craniotomy. This is suggestive of transient cerebral hyperemia (16). Hyperemia may result in hemorrhage or severe edema in 3–12.5% of cases. Patients at especially high risk are those undergoing craniotomy for AVM resection. AVM features associated with a high-risk for postoperative hyperemic complications, including “normal perfusion pressure breakthrough”, are: a) large and deep AVMs, b) low feeding-artery pressures, c) multiple arterial inflows but only a single venous draining vessel, and d) intense steal around the AVM nidus. Strategies used in the management of these precarious include staged therapies (embolization and surgical), barbiturate-based anesthetic regimen continued into the postoperative period, extremely rigorous blood pressure control after surgery, and either invasive or non-invasive cardiovascular monitoring to optimize filling pressure and cardiac performance EXTUBATION IN NEUROSURGICAL PATIENTS The goal of anesthetic emergence and subsequent extubation is to maintain stable respiratory and cardiovascular parameters while preventing adverse CNS effects. One must be cautious as even the physical act of extubation can cause sympathetic discharge via tracheal and laryngeal stimulation (although it relieves the endotracheal tube stimulation itself). On one hand, a delayed emergence with

www.medicalebookpdf.com

deferred extubation in the ICU may achieve better thermal and cardiovascular stability after major neurosurgical procedures (thereby limiting secondary insults). On the other hand, the timely diagnosis of neurosurgical complications is required to limit CNS damage. The diagnosis of complications relies on rapid neurological examination after early awakening and an awake patient is the best and the cheapest neuromonitoring available. However many factors may contribute to delayed emergence including: 1) perioperative opiate analgesia and anxiolytics, 2) metabolic disturbances (electrolyte or acid-base), 3) comorbidity, especially hepatorenal dysfunction that affects drug clearance, 4) stroke, 5) pneumocephalus or CSF hypotension and 6) seizures. Before extubation, airway and swallowing functions should be carefully evaluated and everything should be ready for a possible reintubation. For successful extubation, the patient should be 1) awake, 2) fully reversed from neuromuscular relaxation and spontaneously breathing, 3) hemodynamically stable, and 4) normothermic (Table 11-1 and 112). Rapid awakening and recovery The rationale for “rapid-awakening” after craniotomy with general anesthesia is that an early diagnosis of postoperative neurological complications can limit potentially devastating consequences. But early extubation must be considered in the context of the patient's perioperative neurological status and prognosis, surgical concerns, and respiratory status. After uncomplicated surgery, normothermic and normovolemic patients generally recover from anesthesia with minimal metabolic and hemodynamic changes. Thus, early recovery and extubation in the operating room is the preferred method when the preoperative state of consciousness is relatively normal.

www.medicalebookpdf.com

Delayed recovery In the complicated or unstable patient, the risks of early extubation may outweigh the benefits. Delayed recovery/extubation is appropriate after: long (> 6 hours) surgery, surgery for large tumors or AVM resection, major intraoperative bleeding, preoperative altered consciousness, severe cardiac or respiratory impairment, posterior fossa surgery where there is possible injury to lower cranial nerves, and select cervical procedures where re-intubation may be difficult. It is, however, often possible to perform a brief awakening of the patient without extubation to allow early neurological evaluation, followed by delayed emergence and extubation. If neurological examination is not possible, an immediate postoperative CT may be obtained or an ICP monitor may be placed. Weaning strategies and extubation failure Standard weaning criteria includes normal mental status; therefore these criteria are not always appropriate for neurosurgery patients. To be ready for extubation, the neurosurgical patient should successfully complete a spontaneous breathing trial. The initial trial should last at least 30 minutes and consist of either T-tube breathing or low levels of pressure support (≤8cmH20). A simple “leak test” with cuff deflation may help to identify laryngeal edema. Further details are provided in the recent report of the Task Force on Weaning from Mechanical Ventilation by the 6th International Consensus Conference on Intensive Care Medicine (Table 11-3). Signs of failure of the spontaneous breathing trial are listed in Table 11-4. The GCS and partial pressure of arterial oxygen/fraction of inspired oxygen ratio are factors that may predict extubation.

www.medicalebookpdf.com

For example, the success of extubation is >75% when the GCS is ≥8 but ~33% when GCS is 25/min for 2 hours), clinical signs of muscle fatigue or increased work of breathing, oxygen desaturation (SaO2 < 90%, PaO2 < 80 mmHg on FIO2 >0.5), hypercapnia (PaCO2 >45 mmHg, or an increase by >20%), and acidosis (pH 100 mmHg) and intra-thoracic pressures directly translates into elevated ICP [26]. General risk factors for PONV include: 1) female gender, 2) previous PONV or motion sickness, 3) non-smoker status, 4) duration of surgery >60 minutes, and 5) early post-operative opioid administration [26]. Specific neurosurgical risk factors include: 1) surgery location (i.e., infratentorial surgery near the area postrema at the floor of the fourth ventricle), 2) CSF cisternal space opened (chemical meningitis), 3) awake procedure (vs. general anesthesia), 4) intraoperative CSF leak and subsequent pneumocephalus, 5) use of a fat graft for a CSF leak, and 6) a lumbar intrathecal catheter and intracranial hypotension [27].

www.medicalebookpdf.com

Management of PONV Various pharmaceutical agents can be used to manage PONV. Serotonin (5HT3) antagonists, such as ondansetron, are effective but expensive. Trimethobenzamide is another popular choice and is thought to inhibit the chemoreceptor trigger zone. Cyclizine is a cheap antihistamine commonly prescribed whenever opiates are given. Alternatives include dopamine antagonists, e.g. metoclopramide or droperidol. Steroids also work but there may

www.medicalebookpdf.com

be a ceiling effect (dexamethasone 5-8mg). There are synergistic effects of dexamethasone and ondansetron. Intravenous ondansetron administration (4mg) at the time of dural closure can help reduce the incidence of PONV and the use of rescue antiemetics. Neufeld et al [25] preformed a recent meta-analysis of 7 prospective, randomized, placebo-controlled trials that together included 448 patients and found that ondansetron only had a significant impact on vomiting.

BASIC POSTOPERATIVE NEUROSURGICAL CARE Basic postoperative neurosurgical management is centered on the ABCs of care: 1) Maintain a secure airway, 2) Adequate respiration to maintain oxygen saturation, 3) Hemodynamic stability and fluid management. “Normohomeostasis” may be regarded as neuroprotective [28]. Other aspects of postoperative neurosurgical care (seizure control, prevention and management of infection, venous thromboembolism, ventriculostomy care) are beyond the scope of this review but clearly are fundamental to critical care. The typical postoperative patient probably does not require gastrointestinal prophylaxis unless they are on steroids or remain mechanically ventilated. Respiratory care Adequate oxygenation and ventilation are required to balance oxygen

www.medicalebookpdf.com

delivery to the brain, cerebral blood flow, cerebral perfusion pressure (CPP), and ICP. The Brain Trauma Foundation recommends maintaining PaO2 >60 mmHg and oxygen saturation >90% for TBI patients (BTF) [34]. These are sensible goals that have carried over into postoperative neurosurgical care of all patients. The following respiratory complications may be observed: Airways obstruction: This may be caused by many factors e.g. laryngospasm, soft tissue swelling around the pharynx (especially in children) or laryngeal or glottic edema (anterior cervical surgery, carotid endarterectomy), foreign bodies (loose teeth), hypotonia of pharyngeal muscles from the remaining anesthetic, and viscous fluids (blood after transphenoidal surgery). In all patients who develop airway obstruction, a patent airway must be achieved immediately (head tilt, chin lift, airway adjuncts, or intubation). The signs of airway obstruction include stridor, tachypnea, tracheal tug (downward displacement of the trachea during inspiration), use of accessory muscles, Intercostal and supraclavicular muscle recession, and reduced oxygen saturation (late sign), Hypoventilation A reduced ventilatory capacity can be caused by a depressed neurogenic respiratory drive and neuromuscular disorders. Etiologies include opioid drugs, hypothermia, metabolic alkalosis secondary to intermittent positive pressure ventilation, or mechanical breathing difficulty . Impaired chest expansion may result from parenchymal lung disease (e.g. obstructive airways disease secondary to smoking), muscle weakness (e.g. electrolyte derangement, neuromuscular disorders), hindered diaphragm movement (pain, obesity), and the residual effect of paralyzing agents on the chest wall musculature. Hypoxemia The principal causes of hypoxemia include: 1) a reduced FiO2, 2) hypoventilation associated with a depressed consciousness or airway obstruction and 3) ventilation/perfusion mismatch (e.g. lung collapse, pneumonia, atelectasis, bronchospasm, pulmonary edema, pneumothorax, pulmonary embolism). Thoracic and abdominal surgery often may alter the chest expansibility, and contribute to decreased oxygen saturation. This cause is less frequent after neurosurgery (unless a thoracotomy was performed for thoracic disc or anterior decompression). Neurogenic pulmonary edema Neurogenic pulmonary edema (NPE) is a potential complication of CNS insults, such as intracranial hemorrhage, subarachnoid hemorrhage (SAH), uncontrolled generalized seizures, TBI, and tumors. The postulated cause is sympathetic discharge. The treatment is mainly supportive (mechanical ventilation with careful use of positive end-expiratory pressure [PEEP] if

www.medicalebookpdf.com

tolerated by the ICP, and perhaps alpha-adrenergic blocking agents [35]). Who should be ventilated? Intubation and mechanical ventilation is indicated in neurosurgical patients in the following conditions: inability to protect the airway or manage secretions; need to reduce ICP by ventilation control; PaO2 50 mmHg, pH 40/minute or 33µg/L obtained within the first 72 hours is assigned a FPR of zero with a CI of 0-3 in patients who did not receive TH. Steffen et al [74] have questioned the cut-off value in patients who have undergone hypothermia, where in order to have 100% specificity the cut-off needed to be raised to 78.9µg/L. Two other studies raise important concerns regarding the applications of NSE in neuroprognostication after therapeutic hypothermia where FPR were as high as 10 with CI of 6-16 [64, 65]. EEG and SSEP are the most common electrophysiological modalities utilized in neuroprognostication. EEG has been evaluated in the prognostication of cardiac arrest survivors [66, 75–82], and has also led to some important clinical discoveries. The 2006 AAN practice parameters assign EEG a FPR of 3% with a CI of 0.9-11; making it the least predictive method to predict neurologic outcomes. Abend et al [83] pooled four existing studies on EEG in CA patients who had undergone therapeutic hypothermia and found that 29% of these patients had acute electrographic non-convulsive status epilepticus (NCSE). In contrast to the established guidelines and practice where SSEP is considered the most accurate ancillary method to aid clinical diagnosis of poor neurologic outcome (FPR 0.7% CI 0-3.7), a recent study comparing SSEP and continuous EEG by Cloostermans et al [80] found EEG to be superior in terms of its sensitivity to predict poor neurologic outcomes in CA patients treated with hypothermia. Leithner et al [84] demonstrated that neurologic recovery is possible despite absent or minimally present median nerve N20 responses greater than 24 hours after cardiac arrest. In the study by Bouwes et al [65], the absence of N20 responses on SSEP during hypothermia therapy had a FPR of 3%. Imaging studies have also been employed for prognostication mainly in the form of brain computed tomography (CT) where loss of gray-white matter differentiation and obvious infarction have also been used to bolster clinical prediction. The use of imaging has not yet been formally incorporated into any guidelines, however, and has been used based on individual clinician practice. More recently, quantitative measurements of signal change on both CT and MRI have attempted to improve the predictive abilities of imaging studies in the PCAS. At this time imaging can only provide limited supporting information in an overall multi-modal prognostication strategy, no decisions should be made based on solely one modality, but particularly not based on imaging alone.

www.medicalebookpdf.com

Given the aforementioned uncertainties in prognostication of neurological outcome in patients with HIE following cardiac arrest treated with therapeutic hypothermia, the following points should be kept in mind. There is no good evidence from well-designed studies to support substantial accuracy of early prognostication (< 72 hours post-arrest) in cardiac arrest survivors treated with therapeutic hypothermia. Given our lack of understanding of how therapeutic hypothermia improves outcomes, as well as its effects on emergence from coma and its well described effects in altering drug metabolism and clearance, it is prudent to be more conservative in approaching prognostication. Patients should be observed for a minimum of 72 hours post arrest. However, 5-7 or more days of observation may be necessary to fully account for the effects of therapeutic hypothermia. REFERENCES 1. Go AS, Mozaffarian D, Roger VL, et al.: Heart Disease and Stroke Statistics — 2013 Update A Report From the American Heart Association. Circulation 2013; 127:e1–240 2. Fishman GI, Chugh SS, Dimarco JP, et al.: Sudden cardiac death prediction and prevention: report from a National Heart, Lung, and Blood Institute and Heart Rhythm Society Workshop. Circulation 2010; 122:2335–48 3. Baker SP, Hu G, Wilcox HC, et al.: Increase in suicide by hanging/suffocation in the U.S., 20002010. Am. J. Prev. Med. 2013; 44:146–9 4. Warner DS, Warner MA: Drowning: Update 2009. Anesthesiology 2009; 110:1390–1401 5. Neumar RW, Nolan JP, Adrie C, et al.: Post-cardiac arrest syndrome: epidemiology, pathophysiology, treatment, and prognostication. A consensus statement from the International Liaison Committee on Resuscitation. Circulation 2008; 118:2452–83 6. Norton L, Hutchison RM, Young GB, et al.: Disruptions of functional connectivity in the default mode network of comatose patients. Neurology 2012; 78:175–81 7. Paine MG, Che D, Li L, et al.: Cerebellar Purkinje cell neurodegeneration after cardiac arrest: Effect of therapeutic hypothermia. Resuscitation 2012; 83:1511–6

www.medicalebookpdf.com

8. Wijdicks EF, Campeau NG, Miller GM: MR imaging in comatose survivors of cardiac resuscitation. AJNR 2001; 22:1561–5 9. Sekeljic V, Bataveljic D, Stamenkovic S, et al.: Cellular markers of neuroinflammation and neurogenesis after ischemic brain injury in the long-term survival rat model. Brain Struct. Funct. 2012; 217:411–20 10. Barrett KM, Freeman WD, Weindling SM, et al.: Brain injury after cardiopulmonary arrest and its assessment with diffusion-weighted magnetic resonance imaging. Mayo Clin. Proc. 2007; 82:828–35 11. Brierley J, Graham D, Adams J, et al.: Neocortical death after cardiac arrest. Lancet 1971; 298:560–565 12. Wu O, Sorensen AG, Benner T, et al.: Comatose Patients with Cardiac Arrest: Predicting Clinical Outcome with Diffusion-weighted MR Imaging. Radiology 2009; 252:173–181 13. Wijman CA, Mlynash M, Caulfield AF, et al.: Prognostic value of brain diffusion-weighted imaging after cardiac arrest. Ann. Neurol. 2009; 65:394– 402 14. Peberdy MA, Callaway CW, Neumar RW, et al.: Part 9: post-cardiac arrest care: 2010 American Heart Association Guidelines for Cardiopulmonary Resuscitation and Emergency Cardiovascular Care. Circulation 2010; 122:S768–86 15. Vaagenes P, Cantadore R, Safar P, et al.: Amelioration of brain damage by lidoflazine after prolonged ventricular fibrillation cardiac arrest in dogs. Crit. Care Med. 1984; 12:846–55 16. Group BRCTIS: A randomized clinical study of a calcium-entry blocker (lidoflazine) in the treatment of comatose survivors of cardiac arrest. N. Engl. J. Med. 1991; 324:1225–31 17. Longstreth W, Fahrenbruch C, Olsufka M, et al.: Randomized clinical trial of magnesium, diazepam, or both after out-of-hospital cardiac arrest. Neurology 2002; 59:506–514 18. Jastremski M: Glucocorticoid Treatment Does Not Improve Neurological Recovery Following Cardiac Arrest. JAMA 1989; 262:3427 19. Bernard SA, Gray TW, Buist MD, et al.: Treatment of comatose survivors of out-of-hospital cardiac arrest with induced hypothermia. N. Engl. J. Med. 2002; 346:557–63

www.medicalebookpdf.com

20. Hypothermia after Cardiac Arrest Study Group: Mild therapeutic hypothermia to improve the neurologic outcome after car-diac arrest. N. Engl. J. Med. 2002; 346:549–56 21. Nolan JP, Morley PT, Hoek TL Vanden, et al.: Therapeutic Hypothermia After Cardiac Arrest: An Advisory Statement by the Advanced Life Support Task Force of the International Liaison Committee on Resuscitation. Circulation 2003; 108:118–121 22. Nielsen N, Wetterslev J, Cronberg T, et al.: Targeted temperature management at 33 °C versus 36 °C after cardiac arrest. N. Engl. J. Med. 2013; 369:2197–206 23. Ian J, Nadkarni V: Targeted temperature management following cardiac arrest: An update -ILCOR. 2013. 24. Arrich J, European Resuscitation Council Hypothermia After Cardiac Arrest Registry Study Group: Clinical application of mild therapeutic hypothermia after cardiac arrest. Crit. Care Med. 2007; 35:1041–7 25. Testori C, Sterz F, Behringer W, et al.: Mild therapeutic hypothermia is associated with favourable outcome in patients after cardiac arrest with non-shockable rhythms. Resuscitation 2011; 82:1162–1167 26. Lundbye JB, Rai M, Ramu B, et al.: Therapeutic hypothermia is associated with improved neurologic outcome and survival in cardiac arrest survivors of non-shockable rhythms. Resuscitation 2012; 83:202–7 27. Don CW, Longstreth WT, Maynard C, et al.: Active surface cooling protocol to induce mild therapeutic hypothermia after out-of-hospital cardiac arrest: a retrospective before-and-after comparison in a single hospital. Crit. Care Med. 2009; 37:3062–9 28. Dumas F, Grimaldi D, Zuber B, et al.: Is hypothermia after cardiac arrest effective in both shockable and nonshockable patients?: insights from a large registry. Circulation 2011; 123:877–86 29. Storm C, Nee J, Roser M, et al.: Mild hypothermia treatment in patients resuscitated from non-shockable cardiac arrest. Emerg. Med. J. 2012; 29:100–3 30. Pfeifer R, Jung C, Purle S, et al.: Survival does not improve when therapeutic hypothermia is added to post-cardiac arrest care. Resuscitation 2011; 82:1168–73

www.medicalebookpdf.com

31. Lyden P, Ernstrom K, Cruz-Flores S, et al.: Determinants of effective cooling during endovascular hypothermia. Neurocrit. Care 2012; 16:413– 20 32. Sendelbach S, Hearst MO, Johnson PJ, et al.: Effects of variation in temperature management on cerebral performance category scores in patients who received therapeutic hypothermia post cardiac arrest. Resuscitation 2012; 83:829–34 33. Castrén M, Nordberg P, Svensson L, et al.: Intra-arrest transnasal evaporative cooling: a randomized, prehospital, multicenter study (PRINCE: Pre-ROSC IntraNasal Cooling Effectiveness). Circulation 2010; 122:729–36 34. Deasy C, Bernard S, Cameron P, et al.: Design of the RINSE trial: the rapid infusion of cold normal saline by paramedics during CPR. BMC Emerg. Med. 2011; 11:17 35. Garrett JS, Studnek JR, Blackwell T, et al.: The association between intraarrest therapeutic hypothermia and return of spontaneous circulation among individuals experiencing out of hospital cardiac arrest. Resuscitation 2011; 82:21–5 36. Benz-Woerner J, Delodder F, Benz R, et al.: Body temperature regulation and outcome after cardiac arrest and therapeutic hypothermia. Resuscitation 2012; 83:338–42 37. Leary M, Grossestreuer A V, Iannacone S, et al.: Pyrexia and neurologic outcomes after therapeutic hypothermia for cardiac arrest. Resuscitation 2012; 7–9 38. Badjatia N, Strongilis E, Gordon E, et al.: Metabolic impact of shivering during therapeutic temperature modulation: the Bedside Shivering Assessment Scale. Stroke 2008; 39:3242–7 39. Chamorro C, Borrallo JM, Romera MA, et al.: Anesthesia and analgesia protocol during therapeutic hypothermia after cardiac arrest: a systematic review. Anesth. Analg. 2010; 110:1328–35 40. Sato K, Kimura T, Nishikawa T, et al.: Neuroprotective effects of a combination of dexmedetomidine and hypothermia after incomplete cerebral ischemia in rats. Acta Anaesthesiol. Scand. 2010; 54:377–82 41. Schoeler M, Loetscher PD, Rossaint R, et al.: Dexmedetomidine is

www.medicalebookpdf.com

neuroprotective in an in vitro model for traumatic brain injury. BMC Neurol. 2012; 12:20 42. Doufas AG, Lin C-M, Suleman M-I, et al.: Dexmedetomidine and meperidine additively reduce the shivering threshold in humans. Stroke 2003; 34:1218–23 43. Lenhardt R, Orhan-Sungur M, Komatsu R, et al.: Suppression of shivering during hypothermia using a novel drug combination in healthy volunteers. Anesthesiology 2009; 111:110–115 44. Choi HA, Ko S-B, Presciutti M, et al.: Prevention of shivering during therapeutic temperature modulation: the Columbia anti-shivering protocol. Neurocrit. Care 2011; 14:389–94 45. Rittenberger JC, Popescu A, Brenner RP, et al.: Frequency and timing of nonconvulsive status epilepticus in comatose post-cardiac arrest subjects treated with hypothermia. Neurocrit. Care 2012; 16:114–22 46. Mani R, Schmitt SE, Mazer M, et al.: The frequency and timing of epileptiform activity on continuous electroencephalogram in comatose postcardiac arrest syndrome patients treated with therapeutic hypothermia. Resuscitation 2012; 83:840–7 47. Rossetti AO, Urbano LA, Delodder F, et al.: Prognostic value of continuous EEG monitoring during therapeutic hypothermia after cardiac arrest. Crit. Care 2010; 14:R173 48. Nielsen N, Sunde K, Hovdenes J, et al.: Adverse events and their relation to mortality in out-of-hospital cardiac arrest patients treated with therapeutic hypothermia. Crit. Care Med. 2011; 39:57–64 49. Geocadin RG, Ritzl EK: Seizures and status epilepticus in post cardiac arrest syndrome: therapeutic opportunities to improve outcome or basis to withhold life sustaining therapies? Resus citation 2012; 83:791–2 50. Brophy GM, Bell R, Claassen J, et al.: Guidelines for the evaluation and management of status epilepticus. Neurocrit. Care 2012; 17:3–23 51. Bouwes A, van Poppelen D, Koelman JHTM, et al.: Acute posthypoxic myoclonus after cardiopulmonary resuscitation. BMC Neurol. 2012; 12:1–6 52. Naples R, Ellison E, Brady WJ: Cranial computed tomography in the resuscitated patient with cardiac arrest. Am. J. Emerg. Med. 2009; 27:63–7

www.medicalebookpdf.com

53. Koenig M, Bryan M, Lewin J, et al.: Reversal of transtentorial herniation with hypertonic saline. Neurology 2008; 70:1023–9 54. Neumar RW, Otto CW, Link MS, et al.: Part 8: Adult advanced cardiovascular life support: 2010 American Heart Association Guidelines for Cardiopulmonary Resuscitation and Emergency Cardiovascular Care. Circulation 2010; 122:S729–67 55. Kuisma M, Boyd J, Voipio V, et al.: Comparison of 30 and the 100% inspired oxygen concentrations during early post-resuscitation period: a randomised controlled pilot study. Resuscitation 2006; 69:199–206 56. Kilgannon JH, Jones AE, Shapiro NI, et al.: Association between arterial hyperoxia following resuscitation from cardiac arrest and in-hospital mortality. JAMA 2010; 303:2165–71 57. Kilgannon JH, Jones AE, Parrillo JE, et al.: Relationship between supranormal oxygen tension and outcome after resuscitation from cardiac arrest. Circulation 2011; 123:2717–22 58. Janz DR, Hollenbeck RD, Pollock JS, et al.: Hyperoxia is associated with increased mortality in patients treated with mild therapeutic hypothermia after sudden cardiac arrest. Crit. Care Med. 2012; 40:3135–9 59. Napolitano LM, Kurek S, Luchette F a, et al.: Clinical practice guideline: red blood cell transfusion in adult trauma and critical care. J. Trauma 2009; 67:1439–42 60. Carson J, Carless P, Hebert P: Transfusion thresholds and other strategies for guiding allogeneic red blood cell transfusion (Review). Cochrane Database Syst. Rev. 2012; 1–75 61. Carson JL, Grossman BJ, Kleinman S, et al.: Clinical Guideline Red Blood Cell Transfusion: A Clinical Practice Guideline From the AABB. Ann. Intern. Med. 2012; 157:49–58 62. Wijdicks EFM, Hijdra A, Young GB, et al.: Practice parameter: prediction of outcome in comatose survivors after cardiopulmonary resuscitation (an evidence-based review): report of the Quality Standards Subcommittee of the American Academy of Neurology. Neurology 2006; 67:203–10 63. Samaniego EA, Mlynash M, Caulfield AF, et al.: Sedation confounds outcome prediction in cardiac arrest survivors treated with hypothermia. Neurocrit. Care 2011; 15:113–9

www.medicalebookpdf.com

64. Fugate JE, Wijdicks EFM, Mandrekar J, et al.: Predictors of neurologic outcome in hypothermia after cardiac arrest. Ann. Neurol. 2010; 68:907–14 65. Bouwes A, Binnekade JM, Kuiper M a, et al.: Prognosis of coma after therapeutic hypothermia: A prospective cohort study. Ann. Neurol. 2012; 71:206–12 66. Rossetti AO, Oddo M, Logroscino G, et al.: Prognostication after cardiac arrest and hypothermia: a prospective study. Ann. Neurol. 2010; 67:301–7 67. Oddo M, Rossetti AO: Early multimodal outcome prediction after cardiac arrest in patients treated with hypothermia. Crit. Care Med. 2014; 42:1340– 7 68. Bouwes A, Binnekade JM, Zandstra DF, et al.: Somatosensory evoked potentials during mild hypothermia after cardiopulmonary resuscitation. Neurology 2009; 73:1457–61 69. Sandroni C, Cavallaro F, Callaway CW, et al.: Predictors of poor neurological outcome in adult comatose survivors of cardiac arrest: a systematic review and meta-analysis. Part 2: Patients treated with therapeutic hypothermia. Resuscitation 2013; 84:1324–38 70. Golan E, Barrett K, Alali AS, et al.: Predicting neurologic outcome after targeted temperature management for cardiac arrest: systematic review and meta-analysis. Crit. Care Med. 2014; 42: 1919-30. 71. Al Thenayan E, Savard M, Sharpe M, et al.: Predictors of poor neurologic outcome after induced mild hypothermia following cardiac arrest. Neurology 2008; 71:1535–7 72. Rittenberger JC, Sangl J, Wheeler M, et al.: Association between clinical examination and outcome after cardiac arrest. Resuscitation 2010; 81:1128– 32 73. Lucas JM, Cocchi MN, Salciccioli J, et al.: Neurologic recovery after therapeutic hypothermia in patients with post-cardiac arrest myoclonus. Resuscitation 2012; 83:265–9 74. Steffen IG, Hasper D, Ploner CJ, et al.: Mild therapeutic hypothermia alters neuron specific enolase as an outcome predictor after resuscitation: 97 prospective hypothermia patients compared to 133 historical nonhypothermia patients. Crit. Care 2010; 14:R69 75. Stammet P, Werer C, Mertens L, et al.: Bispectral index (BIS) helps

www.medicalebookpdf.com

predicting bad neurological outcome in comatose survivors after cardiac arrest and induced therapeutic hypothermia. Re suscitation 2009; 80:437– 42 76. Leary M, Fried DA, Gaieski DF, et al.: Neurologic prognostication and bispectral index monitoring after resuscitation from cardiac arrest. Resuscitation 2010; 81:1133–7 77. Legriel S, Bruneel F, Sediri H, et al.: Early EEG monitoring for detecting postanoxic status epilepticus during therapeutic hypothermia: a pilot study. Neurocrit. Care 2009; 11:338–44 78. Wennervirta JE, Ermes MJ, Tiainen SM, et al.: Hypothermia-treated cardiac arrest patients with good neurological outcome differ early in quantitative variables of EEG suppression and epi-leptiform activity. Crit. Care Med. 2009; 37:2427–35 79. Rundgren M, Westhall E, Cronberg T, et al.: Continuous amplitudeintegrated electroencephalogram predicts outcome in hypothermia-treated cardiac arrest patients. Crit. Care Med. 2010; 38:1838–44 80. Cloostermans MC, van Meulen FB, Eertman CJ, et al.: Continuous electroencephalography monitoring for early prediction of neurological outcome in postanoxic patients after cardiac arrest: a prospective cohort study. Crit. Care Med. 2012; 40:2867–75 81. Crepeau AZ, Rabinstein A a, Fugate JE, et al.: Continuous EEG in therapeutic hypothermia after cardiac arrest: Prognostic and clinical value. Neurology 2013; 80:339–44 82. Oh SH, Park KN, Kim YM, et al.: The prognostic value of continuous amplitude-integrated electroencephalogram applied immediately after return of spontaneous circulation in therapeutic hypothermia-treated cardiac arrest patients. Resuscitation 2012; 84:200–205 83. Abend NS, Mani R, Tschuda TN, et al.: EEG Monitoring during Therapeutic Hypothermia in Neonates, Children, and Adults. Am. J. Electroneurodiagnostic Technol. 2012; 51:1–20 84. Leithner C, Ploner CJ, Hasper D, et al.: Does hypothermia influence the predictive value of bilateral absent N20 after cardiac arrest? Neurology 2010; 74:965–9

www.medicalebookpdf.com

HYPOXIC-ISCHEMIC ENCEPHALOPATHY QUESTIONS 1. What are the most common clinical manifestations of HIE? a. Coagulopathies b. Seizure disorders c. Disorders of consciousness d. Stroke e. Autonomic instability 2. According to the AHA guidelines for post resuscitation care, what should the systolic blood pressure (SBP) goal be for post cardiac arrest patients? a. SBP >100 mmHg b. SBP >60 mmHg c. SBP60 mmHg d. SBP 90 3. Which of the following factors allows for accurate prognostication of neurologic outcomes in patients with HIE post cardiac arrest treated with therapeutic hypothermia in the first 24 hours post arrest? a. N20 median nerve somatosensory evoked potentials b. Clinical neurologic examination c. Multimodality prognostication combining imaging, clinical examination and electroencephalography d. Neuron specific enolase levels e. None of the above 4. Based on the landmark 2002 trials on therapeutic hypothermia for comatose survivors of witnessed cardiac arrest with initial rhythms of VT/VF (Bernard et al and HACA Study Group), what is the number needed to treat

www.medicalebookpdf.com

to achieve a good neurologic outcome? a. NNT = 6 b. NNT = 13 c. NNT = 16 d. NNT = 22 e. None of the above 5. What is the principal pathophysiologic mechanism of injury at the cellular level resulting from global cerebral hypoxia-ischemia? a. Cellular peroxidation b. Excitotoxicity c. DNA fragmentation d. Mitochondrial failure e. Errors in transcription 6. What is the current utility of the 2006 Practice Parameter from the Quality Standards Subcommittee of the American Academy of Neurology: Prediction of Outcome in Comatose Survivors after Cardiopulmonary Resuscitation? a. These guidelines are outdated and no longer applicable b. They only apply to comatose survivors of cardiac arrest treated with therapeutic hypothermia c. They only apply to comatose survivors of cardiac arrest who are not treated with therapeutic hypothermia d. They still apply to all comatose survivors of cardiac arrest regardless of use of therapeutic hypothermia e. None of the above 7. Which of the following statements is false ? a. Cardiac arrest is the most common precipitating factor of HIE in adults

www.medicalebookpdf.com

b. HIE can result from cardiac, respiratory, traumatic and a variety of other causes that result in inadequate oxygen and blood flow to the entire brain c. After successful resuscitation, cardiac arrest survivors should be maintained with an SpO2 of ≥94% d. Initiation of therapeutic hypothermia is absolutely contraindicated in comatose cardiac arrest survivors with acute myocardial infarction that are going for emergent coronary revascularization. e. None of the above 8. Which of the following precipitating mechanisms of HIE carries the best prognosis? a. Primary cardiac arrest b. Drowning c. Partial hanging d. Primary respiratory arrest e. No mechanism of HIE has been definitively proven to confer an improved likelihood of a good outcome 9. Which of the following is used to diagnose HIE? a. MRI b. History and clinical examination c. Transcranial Doppler ultrasonography d. EEG e. Radionuclide cerebral blood flow study 10. Which of the following are evidence-based interventions in post cardiac arrest care? a. Therapeutic hypothermia of 30-32 °C for 12-24 hours b. Routine intracranial pressure monitoring c. The use of calcium channel blockers to limit excitotoxicity

www.medicalebookpdf.com

d. Therapeutic hypothermia of 32-34 °C for 12-24 hours e. Routine seizure prophylaxis with anti-epileptic drugs

www.medicalebookpdf.com

HYPOXIC-ISCHEMIC ENCEPHALOPATHY ANSWERS 1. The correct answer is C. Disorders of consciousness are the most characteristic feature of HIE; this may eventually resolve, however, and patients may recover full consciousness. Seizure disorders and autonomic instability may also be present in HIE, however, they are not as common. Stroke is not associated with HIE and coagulopathy may be seen as a complication of TH; however, it is not a feature of HIE. 2. The correct answer is E. According to the AHA post resuscitation guidelines the systolic blood pressure should be maintained greater than 90 mmHg. 3. The correct answer is E. In comatose survivors of cardiac arrest treated with therapeutic hypothermia there is no evidence supporting the use of early (prior to 72 hours post arrest) prognostication. In fact none of the options presented (imaging, electrophysiology, clinical examination or biomarkers), have sufficiently low false positive rates with narrow confidence interval to recommend their independent use in prognostication. 4. The correct answer is A. Based on the Bernard et al. and the Hypothermia After Cardiac Arrest (HACA) Study Group data, the number needed to treat to obtain a good neurologic outcome in witnessed, comatose survivors of VT/VF arrests with less than 30 minutes to return of spontaneous circulation (ROSC) is six (NNT = 6). 5. The correct answer is B. Cellular oxygen deprivation, results in decreased ATP production with resulting cellular energy starvation. This results in excitotoxicity, an uncontrolled release of glutamate, which leads to injury mediated through NMDA receptors. NMDA mediated glutamate excitotoxicity creates intracellular calcium influx that activates second messengers which amplify cellular injury by increasing calcium permeability and increasing glutamate release leading to a vicious cycle and the activation of nitric oxide synthase. Secondary to excitotoxicty, oxygen free radical species are also responsible for cellular injury by direct DNA fragmentation, protein oxidation, lipid peroxidation and disruption of the mitochondrial respiratory chain. 6. The correct answer is C. The current utility of the 2006 Practice Parameter from the Quality Standards Subcommittee of the American Academy of Neurology: Prediction of Outcome in Comatose Survivors after

www.medicalebookpdf.com

Cardiopulmonary Resuscitation is to provide a prognostic algorithm for patients who have not undergone therapeutic hypothermia. These recommendations have not been well validated in patients treated with hypothermia. 7. The correct answer (only false statement) is D. The need for emergent cardiac revascularization is not an absolute contraindication to the initiation of therapeutic hypothermia. Evidence has shown that initiation of hypothermia prior to percutaneous coronary intervention does not delay door to balloon time, or increase complication rates. All other statements are correct. 8. The correct answer is E. Though some authors have postulated that primary cardiac events have the best outcomes, this statement is not definitively proven and remains controversial. At this time no etiologic mechanism can be considered to have a better pre-test probability of a good outcome compared to other causes of HIE. 9. The correct answer is B. The diagnosis of HIE is made on the basis of clinical examination consistent with an encephalopathy, and a history consistent with a precipitating event that could result in a global cerebral hypoxic or ischemic event. The other options (MRI, EEG, TCD) are useful adjuncts in ruling out other processes and can provide relevant information, but they are not necessary to make the diagnosis of HIE. A radionuclide cerebral blood flow test would only be necessary as a confirmatory exam in cases of brain death. 10. The correct answer is D. Therapeutic hypothermia of 32-34 °C for 12-24 hours is the only evidence based intervention listed. There is no evidence for moderate (_ 50% in MCA territory on head CT, (2) early nausea/vomiting, and (3) NIHSS >_ 20 for left and >_15 for right hemispheric strokes [18]. A deteriorating clinical exam with increased somnolence may be reasonably sensitive sign of elevated intracranial pressure (ICP). Outcomes after mMCA are often dismal; despite aggressive medical therapy, mortality rates are reported up to 80% [19]. One intervention that may improve prognosis and outcome for mMCA is hemicraniectomy. Randomized controlled trials evaluating benefit of decompressive hemicraniectomy for mMCA have demonstrated efficacy in the management of malignant brain edema and associated elevated intracranial pressures. Outcomes from pooled analysis of three European clinical trials DECIMAL, DESTINY, and HAMLET demonstrated favorable outcomes in reaching mRS scores of 153, 154 in patients presenting with severe strokes (NIHSS >_21) [20] (Figure 14-1). These outcomes (achieving mRS 3 or 4) were considered relatively good when considering the often dismal natural history for mMCA.. This information can be helpful when counseling patient's families on management options for mMCA infarction. When counseling families, it should be understood what the expectations are for best-case scenario recovery. What might be considered a bad outcome (mRS 3 or 4) to some might be acceptable to others. Finally, posterior circulation strokes are generally associated with poor outcomes. However, in a large registry of posterior circulation strokes [21], outcomes were better than expected with overall mortality rate of 3.6%. They found outcomes worse if the mechanism was cardiac in origin versus penetrating artery disease (relative risk of poor outcome 1.89 vs. 0.82, respectively). Patients with intrinsic basilar artery disease or embolism to the basilar artery had more severe disability or mortality compared to patients with extracranial or intracranial vertebral artery disease. Thus, outcome determination in these patients is best made on a case-by-case basis.

www.medicalebookpdf.com

Figure 14-1. Outcomes from hemicraniectomy versus conservative treatment for malignant middle cerebral artery infarction: pooled analysis of DECIMAL, DESTINY, and HAMLET trials. (Reprinted from Vahedi K, Hofmeijer J, Juettler E, et al. Early decompressive surgery in malignant infarction of the middle cerebral artery: a pooled analysis of three randomized controlled trials. Lancet Neurol. 2007;6:215222, with permission from Elsevier.)

SUBARACHNOID HEMORRHAGE Subarachnoid hemorrhage (SAH) due to ruptured intracranial aneurysms can be a devastating disease with an overall high mortality [22,23]. What are the independent predictors of outcome after SAH? These include older age, large aneurysm size (≥10 mm), initial clinical presentation, and aneurysm rebleeding, particularly in the first 24 hours after presentation (Table 14-5). Older age, higher Hunt and Hess grade score, large aneurysms, and symptomatic vasospasm with delayed cerebral ischemia (DCI) are significant predictors of severe disability and death at 3 months [23]. The Hunt and Hess grading system, initially devised to predict operative risk after SAH, is also useful to predict outcomes. Outcomes have improved since the initial use of this scale [24, 25] (Table 14-6). Symptomatic vasospasm with development of DCI is associated with poor neurological outcome and higher mortality. The ability to predict the risk for vasospasm and DCI in patients after SAH may be partly based on Fisher score [26,27] (see Table 14-7), i.e., amount and location of subarachnoid blood on head CT. In addition, a study by Wartenberg, et. al. [28] identified medical complications in the NICU of anemia, hyperglycemia, and fever in patients with SAH as significant predictors of severe disability and mortality. It is not known

www.medicalebookpdf.com

whether aggressively treating these medical complications improves outcome. However, fever in the NICU is generally associated with worse outcomes. Finally, Samuels, et. al. [29] showed that implementation of neurointensivistlead multidisciplinary care team resulted in significantly improved hospital discharges to home in patients with aneurysmal subarachnoid hemorrhage.

The Simplified Acute Physiology Score (SAPS)-II severity illness score is frequently used in general intensive care units to predict a high risk of mortality [30]. For SAH, the SAPS II score recorded within the first 24 hours of admission was a powerful predictor of poor outcome at 3 months and predictor of DCI. It is a useful scoring system for SAH that incorporates both clinical and laboratory parameters [31].

Continuous electroencephalography (cEEG) may provide information which is independently predictive of prognosis in patients with poor grade Hunt & Hess grade SAH. Findings of absence of EEG reactivity or presence of periodic lateralized epileptiform discharges (PLEDS) independently predicted poor

www.medicalebookpdf.com

outcomes (mRS >4) in poor grade SAH with Hunt & Hess score 3 or worse [32]. Biomarkers that are present in patients with SAH include interleukin-6 (IL6), procalcitonin levels, and elevated leukocyte counts, and these have been associated with unfavorable outcomes and an increased risk for DCI [33].

INTRACEREBRAL HEMORRHAGE Intracerebral hemorrhage (ICH) represents approximately 10-15% of all hospital stroke admissions. The underlying cause of ICH may be classified as primary, originating from rupture of small arterioles due to chronic hypertension, cerebral amyloid angiopathy, secondary due to vascular malformations, hemorrhagic transformation from ischemic strokes, brain tumors, abnormal coagulation, trauma brain injury, or central nervous system vasculitis. In approximately 40% of cases, there can be extension of hemorrhage into the ventricles which significantly increases morbidity and mortality. What are the indicators of poor outcome after ICH? These include ICH volume, extension into ventricles, and location of the hemorrhage (supra- vs. infratentorial compartment). In addition, hematoma expansion or growth in the first 24 to 36 hours after admission is also a determinant of mortality and poor outcomes after ICH [34,35]. Surgical removal of ICH has not been shown to be effective to date in improving survival or functional outcome [STICH I trial; 36].

www.medicalebookpdf.com

However, a recent trial reported a survival advantage with early surgical evacuation of primary ICH near the cortex in patients without intraventricular hemorrhage [STICH II trial; 37]. Medical management involves blood pressure reduction, fever and hyperglycemic control, ICP monitoring and treatment, seizure prevention, and care in specialized stroke unit or NICU. In fact, treatment in NICU improves outcomes after ICH. Diringer and Edwards [38] showed mortality risk after ICH was increased with admission to generalsurgical ICU (as opposed to specialty NICU), with lower GCS scores, and older age. Although improved outcomes of ICH in patients treated in NICU may be from more intensive treatment, the benefit may also be from avoidance of therapeutic nihilism. Perceived views of the practitioner on futility of care in patients with ICH may lead to withdrawal of care and may lead to so-called selffulfilling prophecies [39]. Early do-not-resuscitate (DNR) orders in patients with ICH are associated with doubling the chances of death, even after adjusting for known predictors of mortality [40]. Prognostication of outcome from ICH may include the use of scoring systems such as the ICH score, which incorporates the sum of individual items and factors associated with 30-day mortality. These include Glasgow Coma Scale score, age ≥80 years, infratentorial origin of ICH, presence of intraventricular hemorrhage (IVH), and large ICH volume (see Table 2-2). Intracerebral hemorrhage volume can be divided into small (< 30cm3) vs. large hematomas (>6 0 cm3), calculated by the ABC/2 method in assessing clot size on head CT. Mortality is high with ICH > 60 cm3. However, holding treatment based solely on large hematomas volume is not necessarily justified, particularly in the absence of other poor outcome predictors such as low GCS score, advanced age, or IVH. In a study by Hemphill, et al. [41], all patients with ICH score of 0 survived and those with the highest score, ICH score of 6, died. The grading system also appears valid in predicting functional outcome using mRS at 12 month follow-up. The Charlson Comorbidity Index, which assesses comorbid medical conditions in patients with ICH, independently predicted 12month functional outcome [42]. Scoring systems such as the ICH score and Charlson Comorbid Index provide a framework for the practitioner in discussing prognostic information with patients and families. However, they should not be used as definitive predictors of outcome. TRAUMATIC BRAIN INJURY Traumatic brain injury (TBI) is leading cause of death and disability, particularly in patients younger than 45 years of age, with mortality rates approaching 40%

www.medicalebookpdf.com

[43]. In addition, quality of life post-TBI is impacted with approximately 30% left with serious neurological sequelae or vegetative state. There is significant variability in reported mortality and decisions to withdraw care even across level 1 trauma centers [43]. This partially reflects lack of reliable prognostic models to accurately predict outcomes for these patients and resultant variances in early withdrawal of life-sustaining treatment. Despite this, risk of death at one year after TBI is significantly lower in patients treated in designated trauma versus non-trauma centers [44] Similarly, treatment of head injury patients in a neurosciences intensive care unit results in significant reduction in ICU mortality rates, shorter hospital length of stay (LOS), and greater odds of being discharged to home or rehabilitation unit [45]. Known poor prognostic factors after TBI are the following: (1) initial motor score on GCS of 1 [none] or 2 [extensor posturing], (2) lack of pupil reactivity, (3) hypoxia and hypotension, or (4) head CT characteristics according to Marshal criteria (e.g., mass lesion, SAH, or signs of raised intracranial pressure) [46] The prognostic work-horse in the past, the Glasgow Coma Scale (GCS), has apparently lost its predictive power for TBI since late 1990s [47]. This may be due to aggressive pre-hospital treatment, sedation and intubation obscuring initial neurological assessment of GCS, or progress in clinical TBI management. In addition, inaccurate early assessment of neurological severity in head injury may mistakenly assign poor prognosis to patients who recover, particularly in younger patients and those with milder diffuse injury on initial head CT and preserved brainstem reflexes [48]. Various predictive models and outcome scores have been proposed to help prognosticate patients with TBI. Such examples include APACHE, SAPS, Glasgow Outcome Scale/Extended (GOS, GOSE), and Mortality Prediction Model (MPM). Recently, two prognostic models were developed from evaluation of large data sets: the International Mission on Prognosis and Analysis of Clinical Trials in Traumatic Brain Injury database (IMPACT) and the Corticosteroid Randomization After Significant Head injury (CRASH) trials [42]. From these studies, multiple logistic regression models of variables predicting functional outcome at 6 months were determined (see Table 14-8). Key prognostic factors, similar to above, were age, GCS motor score, pupillary reactivity, Marshal CT classification (epidural hematoma, traumatic SAH), hypoxia, and hypotension. Surprisingly, despite extensive validation of IMPACT and CRASH scoring systems, there is still lack of accuracy needed in making meaningful decisions for the individual patient.

www.medicalebookpdf.com

Diffuse traumatic axonal injury (DAI) involves white matter tracts, indicates severe injury, and occurs in approximately 50% of all severe TBI cases. Magnetic resonance imaging (MRI) with either diffusion-weighted imaging (DWI) or diffusion tensor imaging (DTI) has high sensitivity in detecting traumatic white matter injury. In one study, MRI DTI along with the IMPACT score predicted 1-year functional outcomes better than IMPACT score alone [43]. The sensitivity of serum biomarkers such as S100 protein, NSE, and glial fibrillary acidic protein (GFAP) may be predictive in outcomes after TBI; however, results of studies are mixed in terms of their actual specificity and sensitivity [49]. Similar to other examples provided in this chapter, caution should be taken with early withdrawal after TBI of life-sustaining treatment as self-fulfilling prophecy [43]. This is not to suggest that clinical evaluation by the practitioner for those patients deemed ‘non-recoverable' should be ignored. However, particularly with TBI, cautious early initial assessment of outcome after TBI is warranted given the limited accuracy of current predictive models for TBI and chance for some to improve despite serious initial injury [48]. STATUS EPILEPTICUS Outcomes from status epilepticus (SE) may be variable and dependent on inciting causes such as ischemic stroke, intracerebral hemorrhage, alcohol use,

www.medicalebookpdf.com

non-compliance with antiepileptic medication, or after hypoxic brain injury following cardiac arrest. Status epilepticus is reportedly rare after ischemic stroke or ICH, but when present, is associated with high rates of poor outcomes [50]. Legriel et. al. [51] evaluated predictors of outcome in 248 patients after SE and found several factors independently associated with 90-day functional outcome: age, focal neurological signs, total seizure duration, progression to refractory status epilepticus, and presence of cerebral insult [51]. Approximately 42% patients in the study achieved good recovery, i.e., able to return to formal occupation with or without minor limitations. However, when seizure activity continues despite initial staged therapies, it is termed refractory SE or RSE. Poor outcomes after RSE were noted in patients with prolonged drug-induced coma, burst- or isoelectric electroencephalographic (EEG) suppression, or cardiopulmonary complications [52]. However, some recent studies including that by Kilbride, et al [53], suggest potential for good or excellent outcome in RSE despite age and months of coma. Postanoxic SE almost invariably predicts poor outcome after therapeutic hypothermia but some report favorable outcome in these patients as well particularly with preserved brainstem function, somatosensory evoked potentials (SSEPs), and EEG reactivity [54]. Caution should be taken in treated SE patients who don't ‘wake up' or those in ICU with coma for unknown reason who may be in nonconvulsive SE (NSE). Continuous EEG monitoring can help to identify NCSE and potentially lead to treatment with improved outcomes. OUTCOMES OF POST-ANOXIC BRAIN INJURY AFTER CARDIAC ARREST Neurologic outcomes of patients experiencing sudden cardiac arrest (SCA) is in general poor. The cause of mortality after SCA is primarily related to effects of anoxic brain injury and not necessarily from cardiac complications. One study analyzing outcomes of survival to hospital after SCA in over 12,000 patients treated by emergency medical services (EMS) in one large U.S city, found no difference in survival in patients treated between 1998 and 2001 to those treated between 1977-1981 (15.7 vs. 17.5 %, respectively) [55]. Although trends towards favorable neurological improvement in survivors over the last decade have been reported, overall survival following SCA remains poor [56]. Outcomes differ based on initial rhythm found at scene of SCA. For example, outcomes are poorer with so-called ‘non-shockable' rhythms (asystole, pulseless electrical activity or PEA) compared to ‘shockable' rhythms (ventricular tachycardia [VT]or fibrillation [VF]. Improved survival may be best

www.medicalebookpdf.com

when initial rhythm is VF, but still dependent on prompt delivery of effective cardiopulmonary resuscitation (CPR). Clinical factors identified as predictors of greater likelihood of survival to hospital discharge are witnessed arrest, VT or VF as initial rhythm, return to spontaneous circulation (ROSC) during first 10 minutes, and longer duration of overall resuscitation efforts [57]. Are there definitive guidelines to determine outcome in patients after SCA? In seminal paper in 1986, Levy, et. al. [58] identified specific clinical findings that predicted recovery (or death) in patients from hypoxia-ischemia post-cardiac arrest. The presence of pupillary light reflexes, motor flexor or better response to noxious stimuli, spontaneous or roving eye movements from day one, predicted good outcomes. No patients recovered who had absent pupillary response on day one post-arrest. The American Academy of Neurology in 2006 published predictors of poor neurological outcome to help physicians with prognostication post-cardiac arrest. These predictors, similar to those of Levy, et. al., were absent pupillary, corneal reflexes, motor responses at day 3, absent N20 responses to somatosensory evoked potentials (SSEPs) beginning on day 1, serum NSE > 33 ng/ml on days 1 to 3, and the presence of myoclonic status epilepticus within 24 hours of arrest [59]. Magnetic resonance imaging characteristic abnormalities after global hypoxic-ischemic are also associated with poor outcomes [60]. Clinical bedside examination utilizing the Full Outline of UnResponsiveness (FOUR) score performed 3-5 days after cardiac arrest appears to be an accurate predictor of outcome [61]. With the introduction of therapeutic hypothermia, initial poor neurologic examination or serum biomarkers for brain injury (e.g., NSE, S100B) may no longer be reliable predictors of poor outcome after cardiac arrest. Findings of unreactive an EEG background, status epilepticus, bilaterally absent SSEPs, characteristics changes on MRI DWI and FLAIR imaging, retain their poor predictive value in cardiac arrest even after treatment with therapeutic hypothermia [62]. Biomarkers that appear to correlate with severe brain injury after SCA are serum S100B and interleu-kin-8. Increased S-100B values measured at 12 hrs after insult correlated with unfavorable neurologic outcome at 12 months. What are the cognitive outcomes in patients who survive SCA? In study by Fugate, et. al. [63], longterm cognitive abilities of survivors, with median follow-up at 20 months after arrest, were found to be normal in 60% and abnormal in 40%. Of the patients working at the time of SCA, 79% were able to return to work. This supports the idea that majority of patients who survive cardiac arrest may have preserved cognitive function.

www.medicalebookpdf.com

ETHICAL CONSIDERATIONS IN PROGNOSTICATION IN NEUROCRITICAL CARE The cornerstone of ethical decision making in critical care is careful determination and open communication of prognosis to patient and/or appropriate surrogate. In retrospective review, Boissy et al. identified types of patients and reasons for obtaining formal inhospital ethics consults [65]. The latter, in many cases, is made by the surrogate for the patient. Guidelines in each state exist for legal determination of authorized surrogate for medical decision making, including durable power of attorney, health care proxy, or health care agent. Often decision to proceed with aggressive therapy or withdrawal of care is made by surrogate and dependent on the understanding of physician's communication of diagnosis and prognosis. It is essential that the physician avoid pitfalls in determination of prognosis and be aware that their approach with patient or surrogate might significantly influence their decision regarding care. Framing or biasing how information is presented can have a significant effect of what decisions are made. The fallacy of a self-fulfilling prophecy based on less-than-reliable outcome data can lead to premature withdrawal of lifesustaining therapy that may leads to patients' death. Finally, physicians should exercise sensitivity to specific ethnic and cultural values and beliefs that may affect decision making. Ethical consultation may be needed when there is perception of unresolved conflict. In retrospective review, Boissey et. al. identified types of patients and reasons for obtaining formal in-hospital ethics consults [65]. The majority of patients in whom ethics consults were obtained were stroke patients. Reasons for ethics consultation were primarily physicians' and families' struggle with decisions regarding futility, negotiating family conflict, and capacity determination. Interestingly, most consults were not due to conflict between families and treating physician. REFERENCES 1. Kurtz P, Fitts V, Sumer Z, et al. How does care differ for neurological patients admitted to a neurocritcal care unit versus a general ICU? Neurocrit Care. 2011;15:477-480. 2. Knaus WA, Wagner DP, Draper EA,et al: The APACHEIII prognostic system: risk prediction of hospital mortality for critical ill hospitalized patients. Chest. 1991;10:1619-1636.

www.medicalebookpdf.com

3. Suarez JI. Outcome in neurocritical care: advances in monitoring and treatment and effect of a specialized neurocritical care team. Crit Care Med. 2006;34:S232-S238. 4. Kimberly WT. Biomarkers in neurocritical care. Neurotherapeutics. 2012;9:17-23. 5. Caulfield AF, Galer L, Lansber MG, et al. Outcome prediction in mechanically ventilated neurologic patients by junior neurointensivists. Neurology. 2010;74:1096-1101. 6. Racine E, Dion M-J, Wijman CAC, et al. Profiles of neurological outcome prediction among intensivists. Neurocrit Care. 2009;11:345-352. 7. Jauch EC, Saver JL, Adams HP, et al. Guidelines for the early management of patients with acute ischemic stroke. A guideline for healthcare professionals from the American Heart Association/ American Stroke Association. Stroke. 2013;44:870-947. 8. Seder DB, Mayer SA. Critical care management of subarachnoid hemorrhage and ischemic stroke. Clin Chest Med. 2009;30:103-122. 9. Mayer SA, Copeland D, Bernardini GL, et al. Cost and outcome of mechanical ventilation for life-threatening stroke Stroke. 2000;31:23462353. 10. Kernan WN, Viscoli CM, Brass LM, et al. The stroke prognosis instrument II (SPI-II): a clinical prediction instrument for patients with transient ischemica and nondisabling ischemic stroke. Stroke. 2000;31:456-462. 11. Saposnik G, Guzik A, Reeves M, et al. Stroke prognostication using age and NIH stroke scale. Neurology. 2013;80:21-28. 12. Vora NA, Shook SJ, Schumacher HC, et al. A 5-item scale to predict stroke outcome after cortical middle cerebral artery territory infarction: validation from results of the Diffusion and Perfusion Imaging Evaluation for Understanding Stroke Evolution (DEFUSE) study. Stroke. 2011; 42:645649. 13. Jauch EC, Lindsell C, Broderick J, et al. Association of serial biochemical markers with acute ischemic stroke: the National Institute of Neurological Disorders and Stroke recombinant tissue plasminogen activator Stroke Study. Stroke. 2006;37:2508-2513. 14. Missler U, Wiesmann M, Friedrich C,et al. S-100 protein and neuron-

www.medicalebookpdf.com

specific enolase concentrations in blood as indicators of infarction volume and prognosis in acute ischemic stroke. Stroke. 1997;28:1956-1960. 15. Hand PJ, Wardlaw JM, Rivers CS, et al. MR diffusion-weighted imaging and outcome prediction after ischemic stroke. Neurology. 2006;66:11591163. 16. Johnston KC, Wagner DP, Want XQ. Validation of an acute ischemic stroke model: Does diffusion-weighted imaging lesion volume offer a clinically significant improvement in prediction of outcome? Stroke. 2007;38:18201825. 17. Johnston K, Wagner DP, Haley C, et al. Combined clinical and imaging information as an early stroke outcome measure. Stroke. 2002;33:466-472. 18. Kreiger DW, Demchuk AM, Kasner SE, et al. Early clinical and radiological predictors of fatal brain swelling in ischemic stroke. Stroke. 1999;30:287. 19. Hacke W, Schwab S, Horn M. ‘Malignant' middle cerebral artery territory infarction: clinical course and prognostic signs. Arch Neurol. 1996;53:309. 20. Vahedi K, Hofmeijer J, Juettler E, et al. Early decompressive surgery in malignant infarction of the middle cerebral artery: a pooled analysis of three randomized controlled trials. Lancet Neurol. 2007;6:215-222. 21. Caplan LR, Wityk RJ, Glass TA. New England Medical Center Posterior Circulation Registry. Ann Neurol. 2004;56:389-398. 22. Claassen J, Kreiter KT, Kowalski RG. Effect of acute physiologic derangement on outcome after subarachnoid hemorrhage. Crit Care Med. 2004;32:832-838. 23. Bernardini GL, Mayer SA: Subarachnoid hemorrhage: clinical presentation and neuropsychological outcome. Medical Update for Psychiatrists. 1998;3:71-76. 24. Hunt WE, Hess RM. Surgical risk as related to time of intervention in the repair of intracranial aneurysms. Journal of Neurosurgery 1968; 28:14-20. 25. Mayer SA, Bernardini GL, Solomon RA. Subarachnoid hemorrhage. In: Rowland LP, Pedley TA, eds. Merritt’s Neurology. Philadepphia: Lippincot Williams & Wilkins; 2009: 308-317. 26. Fisher CM, Kistler JP, Davis JM. Relation of cerebral vasospasm to

www.medicalebookpdf.com

subarachnoid hemorrhage visualized by computerized tomographic scanning. Neurosurgery 1980;6:1-9. 27. Frontera JA, Claasen J, Schmidt JM, et al. Prediction of symptomatic vasospasm after subarachnoid hemorrhage: the modified fisher scale. Neurosurgery 2006;59:21-27. 28. Wartenberg KE, Schmidt JM, Claasen J, et al. Impact of medical complications on outcome after subarachnoid hemorrhage. Critical Care Medicine. 2006;34:617-623. 29. Samuels O, Webb A, Culler S, et al. Impact of a dedicated neurocritical care team in treating patients with aneurysmal subarachnoid hemorrhage. Neurocrit Care. 2011;14:334340. 30. Le Gall JR, Lemeshow S, Saulnier F. A new simplified acute physiology score (SAPS II) based on a European/North American multicenter study. JAMA. 1993;270:2957-2963. 31. Schuiling WJ, de Weerd AW, Dennesen PJW, et al. The simplified acute physiology score to predict outcome in patients with subarachnoid hemorrhage. Neurosurgery. 2005;57:230-236. 32. Claassen J, Hirsch LJ, Frontera JA, et al. Prognostic significance of continuous EEG monitoring in patients with poor-grade subarachnoid hemorrhage. Neurocrit Care. 2006;4:103-112. 33. Muroi C, Hugelshofer M, Seule M, et al. Correlation among systemic inflammatory parameter, occurrence of delayed neurological deficits, and outcome after aneurismal subarachnoid hemorrhage. Neurosurgery. 2013;72;367-375. 34. Tuhrim S, Dambrosia JM, Price TR et al. Intracerebral hemorrhage:external validation and extension of a model for prediction of 30-day survival. Ann Neurol. 1991;29:658-663. 35. Broderick JP, Bortt TG, Duldner JE, et al . Volume of intracerebral hemorrhage: a powerful and east-to-use predictor of 30-day mortality. Stroke. 1993;24:987-993. 36. Mendelow AD, Gregson BA, Fernandex HM, et al. Early surgery versus initial conservative treatment in patients with spontaneous supratentorial intracerebral haematomas in the International Surgical Trial in Intracerebral Haemaorrhage (STICH): a randomized trial. Lancet. 2005;365:387-397.

www.medicalebookpdf.com

37. Mendelow AD, Gregson BA, Rowan EN, et. al. Early surgery versus initial conservative treatment in patients with spontaneous supratentorial lobar intracerebral haematomas (STICH II): a randomised trial The Lancet. 2013;382:397 – 408. 38. Diringer MN, Edwards DF. Admission to a neurolgi/neurosurgical intensive care unit is associated with reduced mo9rtality rate after intracerebral hemorrhage. Crit Care Med. 2001;29:635640. 39. Becker KJ, Baxter AB, Cohen WA, et al. Withdrawal of support in intracerebral hemorrhage may lead to self-fulfilling prophecies. Neurology. 2001;56:766-772. 40. Zahuranec DB, Morgenstern LB, Sánchez BN. Do-not-resuscitate orders and predictive models after intracerebral hemorrhage. Neurology. 2010;75:626-633. 41. Hemphill JC III, Farrant M, Neill, TA Jr. Prospective validation of the ICH score for 12-month functional outcome. Neurology. 2009;73:1088-1094. 42. Barak B and Hemphill III JC. Charlson comorbidity index adjustment in intracerebral hemorrhage. Stroke. 2011;42:2944-2946. 43. Turgeon AR, Lauzier F, Burns KEA, et al. Determination of neurologic prognosis and clinical decision making in adult patients with severe traumatic brain injury: a survey of Canadian intensivists, neurosurgeons, and neurologists. Crit Care Med. 2013;41:1087-1093. 44. MacKenzie EJ, Rivara FP, Jorkovich GJ, et al. A national evaluation of the effect of trauma-center care on mortality. N Engl J Med. 2006;354:366-378. 45. Varelas PN, Eastwood D, Yun HJ, et al. Impact of a neurointensivist on outcomes in patients with head trauma treated in a neurosciences intensive care unit. J Neurosurg. 2006;104:713719. 46. Steyerberg EW, Msuhkudiani N, Perel P, et al. Predicting outcome after traumatic breain injury: development and international validation of prognostic socres based on admission characteristics. PLoS Med. 2008;5:e165;discussion e165. 47. Balestreri M, Czosnyka M, Chatfield DA, et al. Predictive value of Glasgow coma scale after brain trauma: change in treand over the past ten years. J Neurol Neurosurg Psychiatry. 2004;75:161162. 48. Stocchetti N, Pagan F, Calappi E, et al. Inaccurate early assessment of

www.medicalebookpdf.com

neurological severity in head injury. J Neurotrauma. 2004;21:1131-1140. 49. Stevens R and Sutter R. Prognosis in severe brain injury. Crit Care Med.2013;41:1104-1123. 50. Bateman BT, Claassen J, Willey JZ. Convulsive status epilepticus after ischemic stroke and intra-cerebral hemorrhage: frequency, predictors, and impact on outcome in a large administrative dataset. Neurocrit Care. 2007;7:187-193. 51. Legriel S, Azoulay E, Resche-Rigon M, et al. Functional outcome after convulsive status epilepticus. Crit Care Med. 2010;38:2295-2303. 52. Hocker SE, Britton JW, Mandrekar JN, et al. Predictors of outcome in refractory status epilepticus. JAMA Neurol. 2013;70:72-77. 53. Kilbride RD, Reynolds AS, Szaflarski JP, Hirsch LJ. Clinical outcome following prolonged refractory status eplilepticus. Neurocrit Care. 2013;18:374-385. 54. Rossetti AO, Oddo M, Liaudet L, Kaplan PW. Predictors of awakening from postanoxic status epilepticus after therapeutic hypothermia. Neurology. 2009;72:744-749. 55. Rea TD, Eisenberg MS, Becker LJ, et al. Temporal trends in sudden cardiac arrest: a 25-year emergency medical services perspective. Circulation. 2003;107:2780. 56. Kitamara T, Iwami T, Kawamura T, et al. Nationwide improvements in survival from out-of-hospital cardiac arrest in Japan. Circulation. 2012;126:2834. 57. Bunch TJ, White RD, Gersh BJ, et al. Outcomes and in-hospital treatment of out-of-hospital cardiac arrest patients resuscitated from ventricular fibrillation by early defibrillation. Mayo Clinic Proc. 2004;79:613. 58. Levy DE, Caronna JJ, Singer BH, et al. Predicting outcome form hypoxicischemic coma. JAMA. 1985;253:1420-1426. 59. Wijdicks EF, Hijdra A, Young GB, et al. Practice parameter: prediction of outcome in comatose survivors after cardiopulmonary resuscitation (an evidence-based review): report of the Quality Standards Subcommittee of the American Academy of Neurology. Neurology. 2006;67:203210. 60. Greer D, Scripko P, Bartscher J, Sims J, et al. Serial MRI changes in

www.medicalebookpdf.com

comatose cardiac arrest patients. Neurocrit Care. 2011; 14: 61-67. 61. Fugate JE, Rabinstein AA, Claasen DO. The FOUR Score predicts outcome in patients after cardiac arrest. Neurocrit Care. 2010;13:205-210. 62. Oddo M and Rossetti AO. Predicting neurological outcome after cardiac arrest. Curr Opin Crit Care. 2011;17:254-259. 63. Fugate JE, Moore SA, Knopman DS, et al. Cognitive outcomes of patients undergoing therapeutic hypothermia after cardiac arrest. Neurology. 2013; 81:40-45. 64. Bernat JL. Ethical aspects of determining and communicating prognosis in critical care. Neuro-crit Care. 2004;1:107-117. 65. Boissy AR, Ford PJ, Edgell RC, et al. Ethics consultations in stroke and neurological disease: a 7-year retrospective review. Neurocrit Care. 2008;9:394-399.

www.medicalebookpdf.com

PROGNOSTICATION IN THE NICU QUESTIONS 1. There is no difference in outcomes in neurologically injured patients treated in NICU versus general ICU. a. True b. False 2. Neurointensivists are skilled at predicting good outcomes for patients treated in the NICU. a. True b. False 3. The following biomarkers may be useful in helping to predict outcomes in patients in NICU except: a. IL-6 b. S-100B protein c. MMP-9 d. Neuron specific enolase (NSE) e. Oligoclonal bands 4. Which of the following risk factors is most important in predicting outcome after acute ischemic stroke? a. Age b. Admission NIHSS score c. Hyperglycemia d. Size of infarct e. Hemorrhagic conversion on head CT 5. Which of the following are risk factors for patients developing malignant MCA infarction?

www.medicalebookpdf.com

a. Proximal MCA occlusion b. Early hypodensity of ≥50% MCA territory on head CT c. Nausea and vomiting d. High NIHSS scores on presentation e. All of the above 6. Which of the following risk factors predict poor outcome in patients with SAH? a. Younger age b. PLEDS on EEG c. Posterior communicating (PCOM) artery aneurysms d. High Hunt and Hess score on initial exam e. B and D 7. In advising patients and/or families regarding outcome after ICH, which of the following factors is predictor(s) of poor outcome: a. Intraventricular extension b. Volume of hemorrhage ≤ 30 cc c. Infratentorial location d. A and C e. Age < 80 years 8. The following risk factors lead to poor prognosis in TBI except: a. Age b. Hypoxia c. Hypertension d. Head CT with SAH e. All of the above 9. Prolonged status epilepticus for more than 1 week predicts poor outcome:

www.medicalebookpdf.com

a. True b. False 10. In patients with post-anoxic brain injury after cardiac arrest, clinical finding(s) most helpful in predicting good recovery is: a. Pupillary reactivity day 1 b. Flexor motor movements day 3 c. Roving eye movements day 1 d. Ventricular fibrillation as initial rhythm e. All of the above 11. Even in those patients who survive cardiac arrest, cognitive outcomes are poor. a. True b. False 12. The key to making sound ethical decisions in the NICU include the following except: a. Informed consent b. Using one's own beliefs to help family make decisions c. Shared decision making d. Obtaining Ethics consult when faced with significant conflicting issues with families

www.medicalebookpdf.com

PROGNOSTICATION IN THE NICU ANSWERS 1. The correct answer is B (False). There is large amount of evidence to support that treatment in the NICU for neurologically injured patients significantly improves outcomes. 2. The correct answer is B (False). Overall, neurointensivists are reasonably good at predicting poor outcomes but less skilled at reliably predicting which patients will do well. 3. The correct answer is E. IL-6, S-100B protein, MMP-9 and NSE are all potential useful biomarkers in determining outcome; oligoclonal bands are useful in diagnosis of multiple sclerosis but serve no role as predictor of severity or outcome of disease. 4. The correct answer is B. Number one predictor of outcomes after acute ischemic stroke is initial clinical presentation. i.e., the neurological examination, as determined by the NIHSS score. 5. The correct answer is E. All of these items listed are predictors of malignant MCA infarction. 6. The correct answer is E. Poor initial Hunt and Hess score and findings of PLEDS on EEG in poor grade SAH patients are predictors of poor outcome. Location of aneurysm and younger age are not associated with determining outcomes. 7. The correct answer is D. Intraventricular extension, infratentorial location, large bleeds (ICH>60cc), age ≥ 80 are all poor outcome risks after ICH. 8. The correct answer is E. All of these factors predict poor outcome after TBI 9. The correct answer is False. There are more reported cases suggesting good or excellent outcome even after prolonged SE. 10. The correct answer is E. All of these findings are positive predictors of outcome after post-anoxic brain injury with cardiac arrest. On the contrary, lack of pupillary reflex at day 7 invariably predicts non-recovery. 11. The correct answer is B (False). In study cited, cognitive outcomes in those who survive cardiac arrest can actually be quite good, with many returning to jobs they held pre-arrest.

www.medicalebookpdf.com

12. The correct answer is B. Using one's own belief system can be flawed, lead to bias or ‘framing' of clinical scenario, and not accurately portray potential outcome to patient and/or family members. Use of informed consent, shared decision making, and ethics consults when needed are all ethically sound ways to approach patients and their families.

www.medicalebookpdf.com

Chapter 15

CLINICAL EVALUATION OF COMA AND BRAIN DEATH David Greer CLINICAL CASE A 62-year-old man with hypertension, diabetes, hyperlipidemia and a prior TIA suffers a cardiac arrest. His initial rhythm is ventricular tachycardia, and he is resuscitated with return of spontaneous circulation (ROSC) after 25 minutes. He remains comatose, and undergoes therapeutic hypothermia (TH) with a temperature maintained between 32-34 °C for 24 hours. He is subsequently rewarmed to normothermia over a 12-hour period. His neurological examination off all sedation after complete rewarming is notable for coma, with no eye opening to noxious stimulation. He has reactive pupillary, corneal and oculocephalic reflexes. He has extensor posturing to noxious stimuli. He has intermittent myoclonic jerking of the face, eyes and upper extremities. You are asked to guide the team through the clinical evaluation, and whether to consider brain death testing. OVERVIEW Coma and brain death are conditions neurointensivists are commonly called upon to evaluate, and the examination can be intimidating and challenging. Proper evaluation requires an understanding of the etiology and pathogenesis of the condition, and examination of the patient under optimal circumstances. Numerous pitfalls exist in the clinical examination, of which the examiner needs to be aware. The brain death examination can be viewed as the coma examination extended: not only must the patient be comatose, but also all brainstem functions must be absent, including respiratory drive as assessed through apnea testing. Herein we will review the specifics of coma and brain death clinical evaluations. DEFINITION Coma is strictly defined as a state of complete unresponsiveness, from which a patient cannot be aroused [1]. Eyes are typically closed, with a notable exception being rare patients post-cardiac arrest with “eyes open coma” but who are

www.medicalebookpdf.com

otherwise completely unresponsive, and who do not close their eyes in response to external stimulation. Cranial nerves and brainstem reflexes may be partially or fully intact, but there can be no purposeful responses to stimulation on the cranium or body. There can be no eye opening to vigorous auditory or tactile stimulation, no grimacing, no blink to visual threat, and no movement of the extremities or trunk reflective of purposeful responsiveness; reflexive or posturing movements are permissible in coma, as they represent brainstem or spinally-mediated responses and not a conscious or purposeful response. EPIDEMIOLOGY/PATHOPHYSIOLOGY Coma can occur due to traumatic or non-traumatic causes. The true incidence of coma is difficult to estimate, as many studies do not stay true to the standard definition presented above. The most common causes of traumatic coma are motor vehicle accidents, physical abuse and falls. Non-traumatic causes include toxic-metabolic insults, global hypoxia/ischemia, ischemic stroke, intracerebral hemorrhage, subarachnoid hemorrhage, brain tumors, CNS infections (meningitis, encephalitis, abscess), inflammatory conditions and psychogenic coma. Comatose patients are at high risk for morbidity and mortality; a rapid and systematic diagnostic work up to evaluate and potentially treat the underlying etiology is paramount. The ascending reticular activating system (ARAS) begins in the lower brainstem as an ill-defined group of nuclei and neurons that extends through the rostral brainstem, projecting into both cerebral hemispheres. Thus, in order to cause coma a process must either affect the brainstem primarily or both cerebral hemispheres concomitantly. Processes that affect both cerebral hemispheres and cause coma are typically diffuse metabolic or toxic insults, such as drug intoxication or severe electrolyte/metabolite abnormalities; other causes of diffuse hemispheric dysfunction include meningoencephalitis, hydrocephalus, multifocal cerebral insults (e.g. multiple large concomitant cerebral emboli) or cardiac or respiratory arrest. A unilateral hemispheric process does not commonly cause coma in the acute setting, unless it is associated with mass effect and compression of the brainstem; coma due to brain-stem compression from an ischemic stroke typically occurs during the time of peak edema, around 3-5 days after the ictus. Processes that primarily affect the brainstem are most commonly structural, such as infarction or hemorrhage. Findings on the coma examination are crucial in determining prognosis. For example, a unilateral dilating pupil, with or without outward/downward deviation of the eye, is indicative of compression of the IIIrd nerve due to an

www.medicalebookpdf.com

expanding hemispheric mass lesion, and portends a poor prognosis if not promptly treated. Classically, comatose patients following a cardiac arrest will have a poor prognosis if they have absent pupillary and corneal reflexes, as well as a motor response of extensor posturing or worse [2,3] However, more recent studies have drawn into question the validity of the motor response, especially in the setting of therapeutic hypothermia [4-6]. CLINICAL FEATURES/DIAGNOSIS The coma exam is straightforward and systematic, and can be separated into broad assessment steps: 1) consciousness, 2) cranial nerves, and 3) brainstem function. Although the examination can be intimidating to inexperienced clinicians, remember these primary principles: ensure a comfortable examination position for yourself, provide strong enough stimuli, and repeat aspects of the testing in which you are uncertain of the response. Minimize potential negative influences to the examination, most notably including sedation, hypothermia, or metabolic abnormalities, whenever possible. When uncertain how to describe a movement or response, rather than trying to force terms like “extensor” or “flexor” or “dyskinetic,” simply describe the response using plain language and as many words as are necessary to convey your thoughts. Taking video of abnormal movements can often be helpful, taking care not to include any identifiable information, or gaining written permission from the family/surrogate decision-maker. The examination begins upon entry to the room, with direct observation of the patient in the resting state. Most patients are ventilated, and careful attention to the respiratory pattern can give clues to brainstem function. Respiratory Patterns: non-volitional respiratory function is mediated primarily through the caudal brainstem and upper cervical spinal cord. Thus, dysfunction at different anatomical levels can cause specific respiratory patterns. However, there are often multiple forces at play, including not just anatomic but also metabolic influences, commonly clouding the picture. Figure 15-1 provides a visual depiction of the descriptions below. Cheyne-Stokes Respiration (CSR) – this pattern manifests as oscillation between periods of hyperventilation and relative apnea. Although common, it is a relatively nonspecific pattern, and can be seen in both cerebral and systemic (e.g. CHF, hypoxia) conditions. If patients cannot voluntarily breathe during the apneic phase or slow down the breathing during the tachypneic phase, the cause of CSR is most likely of cerebral origin.

www.medicalebookpdf.com

Centrally-mediated CSR is most commonly secondary to bilateral hemispheric dysfunction, sometimes extending to the diencephalon. Central Hyperventilation – this manifests as regular, rapid and deep breaths, often resulting in significant hypocapnea and alkalosis. Lesions are typically in the rostral brainstem in the upper pons/lower midbrain region. Central hyperventilation can rarely be seen in noncomatose patients with infiltrative tumors of this region. Apneustic Respiration – clinically this appears as pauses of up to 2-3 seconds at end-inspiration and sometimes end-expiration. It localizes to the mid- to caudal pontine region, most commonly caused by basilar artery occlusion. Ataxic Respiration - this is an irregular and unpredictable pattern of respiration, with both deep and shallow breaths. It is a most ominous sign, commonly seen with medullary lesions as a late manifestation of herniation, and precedes complete apnea. Assessment of Coma: The most common mistakes made by clinicians attempting to assess a comatose patient are 1) not adequately observing, and 2) not providing enough of a stimulus. Some general principles: Uncover the patient's extremities (maintaining decency at all times). Failure to uncover the arms and legs during the examination may preclude the ability to see some responsiveness to stimulation. Observations also should be made for spontaneous movements, including rhythmic movements that may be reflective of seizures, jerking movements that may reflect myoclonus, or dyskinetic movements. Use sufficient stimulation to try to assess consciousness. Start with auditory stimulation, yelling the patients name and telling them to open their eyes. Assume that the patient is deaf (especially if they are older or have previously received ototoxic drugs) until proven otherwise. Note that the command “open your eyes” may not be followed if the patient has an eye opening apraxia, and thus other commands should also be used, such as “stick out your tongue” and “wiggle your thumb” and “wiggle your toes.” If the patient does not respond to maximal auditory stimulation, tactile stimulation should be used. In a patient without a question of C-spine integrity, a vigorous shake from side-to-side can be a very potent stimulus to awaken patients (a technique passed down by Dr. C. Miller Fisher).

www.medicalebookpdf.com

Pressure should be applied to the supra-orbital ridge and temporomandibular joint, as quadriplegic patients may not respond to noxious stimulation below the cranium. A vigorous sternal rub can also be used, as well as deep nail bed pressure in the extremities. More proximal noxious stimulation may be used, and may be helpful when the nail bed pressure response is ambiguous. However, care must be taken to use this sparingly, as it can cause bruising and/ or skin tears.

Figure 15-1. Abnormal respiratory patterns associated with lesions in particular locations in the brain (from Plum F and Posner JB. The Diagnosis of Stupor and Coma. 3rd Ed. (1982) Figure 6 from p. 34, by permission of Oxford University Press, USA)

Upper Cranial Nerves The eye examination is most informative, and can be broken down into several parts. With the eyes held open (or, if already open in situations of “eyes open coma”), a blink to visual threat should be tested. Starting in the peripheral

www.medicalebookpdf.com

visual fields, a flat hand should be used to slowly approach the patient's eye, one at a time, taking care not to create a wind current (and thus unintentionally stimulate a corneal reflex). If there is no response to an approach from the periphery, a central approach should then be used. Each eye should be tested separately. The blink to visual threat should be tested prior to testing pupillary responses. Also with they eyes held open, the position of the eyes, spontaneous eye movements, and the presence of conjugance or dysconjugance should be noted. A flashlight shined directly 1-2 feet from the face should shine approximately equally in both pupils and allow better observation of spontaneous movements or dysconjugance. Horizontal dysconjugance is not uncommon in the setting of a depressed level of consciousness, and does not always reflect cranial nerve and/or brainstem dysfunction. Care should be taken to note the presence of a internuclear ophthalmoplegia, reflective of injury to the medial longitudinal fasciculus. Vertical dysconjugance, or a “skew” deviation, is nearly always pathological, however, and reflects dysfunction of the upper brainstem and/or IIIrd or IVth cranial nerves. With a IIIrd nerve palsy due to a compressive lesion (e.g. herniation syndrome), the affected eye is deviated laterally and inferiorly, and the pupil is dilated and unreactive. A nuclear IIIrd nerve palsy will not occur in isolation, and when due to a midline structural lesion will cause bilateral ptosis and upgaze pareses. Spontaneous eye movements toward one side may reflect damage to the frontal eye fields or paramedian pontine reticular formation (PPRF), or can be reflective of seizure activity. Upward or downward beating eye movements is often reflective of lower brainstem dysfunction. Retraction nystagmus involves spontaneous contraction of all extraocular muscles, and is seen with midbrain tegmental lesions. Roving eye movements are reflective of an intact brainstem to some extent, and cannot be produced voluntarily, thus ruling out psychogenic coma. “Ping-Pong” eye movements are repetitive horizontal movements of the eyes, with pauses of several seconds in the lateral positions, and is a variant of roving eye movements, sometimes seen with structural lesions of the cerebellar vermis. Ocular bobbing is a brief, rapid downward jerk of the eyes with a slower return to the midposition, classically localizing to the ponto-medullary junction. Eye movements may be tested with the oculocephalic reflex (“Dolls Eyes”, OCR) maneuver and oculovestibular reflex (“cold caloric”, OVR) testing.

www.medicalebookpdf.com

The OCR should only be tested when there is no question of C-spine integrity. The head is rapidly rotated horizontally, with a pause at the farthest most point. With an intact brainstem, they eyes should move conjugately in the opposite direction from which the head is turned, with a slow compensatory response toward the midline. Vertical eye movements should also be tested, with the same principle (eyes moving in the opposite direction of head movement). Extreme care must be taken in intubated patients to avoid accidentally extubating the patient or disconnecting the ventilator tubing. The OVR is tested with the following steps. First, the external auditory canal is examined and cleared of any cerumen/debris, and the integrity of the tympanic membrane is confirmed. Second, the head is positioned at 30 ° to ensure the proper orientation of the semicircular canals. Ice water is then instilled into one ear at a time for 60 seconds, and the eyes observed for movement. With an intact brainstem, the eyes will tonically deviate toward the cold-irrigated ear, sometimes accompanied by nystagmus with a fast component in the opposite direction (Figure 15-2). Vertical eye movements can be tested with instillation into both ears at the same time (causing conjugate downward eye movement), but this is rarely performed.

www.medicalebookpdf.com

Figure 15-2. Oculocephalic and oculovestibular reflexes. The top panel illustrates the movements with the brainstem intact. With the oculocephalic (OCR) reflex, the eyes move in the opposite direction from which the head is turned. With the oculovestibular reflex (OVR), the eyes will deviate toward the cold-irrigated ear. Please note that hot water is never used, and simultaneous stimulation of both ears, either with cold or warm water, is rarely performed. In brain death (lowest panel), there are no eye movements with any stimuli. (from Plum F and Posner JB. The Diagnosis of Stupor and Coma. 3rd Ed. (1982) Figure 12 from p. 55, by permission of Oxford University Press, USA)

The pupillary light reflex is tested with a bright light shone into one eye at a time. Note should be made of the direct and consensual response. A magnifying glass should be used if the pupils are small, or if there is a

www.medicalebookpdf.com

question of any response, such as in brain death testing (see below). Automated “pupillometers” are commercially available, but have not been rigorously compared with the bedside magnifying glass examination, and are quite expensive. An important clinical pearl is that toxic-metabolic disorders should not cause pupillary abnormalities (notable exceptions are hypothermia and high-dose barbiturates); thus, when pupillary dysfunction is present it is most likely due to a structural cause. A funduscopic examination should be performed in all patients – it is the one opportunity to look directly at nerves and vessels, and gives evidence of intracranial pressure phenomena. The presence of spontaneous venous pulsations essentially rules out the possibility of raised ICP. A unilateral miotic pupil is often reflective of loss of sympathetic innervation, such as with a Horner syndrome. Horner syndrome is often seen as a constellation also involving ipsilateral ptosis and sometimes anhydrosis, depending on the location of the lesion. Horner syndrome can be seen with lesions in the hypothalamus or lateral medulla. Damage to the midbrain may affect the IIIrd nerve and cause pupillary abnormalities. Lesions in this location often cause midposition pupils that are 4-6 mm, nonreactive to light. However, hippus may be present, and the ciliospinal reflex may be preserved. The ciliospinal reflex is ipsilateral pupillary dilation in with neck, face or upper trunk noxious stimulation, only present with an intact sympathetic pathway. Pontine pupils are small (often described as “pinpoint”), and reflect loss of sympathetic input. Lower Cranial Nerves Facial movement and sensation are tested via various techniques. Painful stimuli should be applied to the cranium to look for responsiveness, either as a facial grimace or with movement elsewhere in the body. A “nasal tickle” is performed by inserting a Q-tip into one nares at a time and shaking/twisting lightly. A corneal reflex is typically tested with brief instillation of sterile saline or water, but this may be an insufficient stimulus in some patients, and thus a more vigorous stimulus may be necessary. A cotton wisp may be applied to the cornea, or for even more vigorous stimulation, a Q-tip may be pressed to the cornea adjacent to the iris (usually reserved for brain death testing, see below).

www.medicalebookpdf.com

Hearing and vestibular testing is performed as above (auditory stimulation, OCR and OVR testing). Palatal movement is tested with stimulation to the posterior pharynx and looking for a gag reflex or palatal movement. This may be challenging in intubated patients, and light tugging on the endotracheal tube may be sufficient stimulation (taking care not to extubate the patient or push the tube into a main stem bronchus). If able, a tongue depressor, suction catheter or Q-tip can be pressed against the soft palate as well to try to elicit a response. A cough reflex is tested by deep bronchial suctioning. Motor and Sensory Examination Comatose patients require noxious stimulation to assess both sensory and motor responses. Tone should be checked first, performed as passive and unpredictable movement at multiple joints in the same limb simultaneously. Note should be made of increased or decreased tone, or of dyskinetic movements. Muscle bulk should also be noted, and any fasciculations or jerking movements. Pain is typically applied first to the nail beds, as this is least likely to cause injury. However, based on the response, further stimulation in other places may be necessary. Monitoring for facial (grimace) and head movements is necessary to ensure that there is not a corticospinal tract lesion that prevents a motor response but with a clear signal that the patient feels the stimulus. In general, one of three motor responses can be expected: purposeful, pathological, or none. Purposeful responses include localization or movement away from the stimulus and nonstereotyped withdrawal of the limb. Discerning whether a withdrawal response is pathological or not often requires stimulation in multiple places on the limb; a pathological response is stereotyped, nearly identical with noxious stimulation in multiple locations. Pathological responses include extensor and flexor posturing. The terms “decerebrate” and “decorticate” posturing, respectively, have been used synonymously with these movements, but these are often inaccurate and misleading. Although animal models have suggested that extensor posturing in the upper extremities reflects lesions at the level of the red nucleus in the midbrain, an anatomical correlation has not been demonstrated in humans. Nonetheless, these signs are often seen as part of a continuum, with extensor movements considered more ominous (and

www.medicalebookpdf.com

reflective of more extensive brainstem dysfunction) than flexor. Extensor posturing in the upper extremities manifests as extension at the elbow and hyper-pronation, with internal rotation at the shoulders. In the legs, extension occurs at the knee, with internal rotation at the hip and plantar flexion at the foot. Flexor posturing in the upper extremities appears as flexion of the elbow, wrist and fingers with adduction at the shoulders. The lower extremities may display “triple flexion” as a pathological response, with flexion at the hip, knee and ankle. Distinguishing triple flexion from purposeful withdrawal can be challenging, and mandates providing noxious stimulation at multiple locations in the lower extremity. A good rule of thumb is that a purposeful response will manifest as movement away from a noxious stimulation; thus, in a patient with triple flexion vs. purposeful withdrawal with nail bed pressure at the toe, painful stimulation at the medial thigh should produce abduction of the leg at the hip. It should be kept in mind that movements are not always easily categorized as flexion or extension; when in doubt simply provide a detailed description of the movement, so that others can see how the movements appear in comparison. Flaccid/absent motor responses may be reflective of injury anywhere along the neuroaxis, and may even reflect severe peripheral nervous system dysfunction (e.g. fulminant Guillian Barré Syndrome). Reflexes Deep tendon reflexes may always be checked, and are graded on a 0-4 scale: 0=absent, 1=hypoactive, 2=normal, 3=hyperactive without clonus, and 4=hyperactive with clonus. Deep tendon reflexes rating 3 or 4 are consistent with an upper motor neuron process. Note that a jaw jerk reflex can be tested as well, and may be helpful in distinguishing cervical spine lesions from processes causing generalized hyporeflexia. Pathological reflexes include the Babinski sign: the lateral aspect of the plantar aspect of the foot is slowly and steadily stroked in one long motion from the heel to the ball of the foot, and then medially across the ball of the foot. A pathological reflex (or “present” Babinski sign) involves two components: the great toe extends as the smaller toes fan out in abduction. A nonpathological response consists of plantar flexion of the great toe and a

www.medicalebookpdf.com

curling in adduction of the smaller toes. A Hoffman sign is produced by flicking the distal end of the middle or ring finger and observing ipsilateral flexion of the thumb. It is not necessarily a pathological sign, as it is often seen in patients who are normal and nonpathologically hyperreflexic. BRAIN DEATH Brain death is defined as the complete and irreversible loss of all brain function, including the brain stem. The American Academy of Neurology Practice Parameters for brain death determination were updated in 2010 [6]. There are 3 cardinal components: coma, brain stem areflexia and apnea. The physical examination includes the techniques described above for the coma examination, but with specific mandatory findings, which will be described below. The evaluation can be broken down into 4 discrete steps: satisfying prerequisites, the clinical examination (including apnea testing), ancillary testing (if necessary) and documentation. A discussion of ancillary testing is beyond the focus of this chapter, and what ensues is a detailed explanation of the clinical evaluation. Prerequisites The cause of the neurological state must be known, and must be known to be irreversible. This is the “do not pass/go” point of the evaluation – if there is any doubt about the diagnosis or reversibility, specific brain death testing should not be undertaken. The cause is typically gleaned through the history and neuroimaging. Occasionally, laboratory testing (such as CSF examination in fulminant bacterial endocarditis) may be used. Drug effect must be excluded. If it is unknown whether the patient has received potential CNS-acting drugs, a toxicology screen must be performed. If medications have been used to treat the patient (e.g. narcotics, benzodiazepines or barbiturates), clearance must be determined by calculating 5 times the drug's half-life, assuming normal hepatic and renal function. It should be kept in mind that hypothermia (including therapeutic hypothermia as used in ICP crisis and cardiac arrest) will delay drug metabolism and neuronal recovery. If a patient has received neuromuscular blocking agents, their continued presence must be excluded by implementation of electrical nerve stimulation. Severe electrolyte, acid-base, or endocrine abnormalities and

www.medicalebookpdf.com

hyperammonemia must be excluded. Normothermia must be achieved, defined as a core temperature > 36 °C. Sufficient blood pressure must be established, with a lowest systolic limit of 100 mm Hg. Clinical Examination Coma – as defined above, with no purposeful response to any and all noxious stimulation. Only spinally-mediated responses are permissible. Pupillary reflexes must be absent. A bright light (and optimally a magnifying glass) should be used. Pupils should be 4-9 mm and are typically midposition. Smaller pupils should suggest the possibility of a medication effect (e.g. narcotics). Ocular movements should be completely absent with OCR and OVR testing. With cold-caloric testing, both ears must be tested separately, with an interval of at least 5 minutes between ears. Absent corneal reflex (with maximal stimulation, as above). Absent facial movement to noxious stimulation in the body and cranium. Facial myokymias may be observed, but should be spontaneous and not in response to stimulation. Cough and gag reflexes must be absent with palatal stimulation and deep bronchial suctioning, respectively. Movement in the trunk and extremities must be absent, other than spinallymediated; these include deep tendon reflexes, Babinski sign, triple flexion and a constellation of features called “Lazarus signs.” Distinguishing these movements as spinally-mediated requires expertise, and sometimes ancillary testing when in doubt. Extensor or flexor posturing in the upper extremities is reflective of a brain stem mediated response, and is not permissible in brain death. Apnea Testing Establish euvolemia – patients with brain death often have diabetes insipidus, and a negative fluid balance will make the patient more prone to hypotension during apnea testing, which may require that the test be aborted.

www.medicalebookpdf.com

Establish eucapnea – the goal pCO2 should be 35-45 mm Hg. In known CO2 retainers, establish the patient's baseline pCO2, if known. Pre-oxygenate the patient for at least 10 minutes with 100% FiO2 to achieve a PO2 > 200 mm Hg. At no point during apnea testing should the patient become hypoxic, as this may lead to hemodynamic compromise and even cardiac arrest. Reduce the PEEP to 5 cm H20. If this results in significant oxygen desaturation, it is an indication that the patient may be unlikely to complete apnea testing. Disconnect the patient from the ventilator, providing a constant source of oxygen via a catheter advanced to the level of the carina. Closely observe the patient for respiratory movements of the chest and abdomen, which should be uncovered. An optimal place for the clinician to stand is at the foot of the bed, where they can observe both the chest and abdomen of the patient as well as the monitor, watching for hypotension, arrhythmias or desaturation. Abort the test if the patient becomes hypotensive, defined as an SBP 60 mm Hg, or >20 mm Hg above the baseline value, and the patient displayed no respiratory effort, the apnea test is positive and the patient is brain dead. If the pCO2 is -20 mmHg, some of these individuals can be safely managed using noninvasive ventilation. Newer noninvasive ventilators with alarms, mask-leak compensation software, and improved orofacial interfaces allow for successful noninvasive ventilation of many patients that previously required intubation, so intubation criteria for patients with acute neuromuscular weakness should take into account disease physiology and the rate of worsening. Many myasthenics in crisis can be managed with closely monitored NPPV, either continuously or at intervals, because their condition often improves rapidly with treatment. Conversely, Guillain-Barre patients or those with acute myelopathy are difficult to manage noninvasively, and frequently progress to require intubation and often tracheostomy. Many patients with chronic neuromuscular weakness that develops over a prolonged period have been safely managed using innovative ventilator strategies for years or even decades. Utilizing a combination of nocturnal NPPV

www.medicalebookpdf.com

with a mask, an aggressive regimen of airway clearance including cough assist, and daytime intermittent ventilation through an angled mouthpiece, one group has reported astounding success at keeping patients with certain forms of chronic neuromuscular weakness such as Duchenne muscular dystrophy and ALS adequately ventilated without tracheostomy and continuous mechanical ventilation. These patients have better quality of life and fewer hospitalizations for pneumonia than similar patients who undergo early tracheostomy [17,18]. CENTRAL AND PERIPHERAL CAUSES OF VENTILATORY FAILURE IN NEUROLOGICAL DISEASE Central ventilatory failure All sorts of severe damage of the central nervous system, i.e. supra- and infratentorial brain or spinal cord lesions due to traumatic, vascular, infectious/inflammatory, metabolic, neoplastic or seizure-related disorders, can cause respiratory failure. The complex connections between the cortical (volitional) respiratory centers and the autonomic centers in pons and medulla, as well as their connections to the phrenic nerve and the upper motor neurons can be affected at every level. Injuries do not only result in changes to the respiratory rate or rhythm, but also dramatically affect airway protective reflexes and airway patency and thus impair ventilation (Table 18-1). Specific patterns of pathologic breathing (e.g. Cheyne-Stokes, Cluster, Biot) have been suggested for topographic diagnosis of lesion level. However, the correlation is not entirely reliable and also many patients with such impaired breathing arrive at the ER/NICU intubated and ventilated thus not allowing for breathing pattern recognition. Peripheral ventilatory failure The connections (phrenic nerve, lower motor neurons) to the respiratory muscles, i.e. diaphragm (80% of ventilatory force), intercostal and accessory muscles, can be damaged by inflammatory, toxic or degenerative disorders (Table 18-2). Inflammatory neuromuscular diseases and myopathies are other causes of peripheral ventilatory failure. It has to be kept in mind that central ventilator drive is intact and only the efferent part of the system is compromised. This has implications for the type and setting of mechanical ventilation, as well as for weaning strategies.

www.medicalebookpdf.com

GENERAL PRINCIPLES OF NEUROCRITICAL CARE AIRWAY MANAGEMENT General principles for airway management in non-neurological ICU patients [19,20] and in the neuro-critically ill [21] have been presented in various

www.medicalebookpdf.com

reviews. Preintubation neurological assessment Especially in the Emergency Department, the intubating physician or provider is likely to be the last person to assess the patient prior to administration of analgesic and sedative medications. This is a grave responsibility, as many critical treatment decisions follow from the neurological findings, and it is incumbent upon the intubator to spend a few minutes, often while preparations for intubation are underway, to rapidly assess and document the neurological examination. Although the neurological examination should never delay an emergency intubation, a routine pre-intubation neurological assessment should never be omitted in a neurologically ill patient when it is reasonable to do so. In such patients, the following assessment is considered acceptable, and should take only 2-3 minutes to perform: 1. General level of arousal and interaction, and the presence or absence of cortical findings such as aphasia, neglect, gaze preference, or visual field deficits 2. Cranial nerve reflexes 3. Motor examination in each extremity and the face 4. Sensory findings in each extremity and sensory level, in the case of suspected or known spinal cord injury 5. Reflexes in each extremity 6. Motor tone 7. The presence or absence of subtle or overt convusions Airway assessment, and consideration of the difficult airway Because of the high risk involved, every airway should be regarded as potentially difficult in the ICU setting, and the situation ”can't ventilate, can't intubate“ anticipated, avoided, or adequately managed. The reader is directed to reviews on this particular topic [22], including the recommendations of the American Society of Anesthesiologists [23] and the Difficult Airway Society [24,25] that contain helpful algorithms. These principles will not be repeated here in detail but their core elements will be empasized. As the guidelines primarily describe operation room (OR) or emergency department (ED) management, they require adaptation to fit the ICU scenario. Airway management in the ICU or the emergency room (ER) is different from the more

www.medicalebookpdf.com

elective OR situation. Twenty percent of all critical incidents in the ICU are airway-related [22]; difficult intubation is encountered in the non-OR setting in about 10% (about twice as often as in the OR setting) [20]. Complications associated with difficult airway management include hypoxemia, hypotension, esophageal intubation, aspiration, cardiac arrest and death, reported at rates between 5 and 40% [20]. In a prospective registry for England and Wales between 2005 and 2007, more than 1000 airway incidents in the ICU were reported, 18% at intubation, 5% during tracheostomy placement and 82% as postprocedural problems [26]. Implementations of difficult airway algorithms have been shown to half the number of intubation-related cardiac arrest [27] and substantially reduce the number of overall complications [28]. The importance of adequate preparation for intubation cannot be overstated. Multiple factors can make routine airway management in the ICU or ED challenging (Tables 18-3 and 18-4).

Five criteria are associated with difficult mask ventilation (DMV) in the OR setting (>2 criteria): age > 55 years, body mass index >26 kg/m2, presence of

www.medicalebookpdf.com

beard, lack of teeth, history of snoring [29]. These can be memorized by the acronym OBESE (Obese, Bearded, Elderly, Snoring history, Edentulous). Although many scores and classifications (i.e. Mallampati, modified Mallampati, Cormack and Lehane, sterno-mental distance, etc.) and the combination of these have been suggested for prediction of the difficult airway, most lack support from confirmatory studies and were insufficiently investigated in the ICU/ ED situation. More helpful in the latter time-pressing situation and validated at least in the emergency setting are easy-to-use scores such as LEMON (3 Look criteria, 3 Evaluate criteria, Mallampati score, airway Obstruction and Neck mobility) [30] or simple maneuvers such as putting three fingers into the mouth and between chin and thyroid bone of the patient. Despite these commonly employed tools, prediction of the difficult airway remains unreliable, and it is important to be prepared, with special equipment and expertise available on short notice (Table 18-5). Two patient groups deserving special awareness and anticipation of airway difficulties are trauma patients with potential facial/pharyngeal injuries or spine fractures (manual and device-supported inline neck stabilisation necessary at intubation) and morbidly obese patients [19]. In both, fiberscopic airway management might be warranted.

Initiating airway protection In Critical Care, it may be necessary to intubate a patient as soon as the option has been considered. Criteria or indications for establishing an airway are shown in Table 18-6. Means of airway protection A multitude of means, techniques and devices for “securing” an airway exist, the choice of which depends on the severity of the situation, the setting, the skills and experience of the team, and patient factors [19,20,22]. Most importantly, the

www.medicalebookpdf.com

techniques by which the airway is secured or established need to be practiced under supervision of an experienced airway manager, and the care team to familiarize itself with airway adjunct devices present in their practice environment (Tables 18-7 and 18-8). Some traditional customs of airway management have recently been questioned. The so called ”sniffing position“ that is meant to align oral, pharyngeal, and laryngeal axes did not appear to be superior to simple head extension in magnetic resonance imaging (MRI) [31] and clinical randomized studies [32]. The sniffing position might have advantages in obese and neck-fixed patients, but optimal positioning has not been clarified for ICU airway management. It is sometimes helpful to remove the board from the head of the bed, put the head of the bed in a ramped (reverse Trendelenberg) position and place support under the occiput of the slightly extended and tilted head. A better view can sometimes be obtained by lifting and moving the head with the right hand while the left manipulates the laryngoscope.

Rapid sequence intubation (RSI) that is traditionally recommended for

www.medicalebookpdf.com

emergency intubation of a patient with a full stomach and involves cricoid pressure to occlude the esophagus (Sellick maneuver) and simultaneous application of the fast-acting muscular blocker succinylcholine (i.e. rendering the patient completely apneic) is also controversial. Contrary to cadaver studies, more recent MRI studies showed that the esophagus is lateral to the larynx in > 50% of cases and left open by cricoid pressure, while the airway is compressed. In terms of cord visualization, cricoid pressure, as well as the other traditional maneuver BURP (thyroid cartilage Back, Upward, Right Pressure) were inferior to bimanual laryngoscopy where the free hand of the intubator (ideally leading the hand of a helper that then remains in place) moves the cords under vision into the perfect place. Overall, there seems to be little evidence that cricoid pressure reduces aspiration during intubation [19]. The drugs commonly used for airway management in the ICU are listed in Table 18-9. Video laryngoscopy has dramatically improved urgent airway management because the camera lens in the distal part of the intubating blade provides an improved view of anterior airways, and eliminates some of the difficulty of poor mouth opening. Used in conjunction with an angled intubating stylet (“Bougie” or “Eschmann stylet”) video laryngoscopy has made accessible to moderately experienced clinicians many airways previously accessible only to experts. Video laryngoscopy also improves the safety of the training environment, as supervising clinicians can see what trainees are doing on the video screen [33].

www.medicalebookpdf.com

Among the many airway skills, high quality non-invasive mask ventilation is the most important. Most patients with respiratory failure can be managed safely for a prolonged time by mask ventilation, and an inexpert intubator can buy time until help arrives and a definitive airway solution can be provided. Nonetheless, many critically ill patients will at one point require endotracheal intubation, so this technique has to be mastered. At least one supraglottic airway device should also be available and familiar to the operator, in case intubation fails (for instance a laryngeal mask airway (LMA) has proved helpful [34]) – if an intubating LMA is utilized, it can later be used as a conduit for endotracheal intubation. Laryngoscope blades for tube loading, augmented by video/fiberscopic or patented lens systems can also be helpful, but have not been evaluated systematically in the ICU setting [19]. Retrograde intubation is another somewhat more complex and less often applied alternative, if antegrade

www.medicalebookpdf.com

intubation fails. Fiberoptic intubation is typically employed in special situations such as anatomical obstructions of the airway, neck injuries, or awake and cooperative patients scheduled for a nasal intubation. It is also a useful technique when the cervical spine is compromised.

www.medicalebookpdf.com

Cricothyroidotomy is the preferred emergency surgical airway strategy, and is

www.medicalebookpdf.com

preferable to tracheostomy in emergency circumstances because of the proximity of the cricothyroid membrane to the skin, and higher likelihood of rapid and successful access. Tracheostomy is sometimes urgently performed when surgeons with extensive tracheostomy experience prefer a familiar and rapid procedure in an emergency to one they rarely perform. A note on emergency airway management Even in an emergency, no clinican should go into an intubation without a well-conceptualized backup plan, in case a straightforward intubation turns out to be a difficult intubation or difficult bag-mask ventilation case. Examples of such backup plans include application of an appropriate-sized laryngeal mask airway, fiberoptic intubation, equipment close at hand for a surgical airway, or other special equipment with which the intubator has experience and expertise . When a difficult airway is predicted or identified, especially in the setting of hypoxia or difficult bag-mask ventilation, a highly expert intubator should be present or immediately available.

Discontinuing airway protection Discontinuing airway protection after re-establishment of spontaneous breathing requires that the patient has regained some airway protective reflexes [35], has an adequate cough and minimal respiratory secretions [36,37]. Extubation should be performed as soon as safely possible, but timing can be difficult to predict in the ICU, particularly if the patient had presented a difficult airway before. Between 5-10% of extubated ICU patients require re-intubation [19,21], and the percentage among patients with neurological injury may be higher, because of the difficulty in predicting airway protective reflexes in an intubated patient. Reasons for extubation failure include abnormal airway reflexes, prolonged effects of analgesics and sedatives, reduced pharyngeal tone,

www.medicalebookpdf.com

occluded airway (blood, secretions etc.), reduced oxygen stores, laryngeal edema, respiratory exhaustion, inadequately treated heart failure or COPD, etc. Both re-intubation after failed extubation and delayed extubation are associated with a longer ICU-length of stay (ICU-LOS), more infections, and higher mortality [36]. Removal of the orotracheal tube or the tracheostomy cannula to allow spontaneous breathing and airway protection requires a successful weaning process and spontaneous breathing trials (Table 18-10), and a back-up strategy for re-intubation. A positive cuff-leak test, i.e. the absence of air leak on deflation of the tube cuff, can indicate laryngeal edema and subsequent extubation failure [38,39], although this criterion for extubation is highly controversial. It is probably a useful additional criterium to guide the extubation decision. Laryngeal edema has been subjected to pre-extubation treatment with steroids. After decades of controversy on this practice, a recent systematic review and Cochrane evidence analysis confirmed that short-term prophylactic corticosteroids reduce extubation failure in the adult critically ill [40,41]. The optimal timing of tracheostomy is still unclear [42], but it is customary in many ICUs to assess the option to discontinue airway protection at the end of the first week of ventilation and to proceed to tracheostomy if this appears unlikely for the following week. The decision to tracheostomize might also follow failed extubation trials. Tracheostomy Tracheostomy is a valuable ventilation weaning procedure required by 10% of all ICU patients. Percutaneous dilational tracheostomy (PDT) has been shown in several studies to be equal to if not advantageous over the surgical technique and can be provided by the intensivist quickly, safely and at low cost at the bedside [43-46]. Early problems with tracheostomy (bleeding, misplacement) arise in about 3%, and late problems (tracheoinnominate fistula, cannula occlusion, tracheal stenosis) in 1% of patients. Although generally safe and more secure than orotracheal tubes, inadvertent dislodgement can be dangerous, and particular care has to be taken when changing tracheostomy tubes, as these situations can be life-threatening, especially if the upper airway is compromised, and intubation cannot be successfully performed [20].

www.medicalebookpdf.com

GENERAL PRINCIPLES OF ICU MECHANICAL VENTILATION General recommendations for ICU mechanical ventilation have been presented in several very helpful reviews [47-49]. Basic principles of modern ICU ventilation are as follows. Basic principles of ICU ventilation 1. Although live-saving and indispensable for the majority of ICU patients, positive pressure mechanical ventilation is not physiological and carries risks, such as barotrauma, volutrauma, and atelectrauma to the lung, ventilation-associated pneumonia, atrophy of respiratory muscles, ventilator dyssynchrony leading to increased work of breathing, and inducing stress and agitation in the patient. Its duration should thus be kept as short as possible. The option to discontinue mechanical ventilation has to be evaluated every day [35]. 2. Except in unusual circumstances (e.g. brain herniation), the aim of mechanical ventilation is maintenance of physiological homeostasis, including adequate but not excessive oxygenation, maintenance of normal pH and pCO2, and decreased work of breathing, without inducing lung injury or metabolic stress. In patients at risk, lung-protective ventilation, i.e. applying low tidal volumes (6 ml/kg IBW) and limited end-inspiratory plateau pressure (< 30 cm H2O), should be adopted whenever possible. This strategy was established to decrease mortality in the first ARDSNet low tidal-volume study [50] and has been the standard of care since then. In patients with elevated lung compliance or elevated ICP, however, “permissive hypercapnea” and respiratory acidosis are NOT recommended, due to the detrimental effects on cerebral vascular tone and ICP [21].

www.medicalebookpdf.com

3. ALI and ARDS are syndromes that identify patients at risk of ventilatorinduced lung injury, and demand special methods of ventilation. The 'openlung“ concept involves low tidal volumes, limited plateau pressure, high levels of positive end-expiratory pressure (PEEP) and recruitment maneuvers [51] although the latter were not confirmed to be clearly beneficial in a recent Cochrane analysis [52]. 4. No particular mode of ventilation has been proved superior in studies on patient outcomes. However, being able to choose from different modes can be helpful in addressing the individual patient's ventilation needs. Whenever clinically advisable, it is beneficial to let the patient participate actively in the ventilation process, i.e. to establish an assisted (as opposed to a fully-controlled) ventilation mode as soon as possible. The reasons are that respiratory muscle atrophy and critical illness myopathy/polyneuropathy (CIM/CIP) can start to develop within the first days of ventilation [53], and fully-controlled ventilation requires more sedation and at times neuromuscular blockade, confounding the neurological evaluation. 5. Non-invasive ventilation can help avoid endotracheal intubation, but is largely reserved for cooperative patients with respiratory compromise caused by exacerbated COPD, asthma, my-asthenic crisis, or cardiogenic pulmonary edema. It can be useful to support patients with early myasthenic crisis. It may also serve to facilitate liberation from mechanical ventilation, or assist with stabilization of spontaneous breathing after extubation [54]. In principle, invasive mechanical ventilation can be pressure-cycled (pre-set inspiratory pressure, varying tidal volume according to compliance of lung and thorax, more common in Europe) or volume-cycled (pre-set tidal volume, varying pressures according to compliance of lung and thorax, more common in the United States). It is not clear that one mode is superior, although each may have benfits in certain populations. Among those used most often in ICUs are synchronized intermittent mandatory ventilation (SIMV) and Pressure support ventilation (PSV) [55]. In practice, these modes are frequently combined – for example, volume cycled ventilation is often set to be pressure limited, meaning that although a certain volume is targeted, pressures above a certain level are not tolerated, and under such circumstances the tidal volume goals are not achieved. A selection of ventilation modes are summarized in Table 18-11 and more detailed information can be found in dedicated monographs [56-58].

www.medicalebookpdf.com

Common lung-protective settings of the ventilator in the ICU are listed in Table 18-12, they can be approached in a step-wise de-escalative fashion [56, www.ardsnet.org ] and have to be adapted according to results of regular blood gas analysis (aims SatO2 > 95%, PaO2 < 60 mmHg, PaCO2 35-45 mmHg) and clinical criteria (comfortable patient-ventilator interaction, no agitation, no signs of excessive work of breathing). Applying low tidal volume-cycled ventilation may result in reduced elimination of CO2, resulting in hypercapnia and respiratory acidosis. While this is accepted in the situation of ARDS as the concept of 'permissive hypercapnia“, it may be problematic in brain-injured patients. Monitoring proximal airway pressure during volume-cycled ventilation can help to recognize causes of acute respiratory deterioration. While the peak inspiratory pressure is unchanged in pulmonary embolism or extrathoracic processes, and decreased in hyperventilation, its increase signals either airway obstruction if plateau pressure is unchanged, or decreased compliance if plateau pressure is increased [56]. When utilizing pressure control ventilation, a patient's tidal volume and minute ventilation depend directly on lung compliance. A patient with pneumothorax or acute pulmonary edema may trigger alarms for “low minute ventilation” – because of decreasing compliance, the same pressure generates a small tidal volume, and minute ventilation declines precipitously.

www.medicalebookpdf.com

www.medicalebookpdf.com

Instrinsic or auto-PEEP develops because of air trapping in the alveoli from certain underlying lung diseases, in association with some ventilation modes, or simply in the combination of high inflation volumes and rapid respiratory rates. Acute increases in auto-PEEP are detected by a sudden rise in proximal airway pressure on occlusion of the tracheal tube at the end of expiraton (or observing the flow curve not returning to baseline after expiration). Although counterintuitive, application of external PEEP can reduce intrinsic PEEP by its stenting effect on small airways that facilitate alveolar emptying. High inflation volumes should be avoided and enough expiration time allowed. Certain special situations, complications, and challenges of mechanical ventilation are listed in Table 18-13.

Weaning Liberating the patient from mechanical ventilation (weaning) can be very challenging, especially in patients with underlying pulmonary disease and after prolonged ventilation. [35,60-63] The weaning period is often exhausting for the patient both physically and mentally, and is associated with a high incidence of delirium. There is no optimal method of weaning. Patients can be put on an assisted ventilation mode and ventilator support gradually reduced (continuous mode of weaning); or mechanical ventilation can be interrupted by periods of spontaneous breathing, and the intervals extended over time (discontinuous mode of weaning). Randomized trials support both gradual reduction in ventilator support [64] and spontaneous breathing trials [65]; in both trials the (S)IMV mode was least helpful. The question of an optimal weaning mode may require testing in physiological subgroups of patients [66]. The application of weaning protocols has consistently produced superior results compared to unsystematic weaning in different subgroups of ICU patients. A particularly effective adjunctive method in difficult ventilator weaning seems to be non-

www.medicalebookpdf.com

invasive ventilation (NIV). Predictors of a successful weaning include a Rapid Shallow Breathing Index (RSBI, RR/Vt) < 105, maximal inspiratory pressure (MIP) > -20 cmH2O and minute volume (Ve) < 10L [67], and a successful spontaneous breathing trial (SBT) [63]. Naturally, the weaning process has to go hand-in-hand with de-escalation of sedation, ideally following a sedation protocol. Steps that can help to successfully wean a patient from the ventilator are summarized in Table 18-14.

www.medicalebookpdf.com

AIRWAY MANAGEMENT IN THE NEUROCRITICALLY ILL

www.medicalebookpdf.com

Many patients with central respiratory failure do not have a primary problem with lung mechanics or gas exchange, but fail due to disordered respiratory control and inability to maintain a patent airway reliably isolated from the digenstive tract. Many brain-injured patients have decreased volitional and reflex clearing of the airways, upper airway obstruction, disordered swallowing apparatus, loss of pharyngeal and glossal sensation, and dysphagia – a dangerous series of disorders that often results in aspiration pneumonitis, airway obstruction, or pneumonia. Peripheral nerve disorders, i.e. neuronal or neuromuscular disease such as Guillain-Barré Syndrome (GBS), amyotrophic Lateral Sclerosis (ALS) or mysthenia gravis (MG) crisis, can cause severe impairment of lung mechanics but may also cause airway compromise by way of reduced capacity to cough, swallow, and close the airway during swallowing – impairing severely the ability to clear saliva, vomitus, or secretions. Non-invasive airway support The airway should be supported by advantageous head positioning, frequent suctioning, application of nasal prongs or an oronasal mask for O2 administration, and possibly the insertion of oro/nasopharyngeal airways – these principles apply to almost all neurological patients with respiratory compromise. These measures may be sufficient to maintain an open and safe airway in some patients with decreased levels of arousal, while others immediately fail and require placement of an invasive airway. Airway patency must be confirmed repeatedly, to verify whether patients remain arousable, cooperative, not respiratorily exhausted, and still have adequate protective reflexes. Otherwise, endotracheal intubation is warranted. Respiratory failure can develop quite rapidly in neuromuscular patients that have appeared stable over long periods, and must be anticipated.

www.medicalebookpdf.com

www.medicalebookpdf.com

Patients with impaired respiratory muscle strength and cough (such as those with myasthenia gravis or muscular dystrophy), should receive aggressive airway clearance techniques, including frequent suctioning, frequent use of cough assist devices, intermittent noninvasive positive pressure ventilation to decrease atelectasis and maintain open basilar lung segments, and advantangeous body positioning. The aggressive and regular application of such measures may prevent intubation in many neuromuscular patients if applied before respiratory muscle exhaustion and pneumonia develop. Intubation The general criteria for intubation listed in Table 18-6 are guidelines, and must be applied using common sense and a broader understanding of the

www.medicalebookpdf.com

patient's anticipated disease course. Neurological patients in a period of decline often require intubation to prevent respiratory arrest, large-volume aspiration, or secondary brain injury due to hypoxia or hypoventilation. Conversely, a stable or recovering patient may sometimes be safely extubated despite impaired airway reflexes or decreased level of arousal. Because of relative volume depletion, the use of vasodilator agents, and impaired venous return due to the institution of positive pressure ventilation, intubation always involves the risk of inducing hypotension or rapid changes in blood pressure and cerebral perfusion. Many neurocritically ill patients, such as those with acute ischemic stroke, depend on steady blood pressure, and cerebral autoregulation is often impaired after brain injury, so that systemic hypotension may result in critically decreased cerebral perfusion pressure (CPP). Conversely, sympathetic surges triggered by discomfort, agitation, or anxiety during intubation may cause tremendous increases in blood pressure, leading to rerupture of aneurysm in SAH, hematoma expansion in ICH, or hemorrhagic transformation in ischemic stroke. Maintaining hemodynamic homeostasis during intubation is therefore a top priority in neurocritical care [68]. Hypotension during pharmacological induction for intubation is more likely in patients with severe underlying disease, a baseline MAP < 70 mmHg, age > 50 years, and with use of propofol or high doses of fentanyl as induction drugs [69]. Thiopental is another agent that causes hypotension. Etomidate is often appropriate for induction of cerebrovascular patients due to its decreased tendency to cause vasodilation, though reports of etomidate lowering the seizure threshold make it a less ppealing induction agent in status epilepticus or when the risk of seizures is high.[70-72] Ketamine does not cause decreased blood pressure, and prior reports of increases in intracranial pressure (ICP), appear to be unfounded. Rapid sequence induction (RSI) is commonly recommended to prevent vomiting and aspiration in non-fasting patients. RSI calls for the administration of a short-acting neuromuscular blocker agent. Succinylcholine provides excellent intubation conditions (i.e. complete oropharyngeal and respiratory muscle relaxation and an open glottis), but intensivists should consider that succinylcholine can induce small, but at times relevant (e.g. in traumatic brain injury) ICP increases, and can cause rhabdomyolysis and hyperkalemia in patients with seizures, neuromuscular disease, or following prolonged immobilization. As the non-depolarizing muscular blocker rocuronium has been found comparable to succinylcholine in a recent Cochrane analysis of almost 40 good-quality studies on the subject [73], the authors recommend rocuronium as an alternative to succinylcholine, and suggest that neuromuscular blockers can

www.medicalebookpdf.com

sometimes be safely avoided using adequate muscle relaxants, analgesics, and sedatives with amnestic properties. Other indications for neuromuscular blockade include invasive procedures (tracheostomy), states of severe muscular overactivity, inabilty to otherwise ventilate the patient, and early ARDS [74]. Intubation of TBI patients in the pre-hospital or early ED phase has been found beneficial in several studies, so this procedure has been included in the guidelines of the Brain Trauma Foundation [75] for TBI patients with a GCS < 8. While there is little doubt in the overall value of these recommendations, some studies suggest that real-life adherence to this guideline is quite low and that improper technical skills and duration of the intubation process as well as early unintentional hyperventilation (with subsequent cerebral vasocontriction and compromised brain perfusion due to hypocapnia) can cause considerable harm. Intubating head trauma patients can be challenging, not only in the case of facial trauma with direct airway involvement, but also because about 10% have associated cervical spine injury. Airway management in all trauma patients must involve in-line traction and stabilization of the neck (by hands of an assistant and then by using a stiff-neck-device). As conventional laryngoscopy and intubation might be difficult or impossible in that situation, awake fiberoptic intubation, if feasible, is a reasonable option, and in cadaver models produces the smallest amount of anterior-posterior spinal displacement. Alternatively, urgent cricothyroidotomy may be indicated if major facial or airway trauma is present. Although these and other options seem appropriate and acceptable, no outcome studies exist to allow favoring one technique over the other [76]. Patients suffering acute ischemic stroke from large vessel occlusion require special consideration, especially those being intubated in anticipation of endovascular revascularization. Many interventionalists prefer to have these patients intubated and put on general aesthesia for several (unproven) technical and safety reasons [77]. A few retrospective studies, however, suggest possible harm when intubation for stroke intervention was compared to noninvasive airway management including conscious sedation [78-80], possibly owing to time delay, hypotension, and unintentional hyperventilation with subsequent detrimental vasoconstriction in the penumbra. A recent consuensus statement of the Society for Neuroscience in Anesthesiology and Critical Care suggested that patients suffering posterior circulation large vessel occlusions might be best managed with endotracheal intubation, due to the tendency of such patients to suffer central respiratory failure, while those with anterior circulation compromise and a widely patent airway might be safely managed using conscious sedation [68]. Patients with status epilepticus need to be intubated if seizures cannot be

www.medicalebookpdf.com

terminated during the first steps of treatment and one has to proceed to the use of general anesthesic agents. However, respiratory depression by benzodiazepines, the accepted first-line treatment agents, can lead to early emergency intubation, especially if standard dose ranges are exceeded. In a study of more than 200 patients with status epilepticus, 45% patients in the group receiving excessive doses of benzodiazepines had to be intubated compared to only 8% in the standard dose group [81]. Lorazepam seems to be associated with less respiratory depression than other benzodiazepines. Extubation Extubation is considered in patients with cardiopulmonary stability and improving or stable neurological status. While classical extubation criteria prefer an awake and cooperative patient, this is not realistic in the neurological ICU, where patients might present with aphasia, anarthria, apraxia, agitation, delirium or a reduced level of consciousness depending on their injuries. Even patients with neuromuscular disease or CIP/CIM that should have preserved cerebral capacity, can develop delirium, psychosis, mutism, cranial nerve-related communication deficits, agitation and especially anxiety (at times a kind of ventilator-dependency), making the extubation decision similarly challenging. Therefore, extubation is often delayed in NICU patients [2]. Classical extubation criteria (Table 18-14) have failed to predict extubation failures in the Neuro-ICU. Extubation failure occurs at a rate of 15-35% in patients with brain lesions [2,82-85] and 30-40% in patients with neuromuscular disease such as MG [86]. Non-specific variables like sputum impaction, secretion load and viscosity, duration of ventilation or underlying diaseases such as COPD or obstructive sleep apnea (OSA) have a predictive role for extubation failure, yet neurological disease adds many additional complex variables related to respiratory regulation, airway patency, protective reflexes, and pharyngeal sensory function. A few disease-specific predictors of extubation success are the ability to follow four simple commands or a higher GCS in patients with brain lesions, and a strong cough in neuromuscular patients. Extubation delay in the neurocritically ill, including patients not meeting classical extubation criteria, may lead to increased rates of ventilator-associated pneumonia (VAP) and prolonged ICU-length of stay (LOS), while patients extubated earlier or later do not seem to differ with regard to re-intubation rate [37]. A small pilot study randomized 16 brain-injured NICU patients and saw similar complications or functional outcome at discharge between patients extubated immediately after meeting respiratory extubation criteria and those reevaluated and extubated later because of coma [87]. Although the timing of extubation in the neurocritically ill requires further prospective research, coma

www.medicalebookpdf.com

should not be the only reason to withhold weaning or extubation from these patients. Rather, particular attention should be paid to the presence of adequate cough and minimal secretions [37]. Following extubation, all measures must be taken to avoid aspiration - a frequent event in the Neuro-ICU population [88]. Endoscopic swallowing tests that do not require cooperation of the patient have been successfully applied in stroke, and might help guide extubation decisions in other neurocritically ill patients [89]. Tracheostomy While 10-20% of ICU patients receive a tracheostomy during their stay, this rate is about 35-45% in NICU patients [2, 90]. This may reflect that NICU patients often do not have compromised pulmonary function but rather lack capacity to protect the airway and handle secretions. A retrospective study suggested that among ICU patients, the neurological/neurosurgical ones were those fastest to be weaned from the ventilator [91]. Two restrospective studies in patients with intracerebral hemorrhage (ICH) found that ganglionic location, hematoma volume, hydrocephalus, midline-shift, low GCS and presence of COPD were predictors of traqueostomy requirement [92,93]. The optimal time point for tracheostomy was restrospectively investigated in cerebrovascular patients [8,94]; the studies suggested that duration of ventilation and ICU-length of stays are reduced in patients recieving earlier tracheostomy. This could not be confirmed in the only prospective randomized trial on early tracheostomy (up to day 3 vs day 7-14 from intubation) in ventilated NICU stroke patients [95]. However, the study showed that early tracheostomy is safe, feasible and reduces sedative demand. Until the potential benefits of early tracheostomy in NICU patients are clarified in larger prospective trials, it is probably reasonable to proceed to tracheostomy as part of a weaning protocol if extubation trials failed or were deemed not feasible. Sometimes it becomes apparent within the first week of ventilation if NICU patients will have to receive a tracheostomy in their clinical course, such as those with severe axonal GBS rapidly proceeding to tetraplegia [96] or with extensive brainstem hemorrhage. If such a situation is not judged futile, if patient's and family's will is in accordance, and if the care team is convinced that tracheostomy is necessary there is no reason why this should be delayed. Compared to an orotracheal tube, a tracheal cannula produces less respiratory dead space and can thus reduce work of breathing. In a small study in TBI patients, tracheostomy led to improved pulmonary mechanics, i.e. reduced peak airway pressure and better dynamic compliance [97]. It may thus serve as an adjunct to a difficult weaning process. The procedure itself, regardless if surgical or percutaneous and regardless of the time point, can be associated with

www.medicalebookpdf.com

transient ICP increases and measures to prevent this should be undertaken (head of bed slightly elevated, avoidance of hypoventilation, sufficient sedation and analgesia etc.). MECHANICAL VENTILATION IN THE NEUROCRITICALLY ILL The optimal method of mechanical ventilation has not been established for the neurocritically ill, and given the spectrum of neurological disease (e.g. central vs peripeheral), different patients ages (e.g. young TBI vs older stroke patients), and comorbidities (e.g. heart failure, COPD, asthma), an optimal method may not exist and must be individualized. Most principles of modern ICU ventilation apply in the NICU as well, and in particular there is no reason to withhold lungprotective (i.e. low tidal volume, limited plateau pressure) ventilation in the neurological patient. However, some pathophysiological aspects should be kept in mind. Starting mechanical ventilation Retrospective studies in cerebrovascular patients have shown mechanical ventilation is more commonly required because of progressive decline in consciousness with loss of airway protection, seizures, and congestive heart failure with subsequent pulmonary edema [3,5,6,98]. The need to mechanically ventilate the NICU patient with central respiratory failure may become obvious rapidly, as criteria of respiratory failure and thus indications for intubation (see above) are often clearly evident. It might be less obvious in peripheral neuronal or neuromuscular disease, where respiratory failure can evolve in a more gradual fashion and then suddenly turn into an emergency situation [99]. These patients with GBS, botulism, myasthenia gravis (MG), Lambert-Eaton myasthenic syndrome (LEMS), ALS, or CIP/CIM need to be monitored very closely as not few of them have lost their lives due to a lack of attention on regular or intermediate care wards. Warning signs of peripheral respiratory failure are presented in Table 18-15. In these patients, non-invasive ventilation (e.g. BiPAP via oronasal mask) may help to compensate a respiratory crisis and avoid intubation. This has been shown particularly in patients with MG crisis [100,101]. Case reports and series on non-invasive ventilation in GBS do also exist. However, most severe GBS cases require long-term invasive ventilation as predicted by rapid progression of weakness in upper and lower limb weakness, neck and bulbar weakness and bilateral facial palsy [102] or the combination of loss of foot flexion ability and sciatic nerve conduction block [103]. Non-invasive ventilation is very timeconsuming, requires a higher therapist-patient ratio, wakefulness and

www.medicalebookpdf.com

cooperation on the side of the patient as well as fairly compensated blood gas and acid-base parameters and the presence of airway protective reflexes. Other than in MG crisis, it might be applicable in ALS, intoxications, as a support in weaning (see below) and to treat mild exacerbations of COPD and cardiogenic pulmonary edema. It can also be used in some less severely afflicted stroke patients to avoid intubation and ICU admission, but apart from these situations it plays a minor role in neurocritically ill patients. In a recent multicenter study on ventilation management in about 4968 ICU patients, non-invasive ventilation was used in only 1% of the 938 neurological patients, as compared to 12% in non-neurological patients [2].

Maintaining mechanical ventilation: physiology and parameter settings After invasive ventilation is initiated, the choice of ventilation modes and parameter settings should follow general principles (see above). Some aspects, however, deserve particular consideration in the neurocritically ill. Oxygenation is the main goal of mechanical ventilation to provide the essential brain nutrient besides glucose. It is important to aim for tissue oxygenation and not for arbitrary levels of oxygen in the blood, however. Toxic levels of oxygen (as caused by FiO2 > 0.6 in patients that are not hypoxemic)

www.medicalebookpdf.com

should be avoided. There ample evidence from experimental and some human studies that hyperoxia is associated with tissue damage resulting from free oxygen radical formation, lipid peroxidation and other mechanisms. It might also impede brain perfusion by a not completely understood process called hyperoxia-related cerebral vasoconstriction that might theoretically even lead to secondary ischemia [104-107]. Although the clinical relevance of this hypothesis has not been confirmed sufficiently in NICU patients [108], it seems reasonable to aim for normoxemia but not hyperoxemia. Improving cerebral oxygenation is not limited to increasing the FiO2 or the aggressiveness of ventilation, but can also be achieved by reducing cerebral oxygen demand via reducing work of breathing, and treating infections, fever, agitation, delirium, shivering, and seizures, and employing certain sedatives that reduce the cerebral metabolic rate of oxygen (CMRO2). Normocarbia may be an even more important aim in the NICU patient, as PaCO2 plays such a prominent role in determining cerebral blood flow (CBF) via pH changes in both directions, as long as cerebral autoregulation is intact (which is often not the case in brain-lesioned patients but difficult to determine). Both hypercarbia with subsequent fall in pH, cerebral vasodilation, increase in CBF, rise in ICP and hypocarbia with subsequent rise in pH, cerebral vasoconstriction, decrease in CBF, risk of secondary ischemia might be detrimental, depending on the extent and duration of the derangement, the specific neurological disease and its stage (acute vs subacute). Changes in PaCO2 may have very different implications for an SAH patient with vasopasm than for a TBI patient with brain edema. The concept of permissive hypercarbia used when treating ARDS may be problematic in brain-injured patients and neuromonitoring should be performed if this is performed. Hyperventilation is used to induce hypocarbia and high pH to reduce raised ICP rapidly. However, if applied chronically or prophylactically, it may be associated with higher morbidity and mortality, as was shown in TBI patients [109,110]. Based on the above considerations and because the ICP-reducing effect is short-lasting [111,112], hyperventilation should only be applied transiently for ICP lowering to gain time before more definitive measures can be undertaken (e.g. while rushing patients to the OR for decompressive surgery). The application of increased PEEP in TBI patients with lung injury can lead to increased ICP, at times to dangerous levels [113]. Indeed, application of higher PEEP seems to be avoided by ICU physicians treating neurological patients [2]. The ICP-increasing effect has been linked to raised intrathoracic pressure with subsequently reduced venous return from the brain, based mainly on animal experiments and pathophysiological considerations. Alternatively or

www.medicalebookpdf.com

additionally, reduced cardiac output and reduced MAP, in itself potentially detrimentral via reduced CPP) can lead to raised ICP indirectly via reductions on cerebral blood flow (CBF) and brain oxygenation. However, the individual patient's ICP reaction to raised PEEP seems to vary greatly, probably according to their lung and ventricular compliance, as those with normal or poor pulmonary compliance seem not to show relevant PEEP-associated ICP crises [114]. For NICU patients with lung injury already at critical baseline ICP levels, high-frequency ventilation (HFOV), separating oxygenation and ventilation without relevant impact on venous outflow, may be an alternative that, however, has hardly been studied in the NICU [115,116]. In patients with severe stroke, raised PEEP did not produce significant ICP changes (but did cause reductions in MAP and thus CPP) [117]. Also, PEEP application can be of paramount value to achieve adequate oxygenation, i.e. the primary prerequisite for brain integrity, and should not be subordinated to potential changes in ICP. Furthermore, no increase in mortality has been linked to the use of PEEP in brain-injured patients so far. In essence, NICU patients that are in need of improved oxygenation should not be denied a higher PEEP. Ideally, neuro-monitoring should be performed in these patients to detect changes in ICP and CPP and be able to take measures to achieve a reasonable compromise, such as raising MAP. Changing the I:E ratio to 1:1 or even higher to improve oxygenation has also been thought to reduce venous return from the brain and raise ICP. Studies in ventilated patients with ischemic stroke, intra-cerebral hemorrhage and TBI have not confirmed this [118-120]. Discontinuing mechanical ventilation: weaning Weaning should certainly not be delayed in NICU patients, whether they are comatose or not, although this seems to be common practice [2]. The best method of weaning, i.e. continuous vs discontinuous, is unclear (as in general ICU patients). Discontinuous weaning methods, however, involve successive spontaneous breathing trials (SBTs) and thus wake-up trials. These have been associated with a release of stress hormones [121] and rises in ICP [122] in brain-injured patients, particularly those with a higher ICP [123]. Overall, it seems prudent to do spontaneous breathing trials (SBTs) in patients fulfilling the general criteria (see above), but refrain from further SBTs if they are accompanied by ICP crises or other physiological derangements; in such cases continuous weaning method is advisable. In a subgroup of NICU patients, such as those with advanced ALS or extensive brainstem injury, weaning will not be successful. In those cases, surgical tracheostomy and application of a homeventilator might be adequate, if this is the patient's or family's will.

www.medicalebookpdf.com

COMMON RESPIRATORY DISEASES AND THEIR BASIC MANAGEMENT Upper airway obstruction Upper airway compromise may result from burns, trauma, infection such as Ludwig's angina, retropharyngeal abscess, or epiglottitis, from aspiration of a foreign body, from an obstructing lesion such as tumor, mucus plug, or airway cast, from vocal cord spasm, laryngeal edema, angioedema, a buildup of scar tissue, or simply poor muscle tone and collapse of normal anatomical structures. Although insensitive, the characteristic flow-volume loop of variable extrathoracic obstruction (such as vocal cord paralysis) on close examination is flat bottomed, while a fixed extrathoracic obstruction (such as tracheal stenosis) may be flat both on top and the bottom (Figure 18-2). Stridor is the hallmark of these conditions, and clinicians must be cautious at all times not to destabilize an already compromised airway. Rapid diagnosis and management are critical to safely manage the disorder. An accurate and detailed history and physical examination can determine the prodrome and circumstances, and may reveal drooling, swelling or deformity of the neck, trismus, signs of infection, or respiratory anxiety. When increased work of breathing or respiratory distress is present, urgent evaluation by otolaryngology and anesthesia consultants is warranted. The critical point in managing upper airway obstruction is that the airway may be lost at any moment, and clinicians must be prepared to secure a definitive (often surgical) airway emergently. If endotracheal intubation is attempted, it should be under optimal conditions and by the most experienced clinician available, since the airway is usually difficult and may be lost entirely with a failed attempt, resulting in respiratory arrest, anoxic brain injury, or death.

Figure 18-2. Classically described normal and abnormal flow-volume loop (FVL) patterns. A. Normal FVL, B. FVL changes suggesting a variable extrathoracic flow restriction (ie: vocal cord paralysis), C. FVL changes suggesting fixed upper airway obstruction (ie: tracheal stenosis), D. FVL changes suggesting neuromuscular weakness.

www.medicalebookpdf.com

Physical examination, neck radiography, CT and MR imaging, and laryngoscopy or tracheoscopy are the critical diagnostic modalities employed in management of an upper airway obstruction. Clinicians should consider the use of intravenous corticosteroids and/or nebulized racemic epinephrine to reduce airway edema. Heliox, a mixture of helium and oxygen that promotes laminar gas flow and markedly relieves dyspnea, can help stabilize a distressed patient prior to definitive therapy. The full array of special airways equipment should be available at the time of intubation to improve visualization of a distorted airway. Rigid bronchoscopy under general anesthesia allows for careful evaluation of the airway while permitting mechanical ventilation and providing surgical access for abscess drainage, tumor debridement, or foreign body removal. Bronchopulmonary infections Pneumonia is common in the neurocritically ill, the leading cause of hospital readmission following stroke, and a major source of morbidity in subarachnoid hemorrhage, intracerebral hemorrhage, TBI, and most other neurological diseases. It requires prompt and expert intervention to prevent a host of complications including septic shock, bacteremia, empyema, end-organ damage (including worsening of neurological injury) and death. For epidemiologic purposes and to correctly select antimicrobial therapies, pneumonia should be characterized at the time of diagnosis as one of the following: 1. Community acquired: Without exposures or comorbid conditions to suggest infection with an unusual or resistant organism. May include atypical (intracellular) bacterial and viral infections. 2. Hospital acquired: occurs 48 hours or more after admission 3. Ventilator associated: occurs 48 to 72 hours or more after endotracheal intubation 4. Healthcare associated: occurs in a non-hospitalized patient with extensive healthcare contact 5. Pneumonia in an immune-compromised or otherwise abnormal (ie, with preexisting lung disease) host. The clinical criteria for pneumonia are typically considered to include: 1. Infiltrates determined by chest imaging or physical examination findings 2. Signs or symptoms of systemic inflammation: fever or hypothermia,

www.medicalebookpdf.com

elevated or low WBC 3. Impairment of gas exchange 4. A change in the quantity or character of secretions Patients with pneumonia typically manifest between two and four of these clinical criteria; no standard exists for the requisite number to make a diagnosis – when all four are required, the sensitivity is low but specificity is high, when only two of four are required the sensitivity is high but specificity is low. Additionally, since microbiology from sputum or blood cultures is positive in half or fewer of patients with proven invasive parenchymal lung infection, the isolation of pathological bacteria may be useful to guide therapy, but is not required for the diagnosis. Because of these diagnostic difficulties, these criteria and others have been codified into a Clinical Pulmonary Infection Score (Table 18-16). The CPIS is a validated, quantitative, reproducible tool for diagnosis of pneumonia [124]. Some authors advocate that it be used to quantify the likelihood of a diagnosis of pneumonia, and repeated at 72h so that antibiotic coverage can be narrowed, broadened, or discontinued based on the results of that re-evaluation [125]. Principles of the diagnosis and management of pneumonia include: 1. Determination of the most likely pathogen, and rapid administration of targeted empiric antimicrobial therapy. 2. Support of oxygenation and ventilation, and verification of adequate systemic tissue perfusion. 3. Cultures of sputum and blood, if possible prior to antibiotic administration, but not delaying antibiotic administration. 4. Consideration of special populations and unusual organisms, with emphasis on a. The immune-compromised host, susceptible to a different spectrum of pathogens, including treatable viruses and fungi that may require special diagnostic techniques b. Multidrug resistant organisms, especially i. Pseudomonas species ii. ESBL producing gram negative rods

www.medicalebookpdf.com

iii. MRSA iv. Mycobacterium tuberculosis c. Consideration of Legionella pneumophilia, an atypical community or institution-acquired pneumonia necessitating fluoroquinolone, macrolide, or tetracycline coverage, with a high case fatality rate in patients receiving inappropriate antimicrobial therapy 5. Reassessment of the need for therapy and the response to empirical therapy after 48-96 hours of treatment. 6. Targeted empirical antimicrobial therapy under a protocol whenever possible, based on local susceptibility patterns, and immediately narrowed to reflect culture results and in-vitro sensitivities of the organism. 7. 7-10 days of therapy in most situations. Exceptions include identification of nonfermenting gram-negative bacilli such as Pseudomonas species, in which case 14-21 days may be superior, or recognition of an incompletely treated source of infection, such as heart valves, pleural space, intraabdominal abscess, or bone) [126]. Chronic Obstructive Pulmonary Disease COPD is a disorder of incompletely reversible airflow obstruction and is usually smoking-related, but may also be caused by asthma, enzyme deficiency, viral infection, bone marrow transplantation, and connective tissue diseases. It is diagnosed by pulmonary function testing including spirometry, lung volumes, diffusion capacity, and bronchodilator responsiveness. The diagnosis of COPD requires an FEV1/FVC ratio less than predicted based on norms related to age, gender, height, and sometimes race. Severity is determined by clinicians according to a variety of factors including the absolute FEV1 as a percentage of predicted, the need for supplemental oxygen, identification of chronic hypoventilation with respiratory acidosis, frequency of hospitalizations, and the extent of disability. A Global Initiative for Chronic Obstructive Lung Disease (GOLD) criteria has been proposed (and criticized) as a simple tool for assessment of COPD severity (Table 18-17) [127].

www.medicalebookpdf.com

www.medicalebookpdf.com

COPD manifests with a variety of different phenotypes. In emphysematous-type, or flow-preserved COPD, airflow obstruction may be minor in comparison to abnormalities of gas exchange and bullae or emphysematous lung parenchyma noted on CT scan. Bronchitic-type COPD is characterized primarily by airflow obstruction. Mucopurulent bronchitis is common, and there may be significant bronchodilator and steroid responsiveness. In both phenotypes, air trapping, hyperinflation, and distorted respiratory mechanics may be a prominent feature of the disease, and may make for difficult mechanical ventilation. COPD exacerbation is a common reason for hospitalization and is associated with progression of the disease. It may be triggered by infection, aspiration, sinus disease, allergies, medication changes, or other irritants. Exacerbation is treated with inhaled bronchodilators (B-agonist or anticholinergic agents), and corticosteroids. Recent data suggest that low dose oral corticosteroids may be as effective as high dose intravenous alternatives. [128] Antibiotics are indicated in moderate and severe COPD exacerbation, or when heavy mucopurulent secretions are present. Oxygen therapy is important in the treatment of COPD exacerbation, and should not be routinely withheld to prevent suppression of the central respiratory drive. Patients at significant risk of having a predominantly hypoxic respiratory drive and developing central hypoventilation in the presence of normal PaO2 are typically those with severe chronic hypercarbia. Most clinicians agree that targeting a PaO2 of 60-70mmHg and oxyhemoglobin saturation of 88-94% is safe. However, when a recent brain injury is present, higher PaO2 may be indicated and should be cautiously pursued even in chronic CO2 retainers, while monitoring for evidence of hypoventilation by frequent arterial blood gas analysis and observation. Noninvasive positive pressure ventilation by face mask has become a standard part of the routine treatment of severe COPD exacerbation, and when administered to patients in whom pCO2 is > 45mmHg and pH < 7.35, it

www.medicalebookpdf.com

decreases mortality by almost half, the need for endotracheal intubation by more than half, and improves many other measures of disease severity. [129,130] Withholding NPPV from patients with serious COPD exacerbation and evidence of early respiratory failure is rarely appropriate, and does not meet the standard of care. Asthma and Status Asthmaticus Unlike COPD, asthma is a disease of airway hyperreactivity and inflammation, and is characterized by largely reversible airway obstruction. When chronic airway inflammation is well controlled, severe asthma exacerbations are rare, but when disease is poorly controlled, flares may be lifethreatening. Risk factors for a fatal asthma exacerbation include previous severe exacerbations, recent hospitalizations, multiple recent emergency department visits, heavy use of short-acting beta agonists, difficulty perceiving symptoms or the severity of exacerbations, various psychosocial issues, and serious medical or psychiatric comorbidities. Severe asthma exacerbations are routinely treated with inhaled B-agonists and glucocorticoids, and sometimes with infusion of intravenous magnesium. Occasionally, anticholinergic agents or antibiotics with atypical coverage, such as fluoroquinolones or macrolides, are additionally employed. When intubation is required, mechanical ventilation may be difficult due to air trapping, high airway pressures, barotrauma, and acidosis, and should be performed with expert guidance. Rescue strategies that have shown some success in these patients include intravenous B-agonist infusions, inhaled helium-oxygen admixture (Heliox), general anesthesia, and even extracorporeal membrane oxygenation (ECMO). Pneumothorax Pneumothorax is categorized as primary or secondary (to underlying lung disease), and as spontaneous or iatrogenic. The diagnosis can be appropriately made based on the physical examination alone (Table 18-18), and under emergency circumstances it should be made without awaiting a chest radiograph. Pneumothorax can be subtle, or may present as a catastrophic event with tension physiology. Diagnosis and treatment depend on many factors including the health of the individual, the severity and acuity of the presentation, and the clinical circumstances. When circumstances allow for a brief delay in diagnosis, then chest radiography may be very helpful. A decubitus view with attention to the “up” lung, or a CT scan will reliably show a small simple pneumothorax, and CT imaging reveals complexities of the pleural space. Stable pneumothorax without evidence of tension can be conservatively managed (without a thoracostomy

www.medicalebookpdf.com

tube) by observation, high FiO2, or air aspiration. Chest tube thoracostomy is usually necessary in unstable patients, those with tension physiology, those with large or complete pneumothorax, and those receiving positive pressure ventilation. Urgent decompression of a tension pneumothorax can be performed by inserting a 14-16 gauge angiocath or needle over the top of the rib and into the 2nd/3rd intercostal space (roughly at the angle of the manubrium) in the anterior midclavicular line.

Chest tube thoracostomy is performed using a variety of techniques including surgical chest tube placement, pigtail catheter insertion over a trocar, and Seldinger-type guidewire-based insertion. Chest tubes are routinely inserted without imaging, or can be placed utilizing CT or ultrasound guidance – particularly useful when the pleural space is complex, and the lung is tethered to the chest wall by scar, infection, tumor, or other pleural disease. Smaller catheters (< 24 Fr) are now routinely used for management of pneumothorax, and nonrandomized data suggest equivalent clinical outcomes to larger bore tubes with less pain [131,132]. The goals of chest tube insertion in the management of acute pneumothorax are to decompress tension and improve respiratory function, encourage reinflation of the injured lung, and provide access to the pleural space in the event that pleuradesis becomes necessary. There is controversy as to the best management of chest tube drainage systems in pneumothorax. When continuous suction is applied to the pleural space, the lung will often reinflate completely, but continuous negative pressure in the pleural space may encourage the bronchopleural fistula (BPF) to remain open – especially when employed in combination with positive pressure ventilation. Conversely, when water seal or a

www.medicalebookpdf.com

Heimlich valve is applied, the lung may remain partially collapsed, though positive pressure cannot accumulate in the pleural space because it escapes through the water seal. Common practice is to transition rapidly from suction to water seal unless the patient is unable to tolerate partial lung collapse, in order to facilitate closure of the BPF. Clinicians responding to respiratory or hemodynamic instability in a patient with a chest tube placed for management of pneumothorax should strongly suspect that the tube has failed and tension pneumothorax has developed – rapid assessment of the function of the tube should ensue by looking for tidal variation of the system, checking all potential sites of air leakage, and flushing air through the system using sterile technique. If after these assessments there remains a high level of concern for recurrent pneumothorax, the chest tube should be immediately replaced. Pleural Effusion and Empyema Pleural effusions are common, and appropriate management is based on a rapid determination of the underlying pathophysiology. When an infectious etiology is likely, thoracentesis should be urgently performed to rule out pleural space infection. When these infections are rapidly treated with definitive drainage, septic shock is unlikely and the pleural space remains simple, rarely requiring surgical decortication. Conversely, delay in diagnosis and drainage can lead to bacteremia, septic shock, formation of loculations leading to the need for surgical decortication, and permanent loss of pulmonary function. These concerns have led to the (oversimplified) adage that “the sun should not be allowed to set on an undrained pleural effusion.” Table 18-19 provides a list of common pleural processes and their corresponding management strategies. Chronic aspiration is associated with anaerobic empyema, and when anaerobic infection is suspected (usually on the basis of odor), it is important to provide anaerobic antibiotic coverage even if these organisms are not recovered on culture. Because the microbiology of pleural space infections is somewhat different than that of pneumonia, and because of various subtleties in management, empyema may be best managed in cooperation with a pulmonary or infectious disease specialist when there is nonresolving infection, unusual or resistant organisms, lack of an identified organism, or a multiloculated (complex) pleural space. Hemoptysis and Pulmonary Hemorrhage Pulmonary hemorrhage may arise from three anatomical sources: bronchial artery bleeding, which is often high-volume due to systemic blood pressure, pulmonary arterial bleeding which occurs at lower pressures and is less often massive, and arteriolar or capillary hemorrhage, due to small artery vasculitis or

www.medicalebookpdf.com

the entities causing diffuse alveolar hemorrhage. In all circumstances, urgent reversal of any underlying coagulation defects is critically important.

Hemoptysis is categorized as massive or non-massive. Massive hemoptysis is life threatening, and defined variously as greater than 100-600cc expectorated blood/24h. Most hemoptysis is caused by infections, including tuberculosis, fungal infection (especially Aspergillus and Mucor), lung abscess, and simple bacterial pneumonia or tracheobronchitis. Other causes include bronchiectasis and bronchogenic carcinoma, vasculitis, chemotherapy and bone marrow transplantation, and foreign bodies. Diffuse alveolar hemorrhage (DAH), a very different clinical entity, results from widespread but heterogeneous leakage of capillary beds into the alveolar spaces, and may result in massive amounts of blood loss and progressive gas exchange abnormalities without much or any expectoration of blood. The primary risk to life in massive hemoptysis is by asphyxia, and routine management of the disorder is characterized by concurrent stabilization of the patient and an accelerated diagnostic workup. Protection of the nonbleeding lung is achieved by placing the patient in a “bleeding side down” lateral decubitus

www.medicalebookpdf.com

position, lung isolation via preferential intubation of the mainstem bronchus of the unaffected lung, intubation with a double-lumen endotracheal tube, or bronchoscopic placement of an endobronchial blocker device. Early elective intubation under controlled circumstances with a large-lumen endotracheal tube is preferred for patient safety and to facilitate bronchoscopy. Diagnostic workup of hemoptysis depends on stability of the patient, clinical circumstances, and other factors. Both CT angiography of the chest and bronchoscopy are helpful in localizing and defining the bleeding source. Although low and moderate volume hemoptysis may resolve with medical therapy, angioembolization is frequently applied and often successful at stopping hemorrhage, infrequently resulting in significant pulmonary infarction due to the lung's dual blood supply. Because of good success with angioembolization, lung resection is now infrequently utilized, but remains an important option in certain circumstances, such as invasive fungal or mycobacterial infection, or bronchogenic carcinoma. DAH should be suspected when diffuse pulmonary infiltrates and gas exchange abnormalities progress in conjunction with a rapid decrease in the serum hemoglobin concentration, especially in patients with hematologic malignancies. The diagnostic gold standard is bronchoscopy with bronchoalveolar lavage, in which sequential lavages with saline return with increasing quantities of heme. Dozens of potentially causative medications, including chemotherapeutic agents have been identified, and clinicians should carefully review all drugs and discontinue any that are associated with the disorder. Treatment is based entirely on supportive care and rapid treatment of the underlying medical condition. Rapid identification of ANCA vasculitis and initiation of immune suppressant therapy may be lifesaving, while urgent plasmaferesis is the treatment of choice in Goodpasture's or anti-GBM disease – associated DAH. REFERENCES 1. Holland MC, Mackersie RC, Morabito D, Campbell AR, Kivett VA, Patel R, Erickson VR, Pittet JF, (2003) The development of acute lung injury is associated with worse neurologic outcome in patients with severe traumatic brain injury. J Trauma 55: 106-111 2. Pelosi P, Ferguson ND, Frutos-Vivar F, Anzueto A, Putensen C, Raymondos K, Apezteguia C, Desmery P, Hurtado J, Abroug F, Elizalde J, Tomicic V, Cakar N, Gonzalez M, Arabi Y, Moreno R, Esteban A, (2011)

www.medicalebookpdf.com

Management and outcome of mechanically ventilated neurologic patients. Crit Care Med 39: 1482-1492 3. Berrouschot J, Rossler A, Koster J, Schneider D, (2000) Mechanical ventilation in patients with hemispheric ischemic stroke. Crit Care Med 28: 2956-2961 4. Steiner T, Mendoza G, De Georgia M, Schellinger P, Holle R, Hacke W, (1997) Prognosis of stroke patients requiring mechanical ventilation in a neurological critical care unit. Stroke 28: 711-715 5. Wijdicks EF, Scott JP, (1997) Causes and outcome of mechanical ventilation in patients with hemispheric ischemic stroke. Mayo Clin Proc 72: 210-213 6. Mayer SA, Copeland D, Bernardini GL, Boden-Albala B, Lennihan L, Kossoff S, Sacco RL, (2000) Cost and outcome of mechanical ventilation for life-threatening stroke. Stroke 31: 2346-2353 7. Roch A, Michelet P, Jullien AC, Thirion X, Bregeon F, Papazian L, Roche P, Pellet W, Auffray JP, (2003) Long-term outcome in intensive care unit survivors after mechanical ventilation for intracerebral hemorrhage. Crit Care Med 31: 2651-2656 8. Rabinstein AA, Wijdicks EF, (2004) Outcome of survivors of acute stroke who require prolonged ventilatory assistance and tracheostomy. Cerebrovasc Dis 18: 325-331 9. Marik PE (2001) Aspiration pneumonitis and aspiration pneumonia. N Engl J Med 344: 665-71 10. Sirvent JM, Torres A, El-Ebiary M, Castro P, de Batlle J, Bonet A, (1997) Protective effect of intravenously administered cefuroxime against nosocomial pneumonia in patients with structural coma. Am J Respir Crit Care Med 155: 1729-34. 11. Acquarolo A, Urli T, Perone G, Giannotti C, Candiani A, Latronico N, (2005) Antibiotic prophylaxis of early onset pneumonia in critically ill comatose patients. A randomized study. Intensive Care Med 31:510-516 12. Marik PE, Zaloga GP, (2003) Gastric versus post-pyloric feeding: a systematic review. Crit Care 7: R46-51 13. White H, Sosnowski K, Tran K, Reeves A, Jones M, (2009) A randomised controlled comparison of early post-pyloric versus early gastric feeding to

www.medicalebookpdf.com

meet nutritional targets in ventilated intensive care patients. Crit Care 13: R187. 14. Rabinstein A, Wijdicks EF, (2002) BiPAP in acute respiratory failure due to myasthenic crisis may prevent intubation. Neurology 59: 1647-9. 15. Seneviratne J, Mandrekar J, Wijdicks EFM, Rabinstein AA, (2008) Noninvasive ventilation in myasthenic crisis. Arch Neurol 65: 54-58. 16. Racca F, Del Sorbo L, Mongini T, Vianello A, Ranieri VM, (2010) Respiratory management of acute respiratory failure in neuromuscular diseases. Minerva Anestesiol 76: 51-62. 17. Gomez-Merino E, Bach JR, (2002) Duchenne muscular dystrophy: prolongation of life by noninvasive ventilation and mechanically assisted coughing. Am J Phys Med Rehabil 81: 411–415. 18. Bach JR, Bianchi C, Finder J, et al, (2007) Tracheostomy tubes are not needed for Duchenne muscular dystrophy. Eur Respir J 30: 179-80. 19. Walls R, Murphy M (2008) Manual of emergency airway management. Lippincott Williams & Wiklins, Philadelphia. 20. Adnet F, Borron SW, Dumas JL, Lapostolle F, Cupa M, Lapandry C, (2001) Study of the “sniffing position” by magnetic resonance imaging. Anesthesiology 94: 83-86 21. Seder DB, Riker RR, Jagoda A, Smith W, Weingart S., (2012) Emergency Neurological Life Support: Airway, Ventilation, and Sedation. Neurocrit Care 17 Suppl 1: 4-20. 22. Lavery GG, McCloskey BV, (2008) The difficult airway in adult critical care. Crit Care Med 36: 2163-2173 23. (1993) Practice guidelines for management of the difficult airway. A report by the American Society of Anesthesiologists Task Force on Management of the Difficult Airway. Anesthesiology 78: 597-602 24. Henderson JJ, Popat MT, Latto IP, Pearce AC, (2004) Difficult Airway Society guidelines for management of the unanticipated difficult intubation. Anaesthesia 59: 675-694 25. Popat M, Mitchell V, Dravid R, Patel A, Swampillai C, Higgs A, (2012) Difficult Airway Society Guidelines for the management of tracheal extubation. Anaesthesia 67: 318-340

www.medicalebookpdf.com

26. Thomas AN, McGrath BA, (2009) Patient safety incidents associated with airway devices in critical care: a review of reports to the UK National Patient Safety Agency. Anaesthesia 64: 358365 27. Mort TC, (2004) The incidence and risk factors for cardiac arrest during emergency tracheal intubation: a justification for incorporating the ASA Guidelines in the remote location. J Clin Anesth 16: 508-516 28. Jaber S, Jung B, Corne P, Sebbane M, Muller L, Chanques G, Verzilli D, Jonquet O, Eledjam JJ, Lefrant JY, (2010) An intervention to decrease complications related to endotracheal intubation in the intensive care unit: a prospective, multiple-center study. Intensive Care Med 36: 248-255 29. Crosby ET, Cooper RM, Douglas MJ, Doyle DJ, Hung OR, Labrecque P, Muir H, Murphy MF, Preston RP, Rose DK, Roy L, (1998) The unanticipated difficult airway with recommendations for management. Can J Anaesth 45: 757-776 30. Walls R, Murphy M (2008) Manual of emergency airway management. Lippincott Williams & Wiklins, Philadelphia 31. Adnet F, Borron SW, Dumas JL, Lapostolle F, Cupa M, Lapandry C, (2001) Study of the “sniffing position” by magnetic resonance imaging. Anesthesiology 94: 83-86 32. Adnet F, Baillard C, Borron SW, Denantes C, Lefebvre L, Galinski M, Martinez C, Cupa M, Lapostolle F, (2001) Randomized study comparing the “sniffing position” with simple head extension for laryngoscopic view in elective surgery patients. Anesthesiology 95: 836-841 33. Griesdale DE1, Liu D, McKinney J, Choi PT. Glidescope® videolaryngoscopy versus direct laryngoscopy for endotracheal intubation: a systematic review and meta-analysis. Can J Anaesth. 2012 Jan;59(1):41-52. 34. Ferson DZ, Rosenblatt WH, Johansen MJ, Osborn I, Ovassapian A, (2001) Use of the intubating LMA-Fastrach in 254 patients with difficult-tomanage airways. Anesthesiology 95: 1175-1181 35. MacIntyre NR, Cook DJ, Ely EW, Jr., Epstein SK, Fink JB, Heffner JE, Hess D, Hubmayer RD, Schein-horn DJ, (2001) Evidence-based guidelines for weaning and discontinuing ventilatory support: a collective task force facilitated by the American College of Chest Physicians; the American Association for Respiratory Care; and the American College of Critical Care Medicine. Chest 120: 375S-395S

www.medicalebookpdf.com

36. Esteban A, Alia I, Gordo F, Fernandez R, Solsona JF, Vallverdu I, Macias S, Allegue JM, Blanco J, Carriedo D, Leon M, de la Cal MA, Taboada F, Gonzalez de Velasco J, Palazon E, Carrizosa F, Tomas R, Suarez J, Goldwasser RS, (1997) Extubation outcome after spontaneous breathing trials with T-tube or pressure support ventilation. The Spanish Lung Failure Collaborative Group. Am J Respir Crit Care Med 156: 459-465 37. Coplin WM, Pierson DJ, Cooley KD, Newell DW, Rubenfeld GD, (2000) Implications of extubation delay in brain-injured patients meeting standard weaning criteria. Am J Respir Crit Care Med 161: 1530-1536 38. Ochoa ME, Marin Mdel C, Frutos-Vivar F, Gordo F, Latour-Perez J, Calvo E, Esteban A, (2009) Cuff-leak test for the diagnosis of upper airway obstruction in adults: a systematic review and meta-analysis. Intensive Care Med 35: 1171-1179 39. Jaber S, Chanques G, Matecki S, Ramonatxo M, Vergne C, Souche B, Perrigault PF, Eledjam JJ, (2003) Post-extubation stridor in intensive care unit patients. Risk factors evaluation and importance of the cuff-leak test. Intensive Care Med 29: 69-74 40. McCaffrey J, Farrell C, Whiting P, Dan A, Bagshaw SM, Delaney AP, (2009) Corticosteroids to prevent extubation failure: a systematic review and meta-analysis. Intensive Care Med 35: 977-986 41. Khemani RG, Randolph A, Markovitz B, (2009) Corticosteroids for the prevention and treatment of post-extubation stridor in neonates, children and adults. Cochrane Database Syst Rev: CD001000 42. Scales DC, Thiruchelvam D, Kiss A, Redelmeier DA, (2008) The effect of tracheostomy timing during critical illness on long-term survival. Crit Care Med 36: 2547-255737. 43. Higgins KM, Punthakee X, (2007) Meta-analysis comparison of open versus percutaneous tracheostomy. Laryngoscope 117: 447-454 44. Seder DB, Lee K, Rahman C, Rossan-Raghunath N, Fernandez L, Rincon F, Claassen J, Gordon E, Mayer SA, Badjatia N, (2009) Safety and feasibility of percutaneous tracheostomy performed by neurointensivists. Neurocrit Care 10: 264-268 45. Polderman KH, Spijkstra JJ, de Bree R, Christiaans HM, Gelissen HP, Wester JP, Girbes AR, (2003) Percutaneous dilatational tracheostomy in the ICU: optimal organization, low complication rates, and description of a new

www.medicalebookpdf.com

complication. Chest 123: 1595-1602 46. Delaney A, Bagshaw SM, Nalos M, (2006) Percutaneous dilatational tracheostomy versus surgical tracheostomy in critically ill patients: a systematic review and meta-analysis. Crit Care 10: R55 47. Tobin MJ, (2001) Advances in mechanical ventilation. N Engl J Med 344: 1986-1996 48. Fan E, Needham DM, Stewart TE, (2005) Ventilatory management of acute lung injury and acute respiratory distress syndrome. JAMA 294: 2889-2896 49. Branson RD, (2005) Functional principles of positive pressure ventilators: implications for patient-ventilator interaction. Respir Care Clin N Am 11: 119-145 50. (2000) Ventilation with lower tidal volumes as compared with traditional tidal volumes for acute lung injury and the acute respiratory distress syndrome. The Acute Respiratory Distress Syndrome Network. N Engl J Med 342: 1301-1308 51. Meade MO, Cook DJ, Guyatt GH, Slutsky AS, Arabi YM, Cooper DJ, Davies AR, Hand LE, Zhou Q, Thabane L, Austin P, Lapinsky S, Baxter A, Russell J, Skrobik Y, Ronco JJ, Stewart TE, (2008) Ventilation strategy using low tidal volumes, recruitment maneuvers, and high positive endexpiratory pressure for acute lung injury and acute respiratory distress syndrome: a randomized controlled trial. JAMA 299: 637-645 52. Hodgson C, Keating JL, Holland AE, Davies AR, Smirneos L, Bradley SJ, Tuxen D, (2009) Recruitment manoeuvres for adults with acute lung injury receiving mechanical ventilation. Cochrane Database Syst Rev: CD006667 53. Latronico N, Bolton CF, (2011) Critical illness polyneuropathy and myopathy: a major cause of muscle weakness and paralysis. Lancet Neurol 10: 931-941 54. Keenan SP, Sinuff T, Burns KE, Muscedere J, Kutsogiannis J, Mehta S, Cook DJ, Ayas N, Adhikari NK, Hand L, Scales DC, Pagnotta R, Lazosky L, Rocker G, Dial S, Laupland K, Sanders K, Dodek P, (2011) Clinical practice guidelines for the use of noninvasive positive-pressure ventilation and noninvasive continuous positive airway pressure in the acute care setting. CMAJ 183: E195-214 55. Esteban A, Anzueto A, Alia I, Gordo F, Apezteguia C, Palizas F, Cide D,

www.medicalebookpdf.com

Goldwaser R, Soto L, Bugedo G, Rodrigo C, Pimentel J, Raimondi G, Tobin MJ, (2000) How is mechanical ventilation employed in the intensive care unit? An international utilization review. Am J Respir Crit Care Med 161: 1450-1458 56. Marino P (2007) Mechanical ventilation. The ICU book. Lippincott, Williams & Wilkins, Philadelphia, Baltimore, New York 57. Wijdicks E (2010) “Short of breath” & Mechanical ventilation. The practice of emergency and critical care neurology. Oxford University Press, New York 58. Stewart NI, Jagelman TA, Webster NR, (2011) Emerging modes of ventilation in the intensive care unit. Br J Anaesth 107: 74-82 59. Gainnier M, Michelet P, Thirion X, Arnal JM, Sainty JM, Papazian L, (2003) Prone position and positive end-expiratory pressure in acute respiratory distress syndrome. Crit Care Med 31: 2719-2726 60. Nevins ML, Epstein SK, (2001) Weaning from prolonged mechanical ventilation. Clin Chest Med 22: 13-33 61. Scheinhorn DJ, Chao DC, Stearn-Hassenpflug M, (2002) Liberation from prolonged mechanical ventilation. Crit Care Clin 18: 569-595 62. Macintyre NR, (2013) The ventilator discontinuation process: an expanding evidence base. Respir Care 58: 1074-1086 63. Boles JM, Bion J, Connors A, Herridge M, Marsh B, Melot C, Pearl R, Silverman H, Stanchina M, Vieillard-Baron A, Welte T, (2007) Weaning from mechanical ventilation. Eur Respir J 29: 10331056 64. Brochard L, Rauss A, Benito S, Conti G, Mancebo J, Rekik N, Gasparetto A, Lemaire F, (1994) Comparison of three methods of gradual withdrawal from ventilatory support during weaning from mechanical ventilation. Am J Respir Crit Care Med 150: 896-903 65. Esteban A, Frutos F, Tobin MJ, Alia I, Solsona JF, Valverdu I, Fernandez R, de la Cal MA, Benito S, Tomas R, et al., (1995) A comparison of four methods of weaning patients from mechanical ventilation. Spanish Lung Failure Collaborative Group. N Engl J Med 332: 345-350 66. Vitacca M, Vianello A, Colombo D, Clini E, Porta R, Bianchi L, Arcaro G, Vitale G, Guffanti E, Lo Coco A, Ambrosino N, (2001) Comparison of two methods for weaning patients with chronic obstructive pulmonary disease

www.medicalebookpdf.com

requiring mechanical ventilation for more than 15 days. Am J Respir Crit Care Med 164: 225-230 67. Yang KL, Tobin MJ, (1991) A prospective study of indexes predicting the outcome of trials of weaning from mechanical ventilation. N Engl J Med 324: 1445-1450 68. Talke PO, Sharma D, Heyer EJ, Bergese SD, Blackham KA, Stevens RD, (2104) Society for Neuroscience in Anesthesiology and Critical Care Expert Consensus Statement: Anesthetic Management of Endovascular Treatment for Acute Ischemic Stroke*. Neurosurg Anesthesiol 26: 95-108. 69. Reich DL, Hossain S, Krol M, Baez B, Patel P, Bernstein A, Bodian CA, (2005) Predictors of hypotension after induction of general anesthesia. Anesth Analg 101: 622-628 70. Modica PA, Tempelhoff R, White PF, (1990) Pro- and anticonvulsant effects of anesthetics (Part II) Anesth Analg 70: 433-44. 71. Hansen HC, Drenck NE, (1988) Generalised seizures after etomidate anaesthesia. Anaesthesia 43:805-6. 72. Krieger W, Copperman J, Laxer KD, (1985) Seizures with etomidate anesthesia. Anesth Analg 64: 1226-7. 73. Perry J, Lee J, Wells G, (2003) Rocuronium versus succinylcholine for rapid sequence induction intubation. Cochrane Database Syst Rev: CD002788 74. Papazian L, Forel JM, Gacouin A, Penot-Ragon C, Perrin G, Loundou A, Jaber S, Arnal JM, Perez D, Seghboyan JM, Constantin JM, Courant P, Lefrant JY, Guerin C, Prat G, Morange S, Roch A, (2010) Neuromuscular blockers in early acute respiratory distress syndrome. N Engl J Med 363: 1107-1116 75. Gabriel EJ, Ghajar J, Jagoda A, Pons PT, Scalea T, Walters BC, (2002) Guidelines for prehospital management of traumatic brain injury. J Neurotrauma 19: 111-174 76. Crosby ET, (2006) Airway management in adults after cervical spine trauma. Anesthesiology 104: 1293-1318 77. McDonagh DL, Olson DM, Kalia JS, Gupta R, Abou-Chebl A, Zaidat OO, (2010) Anesthesia and Sedation Practices Among Neurointerventionalists during Acute Ischemic Stroke Endovascular Therapy. Front Neurol 1: 118

www.medicalebookpdf.com

78. Abou-Chebl A, Lin R, Hussain MS, Jovin TG, Levy EI, Liebeskind DS, Yoo AJ, Hsu DP, Rymer MM, Tayal AH, Zaidat OO, Natarajan SK, Nogueira RG, Nanda A, Tian M, Hao Q, Kalia JS, Nguyen TN, Chen M, Gupta R, (2010) Conscious sedation versus general anesthesia during endovascular therapy for acute anterior circulation stroke: preliminary results from a retrospective, multicenter study. Stroke 41: 1175-1179 79. Jumaa MA, Zhang F, Ruiz-Ares G, Gelzinis T, Malik AM, Aleu A, Oakley JI, Jankowitz B, Lin R, Reddy V, Zaidi SF, Hammer MD, Wechsler LR, Horowitz M, Jovin TG, (2010) Comparison of safety and clinical and radiographic outcomes in endovascular acute stroke therapy for proximal middle cerebral artery occlusion with intubation and general anesthesia versus the nonintubated state. Stroke 41: 1180-1184 80. Davis MJ, Menon BK, Baghirzada LB, Campos-Herrera CR, Goyal M, Hill MD, Archer DP, (2012) Anesthetic management and outcome in patients during endovascular therapy for acute stroke. Anesthesiology 116: 396-405 81. Spatola M, Alvarez V, Rossetti AO, (2013) Benzodiazepine over treatment in status epilepticus is related to higher need of intubation and longer hospitalization. Epilepsia 54: e99-e102. 82. Ko R, Ramos L, Chalela JA, (2009) Conventional weaning parameters do not predict extubation failure in neurocritical care patients. Neurocrit Care 10: 269-273 83. Anderson CD, Bartscher JF, Scripko PD, Biffi A, Chase D, Guanci M, Greer DM, (2011) Neurologic examination and extubation outcome in the neurocritical care unit. Neurocrit Care 15: 490-497 84. Wendell LC, Raser J, Kasner S, Park S, (2011) Predictors of extubation success in patients with middle cerebral artery acute ischemic stroke. Stroke Res Treat 2011: 248789 85. Vallverdu I, Calaf N, Subirana M, Net A, Benito S, Mancebo J, (1998) Clinical characteristics, respiratory functional parameters, and outcome of a two-hour T-piece trial in patients weaning from mechanical ventilation. Am J Respir Crit Care Med 158: 1855-1862 86. Wu JY, Kuo PH, Fan PC, Wu HD, Shih FY, Yang PC, (2009) The role of non-invasive ventilation and factors predicting extubation outcome in myasthenic crisis. Neurocrit Care 10: 35-42 87. Manno EM, Rabinstein AA, Wijdicks EF, Brown AW, Freeman WD, Lee

www.medicalebookpdf.com

VH, Weigand SD, Keegan MT, Brown DR, Whalen FX, Roy TK, Hubmayr RD, (2008) A prospective trial of elective extubation in brain injured patients meeting extubation criteria for ventilatory support: a feasibility study. Crit Care 12: R138 88. Langmore SE, (1996) Dysphagia in neurologic patients in the intensive care unit. Semin Neurol 16: 329-340 89. Dziewas R, Warnecke T, Olenberg S, Teismann I, Zimmermann J, Kramer C, Ritter M, Ringelstein EB, Schabitz WR, (2008) Towards a basic endoscopic assessment of swallowing in acute stroke - development and evaluation of a simple dysphagia score. Cerebrovasc Dis 26: 41-47 90. Kurtz P, Fitts V, Sumer Z, Jalon H, Cooke J, Kvetan V, Mayer SA, (2011) How does care differ for neurological patients admitted to a neurocritical care unit versus a general ICU? Neurocrit Care 15: 477-480 91. Van der Lely AJ, Veelo DP, Dongelmans DA, Korevaar JC, Vroom MB, Schultz MJ, (2006) Time to wean after tracheotomy differs among subgroups of critically ill patients: retrospective analysis in a mixed medical/surgical intensive care unit. Respir Care 51: 1408-1415 92. Huttner HB, Kohrmann M, Berger C, Georgiadis D, Schwab S, (2006) Predictive factors for tracheostomy in neurocritical care patients with spontaneous supratentorial hemorrhage. Cerebrovasc Dis 21: 159-165 93. Szeder V, Ortega-Gutierrez S, Ziai W, Torbey MT, (2010) The TRACH score: clinical and radiological predictors of tracheostomy in supratentorial spontaneous intracerebral hemorrhage. Neurocrit Care 13: 40-46 94. Qureshi AI, Suarez JI, Parekh PD, Bhardwaj A, (2000) Prediction and timing of tracheostomy in patients with infratentorial lesions requiring mechanical ventilatory support. Crit Care Med 28: 1383-1387 95. Bosel J, Schiller P, Hook Y, Andes M, Neumann JO, Poli S, Amiri H, Schonenberger S, Peng Z, Unterberg A, Hacke W, Steiner T, (2012) StrokeRelated Early Tracheostomy Versus Prolonged Orotracheal Intubation in Neurocritical Care Trial (SETPOINT): A Randomized Pilot Trial. Stroke 96. Ali MI, Fernandez-Perez ER, Pendem S, Brown DR, Wijdicks EF, Gajic O, (2006) Mechanical ventilation in patients with Guillain-Barre syndrome.Respir Care 51: 1403-1407 97. Sofi K, Wani T, (2010) Effect of tracheostomy on pulmonary mechanics:

www.medicalebookpdf.com

An observational study. Saudi J Anaesth 4: 2-5 98. Cold G, Enevoldsen E, Malmros R, (1975) Proceedings: Ventricular fluid lactate, pyruvate, bicarbonate and pH in unconscious brain injury patients under controlled ventilation. Acta Neurochir (Wien) 31: 266-267 99. Rabinstein A, Wijdicks EF, (2003) Warning signs of imminent respiratory failure in neurological patients Semin Neurol 23: 97-104 100. Seneviratne J, Mandrekar J, Wijdicks EF, Rabinstein AA, (2008) Noninvasive ventilation in myasthenic crisis. Arch Neurol 65: 54-58 101. Wu YK, Lee CH, Shia BC, Tsai YH, Tsao TC, (2009) Response to hypercapnic challenge is associated with successful weaning from prolonged mechanical ventilation due to brain stem lesions. Intensive Care Med 35: 108-114 102. Paul BS, Bhatia R, Prasad K, Padma MV, Tripathi M, Singh MB, (2012) Clinical predictors of mechanical ventilation in Guillain-Barre syndrome. Neurol India 60: 150-153 103. Fourrier F, Robriquet L, Hurtevent JF, Spagnolo S, (2011) A simple functional marker to predict the need for prolonged mechanical ventilation in patients with Guillain-Barre syndrome. Crit Care 15: R65 104. Iscoe S, Fisher JA, (2005) Hyperoxia-induced hypocapnia: an underappreciated risk. Chest 128: 430-433 105. Ashkanian M, Borghammer P, Gjedde A, Ostergaard L, Vafaee M, (2008) Improvement of brain tissue oxygenation by inhalation of carbogen. Neuroscience 156: 932-938 106. Johnston AJ, Steiner LA, Gupta AK, Menon DK, (2003) Cerebral oxygen vasoreactivity and cerebral tissue oxygen reactivity. Br J Anaesth 90: 774786 107. de Jonge E, Peelen L, Keijzers PJ, Joore H, de Lange D, van der Voort PH, Bosman RJ, de Waal RA, Wesselink R, de Keizer NF, (2008) Association between administered oxygen, arterial partial oxygen pressure and mortality in mechanically ventilated intensive care unit patients. Crit Care 12: R156 108. Diringer MN, (2008) Hyperoxia: good or bad for the injured brain? Curr Opin Crit Care 14: 167171 109. Bouma GJ, Muizelaar JP, Choi SC, Newlon PG, Young HF, (1991) Cerebral

www.medicalebookpdf.com

circulation and metabolism after severe traumatic brain injury: the elusive role of ischemia. J Neurosurg 75: 685-693 110. Muizelaar JP, Marmarou A, Ward JD, Kontos HA, Choi SC, Becker DP, Gruemer H, Young HF, (1991) Adverse effects of prolonged hyperventilation in patients with severe head injury: a randomized clinical trial. J Neurosurg 75: 731-739 111. Unterberg AW, Kiening KL, Hartl R, Bardt T, Sarrafzadeh AS, Lanksch WR, (1997) Multimodal monitoring in patients with head injury: evaluation of the effects of treatment on cerebral oxygenation. J Trauma 42: S32-37 112. McLaughlin MR, Marion DW, (1996) Cerebral blood flow and vasoresponsivity within and around cerebral contusions. J Neurosurg 85: 871-876 113. Nyquist P, Stevens RD, Mirski MA, (2008) Neurologic injury and mechanical ventilation. Neuro-crit Care 9: 400-408 114. Caricato A, Conti G, Della Corte F, Mancino A, Santilli F, Sandroni C, Proietti R, Antonelli M, (2005) Effects of PEEP on the intracranial system of patients with head injury and subarachnoid hemorrhage: the role of respiratory system compliance. J Trauma 58: 571-576 115. Salim A, Miller K, Dangleben D, Cipolle M, Pasquale M, (2004) Highfrequency percussive ventilation: an alternative mode of ventilation for head-injured patients with adult respiratory distress syndrome. J Trauma 57: 542-546 116. Fuke N, Murakami Y, Tsutsumi H, Aruga T, Mii K, Toyooka H, Takakura K, Inada Y, (1984) [The effect of high frequency jet ventilation on intracranial pressure in the patients with severe head injury]. No Shinkei Geka 12: 297-302 117. Georgiadis D, Schwarz S, Baumgartner RW, Veltkamp R, Schwab S, (2001) Influence of positive end-expiratory pressure on intracranial pressure and cerebral perfusion pressure in patients with acute stroke. Stroke 32: 20882092 118. Georgiadis D, Schwarz S, Kollmar R, Baumgartner RW, Schwab S, (2002) Influence of inspiration:expiration ratio on intracranial and cerebral perfusion pressure in acute stroke patients. Intensive Care Med 28: 10891093

www.medicalebookpdf.com

119. Clarke JP, (1997) The effects of inverse ratio ventilation on intracranial pressure: a preliminary report. Intensive Care Med 23: 106-109 120. Stewart AR, Finer NN, Peters KL, (1981) Effects of alterations of inspiratory and expiratory pressures and inspiratory/expiratory ratios on mean airway pressure, blood gases, and intracranial pressure. Pediatrics 67: 474-481 121. Skoglund K, Enblad P, Hillered L, Marklund N, (2012) The neurological wake-up test increases stress hormone levels in patients with severe traumatic brain injury. Crit Care Med 40: 216-222 122. Skoglund K, Enblad P, Marklund N, (2009) Effects of the neurological wake-up test on intra-cranial pressure and cerebral perfusion pressure in brain-injured patients. Neurocrit Care 11: 135-142 123. Jaskulka R, Weinstabl C, Schedl R, (1993) [The course of intracranial pressure during respirator weaning after severe craniocerebral trauma]. Unfallchirurg 96: 138-141 124. Ibrahim EH; Ward S; Sherman G; Schaiff R; Fraser VJ; Kollef MH, (2001) Experience with a clinical guideline for the treatment of ventilatorassociated pneumonia. Crit Care Med 29: 1109-15 125. Singh, N, Rogers, P, Atwood, CW, et al, (2000) Short-course empiric antibiotic therapy for patients with pulmonary infiltrates in the intensive care unit: a proposed solution for indiscriminate antibiotic prescription. Am J Respir Crit Care Med 162:505. 126. Chastre J; Wolff M; Fagon JY; Chevret S; Thomas F; Wermert D; Clementi E; Gonzalez J; Jusserand D; Asfar P; Perrin D; Fieux F; Aubas S, (2003) Comparison of 8 vs 15 days of antibiotic therapy for ventilator-associated pneumonia in adults: a randomized trial. JAMA 290: 2588-98. 127. Global Initiative for Chronic Obstructive Pulmonary Disease, Executive Summary: Global Strategy for the Diagnosis, Management, and Prevention of COPD: www.goldcopd.com 128. Lindenauer PK, Pekow PS, Lahti MC, Lee Y, Benjamin EM, Rothberg MB, (2010) Association of corticosteroid dose and route of administration with risk of treatment failure in acute exacerbation of chronic obstructive pulmonary disease. JAMA 303: 2359-67. 129. Brochard L; Mancebo J; Wysocki M; Lofaso F; Conti G; Rauss A;

www.medicalebookpdf.com

Simonneau G; Benito S; Gasparetto A; Lemaire F, (1995) Noninvasive ventilation for acute exacerbations of chronic obstructive pulmonary disease. N Engl J Med 333: 817-22 130. Ram FS; Picot J; Lightowler J; Wedzicha JA (2004) Non-invasive positive pressure ventilation for treatment of respiratory failure due to exacerbations of chronic obstructive pulmonary disease. Cochrane Database Syst Rev: CD004104 131. Baumann MH, Strange C, Heffner JE, et al, (2001) Management of spontaneous pneumothorax*: an American College of Chest Physicians Delphi consensus statement. Chest 119: 590602. 132. MacDuff A, Arnold A, Harvey J, BTS Pleural Disease Guideline Group, (2010) Management of spontaneous pneumothorax: British Thoracic Society Pleural Disease Guideline. Thorax 65 Suppl 2:ii18. 133. Kandel ER, Schwartz JH, Jessell TM, (2000). Principles of Neural Science. McGraw-Hill, New York.

www.medicalebookpdf.com

AIRWAY AND MECHANICAL VENTILATION QUESTIONS 1. A core element of mechanical ventilation in the ICU that improves outcome is a. The avoidance of PEEP b. The use of low tidal volumes (6 ml/kg IBW) c. The acceptance of unlimited peak pressures d. The avoidance of assisted ventilation modes e. Permissive hyperoxia 2. Mechanical ventilation should a. Be applied as long as possible b. Be using fully-controlled ventilation modes, if feasible c. Involve generous administration of muscular blockers d. Be discontinued as soon as possible e. Be regarded as a form of lung therapy 3. Factors predicting weaning success are a. The use of vasopressors b. A rapid shallow breathing index < 105 c. A maximum inspiratory force < 10 cmH2O d. A temperature > 38 °C e. A vital capacity < 10 ml/kg 4. Factors associated with difficulties in airway management are a. Obesity b. Long teeth c. Short neck

www.medicalebookpdf.com

d. Old age e. A-d 5. Which of the following drugs used for induction cause(s) the least hypotension? a. Thiopental b. Propofol c. High doses of fentanyl d. Etomidate e. A+b 6. PEEP a. Should never be used in TBI patients b. Should never be used in stroke patients c. May lead to ICP elevations in patients with high lung compliance d. Leads to increased mortality in NICU patients e. PEEP counteracts the “open-lung-concept” 7. Warning sings of respiratory failure in myasthenia gravis are a. Weak cough b. Use of accessory breathing muscles c. Bradycardia d. A+b e. A+c 8. What has to be avoided when intubating stroke patients? a. The use of analgesics b. The use of sedatives c. Hypothermia

www.medicalebookpdf.com

d. Hypotension e. Hypophosphatemia 9. Weaning neurocritically ill patients a. Is generally futile b. Has to involve continuous weaning methods c. Has to involve discontinuous weaning methods d. Should not be delayed due to coma e. Is associated with a low incidence of delirium 10. When extubating NICU patients, one should be aware a. That ”classical“ extubation criteria might not work b. Of a high incidence of dysphagia c. That re-intubation may worsen prognosis d. That delayed extubation may worsen prognosis e. A-d Clinical Scenarios 11. A 56 year old male smoker is intubated at the time of admission following a 25cc right thalamic ICH complicated by IVH, hydrocephalus, EVD placement, and atrial fibrillation. On hospital day #6 he develops fever, leukocytosis, purulent sputum, and right apical infiltrate on chest X-ray. What is the diagnosis and appropriate next series of steps? a. He has nosocomial pneumonia. Sputum and blood cultures should be obtained, and treatment initiated with a fluoroquinolone and vancomycin. b. He has nosocomial pneumonia. Sputum and blood cultures should be obtained, along with 3-4 liters of crystalloid resuscitation fluid over 24 hours and the initiation of a carbapenem, vancomycin, and a fluoroquinolone. c. He has ventilator-associated pneumonia. Sputum and blood cultures should be obtained, intravenous fluid administered, and antimicrobial therapy withheld until culture results are available and therapy can be targeted

www.medicalebookpdf.com

according to the microbiology. d. He has ventilator associated pneumonia. Sputum and blood cultures should be obtained, crystalloid administered until tissue perfusion is verified, and empiric broad-spectrum antibiotics initiated according to protocol based on local resistance patterns. 12. A 67 yo woman with severe COPD is admitted after a motor vehicle accident resulting in traumatic brain injury. She was intubated by EMS in the field and remains mechanically ventilated with GCS 6, and intracranial pressure of 20-25 mmHg despite a fairly aggressive medical regimen to control ICP. On hospital day 3 breath sounds are globally diminished, peak airway pressures are increased at 54mmHg, and expiratory wheezing is noted in all lung fields. Air trapping is noted on ventilator waveforms, and the intrinsic PEEP is measured at 16mmHg. The patient appears well synchronized with the ventilator. ABG shows pH 7.30, pCO2 62 mmHg, pO2 65 mmHg, HCO3 28 mmol/L. There is no infiltrate on chest X-ray, and secretions are minimal. The most appropriate medical therapy to introduce at this juncture is: a. Scheduled inhaled anticholinergic and beta-agonist medications, and doxycycline 100mg twice per day. b. Intravenous nitroglycerine titrated to a systolic BP of 120mmHg, furosemide 80mg, and morphine sulfate. c. IV methylprednisolone or enteral prednisone 40mg daily, inhaled anticholinergic and B-agonist medications, and a trial of increased PEEP to decrease air trapping and improve ventilation. d. Pentobarbital infusion to bring the ICP down below 15 mmHg and improve patient-ventilator interactions 13. A 28 year old man with mantle cell lymphoma and pancytopenia following induction chemotherapy is admitted to the neurological intensive care unit following a small spontaneous left frontal ICH. He is transfused multiple units of platelets and FFP to correct severe thrombocytopenia and a moderate coagulopathy, and his ICH remains stable, with minimal neurological disability. The patient develops wheezing, multifocal rales, and dyspnea over the next several days, unresponsive to furosemide. The hemoglobin drops from 12.8 g/dl to 7.3g/dl. Chest

www.medicalebookpdf.com

radiograph demonstrates multifocal bilateral infiltrates. On bronchoscopy the airways are friable, and sequential broncho-alveolar lavage returns increasingly bloody fluid. All cultures from the bronchoscopy are negative, as are viral studies and silver stain for Pneumocystis. The most likely diagnosis is: a. Congestive heart failure due to blood product administration b. Transfusion-related acute lung injury (TRALI) c. Diffuse alveolar hemorrhage d. Legionella pneumophilia or other atypical pneumonia e. None of the above 14. You are called urgently to the bedside to evaluate the respiratory status of a patient receiving mechanical ventilation who had a left subclavian central venous catheter successfully placed one hour ago. On arrival, you note SpO2 84%, BP 78/58 mmHg, HR 124 bests per minute, and RR 38 breaths per minute. The ventilator is alarming because peak airway pressure is 58 despite tidal volume of 450cc, and when you check the plateau pressure you find it is 42 mmHg. On physical examination, the left chest is hyperinflated and hyperresonant to percussion, with near absent breath sounds. The next appropriate move is: a. STAT CXR and if there is evidence of a tension pneumothorax, placement of a thoracostomy tube. b. Urgent surgical consultation c. Needle decompression of the left hemithorax followed by urgent thoracostomy placement d. left lateral decubitus radiograph, discontinue infusion of medications through the left central venous catheter, and request surgical consultation 15. Which of the following organisms can be detected on routine sputum culture? Circle all that apply. a. Legionella pneumophilia b. ESBL-producing Klebsiella pneumoniae c. Pneumocystis jiroveci

www.medicalebookpdf.com

d. Mycoplasma pneumoniae e. Mycobacterium tuberculosis

www.medicalebookpdf.com

AIRWAY AND MECHANICAL VENTILATION ANSWERS 1. The correct answer is B. Lower tidal volumes and limitation of inspiratory pressures constitute lung-protective ventilation that translates in reduced morbidity and mortality. 2. The correct answer is D. Mechanical ventilation, although live-saving and indispensable, is not physiological, and can be a potentially damaging procedure that puts the patient at risk of substantial lung-injury, ventilationassociated pneumonia and other complications. Therefore, discontinuiation has to be considered every day and weaning must be started as soon as is feasible. 3. The correct answer is B. An RSBI < 105 has been shown to predict successful weaning and extubation in the general ICU population. It is only one among other predictors, however. The other distractors are rather associated with weaning failure. 4. The correct answer is E. All of these patient factors are associated with difficult airway management, particularly intubation. 5. The correct answer is D. Etomidate hardly influences circulation and thus does not usually cause hypotension. In contrast, propofol, thiopental and higher doses of fentanyl and have all found to cause potentially detrimental hypotension, especially if baseline MAP is low (< 70 mmHg) already. 6. The correct answer is C. In patients with high lung compliance, higher PEEP might cause considerable reduction of venous return from the brain and thus translate into raised ICP. This problem might have less relevance than thought in the past, however, as most patients requiring high PEEP have a low lung compliance. PEEP application has been questioned in NICU patients prone to ICP elevations, such as those with TBI and large stroke, but more recent studies have not confirmed relevant ICP increases in these groups of patients. PEEP is part of the open-lung concept and often necessary to achieve adequate oxygenation, i.e. one core element of support in brain-lesioned patients. The application of PEEP has not been found to be associated with higher mortality in NICU patients. Neuromonitoring is still recommended in NICU patients with lung injury demanding higher PEEP, and attention to MAP and CPP decreases warranted. 7. The correct answer is D. Weak cough (and thus problems to handle

www.medicalebookpdf.com

secretions) as well as the use of accessory breathing muscles (e.g. the sternocleidomastoid) are typical warning sings of imminent respiratory failure in MG. There are several others, such as tachycardia (not bradycardia). 8. The correct answer is D. Because stroke patients often have compromised cerebral perfusion and oxygenation, even transient hypotension should be avoided when intubating them. Although analgesics and sedatives may cause hypotension, they have to be used for intubation, even if the patient is comatose, as laryngoscopy causes extreme stress. Avoidance of overdosing, choice of less circulatory active agents, and counterbalancing with volume and vasopressores can all help to keep these patients hemodynamically stable. Distractors E and C might be present but are not relevant with regard to the intubation process, C might even be beneficial in stroke. 9. The correct answer is D. NICU patients can often breathe with very little assist even if they have not (yet) regained their baseline level of consciousness. Therefore, weaning can be started in many of these patients without problems. The ability to protect the airway is an issue that may have a lot more to do with the level of consciousness and has to be dealt with later in the weaning process. The optimal method of weaning has not been established in (N)ICU patients. The weaning period has a high incidence of delirium (although this is often neglected or not even recognized), but weaning is certainly not generally futile. 10. The correct answer is E. All these aspects are true and make extubating NICU patients a highly individual and at times difficult decision. 11. The correct answer is D. The pneumonia is a ventilator-associated pneumonia since it occurred after 48 hours. Antimicrobial therapy is empiric, broad-spectrum, and based on local resistance patterns. 12. The correct answer is C. The patient has acute exacerbation of COPD, and should be treated with corticosteroids, bronchodilators, and increased PEEP to stent open airways and augment exhalation, resulting in lower pressures and less air trapping. Antibiotics can be used in this scenario, but are not mandatory, since secretions are minimal and there is no evidence of infection. 13. The correct answer is C. This patient has diffuse alveolar haemorrhage due to coagulopathy and hematologic malignancy. DAH is characterized by

www.medicalebookpdf.com

increasingly bloody return on bronchoscopy and a drop in haemoglobin levels. In contrast, TRALI occurs within 6 hours of a transfusion, and Pneumocystis pneumonia does not cause alveolar haemorrhage. He does not have an appropriate history for Legionella infection or CHF. 14. The correct answer is C. The diagnosis of acute tension pneumothorax does not require chest radiography (though ultrasound can be used to confirm the diagnosis) and the urgent treatment is immediate decompression, typically by placing a large-bore angiocath into the 2nd/3rd intercostal space, over the top of the rib, in the midclavicular line, followed by chest tube placement. 15. The correct answer is B. Klebsiella is an extracellular bacterial pneumonias that grow in routine culture preparations. Legionella can be tested by urinary antigen testing, or by culture on a special growth medium (charcoal agar). Pneumocystis is identified by a silver stain (also used for fungus) of a sputum cytology specimen. Mycoplasma is identified by testing for cold agglutinins, or by PCR for the bacterium. Mycobacteria are seen on acid fast smear (also special-ordered) and cultures may take up to 6 weeks to grow.

www.medicalebookpdf.com

Chapter 19

CARDIOVASCULAR MONITORING AND COMPLICATIONS Jesse Corry and Andrew Naidech CLINICAL CASE A 47 year-old Nepalese male without known past medical history was visiting family when he was found in bed the morning of hospital admission with global aphasia and right sided hemiparesis. Based on time last seen normal, he was not a candidate for either IA or IV therapies. Admission CTA demonstrated an occluded left ICA, and stroke in the corresponding left ACA and MCA distributions. Workup to this point demonstrates an LDL of 127 mg/dL, HbA1c of 8.9%, transthoracic echocardiogram with mild concentric left ventricular hypertrophy, an ejection fraction of 60%, and a large patent foramen ovale. Lower and upper extremity Doppler ultrasound does not show thrombus. Post admission day 2 he became progressively lethargic and developed anisocoria. CT demonstrated increasing midline shift and swelling. The patient was temporized with mannitol and taken for urgent hemicraniectomy. The case was uneventful and he was extubated the following morning. His exam is now notable for facial swelling, mimicking behavior, and right hemiplegia. At the start of physician rounds on post-admission day 4, he is pale, hypotensive with SBP 85 mmHg, and tachycardia. The patient is minimally responsive and poorly protecting his airway. He is bolused with 1 L NS, intubated, cardiac and coagulation profiles are sent, he is pan cultured, and a stat ECG and bedside echo is obtained. The ECG now shows diffuse T wave inversions and a prolonged QT interval. The echocardiogram demonstrates diffuse ventricular hypokinesis. Troponin levels are >80 ng/mL. Cardiology is consulted, the risks of heparin are discussed, and the patient is rushed to the cardiac catheterization laboratory. Coronary angiography demonstrates that the coronary vessels were free of any obstruction or spasm. Meanwhile, It is learned that the patient had become quite agitated and hypertensive overnight, with MAPs >140 mmHg. The patient's sedation is aggressively managed, blood pressure and cardiac index respond with fluids and minimal norepinephrine, and he is extubated the next

www.medicalebookpdf.com

morning. Over the next two days the patient is off hemodynamic support, the cardiac index has returned to normal, and bedside echocardiogram demonstrates resolving wall motion abnormalities. MYOCARDIAL INFARCTION AND UNSTABLE CORONARY SYNDROMES Epidemiology Acute chest pain with paraclinical markers of cardiopulmonary injury presents a wide differential diagnosis that the neurointensivist must frequently confront (Table 19-1). Coronary artery disease (CAD) is the most common cause of death in the developed world. Unstable angina (UA) and non ST-segment elevation myocardial infarction (NSTEMI) occur in about 1.3 million people in the US yearly. ST segment elevation MI (STEMI) occurs in about 400,000 people in the US yearly. For comparison, there are about 750,000 strokes yearly. Risk factors include a history of acute coronary syndrome (ACS), peripheral vascular disease, and/or stroke, age, male sex, diabetes, hypertension, smoking tobacco, and recent cocaine use [1].

www.medicalebookpdf.com

Clinical Features ACS is divided into STEMI, NSTEMI, and UA, with STEMI and NSTEMI demonstrating changes in bio-markers of myocyte damage [2]. STEMI is defined by new ST elevation in two anatomically contiguous leads of > 0.1mV (1mm) (Table 19-2) [3]. A new LBBB (any LBBB not previously documented should be considered new) should be regarded as ischemic until proved otherwise. Early on, NSTEMI and UA are often indistinguishable. ECG findings may demonstrate a new horizontal or down-sloping ST depression > 0.05 mV (0.5mm) in two anatomically contiguous leads and/or T wave inversion > 0.1mV (1mm) in two anatomically continuous leads. These changes will persist in NSTEMI unlike UA, however. Regardless of type, ACS should always be managed in collaboration with a cardiologist. Diagnosis of ACS The likelihood of CAD may be calculated from the medical history and presentation. One-third of myocardial infarctions may be clinically silent or

www.medicalebookpdf.com

without chest pain[4]. A high likelihood is indicated by any of the following: accelerating symptoms in the preceding 48 hours prolonged (>20 minutes) of chest pain at rest findings of congestive heart failure on the physical examination such as pulmonary rales or a new third heart sound transient ST-segment changes at least 0.5 mm or new (or presumed new) bundle-branch block cardiac troponin I > 0.1 ng/mL or similar biomarker age > 75 years An intermediate likelihood is indicated by the absence of high risk features and age > 70 years a known history of CAD or cerebrovascular disease slightly elevated cardiac biomarkers baseline ECG abnormalities angina at rest, now resolved Patients at intermediate likelihood of CAD usually require further diagnostic evaluation to make a determination of ACS or CAD. There is typically a low likelihood of CAD or ACS with atypical symptoms, a normal ECG and no laboratory evidence of myocardial necrosis. Such patients may have a subsequent medical evaluation on an inpatient or outpatient basis, depending on the clinical situation.

Biomarkers. Troponins increase 2-3 hours after MI onset in 80% of patients [5]. Most centers use contemporary troponin (cTnI or TnI), with a highly sensitive TnI (hsTnI) recently available. HsTnI is better in evaluating for myocardial

www.medicalebookpdf.com

ischemia if performed in patients with chest pain of 75 years or older and weight less than 60 kg. For patients at high risk of bleeding, clopidogrel 300 to 600 mg (600 mg is the dose preferred) is preferred to ticagrelor or prasugrel [16]. Glycoprotein IIb/IIIa inhibitors are very potent at blocking platelet aggregation, and their effect is most pronounced in patients undergoing percutaneous coronary intervention (PCI), such as angioplasty [17]. Anticoagulation should be added as soon as possible. Unfractionated heparin reduces event rates in patients with ACS. Most clinical investigations have found that low-molecular weight heparins lead to better outcomes than unfractionated heparin [18]. Recent adverse events and deaths tied to adulterated heparins may accentuate this benefit. Fondaparinux may have a lower risk of bleeding and is a reasonable alternative, although there are fewer data available than for heparin and enoxaparin [19].

www.medicalebookpdf.com

Figure 19-1. Initial Management of Acute Coronary Syndrome

Beta-blockers (e.g. metoprolol) decrease the mortality in ACS, and should be prescribed to all such patients unless there are strong contra-indications such as symptomatic bradycardia, severe heart block or severe reactive airways disease. When beta-blockers are contra-indicated calcium channel blockers such as diltiazem and verapamil are alternatives. ACE-inhibitors are associated with reduced mortality, especially in patients with a reduced ejection fraction and heart failure. When these cannot be given (e.g. hypersensitivity, pulmonary edema, and angioedema of the tongue) angiotensin receptor blockers should be considered. HMG-CoA reductase inhibitors (“statins”) should be given as soon as possible and reduce mortality. Recently, the American College of Cardiology/American Heart Association Task Force on Practice Guidelines released its new guidelines for statin therapy in atherosclerotic cardiovascular disease (ASCVD)[75]. ASCVD includes coronary artery disease, stroke, and

www.medicalebookpdf.com

peripheral arterial disease. The task-force identified 4 groups who most benefited from statin therapy: first, individuals with clinical ASCVD; second, individuals with primary elevations of LDL–C ≥190 mg/dL; third, individuals 40 to 75 years of age with diabetes with LDL-C 70-189 mg/dL; and fourth, individuals without clinical ASCVD or diabetes who are 40 to 75 years of age with LDL-C 70-189 mg/dL and an estimated 10-year ASCVD risk of 7.5% or higher [75]. Figure 19-2 outlines the statin treatment algorithm. Table 19-3 provides statin options based upon need. For patients without clinical ASCVD, well calibrated and verified risk calculators can be found at http://my.americanheart.org/cvriskcalculator -or- http://www.cardiosource. org/science-and-quality/practice-guidelines-and-quality-standards/2013prevention-guideline-tools .aspx [76]. Of note for neurointensivists is the increasing risk of intracerebral hemorrhage with very low LDL and high-dose atorvastatin [20]. Of note, patients on statin therapy who receive intravenous thrombolysis for stroke do demonstrate increased risk for symptomatic intracerebral hemorrhage, directly correlating with dose [77]. However, stating use was associated with greater likelihood of achieving a modified Rankin Score of 0-2 at 3 months than was non-use. In statin naive patients, initiating statin therapy during hospitalization is not associated with and increased risk of intracerebral hemorrhage [78]. Overall, patients benefit from high-dose statin therapy because there is a greater reduction in the risk of coronary events and ischemic stroke. Based on the PROVEIT-TIMI 22 and MIR-ACL trials, 80 mg of atorvastatin should be started as soon as possible [21, 22].

Potassium should be targeted to > 4 meq/L and magnesium ideally > 2

www.medicalebookpdf.com

mg/dL[23]. Nitrates have a coronary vasodilatory effect and provide symptomatic relief. Oxygen should be given and hypoxemia strictly avoided with supplementary oxygen. Percutaneous coronary intervention Percutaneous coronary intervention (PCI) is a life-saving procedure for patients with STEMI and myocardial ischemia in the hands of experienced operators. Intervention is indicated for refractory chest pain hemodynamic instability electrical instability stabilized patients with an elevated risk for cardiovascular events Primary PCI is recommended for STEMI should be performed within 90 minutes by experienced operators. The benefits of reperfusion depend on the time to reperfusion [24]. Mortality is decreased by 50% when myocardial reperfusion is restored within an hour, but unchanged when reperfusion is delayed by 12 hours.

Fibrinolytic therapy. Coronary artery reperfusion with fibrinolytic therapy, when given in a timely manner, improves outcomes compared to no reperfusion therapy in patients STEMI [27]. If the patient is within 12 hours from onset, and no contraindications to fibrinolytic therapy exist, and PCI is not available within an appropriate window (90 minutes or less for patients transported to PCIcapable hospital or 120 minutes or less for patients who are transported first to a non-PCI capable hospital then taken to a PCI capable hospital) then fibrinolysis

www.medicalebookpdf.com

is recommended. In patients between 12 to 24 hours, if PCI is not available and symptoms persist, fibrinolysis should be pursued. Fibrinolytic therapy should be initiated within 30 minutes of STEMI diagnosis. The choice of fibrinolytic agent should be in favor of a fibrin-specific agent (Table 19-5). Patients receiving fibrinolytics benefit from pre-intervention clopidogrel but not from glycoprotein IIb/IIIa inhibition [28]. Anticoagulation with heparin should be given to patients who undergo fibrinolysis with alteplase, reteplase, and tenecteplase. Lowmolecular weight heparins also seem to be effective for this indication.

Complications of fibrinolytic therapy. Fibrinolytic therapy carries the risks of reocclusion of the vessel and distant hemorrhage. GI hemorrhage can be severe but can usually be treated with reversal of anticoagulation and packed red blood cell transfusion. ICH is a devastating complication in approximately 1% of treated patients, and is related to dose, previous cerebral infarction, amyloid angiopathy, structural vascular lesions and higher blood pressure [8].

www.medicalebookpdf.com

Figure 19-2. Recommendations for Statin Therapy

AORTIC DISSECTION Often described as a tearing sensation in the anterior chest, or as sharp, posterior back pain, an aortic dissection may clinically mimic ACS. Horner's syndrome from interruption of sympathetic nerve fibers may be seen. The Stanford and DeBakey systems are used to classify this disease (Table 19-6). Risk factors for spontaneous aortic dissection are similar to those for ACS, but trauma and sudden deceleration (in a restrained head-on motor vehicle collision) may lead to a traumatic dissection. The diagnosis may be made non-invasively with CT angiography of the chest, transesophageal echocardiography, or angiography (aortic catheterization may be precluded by a false lumen).

www.medicalebookpdf.com

Management is largely based on type, with medical management focusing on blood pressure and left ventricle contractile force reductions [29]. Regardless of type, early consultation with a cardiothoracic surgeon for definitive repair is mandatory. Type A dissections are a surgical emergency, with 72% of cases being managed surgically [30]. Type B dissections typically focus on medical management with only 20% typically being managed surgically. Surgical management becomes more likely if the dissection progresses to include endorgan ischemia or hemorrhage. Recently the INSTEAD Trial investigated the effect of aortic stent-grafting in uncomplicated type B dissections on 2 year outcome [31]. While underpowered to answer questions on mortality, the authors found no difference in medical vs. stent-graft treated arms. Targets for medical intervention are SBP 48 hours after ACS is a poor prognostic sign and may imply recurrent ischemia. Monomorphic VT is often from scar tissue, while polymorphic VT is more likely because of recurrent ischemia, abnormal electrolytes or medical therapy. Cardiac anti-arrhythmic medications do indeed suppress VT, but are associated with a higher rate of sudden cardiac death, and are not recommended. An arrhythmia with a depressed ejection fraction should prompt consideration of a pacemaker or defibrillator. Amiodarone and/or lidocaine should be considered if correcting electrolyte abnormalities does not lead to resolution of the arrhythmia. Narrow tachycardias often reflect activation of the ventricles through a normal His-Purkinje system, thus suggesting the arrhythmia starts at or above the AV node. A tachycardia with QRS > 120 ms is found when the activation occurs outside the normal conduction system.

www.medicalebookpdf.com

Bradycardia (heart rate < 60 beats/min) may be due to ischemia of cardiac conduction tissue, pharmacologic beta blockade, irritated vagal receptors or increased vagal tone. Heart block with inferior MI is usually transient with a stable escape rhythm, while heart block with an anterior MI usually indicates severe ischemic disease and requires an implantable pacemaker. Tachycardia (heart rate > 100 beats/min) is commonly from atrial fibrillation and supraventricular tachycardias. Beta-blockers are preferred for rate control. Amiodarone is generally preferred if this is insufficient. See the separate section on tachyarrhythmias for further management of these conditions. STRESS RELATED CARIOMYOPATHY AND “TAKO-TSUBO” CARDIOMYOPATHY A catecholamine surge accompanies acute SAH, but may also occur in patients who use cocaine and after severe emotional stress [32]. Tako-Tsubo Cardiomyopathy (TTC) and stress related cardiomyopathy (SRC) are terms used to describe a spectrum of diseases with common physiologic and imaging findings. They will be used interchangeably here [33]. Investigations into the

www.medicalebookpdf.com

cause of stress related cardiomyopathy vs. Killip class III myocardial infarction suggest a central etiology to this process [34]. Cardiac contractility initially increases, then decreases approximately 24 – 48 hours later. Catecholamines induce calcium overload and elevations of oxidative free-radicals in cardiomyocytes [35]. Histology demonstrates contraction band necrosis, neutrophil infiltration, and myocardial fibrosis suggesting elevations of myocardial calcium [36]. Serum levels of dihydroxyphenylalanine, dihydroxyphenylglycol, and di-hydroxyphenylacetic acid among patients with stress related cardiomyopathy are markedly increased compared to those with MI [35]. This suggests enhanced catecholamine synthesis, neuronal reuptake, and neuronal metabolism. Further, epinephrine and norepinephrine levels are elevated in SRC. Plasma levels of neuropeptide Y, which is stored with catecholamines in postganglionic sympathetic nerves and released during stress, are markedly increased among these patients too, suggesting a central process. Stress related cardiomyopathy demonstrates regional wall motion abnormalities (RWMA) not specific to a coronary distribution and without accompanying CAD [36, 37]. In the case of TTC, a set of diagnostic criteria has been proposed and include transient LV wall motion abnormalities involving the apical and/or mid-ventricular myocardial segments with wall motion abnormalities extending beyond a single epicardial coronary artery distribution, absence of obstructive epicardial coronary artery disease that could be responsible for the observed wall motion abnormality, ECG abnormalities, such as transient ST-segment elevation and/or diffuse T-wave inversion associated with a slight troponin elevation, and the lack of proven pheochromocytoma and myocarditis [38]. Common findings on ECG include QT interval prolongation, ST segment elevation, and T-wave inversions. These are often found in the setting of minimally increased troponin concentration in serum. Cardiac troponin I levels typically peak within 3 days, with elevations of 0.5 – 10 mcg/L predictive of vasospasm, cardiovascular complications, cerebral infarction and death in subarachnoid hemorrhage [39].

www.medicalebookpdf.com

When looking at all-comers with TTC, near complete LV recovery is common [40]. However, while recovery is common, death related to non-cardiac causes is common in this population [41]. In a prospectively followed cohort of SAH patients, partial or full recovery of LV function occurred in 66% of patients [42]. The presence of regional wall motion abnormalities (RWMA) was not associated with increased in-hospital mortality when compared to patients without ventricular dysfunction. A retrospective review of a prospectively collected database found LV dysfunction in 11% of patients in a sample of 481 patients with SAH [40]. The presence of LV dysfunction was associated with cerebral infarction, hypotension, and pulmonary edema, but not with modified Rankin Scale score at day 14. This suggests aggressive neurocritical care confers a protective benefit to this population. What specifically helps is uncertain. Some studies have suggested beta-blockade may be of benefit [43]. A recent trial looking at intraoperative beta-blockade in SAH showed it lowered markers of brain injury [44]. However, a large study investigating the effects of early betablockade on outcome is lacking. The development of delayed cerebral ischemia or vasospasm in SAH in the presence of neurogenic stunned myocardium presents a management dilemma. The combination of TTC and pulmonary edema often is found in patients with high grade SAH and a posterior circulation aneurysm location [72]. Whereas commonly diuresis may be used in treating pulmonary edema, in SAH this practice is not advised. Patients with markers of hypovolemia are at higher risk for the development of delayed cerebral ischemia [73]. While blood pressure can be initially elevated with vasopressors, these are pharmacologic catecholamines and will often lead to a vicious cycle of increased afterload, worse left ventricular performance, and hypotension requiring more vasopressors. If the ejection fraction is depressed, one can improve left ventricular performance with

www.medicalebookpdf.com

dobutamine or milrinone, but these may also lead to hypotension because of their vasodilatory effect[45]. Use of an intra-aortic balloon counter pulsation pump to treat delayed cerebral ischemia has been reported with good results [74]. Patients with vasospasm and an elevated cardiac troponin may be better treated with intra-arterial vasodilators. The syndrome usually resolves spontaneously by 10 – 14 days, but re-challenging with vasopressors may lead to an increase in cardiac troponin and left ventricular failure. PULMONARY EDEMA Pulmonary edema may have different causes in the neuro-ICU compared to medical or surgical ICUs. Causes of pulmonary edema include Severe left ventricular dysfunction leading to pulmonary edema from an inability to move liquid from the lungs into the arterial circulation Mitral regurgitation, whereby blood in the left ventricular is ejected partially into the arterial circulation, and party into the low-pressure pulmonary circulation Neurogenic pulmonary edema, the pathophysiology of which is not entirely clear. Some component is leaking capillaries in the lungs, but this often coexists with neurogenic stunned myocardium and is accompanied by an elevated cardiac troponin measurement. Volume overload, especially from hypervolemic therapy in patients with SAH Acute lung injury or the acute respiratory distress syndrome from volutrauma/barotrauma, sepsis, pneumonia, abdominal processes, etc. The treatment of pulmonary edema depends on the underlying etiology and the effectiveness of its treatment. Neurogenic pulmonary edema usually resolves in 10 – 14 days, but typically much sooner. If oxygenation is acceptable, pulmonary edema may be tolerated for the sake of other goals (e.g. blood pressure augmentation). PULMONARY EMBOLISM Pulmonary embolism (PE) is a common preventable cause of increased length of stay, respiratory distress and poor outcome. Major risk factors for PE including prolonged immobility, enforced bed rest, paresis of a limb and hypercoagulable states are common in neurologically critically ill patients. Acute pulmonary

www.medicalebookpdf.com

embolism (PE) carries a 30% mortality when not treated promptly [46]. The diagnosis of PE is often nebulous. Commonly the findings are nonspecific. Physical exam findings of deep venous thrombosis (new extremity edema, redness or tenderness) may also be seen. The mainstay of diagnosis has become CT angiography of the chest with visualization of large vessels [47]. Among the most concerning complications of PE are increased right ventricular (RV) strain and impaired gas exchange. Typically findings on the blood gas are decreased arterial oxygenation with increased alveolar-arterial oxygen gradient [48]. As blood flow is shunted from the obstructed pulmonary arteries to better ventilated alveoli, the ventilation-perfusion mismatch will increase. Hyperventilation may lead to a respiratory alkalosis. However, as dead space increases, hypercapnia may develop. Therapy for PE is based upon the severity of the underlying thromboembolism. In the resuscitation phase, hemodynamic support requires judicious use of fluids as RV failure, RV ischemia, and leftward septal shift can be precipitated by excessive volume [49]. Vasopressors and inotropes for hemodynamic support should be administered early [49]. However, no randomized trials have definitively demonstrated the optimal vasopressor or inotrope for patients with shock related to PE. Though lacking definitive evidence, in massive PE, unless contraindicated by bleeding risk, thrombolytic therapy is recommended [50]. Patients with an intracranial neoplasm, stroke or intracranial surgery or traumatic brain injury within the previous 2 months, a history of cerebral hemorrhage, and uncontrolled hypertension are contraindicated to receive thrombolysis [51]. A meta-analysis of randomized trials comparing heparin to thrombolysis suggests thrombolysis reduces mortality and recurrence of thromboembolism [52]. Patients receiving thrombolytic therapy demonstrate both short and longterm improvements in pulmonary artery blood pressure and RV function [53]. Thrombolysis should be reserved for patients with persistent hypotension of a systolic BP < 90mmHg [51]. Tissue plasminogen activator should be given at a dose of 100mg over 2 hours and heparin should be administered following the tPA infusion shortly thereafter [50, 51]. In cases with stable blood pressure but hypoxemia, larger ventilation/perfusion mismatch, RV dysfunction, free-floating thrombus, or extensive embolic burden, the role of thrombolysis is less certain [54]. The recently reported Moderate Pulmonary Embolism Treated with Thrombolysis (MOPETT) trial reported while no significant difference in death or recurrent PE with half the standard thrombolytic dose. However, thrombolysis was associated with significant reductions in pulmonary artery pressure and lower rates of

www.medicalebookpdf.com

pulmonary hypertension [55]. In patients who are stable with a submassive PE, anticoagulation is the recommended therapy [56]. Anticoagulation with unfractionated heparin or therapeutic doses of low-molecular weight heparin, followed by warfarin for at least 3 months. The intensity of anticoagulation, however, often must be tempered by a risk of potential CNS hemorrhage. Some authorities avoid the use of low-molecular weight heparins because these agents are not completely reversible with sodium protamine in the event of bleeding, or start unfractionated heparin without the usual bolus at the initiation of therapy. (Neither of these approaches has been validated in clinical trials.) For patients with PE or deep venous thrombosis, inferior vena cava filters are often placed. Such devices reduce the risk of subsequent PE (when combined with anticoagulation), but may not reduce mortality and have a substantial risk of a post-phlebitic syndrome [57]. HYPERTENSIVE URGENCY AND EMERGENCY Severe hypertension may be the cause or the effect of acute neurologic illness, especially cerebral hemorrhage. Hypertensive urgency means end-organ damage may be imminent, while an emergency implies end-organ damage is in progress. Severe hypertension (usually systolic blood pressure > 200 mm Hg) may overwhelm cerebral autoregulation, leading to hypertensive encephalopathy (now referred to as the posterior reversible encephalopathy syndrome, or PRES). Other causes include cocaine use, renal artery stenosis (leading to a high renin state), pheochromocytoma, and hyperaldosteronism. The usual treatment is to lower the blood pressure by 25% or back to the patient's baseline, whichever is a less dramatic change from presentation, and follow the patient's clinical course. A more aggressive goal may be mandated by end-organ damage (encephalopathy, ACS, etc.). A rapidly effective, easily titratable agent is ideal. Intravenous calcium channel blockers and labetalol are most commonly used, followed by repeated doses of non-titratable IV medications (e.g. enalaprilat). Nitrates and vasodilators (e.g. hydralazine) are effective for lowering blood pressure, but they are used less commonly in NeuroICUs because cerebral vasodilatation may lead to increased intracranial pressure. CARDIOVASCULAR MONITORS A number of cardiovascular monitors are present in the neurocritical care environment, ranging from telemetry and pulse-oximetry, arterial line transduction, pulmonary-artery catheterization (PAC), bio-impedance, and

www.medicalebookpdf.com

dilution techniques. It cannot be overstated however, that much of the data for the use of these devices is extrapolated from non-neurologic disease processes. The building blocks of more complicated formulas are stroke volume (amount of blood ejected with each heart beat), cardiac output (stroke volume times number of beats per minute), and body surface area (used to correct for body size). The classic monitoring is PAC. PAC calculates cardiac output using the Fick principle. Briefly, the Fick principle states oxygen consumption must be equal to the difference of arterial and venous oxygen concentrations multiplied by blood flow. This methodology loses efficacy when the FiO2 is > 60%. Enthusiasm for PAC use has waned since the results of the ESCAPE trial were published [58]. Performed between 2000 and 2003, this trial evaluated the use of PAC for patients with advanced heart failure. The use of PAC increased adverse events, and did not affect overall mortality or length of hospitalization [58]. Some authorities feel that after several prospective, randomized clinical trials showing no benefit from pulmonary catheter placement this intervention is no longer justified because of the potential risks of arrhythmia and pulmonary artery hemorrhage [58]. Instead others feel that PAC can still be valuable for titrating vasoactive therapy and assessing the response to treatment in the appropriate setting. Dilutional techniques, via lithium or thermal measurements, have provided another, less invasive means of monitoring. These technologies use modifications in the Stewart-Hamilton equation to calculate cardiac output. Using the systolic portion of the arterial pressure waveform, the dilutional data and waveform data are calibrated to increase accuracy. Particularly with thermal dilution, this technology becomes more variable in the non-ventilated patient were respiratory cycle variation may affect measurements. Further, right-to-left intracardiac shunts and/or pulmonary and tricuspid valve regurgitation may cause falsely high readings of cardiac output. Comparisons of PAC to thermodilution techniques have demonstrated good correlation in cardiac patients [59, 60]. Use of PiCCO (Phillips), which combines pulse contour analysis with intermittent thermodilution measurement via the transpulmonary method, has been demonstrated effective in predicting volume responsiveness and mitigating pulmonary edema in the SAH population [61,62]. However, larger trials are needed before this or similar technology can be endorsed for the management of SAH or other neurocritical care diseases. Lithium dilution techniques have the advantage of allowing for peripheral injection of lithium and requiring a radial arterial line. This technique has been found to provide accurate measurement of CO in critically ill patients [63].

www.medicalebookpdf.com

The more important question is how to interpret the numbers, and how to use them for treatment decisions (if at all). As an assessment of fluid responsiveness, central venous pressure (CVP) has been used. However, the relationship between CVP and hemodynamic response to fluid challenge or between CVP and blood volume is poor [64]. Employing arterial waveform analysis, FloTrac (Edwards Lifesciences Corp, Irvine, CA, USA) attempts to provide information on vascular resistance and cardiac output. Unfortunately, particularly for patients on vasopressors such as norepinephrine, this technique may be unreliable [65]. PERICARDIAL EFFUSION AND TAMPONADE Pericardial effusions most often occur as a result of viral, autoimmune, traumatic, post-operative, or malignant causes. Acute cardiac tamponade is life threatening if not acutely treated. Exam findings suggestive of cardiac tamponade include sinus tachycardia, elevated jugular venous pressure, a pericardial rub on auscultation, and pulsus paradoxus or a >10mmHg reduction in systolic blood pressure with inspiration. Hypotension is common. Patients suspected of having a cardiac tamponade should be studied with an ECG, chest X-ray, and echocardiogram. ECG findings typically show low voltage and/or an electrical alternans pattern. Low voltage may be specific to cardiac tamponade and not effusion [66]. Chest radiography may demonstrate cardiomegaly, but not typically until a volume > 200mL of pericardial fluid is present [67]. Echocardiography is crucial in assessing pericardial effusions and cardiac tamponade [68]. Treatment of cardiac tamponade focuses on removal of the offending fluid and supportive care. When the etiology of tamponade is thought to be the result of cardiac rupture or aortic dissection, emergent surgery in necessary. Rapid recovery of both systemic and cardiac hemodynamics will follow [69]. Fluid may be removed via catheter pericardiocentesis, open surgical drainage, or thoracoscopic pericardiectomy. Fluid recovered from such procedures should be examined for the underlying etiology. Follow-up echocardiography is recommended to ensure recurrence has not occurred. Supportive care with volume in the form of saline or blood products may help stabilize the patient until definitive care can be delivered [70]. The utility of inotropic agents, while theoretically plausible, in practice is uncertain [70]. When possible, efforts to avoid mechanical ventilation should be made to prevent further reductions in preload [71]. When patients are hemodynamically stable and a pericardial effusion is present, urgent drainage may not be needed. Investigation of the etiology and treatment focused on the underlying cause may

www.medicalebookpdf.com

preclude need for surgical or percutaneous drainage. REFERENCES 1. Wilson PW, D'Agostino RB, Levy D, Belanger AM, Silbershatz H, Kannel WB. Prediction of coronary heart disease using risk factor categories. Circulation. 1998; 97:1837-47. 2. Anderson JL, Adams CD, Antman EM, Bridges CR, Califf RM, Casey DE Jr, et al. ACC/AHA 2007 guidelines for the management of patients with unstable angina/non-ST-Elevation myocardial infarction: a report of the American College of Cardiology/American Heart Association Task Force on Practice Guidelines. 2007; 50:e1-e157. 3. Thygesen K, Alpert JS, Jaffe AS, Simoons ML, Chaitman BR, White HD, et al; Joint ESC/ACCF/ AHA/WHF Task Force for the Universal Definition of Myocardial Infarction. Third universal definition of myocardial infarction. Circulation. 2012; 126:2020-35. 4. Canto J, Shlipak M, Rogers W, et al.: Prevalence, clinical characteristics, and mortality among patients with myocaridal infarction presenting without chest pain. JAMA. 2000;283:3223-3229 5. Macrae AR, Kavsak PA, Lustig V, et al. Assessing the requirement for the 6-hour interval between specimens in the American Heart Association Classification of Myocardial Infarction in Epidemiology and Clinical Research Studies. Clin Chem. 2006; 52:812-8. 6. Keller T, Zeller T, Ojeda F, et al. Serial changes in highly sensitive troponin I assay and early diagnosis of myocardial infarction. JAMA. 2011; 306:2684-93. 7. Antman EM, Hand M, Armstrong PW, et al. Focused Update of the ACC/AHA 2004 Guidelines for the Management of Patients With STElevation Myocardial Infarction: a report of the American College of Cardiology/American Heart Association Task Force on Practice Guidelines: developed in collaboration With the Canadian Cardiovascular Society. Circulation. 2008; 117:296329. 8. Sloan M, Price T, Petito C, et al.: Clinical features and pathogenesis of intracerebral hemorrhage after rt-PA and heparin therapy for acute myocardial infarction: the Thrombolysis in Myocardial Infarction (TIMI) II Pilot and Randomized Clinical Trial combined experience. Neurology

www.medicalebookpdf.com

1995;45:649-658. 9. Van den Berghe G, Wilmer A, Hermans G, et al.: Intensive insulin therapy in the medical ICU. New Engl J Med. 2006;354:449-461. 10. Mohr J, Thompson J, Lazar R, et al.: A comparison of warfarin and aspirin for the prevention of recurrent ischemic stroke. New Engl J Med 2001;345:1444-1451. 11. McCord J, Jneid H, Hollander JE, et al. American Heart Association Acute Cardiac Care Committee of the Council on Clinical Cardiology. Management of cocaine-associated chest pain and myocardial infarction: a scientific statement from the American Heart Association Acute Cardiac Care Committee of the Council on Clinical Cardiology. Circulation. 2008;117:1897-907. 12. Catella-Lawson F, Reilly M, Kapoor S, et al.: Cyclooxygenase inhibitors and the antiplatelet effects of aspirin. New Engl J Med. 2001;345:18091817. 13. Yusuf S, Zhao F, Mehta S, et al.: Effects of clopidogrel in addition to aspirin in patients with acute coronary syndromes without ST-segment elevation. New Engl J Med. 2001;345:494-502. 14. Bhatt D, Fox K, Hacke W, et al.: Clopidogrel and aspirin versus aspirin alone for the prevention of atherothrombotic events. New Engl J Med. 2006;354:1706-1717. 15. Breet NJ, van Werkum JW, Bouman HJ, et al.: Comparison of platelet function tests in predicting clinical outcome in patients undergoing coronary stent implantation. JAMA 2010;303:754-762. 16. Wouter Jukema J, Collet JP, De Luca L. Antiplatelet therapy in patients with ST-elevation myocardial infarction undergoing myocardial revascularisation: beyond clopidogrel. Curr Med Res Opin. 2012; 28:20311. 17. Kandzari DE, Hasselblad V, Tcheng JE, et al.: Improved clinical outcomes with abciximab therapy in acute myocardial infarction: a systematic overview of randomized clinical trials. Amer Heart J. 2004;147:457-462. 18. Wong G, Giugliano R, Antman EM: Use of low-molecular-weight heparins in the management of acute coronary artery syndromes and percutaneous coronary intervention. JAMA. 2003;289:331-342.

www.medicalebookpdf.com

19. Yusuf S, Mehta S, Chrolavicius S, et al.: Effects of fondaparinux on mortality and reinfarction in patients with acute ST-segment elevation myocardial infarction: the OASIS-6 randomized trial. JAMA. 2006;295:1519-1530. 20. Goldstein L, Amarenco P, Szarek M, et al.: Hemorrhagic stroke in the Stroke Prevention by Aggressive Reduction in Cholesterol Levels study. Neurology. 2008;70:2364-2370. 21. Cannon CP, Weintraub WS, Demopoulos LA, et al. TACTICS (Treat Angina with Aggrastat and Determine Cost of Therapy with an Invasive or Conservative Strategy)--Thrombolysis in Myocardial Infarction 18 Investigators. Comparison of early invasive and conservative strategies in patients with unstable coronary syndromes treated with the glycoprotein IIb/IIIa inhibitor tirofiban. N Engl J Med. 2001; 344:1879-87. 22. Schwartz GG, Olsson AG, Ezekowitz MD, et al. Myocardial Ischemia Reduction with Aggressive Cholesterol Lowering (MIRACL) Study Investigators. Effects of atorvastatin on early recurrent ischemic events in acute coronary syndromes: the MIRACL study: a randomized controlled trial. JAMA. 2001; 285:1711-8. 23. Goyal A, Spertus JA, Gosch K, et al. Serum potassium levels and mortality in acute myocardial infarction. JAMA. 2012;307:157-64. 24. Nallamothu BK, Bates ER: Percutaneous coronary intervention versus fibrinolytic therapy in acute myocardial infarction: is timing (almost) everything? Am J Cardiol. 2003;82:824-826. 25. Antman EM, Cohen M, Bernink PJ, et al. The TIMI risk score for unstable angina/non-ST elevation MI: A method for prognostication and therapeutic decision making. JAMA. 2000; 284:835-42. 26. Eagle KA, Guyton RA, Davidoff R, et al. American College of Cardiology; American Heart Association. ACC/AHA 2004 guideline update for coronary artery bypass graft surgery: a report of the American College of Cardiology/American Heart Association Task Force on Practice Guidelines (Committee to Update the 1999 Guidelines for Coronary Artery Bypass Graft Surgery). Circulation. 2004;110:e340-437. 27. O'Gara PT, Kushner FG, Ascheim DD, et al. 2013 ACCF/AHA guideline for the management of ST-elevation myocardial infarction: executive summary: a report of the American College of Cardiology

www.medicalebookpdf.com

Foundation/American Heart Association Task Force on Practice Guidelines. Circulation. 2013;127:529-55. 28. Goodman SG, Menon V, Cannon CP, Steg G, Ohman EM, Harrington RA; American College of Chest Physicians. Acute ST-segment elevation myocardial infarction: American College of Chest Physicians EvidenceBased Clinical Practice Guidelines (8th Edition). Chest. 2008;133:708S775S. 29. Erbel R, Alfonso F, Boileau C, et al. Task Force on Aortic Dissection, European Society of ardiology. Diagnosis and management of aortic dissection. Eur Heart J. 2001;22:1642-81. 30. Hagan PG, Nienaber CA, Isselbacher EM, et al. The International Registry of Acute Aortic Dissection (IRAD): new insights into an old disease. JAMA. 2000;283:897-903. 31. Nienaber CA, Rousseau H, Eggebrecht H, et al. INSTEAD Trial. Randomized comparison of strategies for type B aortic dissection: the INvestigation of STEnt Grafts in Aortic Dissection (INSTEAD) trial. Circulation. 2009;120:2519-28. 32. Tung P, Kopelnik A, Banki N, et al.: Predictors of neurocardiogenic injury after subarachnoid hemorrhage. Stroke. 2004;35:548-551. 33. Richard C. Stress-related cardiomyopathies. Ann Intensive Care. 2011;1:39. 34. Wittstein IS, Thiemann DR, Lima JA, et al. Neurohumoral features of myocardial stunning due to sudden emotional stress. N Engl J Med. 2005;352:539-48. 35. Bolli R, Marbán E. Molecular and cellular mechanisms of myocardial stunning. Physiol Rev. 1999;79:609-34. 36. Samuels MA. The brain-heart connection. Circulation. 2007;116:77-84. 37. Akashi YJ, Goldstein DS, Barbaro G, Ueyama T. Takotsubo cardiomyopathy: a new form of acute, reversible heart failure. Circulation. 2008;118:2754-62. 38. Bybee KA, Prasad A. Stress-related cardiomyopathy syndromes. Circulation. 2008; 118:397409. 39. Naidech AM, Kreiter KT, Janjua N, et al.: Cardiac troponin elevation, cardiovascular morbidity, and outcome after subarachnoid hemorrhage.

www.medicalebookpdf.com

Circulation. 2005;112:2851-2856. 40. Parodi G, Bellandi B, Del Pace S, et al. Tuscany Registry of Tako-Tsubo Cardiomyopathy. Natural history of tako-tsubo cardiomyopathy. Chest. 2011;139:887-92. 41. Sharkey SW, Windenburg DC, Lesser JR, et al. Natural history and expansive clinical profile of stress (tako-tsubo) cardiomyopathy. J Am Coll Cardiol. 2010;55:333-41. 42. Banki N, Kopelnik A, Tung P, et al. Prospective analysis of prevalence, distribution, and rate of recovery of left ventricular systolic dysfunction in patients with subarachnoid hemorrhage. J Neurosurg. 2006;105:15-20. 43. Temes RE, Tessitore E, Schmidt JM, et al. Left ventricular dysfunction and cerebral infarction from vasospasm after subarachnoid hemorrhage. Neurocrit Care. 2010;13:359-65. 44. Kawaguchi M, Utada K, Yoshitani K, et al. Intraoperative Landiolol for Intracranial Aneurysm Surgery Trial (ILAST) Investigators. Effects of a short-acting [beta]1 receptor antagonist landiolol on hemodynamics and tissue injury markers in patients with subarachnoid hemorrhage undergoing intracranial aneurysm surgery. J Neurosurg Anesthesiol. 2010;22:230-9. 45. Naidech A, Du Y, Kreiter KT, et al.: Dobutamine versus milrinone after subarachnoid hemorrhage. Neurosurgery. 2005;56:21-26l. 46. Anderson FA Jr, Wheeler HB, Goldberg RJ, et al. A population-based perspective of the hospital incidence and case-fatality rates of deep vein thrombosis and pulmonary embolism. The Worcester DVT Study. Arch Intern Med. 1991;151:933-8. 47. van Strijen MJL, de Monyé W, Schiereck J, et al.: Single-Detector Helical Computed Tomography as the Primary Diagnostic Test in Suspected Pulmonary Embolism: A Multicenter Clinical Management Study of 510 Patients. Annals of Internal Medicine. 2003;138:307-314. 48. Piazza G, Goldhaber SZ. Acute pulmonary embolism: part I: epidemiology and diagnosis. Circulation. 2006;114:e28-32. 49. Piazza G, Goldhaber SZ. The acutely decompensated right ventricle: pathways for diagnosis and management. Chest. 2005;128:1836-52. 50. Kucher N, Goldhaber SZ. Management of massive pulmonary embolism. Circulation. 2005;112:e28-32.

www.medicalebookpdf.com

51. Kearon C, Akl EA, Comerota AJ, et al. American College of Chest Physicians. Antithrombotic therapy for VTE disease: Antithrombotic Therapy and Prevention of Thrombosis, 9th ed: American College of Chest Physicians Evidence-Based Clinical Practice Guidelines. Chest. 2012;141:e419S-94S. 52. Wan S, Quinlan DJ, Agnelli G, Eikelboom JW. Thrombolysis compared with heparin for the initial treatment of pulmonary embolism: a metaanalysis of the randomized controlled trials. Circulation. 2004;110:744-9. 53. Sharma GV, Folland ED, McIntyre KM, Sasahara AA. Long-term benefit of thrombolytic therapy in patients with pulmonary embolism. Vasc Med. 2000;5:91-5. 54. Goldhaber SZ. Modern treatment of pulmonary embolism. Eur Respir J Suppl. 2002;35:22s-27s. 55. Sharifi M, Bay C, Skrocki L, Rahimi F, Mehdipour M; “MOPETT” Investigators. Moderate pulmonary embolism treated with thrombolysis (from the “MOPETT” Trial). Am J Cardiol. 2013;111:273-7. 56. Piazza G, Goldhaber SZ. Fibrinolysis for acute pulmonary embolism. Vasc Med. 2010;15:419-28. 57. The PREPIC Study Group: Eight-Year Follow-Up of Patients With Permanent Vena Cava Filters in the Prevention of Pulmonary Embolism: The PREPIC (Prevention du Risque d'Embolie Pulmonaire par Interruption Cave) Randomized Study. Circulation. 2005;112:416-422. 58. Binanay C, Califf RM, Hasselblad V, et al. ESCAPE Investigators and ESCAPE Study Coordinators. Evaluation study of congestive heart failure and pulmonary artery catheterization effectiveness: the ESCAPE trial. JAMA. 2005;294:1625-33. 59. Zöllner C, Haller M, Weis M, et al. Beat-to-beat measurement of cardiac output by intravascular pulse contour analysis: a prospective criterion standard study in patients after cardiac surgery. J Cardiothorac Vasc Anesth. 2000;14:125-9. 60. Buhre W, Weyland A, Kazmaier S, et al. Comparison of cardiac output assessed by pulse-contour analysis and thermodilution in patients undergoing minimally invasive direct coronary artery bypass grafting. J Cardiothorac Vasc Anesth. 1999;13:437-40.

www.medicalebookpdf.com

61. Mutoh T, Kazumata K, Ishikawa T, Terasaka S. Performance of bedside transpulmonary thermo-dilution monitoring for goal-directed hemodynamic management after subarachnoid hemorrhage. Stroke. 2009;40:2368-74. 62. Watanabe A, Tagami T, Yokobori S, et al. Global end-diastolic volume is associated with the occurrence of delayed cerebral ischemia and pulmonary edema after subarachnoid hemorrhage. Shock. 2012;38:480-5. 63. Linton RA, Jonas MM, Tibby SM, et al. Cardiac output measured by lithium dilution and transpulmonary thermodilution in patients in a paediatric intensive care unit. Intensive Care Med. 2000;26:1507-11. 64. Marik PE, Baram M, Vahid B. Does central venous pressure predict fluid responsiveness? A systematic review of the literature and the tale of seven mares. Chest. 2008;134:172-8. 65. Monnet X, Anguel N, Jozwiak M, Richard C, Teboul JL. Third-generation FloTrac/Vigileo does not reliably track changes in cardiac output induced by norepinephrine in critically ill patients. Br J Anaesth. 2012;108:615-22. 66. Bruch C, Schmermund A, Dagres N, et al. Changes in QRS voltage in cardiac tamponade and pericardial effusion: reversibility after pericardiocentesis and after anti-inflammatory drug treatment. J Am Coll Cardiol. 2001;38:219-26. 67. Spodick DH. Acute cardiac tamponade. N Engl J Med. 2003;349:684-90. 68. Troughton RW, Asher CR, Klein AL. Pericarditis. Lancet. 2004;363:71727. 69. Reddy PS, Curtiss EI, O'Toole JD, Shaver JA. Cardiac tamponade: hemodynamic observations in man. Circulation. 1978;58:265-72. 70. Kerber RE, Gascho JA, Litchfield R, Wolfson P, Ott D, Pandian NG. Hemodynamic effects of volume expansion and nitroprusside compared with pericardiocentesis in patients with acute cardiac tamponade. N Engl J Med. 1982;307:929-31. 71. Little WC, Freeman GL. Pericardial disease. Circulation. 2006;113:162232. 72. Inamasu J, Nakatsukasa M, Mayanagi K, et al. Subarachnoid hemorrhage complicated with neurogenic pulmonary edema and takotsubo-like cardiomyopathy. Neurol Med Chir (Tokyo). 2012;52: 49-55.

www.medicalebookpdf.com

73. Yoneda H, Nakamura T, Shirao S, et al. Multicenter prospective cohort study on volume management after subarachnoid hemorrhage: hemodynamic changes according to severity of sub-arachnoid hemorrhage and cerebral vasospasm. Stroke. 2013 Aug;44: 2155-61. 74. Lazaridis C, Pradilla G, Nyquist PA, Tamargo RJ. Intra-aortic balloon pump counter pulsation in the setting of subarachnoid hemorrhage, cerebral vasospasm, and neurogenic stress cardio-myopathy. Case report and review of the literature. Neurocrit Care. 2010 Aug;13: 101-8. 75. Stone NJ, Robinson J, Lichtenstein AH, et al. 2013 ACC/AHA Guideline on the Treatment of Blood Cholesterol to Reduce Atherosclerotic Cardiovascular Risk in Adults: A Report of the American College of Cardiology/American Heart Association Task Force on Practice Guidelines. Circulation. 2013 Nov 12. 76. Muntner P, Colantonio LD, Cushman M, et al. Validation of the atherosclerotic cardiovascular disease Pooled Cohort risk equations.JAMA. 2014 Apr 9;311(14):1406-15. 77. Scheitz JF, Seiffge DJ, Tütüncü S, et al. Dose-related effects of statins on symptomatic intra-cerebral hemorrhage and outcome after thrombolysis for ischemic stroke.Stroke. 2014 Feb;45(2):509-14. 78. Chen PS, Cheng CL, Kao Yang YH, et al. Impact of early statin therapy in patients with ischemic stroke or transient ischemic attack. Acta Neurol Scand. 2014 Jan;129(1):41-8.

www.medicalebookpdf.com

CARDIOVASCULAR MONITORING QUESTIONS 1. A 28 year-old male graduate student presents with chest pain. His ECG demonstrates T wave flattening in the inferior leads and his troponin is slightly elevated at 0.3 ng/mL. A urine drug screen is positive for cocaine and opiates. Which of the following would be appropriate medical therapy? a. Aspirin, atorvastatin, metoprolol b. Aspirin, lorazepam, morphine, oxygen, atorvastatin c. Sublingual nitro, digoxin, clopidogrel, oxygen d. Lepirudin, nimodipine, morphine 2. A 55 year-old man is involved in an all-terrain vehicle crash. At presentation his GCS is 13, and his CT of head demonstrates a basilar skull fracture and a mild contusion affecting the left frontal lobe. He starts developing ST segment elevations on telemetry, and his ECG demonstrates ST segment elevations of 2 mm in leads I, aVL, V5-6. The nearest PCI capable center is 100 minutes away by air. What is the appropriate option? a. Start streptokinase b. Start tenecteplase c. Bolus with 2000 units heparin, give aspirin d. Metoprolol, oxygen, and flight to PCI center 3. A 77 year-old male presents with a Hunt-Hess 3, Fisher 4 SAH with hydrocephalus. The culprit left posterior communicating 6mm-aneurysm is coiled uneventfully and an EVD is placed. Post procedure his exam is unremarkable except for a minor headache. On post-admission day 6 he develops signs of delayed cerebral ischemia, and responds well to a 15% increase in MAP with phenylephrine. Post-admission day 8 he complains of shortness of breath, chest pain, and becomes mildly hypoxic. His ECG demonstrates diffuse T-wave flattening and his troponin is elevated at 1.4 ng/ mL. He remains hemodynamically stable. Subcutaenous enoxaparin was not started post-coiling. What is the next appropriate test? a. CTA- Pulmonary Embolism Protocol

www.medicalebookpdf.com

b. D-dimer c. Transcranial Doppler d. Transthoracic Echocardiography 4. The above test in Question 3 demonstrates a large saddle PE. Further, it demonstrates venous thrombosis in the right femoral and left popliteal veins. What is the appropriate management for this patient? a. IVC Filter, Alteplase b. Alteplase, heparin drip c. IVC Filter d. Start enoxaparin 5. Central venous pressure is the best measure of preload a. True b. False 6. A 22 year-old female who arrives to the ED after a motor vehicle accident at 30 mph without airbag deployment. She was the driver, and the collision was head on. During assessment, she becomes lethargic, her pulse pressure narrows, and she demonstrates JVD on examination. Chest-X ray demonstrates cardiomegaly. What would be the next most reasonable steps? a. CTA- Pulmonary Embolism Protocol b. Rapid sequence intubation and chest-X ray c. Bolus 1 L NS, US guided pericardiocentesis if fluid present on echo d. Change blood pressure cuff to calf and give 20mg IV furosemide 7. Thermodilution hemodynamic monitoring operates by calibrating what of the following to the systolic portion of the arterial wave form? a. Modified Stewart-Hamilton Equation b. Bladder temperature c. Central venous pressure

www.medicalebookpdf.com

d. Hemoglobin 8. A 24 year old black female presents with a Hunt-Hess 4, Fisher 4 SAH. Her ECG demonstrates prolonged QT intervals and inverted T waves in precordial leads. Bedside echo demonstrates apical ballooning. What is the most likely diagnosis and potential therapy? a. Stanford A aortic dissection/emergent surgical consultation b. NSTEMI/Aspirin, morphine, oxygen, statin, metoprolol c. Tako-Tsubo Cardiomyopathy/beta-blockade d. Cardiac tamponade/emergent pericardiocentesis 9. What is a contraindication to use of prasugrel? a. Lactose intolerance b. Prior stroke c. Prior MI d. Factor V Leiden heterozygote 10. Clopidogrel blocks what portion of which receptor on platelets? a. The NKoTB portion of the KNP receptor b. The COX-2 portion of the NF-kB receptor c. The Mg++ sight of the NMDA receptor d. The P2Y12 portion of the ADP receptor

www.medicalebookpdf.com

CARDIOVASCULAR MONITORING ANSWERS 1. The correct answer is B. In cocaine related ACS, use of beta blockade is not recommended. Use of benzodiazepines and calcium channel blockers is however. In all cases of ACS, use of aspirin, morphine, and statins are recommended. 2. The correct answer is D. The appropriate window for PCI is 90 minutes or less for patients transported to PCI-capable hospital or 120 minutes or less for patients who are transported to a non-PCI capable hospital first. When fibrinolysis is the best option, then typically tenecteplase is recommended. 3. The correct answer is A. PE protocol CTA is the study of choice for diagnosis of PE. 4. The correct answer is C. If patients are hemodynamically stable, then thrombolytics are typically withheld. When anticoagulation is contraindicated, then an IVC filter is a fair option. 5. The correct answer is B. CVP is not a reliable measure of preload in the critically ill. 6. The correct answer is C. This patient most likely has cardiac tamponade. IV fluids should help temporize the patient in preparation for drainage. 7. The correct answer is A. The correlation of arterial, systolic waveform analysis and the modified Stewart-Hamilton equation is the basis for thermodilution hemodynamic monitoring. 8. The correct answer is C. A young female with SAH and echocardiographic demonstration of ballooning is most suggestive of TakoTsubo Cardiomyopathy. Some evidence suggests beta blockade may have efficacy in treatment. 9. The correct answer is B. Prasugrel is contraindicated for patients with stroke. 10. The correct answer is D. Clopidogrel blocks the P2Y12 portion of the ADP receptor.

www.medicalebookpdf.com

Chapter 20

ENDOCRINE DISORDERS IN NEUROCRITICAL CARE Nancy Edwards and Kiwon Lee CLINICAL CASE A twenty year old man without any significant past medical history was brought to the emergency department following a motorcycle accident. His initial Glasgow Coma Score (GCS) was 6; he was intubated for airway protection and a head CT was obtained. He had fractured his left frontal, parietal, and temporal bones, and had a large holohemispheric subdural hematoma in addition to multifocal hemorrhagic contusions. The subdural hematoma was emergently evacuated and the patient was further resuscitated in the neurointensive care unit. Post-operatively, his neurological examination gradually improved and he was able to be extubated 6 days after his initial presentation. The next day, the nurse notified the team that the patient's urine output had abruptly increased to 300 to 600 mL/hr. A serum sodium level was elevated at 156. Additional serum and urine studies were obtained and were notable for a plasma osmolality of 300 mOsm/ kg, urine osmolality of 197 mOsm/kg, and urine specific gravity of 1.004. A diagnosis of diabetes insipidus (DI) was made, and the patient was given 1 microgram of desmopressin intravenously. This resulted in an appropriate decrease in his urine output to 30 mL/hr with a urine specific gravity of 1.019. For the next 3 days, desmopressin was given as needed whenever his urine output was greater than 250 mL/hr for two consecutive hours (approximately 1 dose of desmopressin every 12 hours), and hypo-osmolar intravenous (IV) fluids were given to replace his calculated free water deficit. His serum sodium began to normalize, and IV fluids were weaned and eventually discontinued. No additional desmopressin doses were needed. The patient continued to improve neurologically and was discharged to an acute rehabilitation facility. No further abnormalities in serum sodium or urine output were documented. OVERVIEW Identification of endocrine dysfunction and appropriate intervention can be quite challenging in the severely ill patient, especially those with acute neurological

www.medicalebookpdf.com

injury. Classically, severe illness activates the hypothalamic-pituitary-adrenal (HPA) axis, resulting in a rise in serum cortisol in order to mobilize energy stores and preserve vital organ function [1]. In the brain injured patient, however, multiple factors may impair this classic HPA response. For instance, the hypothalamus and pituitary are located on the floor of the calvarium and are therefore particularly vulnerable to direct mechanical injury in the patient who has sustained traumatic brain injury (TBI). The pituitary gland is perfused by a tenuous network of vessels and the perforating hypothalamic arteries lie within the subarachnoid space [2] – as such, vascular insults ranging from hemorrhage within the subarachnoid space to hypotension may result in HPA dysfunction. In addition, the HPA axis may be disrupted by many of the medications administered in the neurointensive care unit [3]. The goals of this chapter are to describe neuroendocrine dysfunction as it may be encountered in neurologically ill patients, highlight life-saving endocrine interventions, and briefly address several of the controversies in this field today. NEUROENDOCRINE DYSFUNCTION IN TBI PATIENTS TBI-induced endocrinopathies are not rare; published autopsy series have documented injury to the hypothalamic-pituitary region in 26-86% of patients who died acutely in the setting of TBI [4] and cross-sectional studies have documented an incidence of pituitary dysfunction ranging from 16-67% [5]. A large proportion of patients with evidence of endocrine dysfunction during the acute phase of TBI will recover function within 1 year, although additional patients may develop interval endocrine dysfunction during the subacute phase of recovery. When caring for the acutely brain-injured patient, the key is to be able to recognize potentially life threatening endocrinopathies. Disruption of the pituitary-adrenal axis with resultant acute adrenal insufficiency is one lifethreatening endocrinopathy. Another is acute salt and fluid imbalance due to posterior pituitary dysfunction. Pituitary-adrenal axis dysfunction Several studies have examined adrenocorticotropic hormone (ACTH) and cortisol deficiency during the acute phase of TBI. Although the reported incidence ranges from 4 to 53% [6], the exact incidence is unclear as there is considerable controversy regarding the diagnosis of adrenal insufficiency in critically ill patients. For example, there is no consensus cut-off value for the diagnosis of adrenal insufficiency in acute TBI, although several authors have proposed a basal 9 AM cortisol level below 300 nmol/L (11 µg/dL) to be suggestive [7]. Furthermore, even provocative testing such as the ACTH

www.medicalebookpdf.com

stimulation test may be unreliable in TBI patients as critical illness in and of itself may blunt the cortisol response to ACTH stimulation [8]. Nevertheless, it is reasonable to obtain these tests (the basal cortisol level in particular) and urgently initiate repletion therapy in patients with a clinical picture suggestive of deficiency. This includes: Hypotension refractory to volume resuscitation and requiring vasopressor administration Hyponatremia; usually with relative or absolute hyperkalemia Hypoglycemia Nausea, vomiting, abdominal pain, and fever may also be present. If there is a high clinical suspicion of adrenal insufficiency, patients should immediately receive stress dose steroids. A dose of 100 mg of IV hydrocortisone every 6 hours is a typical regimen. Salt and fluid imbalance Acute neurohypophyseal dysfunction typically presents with hypo- or hypernatremia. For instance, central diabetes insipidus (DI) is a well-recognized complication of TBI, with a reported incidence in the medical literature of 3 to 26% [9]. In healthy adults, arginine vasopressin binds to V2 receptors in the renal collecting tubule, stimulating water reabsorption to maintain salt and water homeostasis. In patients with central DI, there is a failure of antidiuretic hormone (ADH) release from the posterior pituitary, resulting in polyuria and hypernatremia. There is a fairly strong correlation of hypernatremia with mortality in this patient population [10], therefore it is essential to accurately diagnose and treat DI. Diabetes insipidus The diagnosis of diabetes insipidus should be considered when a patient excretes large volumes of dilute urine, typically greater than 2.5 mL/kg body weight per hour. Polyuria to this degree can also be caused by hyperglycemia (as glucosuria results in an osmotic diuresis), therefore a serum and urine glucose should be obtained. Post-operative patients receiving aggressive intravenous hydration may also develop polyuria, though in this circumstance, a patient's serum sodium is typically not elevated. In DI, urine studies should reveal hypotonic urine, with a specific gravity < 1.005 and a urine osmolality < 200 mOsm/kg. A treatment strategy for patients with central DI is outlined here in Figure 20-1.

www.medicalebookpdf.com

Figure 20-1. Treatment algorithm for central diabetes insipidus

A variant of DI essential to recognize is the triple phase response [11]. This is characterized by acute DI, typically lasting for 5 to 7 days, followed by a second, antidiuretic phase of the syndrome of inappropriate antidiuretic hormone secretion (SIADH). This second phase is caused by an unregulated release of

www.medicalebookpdf.com

pre-formed arginine vasopressin from degenerating posterior pituitary tissue. In this phase, the urine becomes concentrated and urine output markedly decreases. Continued administration of free water during this period can quickly lead to hyponatremia. The duration of the second phase is variable (2 to 14 days). Then, once arginine vasopressin stores are depleted, the third phase of permanent DI ensues. Because of this, desmopressin doses should be given urgently as needed, rather than regularly scheduled; a positive daily fluid balance of greater than 2 liters suggests SIADH. Hyponatremia independent of the above described triple phase response can also occur in TBI patients. There is controversy regarding the exact mechanism but in general, hyponatremia is due to either SIADH or cerebral salt wasting. The hyponatremia is nearly always transient, and is unrelated to the severity of the head injury [12]. Typically, hyponatremia will manifest within the first 2 days, though there are reports of a delayed SIADH beginning up to 18 days after the TBI. Lastly, one must recall that hyponatremia due to acute glucocorticoid deficiency is biochemically indistinguishable from SIADH; the presence of concurrent hypotension or hypoglycemia is a clue and should prompt rapid repletion with intravenous steroids. NEUROENDOCRINE DYSFUNCTION IN SUBARACHNOID HEMORRHAGE PATIENTS Patients with subarachnoid hemorrhage (SAH) may also develop neuroendocrine dysfunction, though there is far less data in the SAH population as compared with TBI. In one autopsy study, ischemic necrosis and microhemorrhage of the hypothalamus were evident in 68% of SAH patients [13]. Though the HPA axis is frequently intact in the SAH population in general, relative adrenal insufficiency should be considered in those patients with vasopressor-resistant hypotension. One observational study of this subset of SAH patients documented an abnormal cosyntropin stimulation test in 69% [2]. Diabetes insipidus occurs rarely in SAH patients (0.04 to 5%). Adipsic DI is a variant of DI characterized by an impaired thirst response in addition to altered arginine vasopressin release. This can be a complication of anterior communicating artery aneurysm clipping [14]. The vascular supply to the osmoreceptors is derived from small arteries arising from the anterior communicating artery; damage to these arteries results in infarction of the circumventricular organs where the osmoreceptors are positioned. Hyponatremia, on the other hand, is quite frequent in SAH patients. In one study of 316 patients with SAH, 179 (56.6%) developed hyponatremia,

www.medicalebookpdf.com

including 62 (19.6%) with severe hyponatremia [15]. There is considerable controversy regarding the pathophysiology of hyponatremia in SAH. Posited mechanisms include cerebral salt wasting [16, 17] and SIADH, though relative adrenal insufficiency and even inappropriate IV fluid resuscitation may contribute. In one prospective study of low grade SAH patients with hyponatremia, SIADH was determined to be the etiology of hyponatremia in 71.4% of patients [18]. Others have suggested hyponatremia in SAH may be due to both disordered arginine vasopressin secretion and exaggerated natriuresis – the predominant clinical presentation would depend upon the intensities of each as well as the effects of concomitant therapy (e.g. IV fluids, sodium repletion) [19]. A reasonable approach to the hypotonic hyponatremic SAH patient is to first assess the intravascular volume status. Understanding the volume status of the patient may help distinguish the underlying etiology (SIADH versus cerebral salt wasting) and is critical for the general management of the patient. Cerebral salt wasting is defined by salt loss accompanied by a reduced effective arterial blood volume, whereas the hyponatremic patient with SIADH is generally euvolemic (Table 20-1). In cerebral salt wasting, treatment focuses on replenishing both salt and volume. This can be achieved with agents such as fludrocortisone, salt tablets, or hypertonic sodium chloride infusions. On the other hand, SIADH is theoretically treated with fluid restriction. Fluid restriction may not be appropriate in those SAH patients concurrently demonstrating signs of cerebral vasospasm, as fluid restriction in this setting could precipitate infarction.

www.medicalebookpdf.com

PERIOPERATIVE PATIENTS There is one particular subset of neurosurgical patients with a fairly high frequency of neuroendocrine complications: those patients undergoing transsphenoidal surgery for sellar or parasellar pathology. For instance, 18-31% of transsphenoidal surgery patients will develop early postoperative diabetes insipidus [20]. Risk factors for postoperative DI include: Young age Male sex CSF leak Resection of a craniopharyngioma, Rathke-cleft cyst, or ACTH-secreting pitiuitary adenoma Postoperative DI typically manifests during the first 24-48 hours after pituitary surgery. Therefore, in the first several days following transsphenoidal surgery, strict measurement of a patient's intake, output, and daily weight along with frequent serum sodium levels should be obtained to assess for DI. Patients with pre-operative evidence of hypopituitarism should be maintained on stress doses of hormones during the initial perioperative period, followed by physiological maintenance doses. And patients with evidence of hormonal excess typically

www.medicalebookpdf.com

undergo laboratory screening during the first several postoperative days to assess for evidence of early endocrinological remission [21]. ADDITIONAL CAUSES OF ADRENAL INSUFFICIENCY Patients in the neurointensive care unit may develop relative adrenal insufficiency due to several mechanisms other than direct intracranial injury or perioperative complications. For instance, patients with sepsis may develop relative adrenal insufficiency. Patients with profound hypotension due to sepsis may infarct the pituitary or adrenal, resulting in a decrease in glucocorticoid synthesis; alternatively, the inflammatory milieu of septic patients may result in reduced access of glucocorticoids to target tissues and cells [22]. Furthermore, several of the medications used in the intensive care unit can induce pituitary-adrenal axis dysfunction (Table 20-2). For instance, etomidate produces a concentration-dependent blockade of 11-β hydroxylase, the enzyme responsible for the final conversion of cholesterol to cortisol. Although adrenal dysfunction with etomidate had been documented as early as the 1980s, the clinical relevance of this was unclear. Recently, several studies have suggested even a single-dose of etomidate may produce prolonged hypoadrenalism (12-24 hours) in the context of critical illness or sepsis [23]. In the Corticosteroid Therapy of Septic Shock Trial, of the 499 patients enrolled, 96 were administered etomidate during rapid sequence intubation. The use of etomidate was associated with a blunted cortisol response to ACTH (16.4%) and a higher mortality (12.2%) [24]. For this reason, transient administration of stress dose corticosteroids may be considered in those patients intubated with etomidate who subsequently develop vasopressor-resistant hypotension. Phenytoin and phenobarbital can upregulate the metabolism of cortisol, primarily via induction of cytochrome P-450 activity, theoretically predisposing to adrenal insufficiency. In patients with epilepsy, transient decreases in cortisol concentrations have been reported with both of these drugs [25]. The clinical significance of this is uncertain, though it does raise concern about routine phenytoin use for seizure prophylaxis in those TBI patients already predisposed to endocrine dysfunction.

www.medicalebookpdf.com

THYROID DISORDERS IN THE NEUROINTENSIVE CARE UNIT Thyroid storm In the spectrum of endocrine emergencies, thyroid storm ranks as one of the most critical illnesses. Early recognition and appropriate management is paramount and if treatment is delayed, mortality can exceed 90% [26]. The most frequent underlying cause of thyrotoxicosis in cases of thyroid storm is Graves' disease. Thyroid storm can also occur with a solitary toxic adenoma or toxic multinodular goiter. Various acute stressors can precipitate a thyroid storm, including surgery, trauma, myocardial infarction, pulmonary thromboembolism, diabetic ketoacidosis, and sepsis. Certain medications such as iodine, amiodarone, salicylates, and pseudoephedrine can induce a storm [27]. One hypothesis to explain the cause of thyroid storm is an increase in the amount of free thyroid hormones. Elevated serum and intracellular T4 and T3 suppress thyrotropin, so that serum thyrotropin should be undetectable, except in those patients with a pituitary thyrotropin-secreting adenoma. The diagnosis of thyroid storm should be a clinical one, as treatment should not be delayed for the results of confirmatory thyroid function tests (TFTs). Classic signs and symptoms of thyrotoxicosis include: Fever, diaphoresis Tachycardia Cardiac arrhythmias Tremulousness Anxiety/confusion Nausea, vomiting, diarrhea

www.medicalebookpdf.com

Patients may progress to high-output heart failure, shock, and even coma. The pattern of thyroid function tests is one of an elevated free T4 and free T3 with a depressed thyrotropin (also known as thyroid-stimulating hormone [TSH]). Free hormone concentrations are preferable to diagnose thyrotoxicosis as total concentrations may be significantly altered by conditions that alter protein binding. The medical management of thyroid storm includes a multi-drug approach to halt the synthesis, release, and peripheral effects of thyroid hormone [27]: 1. Inhibit new hormone production with methimazole (20-25 mg orally [PO] q6H) or propylthiouracil (200-400 mg PO q6H) 2. Inhibit thyroid hormone release with saturated solution of potassium iodide (5 drops PO q6H), IV sodium iodide (0.25 grams q6H), or Lugol solution (4-8 drops PO q6-8H) 3. Decrease the conversion of T4 to its active T3 form with glucocorticoids (hydrocortisone 100 mg IV q8H) 4. Blunt the adrenergic effects of thyroid hormone with beta blockade (propranolol, 1-2 mg IV q4H or 20-120 mg PO q4-6H); cautious use in patients with heart failure Additional supportive therapies such as antipyretics (acetaminophen rather than salicylates), external cooling measures, and intravenous hydration with dextrosecontaining fluids are also helpful. Myxedema coma In contrast to thyroid storm, myxedema coma results from decompensated hypothyroidism. Numerous factors can trigger myxedema coma, including stroke, trauma, infection, congestive heart failure, gastrointestinal bleeding, anesthetics, sedatives, tranquilizers, narcotics, amiodarone, lithium, hypoxemia, and hypercapnia. The classic presentation is one of lethargy progressing to stupor and then coma, with respiratory failure and hypothermia. Additional symptoms include bradycardia, hypotension, hypoventilation, anorexia, nausea, abdominal pain, and decreased gastrointestinal motility. Abnormal laboratory tests demonstrate hyponatremia and hypoglycemia. All patients have low serum total and free T4 and T3 concentrations, frequently with an elevated TSH [28]. Given a reasonable index of suspicion, therapy with thyroid hormone should begin immediately (while awaiting the results of TFTs). Thyroid repletion alone without addressing all the additional metabolic derangements would likely be inadequate for recovery; frequent monitoring of pulmonary and cardiac status

www.medicalebookpdf.com

(typically in an intensive care unit) is paramount. Treatment should include: Ventilatory support if needed Intravenous T4 with a loading dose of 400-500 micrograms once and then a maintenance dose of 50-100 micrograms per day, or intravenous T3 with a loading dose of 10-20 micrograms once and then a maintenance dose of 10 micrograms q4-6H Intravenous volume repletion External warming of patients (caution as hypotension may result from vasodilatation) Steroid therapy (hydrocortisone 50-100 mg IV q6-8H) is indicated in patients with myxedema coma attributable to pituitary or hypothalamic disease because they may have corticotropin deficiency as well as TSH deficiency Nonthyroidal illness syndrome (NTIS) NTIS, formerly referred to as sick euthyroid syndrome, refers to the alterations in serum thyroid hormone levels observed in critically ill patients in the absence of hypothalamic-pituitary-thyroid primary dysfunction. The acute phase of critical illness is marked by low total and free T3 and high reverse T3 (rT3). As the disease progresses, additional decreases in T3 and further reductions in the T3/rT3 ratio are observed, whereas TSH levels are often within the normal range or slightly elevated. Several investigators posit NTIS represents a possibly adaptive response to oxidative stress, likely through pathways mediated by interleukin-6 [29]. Although the vast majority of patients with NTIS have a return of normal thyroid function, critically ill patients with very low T4 concentrations do have an increased risk of death. Whether or not NTIS patients in general would benefit from thyroid hormone repletion continues to be a controversial subject. SUMMARY In summary, endocrine derangements are not rare in the critically ill and acutely brain-injured populations. Early recognition and correction of life-threatening endocrinopathies are essential; if left unchecked, they can result in considerable morbidity and mortality. This includes adrenal insufficiency with refractory hypotension, disorders of salt and fluid balance resulting in severe hyper- or

www.medicalebookpdf.com

hyponatremia, and thyrotoxicosis and myxedema coma. REFERENCES 1. Vermes I, Beishuizen A. The hypothalamic-pituitary-adrenal response to critical illness. Best Practice and Research: Clinical Endocrinology and Metabolism 2001 Dec; 15(4): 495-511. 2. Vespa P. Hormonal dysfunction in neurocritical patients. Curr Opin Crit Care 2013; 19: 107-112. 3. Thomas Z, Bandali F, McCowen K, Malhotra A. Drug-induced endocrine disorders in the intensive care unit. Crit Care Med 201; 38(6): s219-s230. 4. Benvenga S, Campenni A, Ruggeri RM et al. Clinical review 113: hypopituitarism secondary to head trauma. J Clin Endocrin Metab 2000 Apr; 85(4): 1353-1361. 5. Schneider HJ, Kreitschmann-Andermahr I, Ghigo E et al. Hypothalamopituitary dysfunction following traumatic brain injury and aneurysmal subarachnoid hemorrhage: a systematic review. JAMA 2007; 298: 1429-1438. 6. Hannon MJ, Sherlock M, Thompson CJ. Pituitary dysfunction following traumatic brain injury or subarachnoid haemorrhage. Best Practice and Research Clinical Endocrinology and Metabolism 2011; 25: 783-798. 7. Olivecrona Z, Dahlqvist P, Koskinen LD. Acute neuro-endocrine profile and prediction of outcome after severe brain injury. Scandinavian Journal of Trauma, Resuscitation and Emergency Medicine 2013, 21: 33. 8. Hamilton DD, Cotton BA. Cosyntropin as a diagnostic agent in the screening of patients for adrenocortical insufficiency. Clinical Pharmacology: Advances and Applications 2010; 2: 77-82. 9. Agha A, Thornton E, O'Kelly P et al. Posterior pituitary dysfunction after traumatic brain injury. J Clin Endocrin Metab 2004; 89(12): 5987-5992. 10. Maggiore U, Picetti E, Antonucci E et al. The relation between the incidence of hypernatremia and mortality in patients with severe traumatic brain injury. Crit Care 2009; 13(4): R110. 11. Loh JA, Verbalis JG. Diabetes insipidus as a complication after pituitary surgery. Nature Clinical Practice: Endocrinology and Metabolism 2007; 3(6): 489-494.

www.medicalebookpdf.com

12. Agha A, Sherlock M, Phillips J et al. The natural history of post-traumatic neurohypophysial dysfunction. European Journal of Endocrinology 2005; 152(3): 371-377. 13. Crompton MR. Hypothalamic lesions following the rupture of cerebral berry aneurysms. Brain 1963; 86: 301-314. 14. Crowley RK, Sherlock M, Agha A et al. Clinical insights into adipsic diabetes insipidus: a large case series. Clin Endocrinol 2007; 66:4): 475482. 15. Sherlock M, O'Sullivan E, Agha A et al. The incidence and pathophysiology of hyponatremia after subarachnoid hemorrhage. Clin Endocrinol 2006; 64: 250-254. 16. McGirt MJ, Blessing R, Nimjee SM. Neurosurgery 2004; 54(6): 1369-73. 17. Tomida M, Muraki M, Uemura K, Yamasaki K. Plasma concentrations of brain natriuretic peptide in patients with subarachnoid hemorrhage. Stroke 1998; 29: 1584-1587. 18. Hannon MJ, Behan AMM, O'Brien C et al. Hyponatremia following mild/moderate subarachnoid hemorrhage is due to SIAD and glucocorticoid deficiency and not cerebral salt wasting. J Clin Endocrinol Metab 2014; 99: 291-298. 19. Verbalis JG. Hyponatremia with intracranial disease: not often cerebral salt wasting. J Clin En-docrinol Metab 2014; 99: 59-62. 20. Nemergut EC et al. Predictors of diabetes insipidus after transsphenoidal surgery: a review of 881 patients. J Neurosurg 2005; 103: 448-454. 21. Zada G, Woodmansee WW, Iuliano S, Laws ER. Perioperative management of patients undergoing transsphenoidal pituitary surgery. Asian J Neurosurg 2010; 5(1): 1-6. 22. Prigent H, Maxime V, Annane D. Science Review: Mechanisms of impaired adrenal function in sepsis and molecular actions of glucocorticoids. Crit Care 2004; 8: 243-252. 23. Thomas Z, Bandali F, McCowen K, Malhotra A. Drug-induced endocrine disorders in the intensive care unit. Crit Care Med 2010; 38(6): S219-S230. 24. Cuthbertson BH, Sprung CL, Annane D et al. The effects of etomidate on adrenal responsiveness and mortality in patients with septic shock.

www.medicalebookpdf.com

Intensive Care Med 2009; 35: 1868-1876. 25. Ostrowska Z, Buntner B, Rosciszewska D et al. Adrenal cortex hormones in male epileptic patients before and during a 2-year phenytoin treatment. J Neurol Neurosurg Psychiatry 1988; 51: 374-378. 26. Fauci AS, Braunwald E, Kasper DL et al. Harrison's principles of internal medicine, 17th edition, McGrawHill, 2008, vol. II, 2236-2237. 27. Nayak B, Burman K. Thyrotoxicosis and thyroid storm. Endocrinol Metab Clin N Am 2006; 35: 663-686. 28. Wartofsky L. Myxedema coma. Endocrinol Metab Clin N Am 2006; 35: 687-698. 29. Wajner SM, Maia AL. New insights toward the acute non-thyroidal illness syndrome. Front En-docrinol 2012; 3: 8.

www.medicalebookpdf.com

ENDOCRINE IN NEUROCRITICAL CARE QUESTIONS 1. Regarding the management of thyroid storm, which of the following should NOT be administered? a. Temperature modulation for hyperpyrexia with salicylates b. Methimazole 20-25 mg PO q6H c. Hydrocortisone 100 mg q8H d. Saturated solution of potassium iodide 2. A 34 year old female with a history of anorexia is brought to the emergency department by her roommate for altered mental status. Upon arrival, blood pressure is 90/62, heart rate is 50, and she is hypothermic at 35.5oC. She is somnolent but arouses briefly to loud name calling; she is generally weak, but able to lift her extremities off the bed. Hyporeflexia is noted. What laboratory tests should be ordered? a. Serum sodium, creatinine and urine sodium, creatinine b. LH and FSH c. Free and total T3, T4, TSH d. Alcohol level 3. True or false: Hyponatremia is infrequent in subarachnoid hemorrhage patients 4. Which of the following is NOT a typical feature of diabetes insipidus? a. Urine osmolality less than 200 mOsm/kg b. Thirst c. Urine specific gravity greater than 1.005 d. Polyuria 5. Adrenal insufficiency has NOT been linked to which of the following medications?

www.medicalebookpdf.com

a. Etomidate b. Phenytoin c. Phenobarbital d. Dopamine 6. A 53 year old male is transferred to the neurointensive care unit for subarachnoid hemorrhage. He was intubated at an outside hospital. Review of the transfer records is notable for the administration of etomidate for intubation. An external ventricular drain is placed; the patient's neurological examination improves and intracranial pressure remains normal. His vital signs are notable for a blood pressure of 70/42 and a heart rate of 120. He is volume resuscitated with multiple liters of normal saline but continues to be hypotensive, requiring the initiation of vasopressors. A transthoracic echocardiogram is normal, as are cardiac enzymes. There is no evidence of systemic infection or sepsis. What is the likely diagnosis and treatment? a. Urosepsis; empiric antibiotics b. Neurocardiogenic injury; beta blockade c. Medullary infarct; aspirin d. Adrenal insufficiency; stress dose hydrocortisone 7. In a patient undergoing transsphenoidal resection of a pituitary mass, what is NOT a risk factor for the development of diabetes insipidus? a. Craniopharyngioma b. Rathke-cleft cyst c. Female gender d. Young age 8. True or false: The classic thyroid function test profile of nonthyroidal illness syndrome is a low free T3, high reverse T3, and normal TSH level 9. Which of the following is correct regarding vasopressin receptors? a. V1a vasopressin receptors primarily mediate the antidiuretic effect of

www.medicalebookpdf.com

AVP b. Conivaptan antagonizes both V1a and V2 receptors c. Platelets express V1b vasopressin receptors d. Smooth muscle cells do not express vasopressin receptors 10. All of the following are true regarding the syndrome of inappropriate diuresis (SIADH) EXCEPT: a. SIADH is frequently caused by a lesion in the anterior pituitary b. The syndrome is due to inappropriate release of arginine vasopressin c. Patients are generally euvolemic d. Hypotonic hyponatremia is the classic finding

www.medicalebookpdf.com

ENDOCRINE IN NEUROCRITICAL CARE ANSWERS 1. The correct answer is A. Salicylates should not be used for pyrexia in thyroid storm as they can disproportionately increase free thyroid hormone levels. 2. The correct answer is C. The patient's clinical history and examination findings are suggestive of myxedema coma; thyroid function tests should be obtained. 3. The correct answer is False. Hyponatremia (due to either SIADH or cerebral salt wasting) is frequent in SAH patients. 4. The correct answer is C. In DI, urine specific gravity is typically less than 1.005. 5. The correct answer is D. Dopamine has not been associated with adrenal insufficiency. 6. The correct answer is D. The patient may have relative adrenal insufficiency given the etomidate exposure in conjunction with the severity of his critical illness. Short-term stress dose steroid administration should be considered as the patient has vasopressor-refractory hypotension. 7. The correct answer is C. Male gender rather than female gender is a risk factor for perioperative DI. 8. The correct answer is True. 9. The correct answer is B. Conivaptan is a dual V1a/V2 receptor antagonist. 10. The correct answer is A. If SIADH is due to a pituitary lesion, it is of the posterior pituitary.

www.medicalebookpdf.com

Chapter 21

PEDIATRIC NEUROCRITICAL CARE Jose Pineda and Mark Wainwright CLINICAL CASE A 3 month old male infant presents to the Emergency Department after being found unresponsive by his mother at home. The mother reports she had left to go to work about 12 hours earlier, and left the child under the care of her boyfriend. Prior to her departure, the child was acting normal for age and was able to eat without any difficulty. The patient was born at term gestational age and had no significant past medical history. The mother reported no history of trauma, and her boyfriend was not available for interview. In the Emergency Department he had agonal breathing, hypotension and severe hypoglycemia. His Glasgow Coma Scale (GCS) score was 3, his left pupil was fixed and dilated and his fontanel was bulging. He was intubated and received 3% saline solution and intravenous (IV) glucose, with normalization of his pupillary function and blood pressure. A CT scan revealed a left-sided 15 mm epidural hematoma with midline shift. The patient was taken to the operating room where the hematoma was evacuated, a left decompressive hemicraniectomy was performed, and an intraparenchymal intracranial pressure monitor was placed. Upon arrival to the Pediatric Intensive Care Unit (PICU) the patient was intubated, receiving mechanical ventilation and an infusion of epinephrine at 0.05 mcg/Kg/minute. His pupils were 2 mm in diameter bilaterally and had sluggish reaction to light. Neurological examination revealed minimal response to painful stimulation but no verbal response or eye opening. His intracranial pressure (ICP) was initially 12 mmHg but increased over the next 48 hours and remained over 30 mmHg despite maximal escalation of medical therapy. He also developed clinical seizures and non-convulsive epileptiform activity on EEG, requiring therapy with multiple antiepileptic medications including, levetiracetam, midazolam and phenobarbital. After 2 weeks in the ICU the patient was extubated and subsequently transferred to the Pediatric Rehabilitation Unit. He continued to make progress but was discharged with significant disability. Further investigation revealed that inflicted trauma was the cause of the patient's brain injury.

www.medicalebookpdf.com

Traumatic brain injury (TBI) is a leading cause of death and disability in children, and abusive head trauma (child abuse) is the most common cause of traumatic brain injury in infants. This case illustrates one of the most challenging cases in Pediatric Neurocritical Care for several reasons. The infant brain may be more prone to developing neuronal death and to neurotoxicity from frequently used sedatives and anticonvulsants. Victims of inflicted trauma have delayed presentation, the exact time of injury is frequently unknown, and they frequently experience secondary insults prior to arrival to the hospital. Despite aggressive surgical and medical therapy, many develop progressive brain edema and secondary ischemia (in part possibly due to neurovascular dysfunction), with intracranial hypertension despite having open fontanels and non-fused cranial sutures. There is limited evidence for age appropriate intracranial pressure and cerebral perfusion pressure targets and intracranial monitoring can be challenging due to the lack of pediatric specific technology. Consequently, in contrast to the significant progress being made improving outcomes for children with acute neurological disorders (including non-abusive traumatic brain injury), the rates of mortality and long-term disability for infants with abusive head trauma injury remain extremely high. OVERVIEW Critically ill children with acute neurological disorders have great potential for recovery. Caring for this patient population requires a high level of expertise that is best provided by a multidisciplinary team using specialized equipment. To achieve optimal outcomes, continuous functional monitoring of the nervous system, clinical strategies designed to detect secondary insults to the nervous system, and guided interventions crafted to reverse evolving pathology while protecting uninjured nervous tissue are necessary. The team is composed of physicians, nurses and support staff with extensive experience and expertise in pediatric critical care, neurology and neurosurgery. The multidisciplinary nature of such a team requires careful coordination of services and resources, development and implementation of best clinical practice guidelines, and a specialized quality improvement program. Because of the nature of the patient case mix and available therapeutic interventions, the current number and size of Pediatric Neurocritical Care programs is much smaller compared with adult programs [1]. There are fewer large clinical trials and such studies may be more difficult to conduct in critically ill children with neurological disorders. However, as advances in diagnoses and therapies for lung, heart, kidney, and infectious disease have dramatically

www.medicalebookpdf.com

reduced mortality rates in the PICU, the relative contribution of neurological illness to mortality and long-term morbidity after critical illness has grown [2]. Consequently, there is growing momentum for developing Neurocritical Care for children. As new therapeutic approaches become available, models of care and training are being developed [35]. Interdisciplinary collaboration between pediatric critical care medicine, neurosurgery and neurology constitutes the foundation on which advancements leading to improved outcomes will develop. EPIDEMIOLOGY Regardless of the primary diagnosis, quality of life in children who survive critical illness is mainly determined by the child's degree of neurological recovery. Furthermore, approximately 1/3 of children admitted to Pediatric Intensive Care Units have a neurological or neurosurgical disorder as their primary diagnosis. Amongst critically ill children with acute neurological disorders, children suffering from traumatic injuries, recovering from complex surgical procedures such as brain tumor resections or cranial reconstructions, or suffering from epilepsy or respiratory disorders complicating neuromuscular disease represent a significant proportion of the Pediatric Neurocritical Care population. Additional diagnoses are included in Table 21-1. Although less common, most of these clinical entities are amenable to treatment, and timely, efficient implementation of effective treatment frequently results in low mortality and meaningful functional recovery.

www.medicalebookpdf.com

PATHOPHYSIOLOGY, CLINICAL FEATURES, DIAGNOSIS, AND TREATMENT Neurological Assessment and Pediatric Critical Care Considerations A variety of tools are available for the monitoring of neurological function and brain physiology in critically ill children. These range from EEG monitoring, which is well-established as a modality essential for the detection of non-convulsive seizures, to intracranial pressure monitoring, transcranial

www.medicalebookpdf.com

Doppler ultrasonography (TCD), tissue oxygen monitoring and pupillary reactivity. While thresholds for detection of cerebral ischemic injury have been proposed for brain tissue oxygen tension [6] and near-infrared spectroscopy (NIRS) [7], there is no consensus on normal values and age-dependence of these endpoints. Furthermore, most of these studies focus on traumatic brain injury and there is practically no experience on the application of these data to critically ill children with other diagnoses such as stroke, refractory status epilepticus (RSE), hepatic encephalopathy or central nervous system infections. The importance of the bedside neurological exam in the assessment of initial neurological injury and progression of the insult as well as the need for serial examinations by the entire medical team in this assessment cannot be overstated. In this context, the GCS is simple to perform and produces reproducible results between observers [8], but the GCS is not a comprehensive neurological exam and does not assess brainstem (particularly pupillary reactivity) function or the presence of focal findings. The neurological exam in the ICU should include many of the components (e.g. mental status, cranial nerves, muscle tone and strength, reflexes) of the complete neurologic examination with particular focus on changes in mental status, loss of reflexes, development of pathological reflexes or asymmetries of tone or strength. The initial neurological exam serves to establish the baseline against which serial examinations are compared. The involvement of the entire team is an essential step in ensuring that pathologic changes in neurologic function are detected as early as possible. The neurological exam in a critically ill child may be limited, particularly when administration of sedatives and analgesics is unavoidable. In contrast, this patient population is at risk for development of new or progressive acute neurologic insults, making attention to subtle changes in the exam essential. The resulting interventions (for example, to minimize secondary injury), additional diagnostic studies (brain or spinal cord imaging, EEG monitoring, ICP monitoring), or therapeutic interventions will be contingent on the nature of the primary insult and mechanism of the injury. Physiological decompensation due to critical illness can expose the brain to systemic secondary insults such as hypotension, hypoxia, hypercapnea and hyperthermia. These secondary insults may aggravate neurological injury, especially in the presence of intracranial hypertension or disrupted cerebral blood flow autoregulation. Respiratory decompensation in children represents the most common cause of cardiorespiratory arrest. Respiratory decompensation creates additional challenges for critically ill children with neurological disorders. Abnormalities in ventilation and oxygenation impact cerebral hemodynamics and metabolism and

www.medicalebookpdf.com

can result in secondary insults to the brain. For example, hyperventilation can lead to hypocapnea and detrimental changes in cerebral blood flow, as there is approximately a 3% reduction in cerebral blood flow for each 1 mmHg reduction in PaCO2. While targeted hyperventilation is considered a treatment option during periods of severe intracranial hypertension and cerebral herniation, unintentional hyperventilation has been independently associated with increased mortality [9]. Avoidance of unintentional hyperventilation is especially important in the immediate post-injury period after traumatic brain injury when cerebral blood flow may already be markedly reduced or mismatch between oxygen demand and supply may be present [10,11]. This principle should also be kept in mind when treating patients with other acute neurological disorders complicated by marginal brain perfusion. Hypoventilation can lead to detrimental increases in PaCO2. Hypercapnea will increase cerebral blood volume and can lead to severe intracranial hypertension in patients with decreased cerebral compliance. Causes of hypoventilation include neurological deterioration due to side effects of narcotic therapy. Children under 6 months of age are at higher risk for respiratory depression related to narcotics due to their immature liver metabolism. For example, the mean half-life of morphine has been reported to be 5.4 + 3.4 hours in infants 1 week to 2 months of age and 2.6 + 1.7 hours in infants aged from 2 to 6 months [12]. Developmental differences in pharmacokinetics should be taken into consideration when dosing narcotics in the PICU to minimize the risk of respiratory depression. While uncommon, severe respiratory failure may develop in critically ill children with primary neurological diagnoses. Current management strategies for critically ill children with severe respiratory failure include permissive hypoxia (systemic oxygen saturations of 85% and above) and permissive hypercapnea. The potential benefits of these strategies should be balanced with the risk of secondary insults to the brain, especially in children in whom the threshold for brain hypoperfusion is unknown. Prevention of Secondary Brain Insults The main objective of intensive care management of children with or at risk of brain injury is to prevent secondary insults to the brain. Neurocritical care in children aims at preventing cerebral hypoxia and ischemia through the avoidance of intracranial hypertension and systemic hypoxia, hypotension, hypoglycemia and hypocarbia. Inadequate perfusion may have a direct and deleterious effect on the pathophysiology of brain injury. A cerebral perfusion pressure (CPP) between 40 and 65 mmHg has been recommended for children with severe TBI [13].

www.medicalebookpdf.com

Children with a CPP below these thresholds have worse outcome. Optimal CPP levels for children under 2 years of age have not been established, but several studies demonstrate that a CPP of 40 mmHg or less is associated with higher mortality and worse outcome in children of any age. Therefore 45 mmHg is reasonably considered a critical threshold for CPP in children less than 2 years of age. CPP values should be interpreted in the context of other determinants of brain tissue perfusion such as cerebral blood flow and autoregulation [14-18]. Adjustments of the mean arterial pressure (MAP) to maintain CPP above critical thresholds may be necessary due to disrupted cerebral pressure autoregulation. In critically ill children with acute neurological disorders other than TBI, ICP monitoring is much less commonly performed, making estimations of critical thresholds for brain perfusion difficult. Even in the absence of increased ICP, hypotension is a predictor of poor outcome in patients with brain injury [19]. Normal systolic blood pressure and MAP values change with age (Table 21-2). The formula 70 mmHg + (2 X age in years) allows easy calculation of the lower limit (5th percentile) of systolic blood pressure for age. Different definitions of hypotension have been used. A recent study described an association between early hypotension and outcome using the 75th percentile for age appropriate systolic blood pressure (AASBP) as the threshold for hypotension in children with TBI [20]. Maintaining AASBP during the initial stabilization phase is an important goal and may influence outcome in children with traumatic brain injury, and possibly in other types of acute brain injury.

Other factors that may result in secondary insults to the brain include hyperthermia, anemia, seizures, pain and agitation, cerebral edema and cerebral arterial or cerebral venous thrombosis. Hyperthermia is known to correlate with worse outcome and longer ICU stay in patients with brain injury [21-23]. Avoidance of hyperthermia is important during periods of brain ischemia.

www.medicalebookpdf.com

External cooling and antipyretics are commonly used; intravascular cooling methods are more effective but currently unavailable for children. There is no clear threshold for transfusion in children, and while hemoglobin concentrations of 7 g/dL are well tolerated in stable children in the intensive care unit [24], no data is available for children with marginal brain perfusion. Recent information has also raised concern about the effects of stored blood transfusion on brain tissue perfusion [25]. While coronary artery disease is much less common in children, pediatric patients can suffer from low cardiac output due to systemic hypoperfusion, severe brain injury, or the effect of medications such as pentobarbital, narcotics, and benzodiazepines. Propofol in particular has been reported to rarely result in refractory shock and metabolic acidosis. This medication is not recommended for continuous sedation in the pediatric intensive care unit ( http://www.fda.gov/medwatch/SAFETY/2001/safety01 .htm#dipriv ). Pain and agitation resulting in hypermetabolism and increased ICP can also aggravate brain injury. The beneficial effects of sedatives and analgesics are balanced against the effects of these medications on the child's neurological exam. In patients undergoing mechanical ventilation and ICP monitoring, continuous sedation and analgesia is more desirable than intermittent administration. In patients recovering from intracranial procedures such as resection of posterior fossa tumors, intermittent careful titration of analgesics is preferred, and consideration can be given to non-narcotic analgesics such as ketorolac, in the absence of obvious contraindications [26-28]. Ketorolac appears to be well tolerated in children after significant surgical procedures without increasing the risk of bleeding and renal dysfunction, but safety and efficacy in neurosurgical patients has not been established. Ketorolac is typically prescribed for 48 hours or less. Finally, while less common, deep venous thrombosis is possible in children with brain pathology [29]. Risk factors include immobility, obesity, thoracic or abdominal surgery, need for central venous access and pro-coagulant states [30,31]. As children recover and tolerate external stimulation, physical and occupational therapy interventions will increase mobility and decrease the risk of venous thrombosis. Anticoagulation prophylaxis is indicated in selected cases. The treatment of cerebral venous thrombosis, a life threatening condition in critically ill children is rather complex and more studies are needed [32]. The risk of bleeding should be balanced against the risk of expanding thrombosis, which can lead to severe brain injury or systemic complications such as pulmonary embolism.

www.medicalebookpdf.com

STROKE There are a number of excellent reviews on this topic, including consensus statements that also address the controversies regarding treatment [33-36]. This summary will focus on acute vascular insults but it is important to consider that other disorders such as metabolic or mitochondrial disease, migraine, or seizures may mimic stroke [37]. Timely and efficient evaluation is therefore essential. Because of the rarity of stroke in children and the challenges of clinical diagnosis, delayed diagnosis is a major problem, with the average interval between onset of initial symptoms and presentation for medical evaluation at 34 hours [38]. The mortality and neurologic morbidity associated with stroke in the young is substantial. Stroke is among the top 10 causes of death in children and 60% of survivors have residual neurological deficits [39-42]. Data from arterial ischemic stroke (AIS) cases which were not treated with anti-thrombotic medications indicate a recurrence rate of nearly 50% [34]. This rate may be reduced since studies in which most children were treated with aspirin or anticoagulation report recurrence rates of 5% to 25%, although no controlled trials have been performed [43,44]. The causes of stroke in children differ substantially from adults and large artery atherosclerosis or small vessel occlusive disease are not common pathologies in pediatric cerebrovascular disease [45]. For AIS, the most common risk factors are congenital or acquired heart disease and, less frequently, hypercoagulable or autoimmune disorders and arteriopathies (e.g. arterial dissection, moya moya syndrome, and vasculitis) [46], [45], [47,48]. In the United States, in the largest population-based study of AIS in children, the most common causes were cerebral arteriopathy (24%), infection (23%) and cardiac disease (12%) while sickle cell disease (SCD) accounted for only 3% of cases [49]. Among the arteriopathies, an association between antecedent varicella zoster infection and increased risk of AIS has been reported, often involving the basal ganglia [50]. Cervicocephalic arterial dissection is relative higher in the young, accounting for 8% [51,52] to 20% [41] of cases in different series. The causes of moyamoya disease are not known but there is a robust association with progressive neurologic dysfunction, occurring in up to 60% of cases [53] which may be reduced by revascularization surgery.[54] The contribution of thrombophilias to stroke in the young has been addressed in multiple studies [46,55,56]. The contribution of a single biochemical or genetic risk factor to elevated risk for stroke is not precisely defined and likely to be low, given the relative rarity of childhood stroke [55]. In practice, this means

www.medicalebookpdf.com

that other factors should be investigated even in the presence of a biochemical or genetically determined pro-coagulant state [33]. There are limited data and no controlled trials addressing the management of hemorrhagic stroke in children. Intracerebral hemorrhage (ICH) in the young presents with similar symptoms as in adults, most commonly (59%) with headache or vomiting and is associated with seizures in 37% of cases in one study [57]. For non-traumatic ICH, the most common risk factor in this study was an arteriovenous malformation or arteriovenous fistula, occurring in 32% (23 of 68) of cases. Guidelines for the approach to diagnostic studies are published [35,58]. Venous sinus thrombosis is underdiagnosed, and may present with subtle signs including seizures and subarachnoid hemorrhage or ICH [59,60]. If the stroke is not in a specific arterial vascular distribution or mitochondrial disorders are a consideration for any reason, MR spectroscopy should by obtained. Cardiac echocardiogram is indicated in the evaluation of new stroke and should include bubble contrast to evaluate for a patent foramen ovale or right-to-left shunt. Based on the index of suspicion, a transesophogeal echocardiogram may be needed. Laboratory studies should include measures of coagulation, platelets, homocysteine, fasting cholesterol, triglycerides and Lp(a) lipoprotein. Screening for infection may include antibodies to varicella zoster [47,50], mycoplasma and, if indicated, chlamydia, helicobacter and borrelia titers. Studies for genetic and biochemical risk factors implicated in childhood stroke together with von Willebrand factor antigen and plasminogen, may be obtained along with other screening studies for autoimmune disorders including lupus anticoagulant, and erythrocyte sedimentation rate. Anticardiolipin antibodies may be abnormal acutely and need to be repeated 8-12 weeks after the stroke along with any abnormal biochemical measures of coagulation. If a metabolic disorder is suspected, then lactate, pyruvate, serum amino and urine organic acids should be obtained during the acute phase of the stroke together with measurement of lactate levels in cerebrospinal fluid if available. Stroke in Children with Sickle Cell Disease Children with sickle cell disease (SCD) pose a separate set of challenges for diagnosis and management of acute neurologic deficits (for review, see references [61] and [62]). By age 45, 25% of patients with hemoglobin (Hb)SS disease and 10% of patients with HbSC disease have had a stroke [63]. The incidence of stroke in children with SCD is at least 20 times higher than the incidence in general pediatric population [64]. In practice therefore, stroke should be the leading differential diagnosis in children with SCD presenting with new neurologic findings.

www.medicalebookpdf.com

Strokes in children with SCD may occur as the consequence of multiple mechanisms including arterial ischemia, subarachnoid or intracerebral hemorrhage [65], aneurysm rupture, dissection, moyamoya syndrome [66] or cerebral venous sinus thrombosis [59]. If imaging studies do not identify stroke as the mechanism, there should be a high index of suspicion for seizures, as these occur 10 times more frequently in SCD than in the general population [67]. Specific risk factors which increase the risk of stroke in SCD have been identified, including previous transient ischemic attack, low hemoglobin, elevated peripheral white blood cell count, hypertension and acute chest syndrome (ACS) [63]. For children with SCD with acute stroke, the initial focus of clinical management is exchange transfusion and hydration [33,61]. Data on pharmacologic prophylaxis for stroke prevention in SCD is limited. TCD mean blood-flow velocity in the internal carotid or middle cerebral artery of 200 cm/second or higher has been used as criteria to initiate chronic transfusion therapy to prevent stroke in children with SCD [68]. Stroke – Anticoagulation and Supportive Therapy In contrast to adults where there are evidence-based recommendations for treatment of AIS [69], evidence-based data for children is more limited. There is a universal consensus on the general goals of management, to reestablish flow to the ischemic brain, to minimize secondary injury, and the principle that “time lost is brain lost”. Guidelines for the management of children with AIS and ICH are available [33,70]. Notably, short-term anticoagulation may be considered for pediatric AIS, pending the determination of the cause of stroke. Intravenous t-PA for treatment of AIS has been employed safely in small pediatric series [71] and has promise as a therapy for pediatric stroke. Strategies which aggressively treat fever, maintain euglycemia and normotension and prevent hypoxia are adapted from the management of adults with AIS [70]. While these recommendations for children are based on limited pediatric data [33], there is broad agreement that such management is indicated [33,35,58]. FLUID AND ELECTROLYTE ABNORMALITIES Children with acute brain injury and those recovering from neurosurgical procedures commonly require intravenous fluids early in the ICU course. Maintenance IV fluids (1500 ml/m2/day) are recommended to avoid dehydration and the consequent hemodynamic instability. The composition of maintenance IV fluids is tailored to avoid glucose and electrolyte abnormalities. While hyperglycemia (blood glucose over 180 mg/dL) is generally avoided because of its potential to worsen brain injury [72,73], care should be taken to avoid

www.medicalebookpdf.com

hypoglycemia, especially in neonates and infants. Hypotonic fluids such as Ringer's Lactate solution and 1/4 normal saline solutions are avoided because of the risk of hyponatremia which can lead to worsening brain edema and even herniation [74,75]. In patients requiring fluid resuscitation to treat hypotension, 0.9% (normal) saline solution is used as first line therapy. While the optimal time for initiation of enteral nutrition has not been established in children with brain injury, enteral nutrition is usually started as soon as possible and as tolerated based on the patient's neurological status. While gastric feeding is commonly well tolerated, transpyloric (jejunal) feeding is an option in children with brain injury who have gastric intolerance due to gastroparesis. It is important to note that transpyloric feeding does not decrease the risk of vomiting and aspiration in critically ill children or adult patients with brain injury [76-78]. Fluid balance and electrolyte abnormalities can be seen in children with acute brain injury and in the immediate post-operative period after neurosurgical procedures. Diabetes insipidus (DI) results from a deficiency of arginine vasopressin and can result in severe water and electrolyte imbalance [79]. It develops in approximately 75% of patients following transcranial resection of a pituitary tumor and 10% to 44% after transsphenoidal pituitary surgery [80,81]. Patients can also develop DI after other neurosurgical procedures such as ventricular fenestration, as well as after traumatic brain injury or cardiac arrest. Some patients may develop transient syndrome of inappropriate antidiuretic hormone secretion (SIADH) as part of a tri-phasic presentation (DI-SIADH-DI) or symptoms of cerebral salt wasting syndrome (CSWS) [81-87]. Diagnostic criteria for post-operative DI are: polyuria (urine output > 3 mL/kg/h) serum sodium > 145 mEq/L increased plasma osmolarity (>300 mOsm/kg) and hypotonic urine (urine Osm 40 mEq/L) with normal salt and water intake and clinical euvolemia. A less common but important disorder, CSWS is characterized by hyponatremia, hypovolemia, natriureis and diuresis [88]. Patients with CSWS can develop severe hyponatremia and require careful titration of fluid intake and sodium replacement.

www.medicalebookpdf.com

Figure 21-1. Management of post-operative Diabetes Insipidus

INTRACRANIAL HYPERTENSION The most common indication for ICP monitoring in critically ill children is traumatic brain injury (TBI). Children of all ages, including infants, are at risk of developing intracranial hypertension and the consequent cerebral hypoperfusion and cerebral herniation. Table 21-3 summarizes a tiered therapeutic approach to ICP directed therapy in children with severe traumatic brain injury. This approach is consistent with published TBI guidelines as well as general principles of pediatric critical care medicine, and it may result in improved outcomes [89-91]. Monitoring of ICP in medical conditions other than traumatic brain injury remains controversial. Examples of reversible brain pathologies that result in increased ICP include meningitis and liver failure with encephalopathy. In patients with meningitis, ICP monitoring is used in an estimated 7% of patients [92]. Mortality is higher in patients with meningitis with a mean cerebral perfusion pressure less than 50 mmHg in spite of CPP directed therapies [93]. In

www.medicalebookpdf.com

patients with liver failure and encephalopathy, the potential benefit of ICP directed therapy should be balanced against the risk of complications [94-96]. Global ischemia from near drowning or cardiac arrest with resultant brain edema can result in increased ICP but the incidence in children is not known and, since these patients are not routinely ICP monitored, there is no data on the benefit of monitoring ICP and other parameters with the goal of optimizing cerebral perfusion and balancing cerebral metabolism. Limited reports suggest that ICP monitoring and interventions aimed at lowering ICP are not beneficial in patients after global ischemia [97]. Overall, although ICP monitoring is feasible in conditions other than TBI, very limited information on the safety and efficacy of ICP directed therapies is available in these conditions [98]. SEIZURES The approach to seizures in the PICU is based on the need to detect their occurrence, terminate them, and to minimize any secondary injury they may produce. Seizures may be the presenting symptom of new ischemic (venous or arterial) or hemorrhagic injury in the critically ill child. In parallel with acute management, investigation of common ICU causes of seizures (e.g. metabolic derangement, drug reaction, vascular injury, infection) should begin immediately. Time is of the essence for the treatment of status epilepticus (SE) because repetitive seizures may become resistant to therapy. The duration of seizures required for the definition of SE has been shortened to 30 minutes [99] and a duration as short as 5 minutes has been proposed [100]. A further refinement of the definition of SE has been proposed, to include ‘impending' SE (continuous or intermittent seizures lasting more than 5 minutes, without full recovery of consciousness between seizures), and “established” SE (clinical or electrographic continuous seizures lasting more than 30 minutes without full recovery of consciousness between seizures) [101]. The criteria for refractory status epilepticus (RSE) include persistence of seizure activity despite appropriate medical and anticonvulsant drug (ACD) therapy, although the duration of seizures (1-2 hours) [99,102] and number of drugs (2 or 3) varies between studies [103]. Early recognition of clinical or electrographic seizures is an important factor in enhancing the efficacy of ACDs to terminate seizures and thereby to prevent additional metabolic stress. Non-convulsive seizures (NCS) or status epilepticus (SE) may be difficult to diagnose and are often unrecognized in comatose patients in both the adult and pediatric setting [104,105]. Convulsive or NCS are

www.medicalebookpdf.com

reported in up to 50% of patients undergoing treatment in an ICU when examined with continuous EEG (cEEG) monitoring [106-108]. Protocols for the treatment of SE are well-established [109,110]. The typical protocol involves intravenous, rectal, or buccal administration of a short-acting benzodiazepine followed by phenytoin, phenobarbital or valproic acid, depending on age of the child, current ACD use, and seizure type.

www.medicalebookpdf.com

www.medicalebookpdf.com

Many patients in the ICU will meet the criteria for RSE. In contrast to SE, approach to the management of RSE is not as well defined, although treatment protocols have been proposed [111,112]. Careful avoidance of and treatment of respiratory failure, cardiovascular failure, liver dysfunction and secondary infections is required to maximize the probability of favorable outcome. Electrographic seizures occur in approximately one third of children with structural brain injuries, prior in-hospital convulsive seizures, or with interictal EEG abnormalities [113]. Similarly, in children with acute encephalopathy, electrographic seizures occurred in 36 to 46% [114,115], and the main risk factor was young age. Infants and children have lower seizure thresholds [116], adding to the challenge of recognizing subtle clinical seizures in critically ill children [114,117]. The increasing availability and use of cEEG in pediatric neurotrauma has lead to greater appreciation for the burden of NCS after TBI. In a prospective study of 87 unselected children with mild-severe TBI at 2 centers, 43% of patients had seizures [118]. In children with TBI, it is likely that the true seizure burden, when accounting for clinical and electrographic seizures, is underestimated.

www.medicalebookpdf.com

A number of lines of evidence now suggest that electrographic seizures are associated with worse outcome [115,119]. Previously healthy children with acute neurologic disorders admitted to the PICU and who develop electrographic status epilepticus are at an increased risk of unfavorable global outcome, lower health-related quality of life scores, and subsequent epilepsy [119]. In a study of 259 children with various neurological insults that were admitted to the pediatric and cardiac ICU and underwent continuous video-EEG monitoring seizure burden was strongly associated with neurological deterioration [115]. The optimal treatment for electrographic seizures in critically ill children has not been determined. INFECTIONS Mortality from bacterial meningitis in children has decreased to as low as 2% [120]. Treatment with dexamethasone improves outcome in children with Hemophilus influenzae [121,122] and pneumococcal bacterial meningitis [123]. However, the use of adjunctive steroids in the treatment of pneumococcal meningitis in children is controversial and practice varies in the US [120]. The fundamental principles of prevention of secondary neurological injury also apply to the management of meningitis. The maintenance of normal oxygenation, adequate cerebral perfusion pressure, prevention of hypoglycemia and hyponatremia, early detection of seizures, AIS, ICH or cerebral sinus venous thrombosis are essential in optimizing neurological outcome [124,125]. Taken together, the variations in practice and lack of demonstrated effect on outcome render impossible any clear recommendation for or against ICP monitoring in pediatric bacterial meningitis [126]. Tuberculous meningitis (TBM) may occur as result of reactivation of latent infection or as a new infection [127]. The neurological complications of TBM are myriad, but hydrocephalus, stroke, and seizures should be considered in the differential diagnosis for any neurological deficits [128]. Hydrocephalus is more frequent in children (80-90%) than adults (12%) with TBM [129]. Accordingly, serial lumbar punctures or external ventricular drainage may be considered both to prevent the need for ventricular shunting and to increase the potential for benefit from shunt placement [130,131] although there are no data from controlled trials. The use of adjunctive corticosteroid treatment remains controversial but cumulative evidence [132] suggests that dexamethasone reduces morbidity but not mortality and should be administered to all patients regardless of age or disease severity [127]. In the developing world, cysticercosis is the most common cause of adult

www.medicalebookpdf.com

onset epilepsy [133]. For patients from regions with endemic disease, neurocysticerosis should be considered in the differential diagnosis of brain calcifications and seizures. The criteria for treatment with antihelmintic agents and the optimal agents for such use are not established [134]. Patients with viral encephalitis most commonly present in the PICU setting with seizures, which may be non-convulsive. The most common agents are the non-polio viruses, enteroviruses, respiratory viruses and herpes group viruses including Epstein-Barr virus, although a specific agent may be identified in only 40% of cases [135]. In children, the risk factors associated with a poor outcome after encephalitis include status epilepticus during the acute phase of encephalitis, and herpes simplex as the etiologic agent [136]. The neurological complications of the common virus, respiratory syncytial virus, are rare but include seizures, cardiac arrest and postulated brainstem dysfunction [137]. Lastly, new central nervous system infection or reactivation of latent human herpes virus-6 (HHV)6 should be considered in any immune-suppressed patient in the PICU setting with a change in neurological function [138]. Limbic encephalitis in these patients may manifest as inability to sleep and short-term memory loss [139]. A high index of suspicion is needed in any transplant recipient with these symptoms given the risk of long-term cognitive impairment associated with HHV6 encephalitis [140]. DEATH BY NEUROLOGICAL CRITERIA IN CHILDREN Guidelines for determining death, including brain death, have developed over time at many institutions largely based on the Harvard criteria [141]. In 1981, both national policy and state law regarding the determination of death were established [142]. These directives indicate that, as with irreversible failure of circulation and respiration, irreversible failure of brain function (including the brainstem) is death. The criteria for establishing brain death in children were published in 1987 in a statement issued by the American Academy of Pediatrics [143]. A multidisciplinary group of experts recently published updated guidelines that attempt to facilitate a standardized approach and address aspects of the determination of death by neurological criteria that are unique to children [144]. Prior to the determination of brain death, the patient must be known to have an irreversible disease that can cause brain death. Reversible disorders that may result in loss of brain function must be excluded. Examples of such recoverable disorders include drug poisoning, toxins, metabolic disorders, severe electrolyte disturbances, hypothermia and shock. The degree of hypothermia that could

www.medicalebookpdf.com

interfere with a brain death exam is in general not well defined. While defining such threshold at a given institution, consideration should be given to the fact that brain stem reflexes can be lost at 32oC [145]. Additionally, a temperature of 36.5º C or lower can create difficulty in the interpretation of target partial pressure of carbon dioxide (PaCO2) values. Controversy surrounding the need to correct arterial blood gas values to the patient's temperature is avoided at normothermia. Given the difficulty in achieving normothermia in children with loss of brain function and the paucity of data supporting a specific lower limit for temperature, a threshold of 35oC can be considered feasible and appropriate. Current recommendations for determination of death by neurological criteria include conducting an apnea test with each neurological exam. While different methods have been described, a technique involving transition from the mechanical ventilator to a self-inflating resuscitation bag may reduce the risk of barotrauma and hypoxia [146]. The bag is set with 100% oxygen and positive end expiratory pressure (PEEP) that approximates the patient's current mechanical ventilator settings. If severe hypoxemia or circulatory instability occurs at any time, the patient should be reconnected to the ventilator. In an effort to facilitate a standardized approach to the determination of death by neurological criteria in children, the updated guidelines also provide updated recommendations on the following aspects of the process: Sedative and anticonvulsant medication levels should be in the low or midtherapeutic range. If there is uncertainty, an ancillary test should be performed. In patients presenting after cardiac arrest or severe acute brain injury, it is recommended that the exam be deferred for 24-48h if there are concerns or inconsistencies. The two exams are to be conducted by different attending physicians (experienced clinicians with specific training in neurocritical care). Guidance for evaluation of oculovestibular reflexes provided: head of the bed elevated 30 degrees, 1 min of observation, and instillation of 10-50 mL of cold normal saline solution Two apnea tests are required, one with each neurological examination. If unable to conduct or complete an apnea test, an ancillary test should be performed. Only EEG and radionuclide cerebral blood flow testing are recommended for ancillary testing. Angiogram is the gold standard but is less feasible.

www.medicalebookpdf.com

Ancillary testing may be used when there is a need to shorten the examination interval period. Examination interval period: born at term gestational age and up to 30 days old: 24 hours between the first and the second examination 31 days to 18 years old: 12 hours Ancillary tests are not required and cannot substitute for the neurological examination, but may assist clinicians in making the diagnosis under the circumstances described above. Once the determination of brain death is made, appropriate documentation in the patient's medical record and communication with the patient's family are important [147]. At this point it may also be appropriate to discuss the recovery of organs for humanitarian purposes. Permission from the family may be requested for evaluation of the patient by an organ and tissue procurement agency. Appropriate resources should be allocated to provide support and information to the patient's family during this difficult time. CONCLUSIONS Critically ill children with neurological conditions benefit from a structured approach that focuses on the timely application of best clinical practices and prevention of secondary insults to the brain [148-150]. As new monitoring technology becomes available, the contribution to outcome of poorly characterized physiological variables such as cerebral blood flow autoregulation will be better understood, allowing development of new strategies to optimize patient management. There is also tremendous need for clinical trials testing pharmacological neuroprotective agents and rehabilitation interventions. REFERENCES 1. Cappell J, Kernie SG. Advances in pediatric neurocritical care. Pediatric clinics of North America. 2013;60:709-24. 2. Murphy S. Pediatric neurocritical care. Neurotherapeutics : the journal of the American Society for Experimental NeuroTherapeutics. 2012;9:3-16. 3. Tasker RC. Pediatric neurocritical care: is it time to come of age? Current opinion in pediatrics. 2009;21:724-30.

www.medicalebookpdf.com

4. LaRovere KL, Graham RJ, Tasker RC. Pediatric neurocritical care: a neurology consultation model and implication for education and training. Pediatric neurology. 2013;48:206-11. 5. Scher M. Proposed cross-disciplinary training in pediatric neurointensive care. Pediatric neurology. 2008;39:1-5. 6. van_den_Brink W, van_Santbrink H, Steyerberg E, et al. Brain oxygen tension in severe head injury. Neurosurgery. 2000;46:868-76. 7. Al-Rawi P, Kirkpatrick P. Tissue oxygen index: thresholds for cerebral ischemia using near-infrared spectroscopy. Stroke. 2006;37:2720-5. 8. Teasdale G, Jennett B. Assessment of coma and impaired consciousness: a practical scale. Lancet. 1974;2:81-4. 9. Curry R, Hollingworth W, Ellenbogen RG, Vavilala MS. Incidence of hypoand hypercarbia in severe traumatic brain injury before and after 2003 pediatric guidelines. Pediatr Crit Care Med. 2008;9:141-6. 10. Adelson PD, Clyde B, Kochanek PM, Wisniewski SR, Marion DW, Yonas H. Cerebrovascular response in infants and young children following severe traumatic brain injury: a preliminary report. Pediatr Neurosurg. 1997;26:200-7. 11. Sharples PM, Stuart AG, Matthews DS, Aynsley-Green A, Eyre JA. Cerebral blood flow and metabolism in children with severe head injury. Part 1: Relation to age, Glasgow coma score, outcome, intracranial pressure, and time after injury. J Neurol Neurosurg Psychiatry. 1995;58:14552. 12. Pokela ML, Olkkola KT, Seppala T, Koivisto M. Age-related morphine kinetics in infants. Dev Pharmacol Ther. 1993;20:26-34. 13. Adelson PD, Bratton SL, Carney NA, et al. Guidelines for the acute medical management of severe traumatic brain injury in infants, children, and adolescents. Chapter 8. Cerebral perfusion pressure. Pediatr Crit Care Med. 2003;4:S31-3. 14. Freeman SS, Udomphorn Y, Armstead WM, Fisk DM, Vavilala MS. Young age as a risk factor for impaired cerebral autoregulation after moderate to severe pediatric traumatic brain injury. Anesthesiology. 2008;108:588-95. 15. Tontisirin N, Armstead W, Waitayawinyu P, et al. Change in cerebral autoregulation as a function of time in children after severe traumatic brain

www.medicalebookpdf.com

injury: a case series. Childs Nerv Syst. 2007;23:1163-9. 16. Udomphorn Y, Armstead WM, Vavilala MS. Cerebral blood flow and autoregulation after pediatric traumatic brain injury. Pediatric neurology. 2008;38:225-34. 17. Vavilala MS, Tontisirin N, Udomphorn Y, et al. Hemispheric differences in cerebral autoregulation in children with moderate and severe traumatic brain injury. Neurocrit Care. 2008;9:4554. 18. Allen BB, Chiu YL, Gerber LM, Ghajar J, Greenfield JP. Age-specific cerebral perfusion pressure thresholds and survival in children and adolescents with severe traumatic brain injury*. Pediatr Crit Care Med. 2014;15:62-70. 19. Adelson PD, Bratton SL, Carney NA, et al. Guidelines for the acute medical management of severe traumatic brain injury in infants, children, and adolescents. Chapter 4. Resuscitation of blood pressure and oxygenation and prehospital brain-specific therapies for the severe pediatric traumatic brain injury patient. Pediatr Crit Care Med. 2003;4:S12-8. 20. Vavilala MS, Bowen A, Lam AM, et al. Blood pressure and outcome after severe pediatric traumatic brain injury. J Trauma. 2003;55:1039-44. 21. Jonas RA. Cardiac surgery and neurological injury in children. Heart Lung Circ. 2000;9:16-22. 22. Laptook A, Tyson J, Shankaran S, et al. Elevated temperature after hypoxicischemic encephalopathy: risk factor for adverse outcomes. Pediatrics. 2008;122:491-9. 23. Natale JE, Joseph JG, Helfaer MA, Shaffner DH. Early hyperthermia after traumatic brain injury in children: risk factors, influence on length of stay, and effect on short-term neurologic status. Crit Care Med. 2000;28:260815. 24. Lacroix J, Hebert PC, Hutchison JS, et al. Transfusion strategies for patients in pediatric intensive care units. N Engl J Med. 2007;356:1609-19. 25. Leal-Noval SR, Munoz-Gomez M, Arellano-Orden V, et al. Impact of age of transfused blood on cerebral oxygenation in male patients with severe traumatic brain injury. Crit Care Med. 2008;36:1290-6. 26. Gupta A, Daggett C, Ludwick J, Wells W, Lewis A. Ketorolac after congenital heart surgery: does it increase the risk of significant bleeding

www.medicalebookpdf.com

complications? Paediatr Anaesth. 2005;15:139-42. 27. Kokki H. Nonsteroidal anti-inflammatory drugs for postoperative pain: a focus on children. Paediatr Drugs. 2003;5:103-23. 28. Vitale MG, Choe JC, Hwang MW, et al. Use of ketorolac tromethamine in children undergoing scoliosis surgery. an analysis of complications. Spine J. 2003;3:55-62. 29. Levy ML, Granville RC, Hart D, Meltzer H. Deep venous thrombosis in children and adolescents. J Neurosurg. 2004;101:32-7. 30. Vu LT, Nobuhara KK, Lee H, Farmer DL. Determination of risk factors for deep venous thrombosis in hospitalized children. J Pediatr Surg. 2008;43:1095-9. 31. Sandoval JA, Sheehan MP, Stonerock CE, Shafique S, Rescorla FJ, Dalsing MC. Incidence, risk factors, and treatment patterns for deep venous thrombosis in hospitalized children: an increasing population at risk. J Vasc Surg. 2008;47:837-43. 32. Sebire G, Tabarki B, Saunders DE, et al. Cerebral venous sinus thrombosis in children: risk factors, presentation, diagnosis and outcome. Brain. 2005;128:477-89. 33. Roach E, Golomb M, Adams R, et al. Management of stroke in infants and children. Stroke. 2008;39:2644-91. 34. deVeber G. In pursuit of evidence-based treatments for paediatric stroke: the UK and Chest guidelines. Lancet Neurology. 2005;4:432-6. 35. Stroke in childhood: clinical guidelines for diagnosis, management and rehabilitation. Clinical Effectiveness & Evaluation Unit. 2004;London: Royal College of Physicians. 36. Monagle P, Chan A, Massicotte P, Chalmers E, Michelson A. Antithrombotic therapy in children: the Seventh ACCP conference on antithrombotic and antithromoblytic therapy. Chest. 2004;126:645S-87S. 37. Shellhaas R, Smith S, O'Tool E, Licht D, Ichord R. Mimics of childhood stroke: characteristics of a prospective cohort. Pediatrics. 2006;118:704-9. 38. Gabis L, Yangala R, Lenn N. Time lag to diagnosis of stroke in children. Pediatrics. 2002;110:9248. 39. Ganesan V, Hogan A, Shack N, Gordon A, Isaacs E, Kirkham F. Outcome

www.medicalebookpdf.com

after ischaemic stroke in childhood. Develoopmental Medicine & Child Neurology. 2000;42:455-61. 40. deVeber G, MacGregor D, Curtis R, Mayank S. Neurologic outcome in survivors of childhood arterial ischemic stroke and sinovenous thrombosis. J Child Neurol. 2000;15:316-24. 41. Chabrier S, Husson B, Lasjaunias P, Landrieu P, Tardieu M. Stroke in childhood: outcome and recurrence risk by mechanism in 59 patients. J Child Neurol. 2000;15:290-4. 42. Lanthier S, Carmant L, David M, Larbrisseau A, De_Veber G. Stroke in children: the coexistence of multiple risk factors predicts poor outcome. Neurology. 2000;54:371-8. 43. Sträter R, Becker S, von_Eckardstein A, et al. Prospective assessment of risk factors for recurrent stroke during childhood - a 5-year follow-up study. Lancet. 2002;360:1540-5. 44. Ganesan V, Prengler M, Wade A, Kirkham F. Clinical and radiological recurrence after childhood arterial ischemic stroke. Circulation. 2006;114:2170-2. 45. Lynch J, Hirtz D, deVeber G, Nelson K. Report of the National Institute of Neurological Disorders and Stroke workshop on perinatal and childhood stroke. Pediatrics. 2002;109:116-23. 46. Nowak-Gottl U, Gunther G, Kurnik K, Strater R, Kirkham F. Arterial ischemic stroke in neonates, infants and children: an overview of underlying conditions, imaging methods, and treatment modalities. Sem Thrombosis Hemostasis. 2003;29:405-14. 47. Ganesan V, Prengler M, McShane M, Wade A, Kirkham F. Investigation of risk factors in children with arterial ischemic stroke. Ann Neurol. 2003;53:167-73. 48. Kirkham F, Prengler M, Hewes D, Ganesan V. Risk factors for arterial ischemic stroke in children. J Child Neurol. 2000;15:299-307. 49. Fullerton H, Wu Y, Sidney S, Claiborne_Johnston S. Risk of recurrent childhood arterial ischemic stroke in a population-based cohort: the importance of cerbrovascular imaging. Pediatrics. 2007;119:495-501. 50. Gilden D, Kleinschmidt-DeMasters B, LaGuardia J, Mahalingam R, Cohrs R. Neurologic complications of the reactivation of varicella-zoster virus. N

www.medicalebookpdf.com

Engl J Med. 2000;342:635-45. 51. Chabrier S, Lasjaunias P, Husson B, Landrieu P, Tardieu M. Ischaemic stroke from dissection of the craniocervical arteries in childhood: report of 12 patients. Eur J Paediatr Neurol. 2003;7:39-42. 52. Rafay M, Armstrong D, deVeber G, Domi T, Chan A, MacGregor D. Craniocervical arterial dissection in children: clinical and radiographic presentation and outcome. J Child Neurol. 2006;21:816. 53. Kurokawa T, Tomita S, Ueda K, et al. Prognosis of occlusive disease of the circle of Willis (moyamoya disease) in children. Pediatric neurology. 1985;1:274-7. 54. Fung L, Thompson D, Ganesan V. Revascularization surgery for paediatric moyamoya: a review of the literature. Childs Nervous System. 2005;21:358-64. 55. Lynch J, Han C, Nelson K. Prothrombotic factors in children with stroke or porencephaly. Pediatrics. 2005;116:447-53. 56. Strater R, Vielhaber H, Kassenbohmer R, von_Kries R, Gobel U, NowakGottl U. Genetic risk factors of thrombophilia in ischaemic stroke of cardiac origin: A prospective ESPED study. Eur J Pediatr. 1999;158:S122S5. 57. Al-Jarallah A, Al-Rifai M, Riela A, Roach E. Nontraumatic brain hemorrhage in children: etiology and presentation. J Child Neurol. 2000;15:284-9. 58. Kirkham F. Stroke in childhood. Arch Dis Child. 1999;81:85-9. 59. Sebire G, Tabarki B, Saunders D, et al. Cerebral venous sinus thrombosis in children: risk factors, presentation, diagnosis and outcome. Brain. 2005;128:477-89. 60. Adaletli I, Sirikci A, Kara B, Kurugoglu S, Ozer H, Bayram M. Cerebral venous sinus thrombosis presenting with excessive subarachnoid hemorrhage in a 14-year-old boy. Emerg Radiol. 2005;12:57-9. 61. Kirkham F. Therapy insight: stroke risk and its management in patients with sickle cell disease. Nature Clin Practice Neurology. 2007;3:254-78. 62. Switzer J, Hess D, Nichols F, Adams R. Pathophysiology and treatment of stroke in sickle-cell disease: present and future Lancet Neurol. 2006;5:501-

www.medicalebookpdf.com

12. 63. Ohene-Frempong K, Weiner S, Sleeper L, et al. Cerebrovascular accidents in sickle cell disease: rates and risk factors. Blood. 1998;91 and the Cooperative Study of Sickle Cell Disease 288-94. 64. Earley C, Kittner S, Feeser B, et al. Stroke in children and sickle-cell disease. Neurology. 1998;51:169-76. 65. Strouse J, Hulbert M, DeBaun M, Jordan L, JF Casella. Primary hemorrhagic stroke in children with sickle cell disease is associated with recent transfusion and use of corticosteroids. Pediatrics. 2006;118:1916-24. 66. Dobson S, Holden K, Nietert P, et al. Moyamoya syndrome in childhood sickle cell disease: a predictive factor for recurrent cerebrovascular events. Blood. 2002;99:3144-50. 67. Prengler M, Pavlakis S, Boyd S, et al. Sickle cell disease: ischemia and seizures. Ann Neurol. 2005;58:290-302. 68. Adams R, McKie V, Hsu L, et al. Prevention of a first stroke by transfusion in children with sickle cell anemia and abnormal results on transcranial doppler ultrasonography. N Engl J Med. 1998;338:5-11. 69. Adams H, del_Zoppo G, Alberts M, et al. Guidelines for the management of adults with ischemic stroke. Stroke. 2007;38:1655-711. 70. Goldstein L. Acute ischemic stroke treatment in 2007. Circulation. 2007;116:1504-14. 71. Carlson M, Leber S, Deveikis J, Silverstein F. Successful use of rt-PA in pediatric stroke. Neurology. 2001;57:157-8. 72. Chiaretti A, De Benedictis R, Langer A, et al. Prognostic implications of hyperglycaemia in paediatric head injury. Childs Nerv Syst. 1998;14:455-9. 73. Cochran A, Scaife ER, Hansen KW, Downey EC. Hyperglycemia and outcomes from pediatric traumatic brain injury. J Trauma. 2003;55:1035-8. 74. Moritz ML, Ayus JC. Preventing neurological complications from dysnatremias in children. Pediatr Nephrol. 2005;20:1687-700. 75. Carpenter J, Weinstein S, Myseros J, Vezina G, Bell MJ. Inadvertent hyponatremia leading to acute cerebral edema and early evidence of herniation. Neurocrit Care. 2007;6:195-9. 76. Meert KL, Daphtary KM, Metheny NA. Gastric vs small-bowel feeding in

www.medicalebookpdf.com

critically ill children receiving mechanical ventilation: a randomized controlled trial. Chest. 2004;126:872-8. 77. Montejo JC, Grau T, Acosta J, et al. Multicenter, prospective, randomized, single-blind study comparing the efficacy and gastrointestinal complications of early jejunal feeding with early gastric feeding in critically ill patients. Crit Care Med. 2002;30:796-800. 78. Spain DA, DeWeese RC, Reynolds MA, Richardson JD. Transpyloric passage of feeding tubes in patients with head injuries does not decrease complications. J Trauma. 1995;39:1100-2. 79. Verbalis JG. Disorders of body water homeostasis. Best Pract Res Clin Endocrinol Metab. 2003;17:471-503. 80. Wise-Faberowski L, Soriano SG, Ferrari L, et al. Perioperative management of diabetes insipidus in children. J Neurosurg Anesthesiol. 2004;16:220-5. 81. Smith D, Finucane F, Phillips J, et al. Abnormal regulation of thirst and vasopressin secretion following surgery for craniopharyngioma. Clin Endocrinol (Oxf). 2004;61:273-9. 82. Bussmann C, Bast T, Rating D. Hyponatraemia in children with acute CNS disease: SIADH or cerebral salt wasting? Childs Nerv Syst. 2001;17:58-62; discussion 3. 83. Jimenez R, Casado-Flores J, Nieto M, Garcia-Teresa MA. Cerebral salt wasting syndrome in children with acute central nervous system injury. Pediatric neurology. 2006;35:261-3. 84. Lehrnbecher T, Muller-Scholden J, Danhauser-Leistner I, Sorensen N, von Stockhausen HB. Perioperative fluid and electrolyte management in children undergoing surgery for craniopharyngioma. A 10-year experience in a single institution. Childs Nerv Syst. 1998;14:276-9. 85. Levine JP, Stelnicki E, Weiner HL, Bradley JP, McCarthy JG. Hyponatremia in the postoperative craniofacial pediatric patient population: a connection to cerebral salt wasting syndrome and management of the disorder. Plast Reconstr Surg. 2001;108:1501-8. 86. Palmer BF. Hyponatremia in patients with central nervous system disease: SIADH versus CSW. Trends Endocrinol Metab. 2003;14:182-7. 87. Sata A, Hizuka N, Kawamata T, Hori T, Takano K. Hyponatremia after transsphenoidal surgery for hypothalamo-pituitary tumors.

www.medicalebookpdf.com

Neuroendocrinology. 2006;83:117-22. 88. Berkenbosch JW, Lentz CW, Jimenez DF, Tobias JD. Cerebral salt wasting syndrome following brain injury in three pediatric patients: suggestions for rapid diagnosis and therapy. Pediatr Neurosurg. 2002;36:75-9. 89. Kochanek PM, Carney N, Adelson PD, et al. Guidelines for the acute medical management of severe traumatic brain injury in infants, children, and adolescents--second edition. Pediatr Crit Care Med. 2012;13 Suppl 1:S1-82. 90. Carney NA, Chesnut R, Kochanek PM. Guidelines for the acute medical management of severe traumatic brain injury in infants, children, and adolescents. Pediatr Crit Care Med. 2003;4:S1. 91. Pineda JA, Leonard JR, Mazotas IG, et al. Effect of implementation of a paediatric neurocritical care programme on outcomes after severe traumatic brain injury: a retrospective cohort study. Lancet Neurol. 2013;12:45-52. 92. Shetty R, Singhi S, Singhi P, Jayashree M. Cerebral perfusion pressure-targeted approach in children with central nervous system infections and raised intracranial pressure: is it feasible? J Child Neurol. 2008;23:192-8. 93. Odetola FO, Tilford JM, Davis MM. Variation in the use of intracranialpressure monitoring and mortality in critically ill children with meningitis in the United States. Pediatrics. 2006;117:1893900. 94. Raschke RA, Curry SC, Rempe S, et al. Results of a protocol for the management of patients with fulminant liver failure. Crit Care Med. 2008;36:2244-8. 95. Wendon J, Lee W. Encephalopathy and cerebral edema in the setting of acute liver failure: pathogenesis and management. Neurocrit Care. 2008;9:97-102. 96. Kamat P, Kunde S, Vos M, et al. Invasive intracranial pressure monitoring is a useful adjunct in the management of severe hepatic encephalopathy associated with pediatric acute liver failure. Pediatr Crit Care Med. 2012;13:e33-8. 97. Le Roux PD, Jardine DS, Kanev PM, Loeser JD. Pediatric intracranial pressure monitoring in hypoxic and nonhypoxic brain injury. Childs Nerv Syst. 1991;7:34-9. 98. Wainwright M. Disorders of intracranial pressure. In: Swaiman K AS,

www.medicalebookpdf.com

Ferriero D, Schor N, ed. Pediatric Neurology: Principles and Practice. 5th ed: Elsevier; 2011:1185-97. 99. Treatment of convulsive status epilepticus. Recommendations of the Epilepsy Foundation of America's Working Group on Status Epilepticus. JAMA. 1993;270:854-9. 100. Meldrum B. The revised operational definition of generalized statusepilepticus in adults. Epilepsia. 1999;40:123-4. 101. Chen J, Wasterlain C. Status epilepticus: pathophysiology and management in adults. Lancet Neurol. 2006;5:246-56. 102. Lowenstein D, Alldredge B. Status epilepticus. N Engl J Med. 1998;338:970-6. 103. Gilbert D, Gartside P, Glauser T. Efficacy and mortality in treatment of refractory generalized convulsive status epilepticus in children. J Child Neurol. 1999;14:602-9. 104. Towne A, Waterhouse E, Boggs J, et al. Prevalence of nonconvulsive status epilepticus in comatose patients. Neurology. 2000;54:340-5. 105. Abend NS, Arndt DH, Carpenter JL, et al. Electrographic seizures in pediatric ICU patients: Cohort study of risk factors and mortality. Neurology. 2013;81:383-91. 106. Vespa P, O'Phelan K, Shah M, et al. Acute seizures after intracerebral hemorrhage: a factor in progressive midline shift and outcome. Neurology. 2003;60:1441-6. 107. Vespa P, Nuwer M, Nenov V, et al. Increased incidence and impact of nonconvulsive and convulsive seizures after traumatic brain injury as detected by continuous electroencephalographic monitoring J Neurosurg. 1999;91:750-6. 108. Claassen J, Mayer S, Kowalski R, Emerson R, Hirsch L. Detection of electrographic seizures with continuous EEG monitoring in critically ill patients. Neurology. 2004;62:1743-8. 109. Walker D, Teach S. Update on the management of status epilepticus in children. Current opinion in pediatrics. 2006;18:239-44. 110. Glauser T, Ben-Menachem E, Bourgeois B, et al. ILAE treatment guidelines: Evidence-based analysis of antiepileptic drug efficacy and

www.medicalebookpdf.com

effectiveness as initial monotherapy for epileptic seizures and syndromes. Epilepsia. 2006;47:1094-120. 111. Abend N, Dlugos D. Treatment of refractory status epilepticus: literature review and a proposed protocol. Pediatric neurology. 2008;38:377-90. 112. Wilkes R, Tasker RC. Pediatric intensive care treatment of uncontrolled status epilepticus. Critical care clinics. 2013;29:239-57. 113. McCoy B, Sharma R, Ochi A, et al. Predictors of nonconvulsive seizures among critically ill children. Epilepsia. 2011;52:1973-8. 114. Abend N, Gutierrez-Colina A, Topjian A, et al. Nonconvulsive seizures are common in critically ill children. Neurology. 2011;76:1071-7. 115. Payne E, Zhao X, Frndova H, et al. Seizure burden is independently associated with short term outcome in critically ill children. Brain. 2014;137:1429-38. 116. Holmes G. Effects of seizures on brain development: lessons from the laboratory. Pediatric neurology. 2005;33:1-11. 117. Abend N, Dlugos D, Hahn C, Hirsch L, Herman S. Use of EEG monitoring and management of non-convulsive seizures in critically ill patients: a survey of neurologists. Neurocrit Care. 2010;10.1007/s12028-010-9337-2. 118. Arndt D, Lerner J, Matsumoto J, et al. Subclinical early posttraumatic seizures detected by continuous EEG monitoring in a consecutive pediatric cohort. Epilepsia. 2013;54:1780-8. 119. Wagenman K, Blake T, Sanchez S, et al. Electrographic status epilepticus and long-term outcome in critically ill children. Neurology. 2014;82:396404. 120. Saez-Llorens X, McCracken G. Bacterial meningitis in children. Lancet. 2003;361:2139-48. 121. Odio C, Faingezicht I, Paris M, et al. The beneficial effects of early dexamethasone administration in infants and children with bacterial meningitis. N Engl J Med. 1991;324:1525-31. 122. Lebel M, Freij B, Syrogiannopoulos G, et al. Dexamethasone therapy for bacterial meningitis. Results of two double-blind, placebo-controlled trials. N Engl J Med. 1988;391:964-71. 123. McIntyre P, MacIntyre C, Gilmour R, Wang H. A population based study of

www.medicalebookpdf.com

the impact of corticosteroid therapy and delayed diagnosis on the outcome of childhood pneumoccal meningitis. Arch Dis Child. 2005;90:391-6. 124. Ashwal S, Perkin R, Thompson J, Schneider S, Tomasi L. Bacterial meningitis in children: current concepts of neurologic management. Curr Prob Pediatr. 1994;24:267-84. 125. Kaplan S. Adjunctive therapy in meningitis. Adv Pediatr Infect Dis. 1995;10:167-86. 126. Odetola F, Tilford J, Davis M. Variation in the use of intracranial-pressure monitoring and mortality in critically ill children with meningitis in the United States Pediatrics. 2006;117:1893900. 127. Thwaites G, Hien T. Tuberculous meningitis: many questions, too few answers. Lancet Neurol. 2005;4:160-70. 128. Leonard J, Des_Prez R. Tuberculous meningitis. Infect Dis Clin North Am. 1990;4:769-87. 129. Thwaites G, Chau T, Mai N, Drobniewski F, McAdam K, Farrar J. Neurological aspects of tropical disease: tuberculous meningitis. J Neurol Neurosurg Psych. 2000;68:289-99. 130. Palur R, Rajshekhar V, Chandy M, Joseph T, Abraham J. Shunt surgery for hydrocephalus in tuberculous meningitis: a long-term follow up study. J Neurosurg. 1991;74:64-9. 131. Lamprecht D, Schoeman J, Donald P, Hartzenberg H. Ventriculoperitoneal shunting in childhood tuberculous meningitis. Br J Neurosurg. 2001;15:119-25. 132. Schoeman J, Van_Zyl L, Laubscher J, Donald P. Effect of corticosteroids on intracranial pressure, computed tomographic findings and clincal outcome in young children with tuberculous meningitis. Pediatrics. 1997;99:226-31. 133. White A. Neurocysticercosis: a major cause of neurological disease worldwide. Clin Inf Dis. 1997;24:101-14. 134. Nash T, Singh G, White A, et al. Treatment of neurocysticercosis. Neurology. 2006;67:1120-7. 135. Kolski H, Ford-Jones E, Richardson S, et al. Etiology of acute childhood encephalitis at The Hospital for Sick Children, Toronto, 1994-1995. Clin Inf Dis. 1998;26:398-409.

www.medicalebookpdf.com

136. Chen Y, Fang P, Chow J. Clinical characteristics and prognostic factors of postencephalitic epilepsy in children. J Child Neurol. 2006;12:1047-51. 137. Millichap J, Wainwright M. Neurological complications of respiratory syncytial virus infection: case series and review of literature. J Child Neurol. 2009; 24:1499-503. 138. Seeley W, Marty F, Holmes T, et al. Post-transplant acute limbic encephalitis: Clinical features and relationship to HHV6. Neurology. 2007;69:156-65. 139. Wainwright MS, Martin PL, Morse RP, et al. Human herpesvirus 6 limbic encephalitis after stem cell transplantation. Ann Neurol. 2001;50:612-9. 140. Theodore W, Epstein L, Gaillard W, Shinnar S, Wainwright M, Jacobson S. Human Herpes Virus 6B: A possible role in epilepsy? . Epilepsia. 2008;49:1828-37. 141. A definition of irreversible coma. Report of the Ad Hoc Committee of the Harvard Medical School to Examine the Definition of Brain Death. JAMA. 1968;205:337-40. 142. Guidelines for the determination of death. Report of the medical consultants on the diagnosis of death to the President's Commission for the Study of Ethical Problems in Medicine and Biomedical and Behavioral Research. JAMA. 1981;246:2184-6. 143. Report of special Task Force. Guidelines for the determination of brain death in children. American Academy of Pediatrics Task Force on Brain Death in Children. Pediatrics. 1987;80:298-300. 144. Nakagawa TA, Ashwal S, Mathur M, et al. Guidelines for the determination of brain death in infants and children: an update of the 1987 Task Force recommendations. Crit Care Med. 2011;39:2139-55. 145. Wijdicks EF. The diagnosis of brain death. N Engl J Med. 2001;344:121521. 146. Monterrubio-Villar J, Cordoba-Lopez A. Barotrauma during apnoea testing for brain death determination in a five-year-old boy. Anaesth Intensive Care. 2008;36:462-3. 147. Mathur M, Petersen L, Stadtler M, et al. Variability in pediatric brain death determination and documentation in southern California. Pediatrics. 2008;121:988-93.

www.medicalebookpdf.com

148. Bell MJ, Carpenter J, Au AK, et al. Development of a Pediatric Neurocritical Care Service. Neurocrit Care. 2008. 149. Diringer MN, Edwards DF. Admission to a neurologic/neurosurgical intensive care unit is associated with reduced mortality rate after intracerebral hemorrhage. Crit Care Med. 2001;29:63540. 150. Fakhry SM, Trask AL, Waller MA, Watts DD. Management of braininjured patients by an evidence-based medicine protocol improves outcomes and decreases hospital charges. J Trauma. 2004;56:492-9; discussion 9-500.

www.medicalebookpdf.com

PEDIATRIC NEUROCRITICAL CARE QUESTIONS 1. The most common cause of cardiac arrest in children is: a. Respiratory decompensation b. Cardiac arrhythmia c. Intracerebral hemorrhage d. Traumatic brain injury e. Infection 2. Regarding stroke in children, the following statements are true except: a. Stroke is one of the top ten causes of death in children b. Average time from onset of symptoms to presentation for medical attention is 18 hours c. There is no data on the safety and efficacy of anti-thrombotic medications d. Over 60% of children diagnosed with stroke have residual neurological deficits e. Large artery atherosclerosis or small vessel occlusive disease are uncommon in pediatric cerebrovascular disease 3. The following statement is true regarding cerebrovascular disease in children with sickle cell disease (SCD): a. Central nervous system venous thrombosis is a rare but important cause of stroke in children with SCD b. Patients with acute onset of focal neurological symptoms may benefit from emergency exchange transfusion c. Imaging studies may demonstrate evidence of previous silent strokes on presentation d. Acute chest syndrome is a risk factor e. All of the above

www.medicalebookpdf.com

4. Criteria for post-operative Diabetes Insipidus include: a. Polyuria (urine output > 3 mL/kg/h) b. Serum sodium > 145 mEq/L c. Increased plasma osmolarity (>300 mOsm/kg) d. Hypotonic urine (urine Osm 35 °C is required d. Children should be maintained at normal temperature (37 °C) during the determination of death by neurological criteria

www.medicalebookpdf.com

e. The neurological exam can be done at any body temperature, provided an ancillary test is conducted

www.medicalebookpdf.com

PEDIATRIC NEUROCRITICAL CARE ANSWERS 1. The correct answer is A. Cardiac arrest in children commonly results in death or significant brain injury. The risk of brain injury is present after even a short period of chest compressions. Respiratory decompensation is the most common cause of cardiac arrest in children. 2. The correct answer B. The average interval between onset of initial symptoms and presentation for medical evaluation is 34 hours. Delayed presentation is one of the challenges precluding testing and effective implementation of pharmacological therapies for pediatric stroke. 3. The correct answer E. Cerebrovascular disease is an important cause of morbidity in children with SCD. Since central venous thrombosis requires a different therapeutic approach (anticoagulation), this rare but important diagnosis should always be kept in mind and justifies the need for MRI studies as soon as safe and feasible. 4. The correct answer is E. To avoid confusion with other conditions such as normal post-operative diuresis, patients should meet al inclusion criteria before treatment is initiated. 5. The correct answer is D. Patients with all the diagnoses above are at risk of developing Diabetes Insipidus, but trancranial resection of a pitituary tumor represents the highest risk. 75% of patients develop this, usually transient, neuroendocrine abnormality in the immediate post-operative period. 6. The correct answer is E. While intracranial pressure monitoring appears to be safe and at times effective in children with non-traumatic brain injury, severe acute trauma in children with a Glasgow Scale Core of 8 or less and abnormalities in CT scan is the only widely used indication. Even in patients with traumatic brain injury there is large practice variability (1998% of children are reported to undergo ICP monitoring). 7. The correct answer is E. The incidence of meningitis has been markedly reduced through successful immunization programs. However, it continues to be a serious condition in children. The maintenance of normal oxygenation, adequate cerebral perfusion pressure, prevention of hypoglycemia and hyponatremia, early detection of seizures, AIS, ICH or cerebral sinus venous thrombosis are essential in optimizing neurological

www.medicalebookpdf.com

outcome. 8. The correct answer is C. Fever is associated with worsened outcome in pediatric TBI. Hyperventilation decreases cerebral blood flow in children and may worsen outcome. While the 5th percentile systolic blood pressure for age is commonly used as the threshold to define hypotension, optimal blood pressure thresholds for children with acute brain injury have not been established. Tight glucose control after TBI has not been studied in prospective trials. 9. The correct answer is E. Updated guidelines changed the recommendation for interval between exams to be: term gestational age and up to 30 days old, 24 hours; 31 days old to 18 years of age, 12 hours. If necessary, the interval between exams may be shortened (ancillary testing is recommended in such cases). 10. The correct answer is C. Small children and infants are at particular risk of rapidly developing hypothermia. Body temperature (rectal or bladder temperature) >35 °C should be reached and maintained prior to and during the determination of death by neurological criteria.

www.medicalebookpdf.com

Chapter 22

PHARMACOLOGY IN THE NEUROINTENSIVE CARE UNIT Shaun Rowe and Theresa Murphy-Human CLINICAL CASE A 35-year-old African-American male arrived at the emergency department after a motor vehicle collision. He was unrestrained and had been ejected from the vehicle on collision with a concrete barrier. On presentation, he had a Glasgow Coma Score (GCS) of 4T (intubated, abnormal flexion to painful stimulus, did not open eyes), pupils were 5 mm (reactive) on the right and 2 mm (reactive) on the left. His past medical history was significant for epilepsy, hypertension and diabetes mellitus. His wife reported that he smokes approximately one pack of cigarettes per day and has an occasional alcoholic beverage. His home medications prior to admission included phenytoin 300 mg po at each bedtime, amlodipine 10 mg po each day, and hydrochlorothiazide 25 mg po each day Admission Laboratory studies showed normal electrolytes, BUN, creatinine, and glucose. Hemoglobin was 15 and platelet count was 200,000. Arterial blood gas was 7.2/60/150. Total phenytoin level was < 1. Initial non-contrast computed tomography (CT) of the head revealed scattered subarachnoid hemorrhage, a subdural hematoma on the left, and 5 mm of midline shift. He was admitted to the neurointensive care unit. After placement of an external ventricular drain (EVD) his intracranial pressure (ICP) was 20 mmHg. He received mannitol 50 g IV q2 h for ICPs greater than 20 mmHg along with cerebrospinal fluid (CSF) diversion. The patient was sedated with propofol at 35 mcg/kg/min and fentanyl 100 mcg/hour. He was started on intravenous (IV) levetiracetam 1000 mg twice daily for seizure prophylaxis secondary to his traumatic brain injury. After 24 hours, it was noted that his osmolar gap was 20 mOsm/kg and his sodium was 142 mmol/L. It was decided to change to 23.4% saline administered intravenously for osmotic therapy. Continuous EEG showed slowing on the left side without evidence of seizure activity. On hospital day 4 his ICPs were consistently in the 10 to 15 mmHg range and he did not require 23.4% saline. He localized to painful stimuli and opened his eyes to voice when his analgesia and sedation were titrated down (GCS 8T).

www.medicalebookpdf.com

His pupils were 5 mm, equal and reactive. The nurse reported that he had a 24hour maximum temperature of 102.5 ° F (39.2 ° C) with increased sputum from his endotracheal tube. His chest xray was significant for consolidation of the right lower lobe. He was empirically started on vancomycin 1000 mg IV q12h and piperacillin/tazobactam 4.5g IV every 6 hours (q6 hrs). On the fourth dose of vancomycin, the vancomycin trough was 5 mcg/ml. His dose was increased to 1500 mg IV q8 hours (20 mg/kg). The EVD was discontinued by hospital Day 10. Attempts to extubate him failed due to increased anxiety and combativeness. He had received multiple boluses of lorazepam in addition to the propofol during this time period. Due to the amount of lorazepam and propofol that he had received, he failed his spontaneous breathing trial. The medical team decided to change the propofol infusion to dexmedetomidine (4mg/kg/hr, titrate to RASS 0). He tolerated dexmedetomidine with no hemodynamic changes and was successfully extubated 24 hours after initiation. On hospital day 12, he was transferred to a monitored acute care floor with plans for discharge to a rehabilitation center. On transfer he had a GCS of 14 with occasional periods of confusion. OVERVIEW The appropriate and optimal use of pharmacologic agents in neurocritical care is a fundamental topic that spans the treatment of essentially every disease encountered in this setting. Many aspects regarding specific treatments are covered in other disease-related chapters. The purpose of this chapter is to provide broad substantial information regarding pharmacokinetics, monitoring, side effects, and drug interactions for many of the agents commonly used in the neurointensive care unit. Specific areas are highlighted in outline form and numerous tables are included that should serve as reference when considering pharmacological aspects of patient treatment. BASIC PHARMACOKINETICS Major Factors That Influence Bioavailability in Critically Ill Patients [1,2] 1.Route of Administration a. Subcutaneous (SQ), intramuscular r(IM), transdermal, and oral administration (PO) may have bioavailability concerns in critically ill patients. i. Hypoperfusion (hypotension/hypovolemia) generally cause reduction

www.medicalebookpdf.com

in medication absorption secondary to reduced blood flow ii. Fever and external warming devices may cause rapid absorption of transdermally administered agents 2. Gastrointestinal Motility a. Alterations in GI motility have unpredictable effects on drug absorption. b. Increased GI motility increases absorption of acetaminophen and lithium but decreases absorption of cimetidine and digoxin. 3. Drug-Drug Interactions a. Ketoconazole, tetracycline and sucralfate require an acidic environment for absorption/activity, but when given with concurrent GI stress ulcer prophylaxis (H2 blockers or PPIs), their bioavilability/activity are greatly reduced. b. Glyburide absorption is increased when given with concomitant gastric acid reducing medications. c. Substrates, inhibitors and inducers of the cytochrome P450 enzymatic system must be identified and considered when dosing and monitoring. Isoenzyme 3A4 is most commonly affected, but others may also have significant effects. (see appendix) 4. Protein Binding a. Protein binding may be altered in critically ill patients b. Multiple highly protein bound medications may compete for binding sites, with resultant increases in “free” or unbound active medication (e.g. warfarin, valproate, phenytoin, diazepam) 5. Metabolism a. Higher doses of medications may be necessary in the acute period of critical illness when metabolism may be dramatically increased. i. Traumatic Brain Injury (TBI), Spinal Cord Injury (SCI), Burns

www.medicalebookpdf.com

6. Hypothermia a. Slows drug absorption and delays time to maximum concentration b. General decrease in volume of distribution c. Decreased hepatic metabolism and decreased renal clearance lead to a decrease in total clearance of many medications 7. Therapeutic Drug Monitoring (Table 22-1)

www.medicalebookpdf.com

SEDATION, ANALGESIA AND DELIRIUM [3-5] Clinical Pearls 1. “Analgesia-first” sedation should be used if possible. However, due to an increased risk of physiological stress, light sedation may not be appropriate in all neurointensive care unit patient 2. If a benzodiazepine is needed, midazolam is the most appropriate for shortterm sedation. Midazolam has the highest lipid solubility, fastest onset, and shortest duration of currently available benzodiazepines. 3. Continuous infusion of midazolam lasting more than a few hours can produce prolonged sedation due to drug accumulation in the CNS and accumulation of the active metabolite. Medications that inhibit cytochrome P450 metabolism can also contribute to prolonged sedation with midazolam. 4. Lorazepam contains the solvent propylene glycol. Prolonged administration or large doses can cause paradoxical agitation, metabolic acidosis. A clinical syndrome can resemble sepsis and acute kidney injury. The serum osmolar gap can be used to monitor for propylene glycol accumulation. A gap of greater than 12 mOsmol/L may be indicative of accumulation. When large doses or prolonged administration of a benzodiazepine are necessary, midazolam should be considered. 5. Propofol Related Infusion Syndrome (PRIS) is a rare, but potentially serious complication of propofol. It is characterized by rapid onset of heart failure, bradycardia, lactic acidosis, hyperlipidemia, hyperkalemia, and rhabdomyolysis. Risk factors associated with this syndrome include high doses, prolonged infusions, low BMI, younger age, and concomitant vasopressor use. Mortality has been reported as high as 80% despite supportive care. 6. Dexmedetomidine is the can be used for sedation in a non-mechanically ventilated patient or in patients weaning from mechanical ventilator. 7. Dexmedetomidine has added benefit in shivering control. 8. Dexmedetomidine may decrease the duration of delirium. 9. Bolus dosing of dexmedetomidine is rarely recommended due to the increased risk of hypotension and bradycardia.

www.medicalebookpdf.com

10. Benzodiazepine reversal: flumazenil – Works as a competitive inhibitor at the benzodiazepine receptor. General dosing is 0.2 mg IV pushed over 15 seconds. Due to the risk of seizures, use with caution in patients who are receiving chronic benzodiazpepines. Use the lowest effective dose.

Clinical Pearls 1. Fentanyl is 600 times more lipid soluble than morphine and therefore is taken up by the CNS more readily which produces a quicker onset of action. 2. Morphine has active metabolites that can accumulate in renal failure. Dose reduction is necessary. 3. Morphine promotes the release of histamine and can cause vasodilation and hypotension. Because fentanyl does not promote histamine release, it is the preferred agent for patients with hemodynamic compromise. 4. Secondary to normeperidine accumulation, which may increase the risk of neurotoxicity (seizures) in critically ill patients, meperidine is not advised for pain control in ICU patients. 5. Transdermal fentanyl has a slow onset of action (12-24 hours), therefore an adequate overlap of dosing of oral or IV opioids to maintain pain relief must occur until the onset of activity occurs. 6. Transdermal fentanyl should not be used in patients with intense sweating because delivery of medication is unpredictable. 7. Opioid Reversal: Naloxone – works via antagonism and displacement of opioids at the opioid receptor site. Can be administered IV, IM or SubQ at a

www.medicalebookpdf.com

dose of 0.4mg to 2 mg. The dose may need to be repeated every 2 to 3 minutes. This can cause acute opioid withdrawal and subsequent hyperdynamic response (tachycardia, hypertension, irritability, sweating, etc.). Respiratory suppression can reoccur depending on the half-life of the opioid.

NEUROMUSCULAR BLOCKING AGENTS (NMBA) [6] Indications for the use of neuromuscular blocking agents include the performance of tracheal intubation, to facilitative mechanical ventilation, management of elevated intracranial pressure, management of pathologic tetany, and to abolish profound refractory shivering during therapeutic temperature management. Dosing of various NMBA are provided in Table 22-5 and pharmacokinetics are provided in Table 22-6. Mechanism of Action Depolarizing NMBA mimic the effects of acetylcholine by binding and causing Na+ and Ca+ influx resulting in membrane depolarization and muscle contraction. Continued stimulation inhibits repolarization, resulting in muscle paralysis. Succinlycholine is currently the only available depolarizing NMBA. The depolarization of muscle cells produced by this agent may be accompanied by potassium efflux out of the cells resulting in a rise in serum K+ of 0.5-1 mEq/L. Caution for use in head or spinal cord injury and should be avoided in patients with rhabdomyolysis, hemorrhagic shock, thermal injury and chronic immobility because life threatening increases in serum K+ can occur. Nondepolarizing NMBAs prevent acetylcholine from binding to the receptor through competitive antagonism. Conditions where fewer acetylcholine receptors are present will lead to an increase in sensitivity to nondepolarizing agents (e.g. myasthenia gravis).

www.medicalebookpdf.com

Clinical Pearls 1. NMBA's do not produce analgesia or sedation; therefore, it is imperative to establish desired levels of both prior to and continuously during paralysis. 1. Eye lubricant must be scheduled to prevent corneal abrasions. 2. Secretion management 3. DVT prophylaxis 4. Repositioning, when possible, to avoid pressure injuries. 5. Cisatracurium (or atracurium) is recommended in patients with significant hepatic and/or renal impairment.

www.medicalebookpdf.com

6. Due to the potential risk of prolonged weakness, attempts should be made to avoid concomitant administration of corticosteroids with aminosteroidal NMBA (vecuronium, pancuronium, etc.). 7. Monitor effect with peripheral nerve stimulator. OSMOTIC AGENTS Mannitol [7-9] Osmotic Effects Proposed Mechanism of Action of Mannitol Decreases blood viscosity and increases intravascular volume; thus increases cerebral blood flow Extract water from the brain by creating an osmotic gradient that pulls fluid from intracellular and interstitial spaces into the intravascular space. Mannitol does not cross an intact blood brain barrier (BBB), but like all osmotic agents, it can cross a damaged BBB and may accumulate in an injured brain, causing a theoretical concern for worsening edema in the injured areas. Additionally, mannitol removes water less effectively from injured brain than noninjured brain, which raises concerns for rebound cerebral edema. Hemodynamic Effects Movement of water into the extracellular compartments increases blood volume, cardiac output and blood pressure. Caution should be employed in patients with poor cardiac reserve. Without adequate volume repletion, patients may become hypovolemic and hypotensive secondary to the osmotic diuresis. Pharmacodynamics A clinically effective dose of mannitol ranges between 0.5-1.5 g/kg. Distribution is rapid with a half-life of 0.5-2.5 hours. Mannitol is eliminated unchanged in the urine with the most important determinate of mannitol clearance being renal function. Monitoring Mannitol Use 1. Mannitol accumulation in the injured brain tissue appears to be directly related to clearance of the drug from the blood between doses. To assess clearance of mannitol, calculate the osmolality gap prior to the dose (trough) osmolality gap= measured serum osmolality – calculated serum osmolality

www.medicalebookpdf.com

Calculated serum osmolarity = (2* Na)+(BUN/2.8)+(glucose/18) + (ethanol/4.6) For SI units no need to divide by 2.8, 18 or 4.6 Osmolal gap of 15-20 indicates incomplete clearance of mannitol and could predispose the patient to renal failure. If the osmolal gap is elevated, the dose of mannitol may be held and the dose should be reduced or the dosing interval extended to allow for complete removal of the drug prior to repeat dosing. 2. Routine monitoring of electrolytes (K+, Mg+, Phos, Ca+) must be employed to prevent depletion due to osmotic diuretic effects. 3. Monitoring fluids and outputs should be followed to maintain adequate intravascular volume. Hypertonic Saline (HS) [8,10-12] Osmotic Effects Proposed Mechanism of Action of HS Optimization of blood viscosity, improved cerebral blood flow, and reduction in ICP. Extract water from the brain by creating an osmotic gradient that pulls fluid from the cerebral tissues. Hemodynamic Effects 1. HS lacks the diuretic effect that mannitol possesses; therefore, potentially may reduce the risk of dehydration and other electrolyte disturbances seen with mannitol. 2. HS is a rapid volume expander and may lead to heart failure in patients with cardiopulmonary or renal dysfunction. Pharmacodynamics 1. 0.686 ml of 23.4% sodium chloride is equiosmolar to 1g mannitol. 2. Reflection coefficient is 1, indicating that HS is more likely to stay within the intravascular space and does not cross the intact blood-brain barrier. Mannitol has a reflection coefficient of 0.9 which suggests HS has a greater ability to maintain intravascular osmotic effect.

www.medicalebookpdf.com

Monitoring Hypertonic Saline Use 1. Serum Sodium Levels (trough) q 6-12 hours until the goal serum sodium is achieved. Rapid increases in serum sodium/osmolality may lead to demyelination of neurons in the deep white matter called osmotic demyelinating syndrome (ODS) in patients with more chronic hyponatremia (>48 hours). 2. Strict in/out along with other modalities to assess intravascular volume status to assess rapid volume expansion. 3. Hypokalemia may occur secondary to the large sodium load presented to the kidney and its need to exchange for K+ in the distal tubule. Clinical Pearls 1. There is not enough evidence to suggest one osmotic agent over another. 2. Consider calculating an osmolar gap when administering repeated doses of mannitol dose, especially in patients with suspected renal dysfunction. INOTROPES AND VASOPRESSORS

Clinical Pearls 1. Norepinephrine and dopamine are the vasopressor options for patients with both bradycardia and hypotenstion. 2. Higher doses of vasopressors are generally necessary to augment blood pressure in patients being treated for vasospasm. 3. Tachyphylaxis may occur with any of the agents and may require higher doses or change in vasopressor agent.

www.medicalebookpdf.com

a. Combining two vasopressors may be more beneficial than escalating the dose of one agent. 4. Reflex bradycardia may occur with phenylephrine use. 5. Not enough data to suggest one agent over another for increasing/maximizing cerebral blood flow. ANTIEPILEPTIC DRUGS [13]

www.medicalebookpdf.com

www.medicalebookpdf.com

ENZYME INDUCING SUBSTRATES AND INHIBITORS Numerous agents either act as substrates, inhibitors, or inducers of various enzymes that result in drug-drug interactions or may lead to alterations in serum levels of various medications. Tables 22-9, 22-10, 22-11, and 22-12 provide references for several specific enzymes [14-16]. Inhibitors will decrease metabolism of substrates and generally lead to increased drug effect (unless the substrate is a pro-drug). Inducers will increase metabolism of substrates and generally lead to decreased drug effect (unless the substrate is a pro-drug).

www.medicalebookpdf.com

www.medicalebookpdf.com

REFERENCES

www.medicalebookpdf.com

1. Boucher BA, Wood GC, Swanson JM. Pharmacokinetic changes in critical illness. Critical care clinics 2006;22:255-71, vi. 2. Varghese JM, Roberts JA, Lipman J. Pharmacokinetics and pharmacodynamics in critically ill patients. Current opinion in anaesthesiology 2010;23:472-8. 3. Martin J, Heymann A, Basell K, et al. Evidence and consensus-based German guidelines for the management of analgesia, sedation and delirium in intensive care--short version. German medical science : GMS e-journal 2010;8:Doc02. 4. Mirski MA, Hemstreet MK. Critical care sedation for neuroscience patients. Journal of the neurological sciences 2007;261:16-34. 5. Barr J, Fraser GL, Puntillo K, et al. Clinical practice guidelines for the management of pain, agitation, and delirium in adult patients in the intensive care unit. Critical care medicine 2013;41:263-306. 6. Nasraway SA, Jr., Jacobi J, Murray MJ, Lumb PD, Task Force of the American College of Critical Care Medicine of the Society of Critical Care M, the American Society of Health-System Pharmacists ACoCP. Sedation, analgesia, and neuromuscular blockade of the critically ill adult: revised clinical practice guidelines for 2002. Critical care medicine 2002;30:117-8. 7. Diringer MN, Zazulia AR. Osmotic therapy: fact and fiction. Neurocritical care 2004;1:219-33. 8. Lazaridis C, Neyens R, Bodle J, DeSantis SM. High-osmolarity saline in neurocritical care: systematic review and meta-analysis. Critical care medicine 2013;41:1353-60. 9. Oddo M, Levine JM, Frangos S, et al. Effect of mannitol and hypertonic saline on cerebral oxygenation in patients with severe traumatic brain injury and refractory intracranial hypertension. Journal of neurology, neurosurgery, and psychiatry 2009;80:916-20. 10. Elliott MB, Jallo JJ, Barbe MF, Tuma RF. Hypertonic saline attenuates tissue loss and astrocyte hypertrophy in a model of traumatic brain injury. Brain research 2009;1305:183-91. 11. Kerwin AJ, Schinco MA, Tepas JJ, 3rd, Renfro WH, Vitarbo EA, Muehlberger M. The use of 23.4% hypertonic saline for the management of elevated intracranial pressure in patients with severe traumatic brain injury:

www.medicalebookpdf.com

a pilot study. The Journal of trauma 2009;67:277-82. 12. Rockswold GL, Solid CA, Paredes-Andrade E, Rockswold SB, Jancik JT, Quickel RR. Hypertonic saline and its effect on intracranial pressure, cerebral perfusion pressure, and brain tissue oxygen. Neurosurgery 2009;65:1035-41; discussion 41-2. 13. Rowe AS, Goodwin H, Brophy GM, et al. Seizure prophylaxis in neurocritical care: a review of evidence-based support. Pharmacotherapy 2014;34(4):396-409. 14. Drug Development and Drug Interactions: Table of Substrates, Inhibitors and Inducers. 2011. (Accessed July 10, 2013, at http://www.fda.gov/Drugs/DevelopmentApprovalProcess/DevelopmentResources/DrugInteractionsLabeling/ucm093664.htm.) 15. P450 Drug Interaction Table. at https://cisweb1.utmck.edu/cvpn/cBhp-tFql3QoScrBZ -Rqo7FQoACIGfc_-w/clinpharm/ddis/main-table/.) 16. Online L-C. Cytochrome P450 Enzymes: Substrates, Inhibitors, and Inducers. 2013.

www.medicalebookpdf.com

NEUROINTENSIVE CARE PHARMACOLOGY QUESTIONS 1. OM is a 56 year old female admitted for complaints of dizziness and ataxia. Her home medication list includes phenytoin 300mg at bedtime, and lacosamide 100mg twice daily. Upon admission her total phenytoin level was 7.2 and free phenytoin was 3.5. What is the rational for the lab discrepancy? a. Erratic absorption of her mediations b. Higher percentage of free (unbound) drug than expected due to alterations in protein binding. c. Noncompliance d. Drug interaction between phenytoin and lacosamide 2. Upon lowering OM's phenytoin dose she experienced recurrent seizures. Upon further evaluation of the EEG and discussion among all team members it was decided to start a third agent to better control her seizures. Valproic Acid (VP) was initiated. What drug interaction should be considered and how should the medications be adjusted? a. VP is an inhibitor of the CYP450 (3A4 isoenzyme) which will inhibit the metabolism of phenytoin. The phenytoin dose must be reduced to offset this drug-drug interaction and reduce the toxicity. b. There is no interaction c. Phenytoin is an inducer of the same CYP450 enzymatic process and therefore more VP will need to be given to overcome the increased metabolism caused by the phenytoin. d. Both A and C are correct 3. NM is a 23 year old who presents to the ED after a motorcycle accident. Upon arrival to the ED his GCS was 3, he was intubated, an intracranial monitor was placed and he was admitted to the Neurointensive Care Unit. Mannitol 100gm every 6 hours was initiated for elevated ICP's (range 4050). Two days later his creatinine tripled and his urine output dramatically declined. Na+ 139; K+3.4; Osmolal gap 28. What is the most appropriate action at this time?

www.medicalebookpdf.com

a. Discontinue all osmotic agents b. Continue Mannitol at the same dose c. Discontinue Mannitol and start 23.4% NaCl 50ml IV q 6 hours to goal Na+145. d. Hold Mannitol and recheck osmolar gap in the morning and restart Mannitol 75g every day. 4. Day 5 after admission to the ICU, NM began exhibiting sympathetic storming symptoms; including posturing of all four extremities, tachycardia and elevated core temperature with simultaneous sweating. The ICU team decided to initate opioids. Fentanyl 100mcg/hr trans-dermal patch was placed with instructions to change every 72 hours. Why is this route of administration not optimal in this patient? a. The patient was at risk for constipation b. The patient's excessive sweating compromised the medication delivery system. c. There is no problem d. You should never administer fentanyl in this manner. 5. CH is a 19 year old female admitted to the Neurointensive Care Unit for status epilepticus. She was considered refractory after her EEG continued to reveal epileptiform after administration of lorazepam, phenytoin, levetiracetam, and phenobarbital. A propofol infusion was initiated at 50 mcg/kg/min and increased every 30 minutes until seizure cessation. The next morning CH was seizure free on propofol 250 mcg/kg/min. What should be monitored in this patient for signs of PRIS (propofol related infusion syndrome)? a. SVR, Central venous pressure, heart rate b. pH, potassium, triglycerides, EKG c. urine output, serum sodium, urine specific gravity d. Propofol is an illegal substance 6. JH is a 42 year male who presents to the local ED by EMS after prolonged

www.medicalebookpdf.com

extraction following MVA. After thorough work-up he is found to have a complete C3-C4 spinal cord injury and is admitted to the Neurointensive Care Unit. His vital signs upon admission to the ICU are MAP 50; HR 41; Tmax 36.4. What vasopressor would be the most appropriate agent? a. Phenylephrine b. Dobutamine c. Dopamine d. I would not start a vasopressor 7. JH is a 44 year old male admitted to the Neurointensive Care Unit secondary to ICH. He was intubated upon admission for airway protection but self extubated on day two. He appears agitated and restless but his airway appears stable. What sedation agent would be most appropriate in this extubated patient? a. Meperidine 25mg IVP q 4 hours PRN b. Lorezepam infusion c. Nothing d. Dexmedetomidine infusion 8. A 35-year-old male is in the Neurointensive Care Unit secondary to a severe traumatic head injury. He is intubated, receiving propofol 50 mcg/kg/hr and fentanyl 150 mcg/hr. Despite receiving scheduled mannitol and an external ventricular drain, his ICP remain elevated (ICP 25 to 30). He is noted to have significant ventilator dyssynchrony. Which of the following pharmacologic interventions would be most appropriate in this patient. a. Give vecuronium bolus of 0.1 mg/kg and monitor for ICP response b. Increase propofol to 75 mcg/kg/min and monitor for ICP response c. Increase fentanyl to 200 mcg/hr and monitor for ICP response d. Start vecuronium infusion at 0.5 mcg/kg/min and monitor for ICP response 9. After initiation of mannitol, which of the following should be monitored?

www.medicalebookpdf.com

a. Sodium b. Potassium c. Serum Osmolal Gap d. All of the above 10. As compared to mannitol, hypertonic saline has which of the following properties? a. Can be administered via a peripheral IV b. Does not have a significant diuretic effect c. Has no affect on a patient's sodium level d. Has a lower reflection coefficient

www.medicalebookpdf.com

NEUROINTENSIVE CARE PHARMACOLOGY ANSWERS 1. The correct answer is B. Phenytoin is a highly protein bound drug. Protein binding in ICU patients can be highly variable. 2. The correct answer is D. Both phenytoin and valproic acid are processed through the same CYP isoenzyme; thus, significant drug-drug interactions are possible. 3. The correct answer is C. Mannitol cannot easily be measured in the plasma; however, by monitoring the osmolal gap, accumulation can be assessed. Mannitol accumulation can cause acute kidney injury. 4. The correct answer is B. The absorption of medications administered transdermally can be alterted significantly by many host factors (e.g. sweating, decreased subcutaneous blood flow). 5. The correct answer is B. The risk of PRIS is higher in patients on large doses of propofol. 6. The correct answer is C. Dopamine can increase the heart rate as well as increase the blood pressure. Unlike phenylephrine, dopamine does not cause reflex bradycardia. 7. The correct answer is D. Unlike benzodiazepines, dexmedetomidine does not cause respiratory suppression. 8. The correct answer is A. the patient is noted to have ventilator dyssynchrony despite adequate sedation. A trial of NMBA is warranted. If the patient's ICP respond to the bolus, an infusion would be appropriate. 9. The correct answer is D. Mannitol has significant diuretic effects that can cause alterations to the patient's plasma chemistry. The Osmolal gap should be used to monitor for mannitol accumulation. 10. The correct answer is B. Unlike mannitol, HTS does not cause a significant diuretic effect. It should generally be administered via a central line due to venous irritation and damage.

www.medicalebookpdf.com

Chapter 23

NEUROCRITICAL CARE NURSING SPECIAL CONSIDERATIONS Jennifer Robinson CLINICAL CASE A 45-year-old woman with an aneurysmal subarachnoid hemorrhage (aSAH), Hunt-Hess Grade 3, Fisher Grade III SAH, who is seven days out from the initial rupture develops new onset right-sided arm drift and word finding difficulties. The neurocritical care nurse quickly communicates this information to the treating team. A decision is made to elevate the systolic blood pressure (SBP) and obtain a stat head CT and CT Angiogram (CTA). The nurse mixes and starts a vasopressor continuous infusion, closely monitors the exam more to detect any improvement as the BP is increased and packs up the patient quickly for transport to CT. They rapidly go to CT scan in under thirty minutes. The same patient and same scenario, but now with a less experienced neurocritical care nurse. The team is notified the patient status has changed, but provides the team with minimal details. Several minutes later, a team member examines the patient and discovers the new right sided drift and word finding difficulties. Orders are placed for a stat head CT/CTA and an increased BP goal. The nurse waits for pharmacy to send the vasopressor and waits for CT to call. One hour later, the exam has deteriorated, the BP has not been elevated to augment cerebral perfusion, and the CT/CTA has not been completed. What are the key differences in these scenerios which potentially will impact the patient's outcome? OVERVIEW In the United States alone, over 55,000 critically ill patients are cared for every day. Approximately 5,800 hospitals house over 67,000 adult intensive care beds [1-4]. Critically ill patients are best cared for by a multidisciplinary team composed of intensivists, critical care nurses, pharmacists, respiratory therapists, advanced practice providers such as nurse practitioners and physician assistants, and physical, speech, and occupational therapists. Currently more than 10,000 intensivists and over 500,000 critical care nurses practice in the United States [5-

www.medicalebookpdf.com

8]. Of these critical care nurses, a smaller subset practice in neuroscience ICUs. Like other ICUs, neuroscience ICU nurses have patient ratios of 1:2. Patient acuity plays the largest role in determining staffing, such as if a high acuity patient is paired with a second, lower acuity patient. With extremely high acuity, clinically demanding patients, a 1:1 nurse to patient ratio is required. Appropriate staffing in the ICU can improve patient outcome [9-10]. Subspecialty neuroscience certification is available for neuroscience ICU nurses which demonstrate additional expertise in all areas of neuroscience nursing from rehabilitation to critical care. At present, just over 4,200 nurses have obtained board certification as a certified neuroscience RN. No exam exists for neurocritical care alone. Certification is optional for nurses and is not always associated with a financial incentive. An excellent neuroscience ICU nurse is distinguished by both an increased knowledge base and technical skills. The focus of this chapter is how nursing excellence translates to best practice for the following: transporting patients safely, sedation control, ICP monitoring, temperature management, detection of neurologic changes, prevention of infection, and nursing involvement on a multidisciplinary team. Nurses are primarily responsible for the above interventions and practice is often variable at an institutional level, even among nurses in the same unit. Understanding these practice variations and optimizing and standardizing the best practice is necessary for all medical professions caring for neurocritical care patients. THE ART OF TRAVELING Critical care nurses are responsible for the safety of the patient while traveling and are often the sole medical professional accompanying the patient during the transport. Traveling is the transportation of a critically ill patient to another destination in the hospital. Traveling in the neuroscience ICU has become increasingly more common with the availability of varying imaging options. Despite the use of portable CTs, patients are transported off the ICU regularly. Transport of critically ill patients off the ICU presents a myriad of challenges for ICU nurses. Less personnel and resources are immediately available outside of the ICU and much planning is necessary to ensure a safe and timely transport. Special consideration should be given to emergency versus planned travels as emergency travels naturally have much less time for planning and typically involve a more critically ill and possible unstable patient. Neuroscience ICU patients, not surprisingly, are transported to CT most

www.medicalebookpdf.com

commonly. Other typical destinations include the angiography suite, operating room, MRI, and nuclear medicine [11]. For emergency travels, head CT is also the most common location [11-12]. Personnel involved in transport include at the minimum one ICU nurse and can include as many team members as two nurses, a respiratory therapist, physician, nurse practitioner, physician assistant, or surgeon. While traveling with many medical professionals is less common, it represents a situation of an acute deterioration where treatment decisions in real time from the CT scanner are required. All critically ill patients are transported on a monitor that provides continuous electrocardiographic monitoring, continuous pulse oximetry, intermittent blood pressure, and respiratory rate. Some patients also have a continuous arterial blood pressure and intracranial pressure monitored. However, monitoring capabilities decline substantially in MRI. Not only is a MRI study longer than CT, but visualization of the patient's airway and lines is generally not possible from the control room outside the MRI. Due to the high field strength of the MRI, electrocardiographic monitoring is highly distorted and sometimes inaccurate. At times during MRI, the nurse is unable to monitor heart rate and rhythm. A pulse oximetry probe can also easily dislodge during the scan, further hindering monitoring capabilities. Some intracranial monitors are not MRI compatible and must be disconnected from the monitor prior to entering the MRI. For these reasons, traveling to MRI is generally limited to stable patients. However, if a MRI must be obtained on a more acutely ill patient, the team must be on heightened awareness of the challenge to safely obtain imaging. From a nursing standpoint, traveling to MRI is often the most unnerving as monitoring capabilities are so limited. In routine travel, clinical decision making is often performed by the bedside nurse. For example, patients on continuous infusions of sedatives, the amount of bolus or rate of sedation necessary to obtain the scan and limit agitation is dependent on the nurse's judgment. The organization and securement of endotracheal tubes and invasive lines often falls to the bedside nurse throughout the travel experience. Nurses must be on heightened awareness to ensure lines and tubes remain in place during scans. If the patient is unable to tolerate the scan or the scan is deemed too dangerous, it is often the nurse who makes the decision to abort with a quick phone call to the ICU provider explaining why the scan is to be aborted. The ultimate decision to abort a scan, depends largely on the accompany nurse's comfort level for how high to let the ICPs ride or how much sedative to give without compromising the airway. For emergent scans, less time is available to prepare the patient and more haste is needed to quickly

www.medicalebookpdf.com

obtain the desired imaging. Safe but efficient transport becomes more challenging when travel is for an acute change in a neurologic exam. Physicians are more likely to accompany a patient on an emergent scan [11]. Patients undergoing an emergent scan are also more likely to be intubated and have a higher acuity. Nurses are responsible for traveling with any potentially necessary medications to ensure the patient's blood pressure, heart rate, and ICP remain within parameters. Adverse outcomes related to transport include unintentional extubation or endotracheal tube maladjustment, loss of intracranial monitor, hypotension, hypertension, pain, stress and added work for the nursing staff covering for other patients still in the ICU [13-14]. Traveling well is not taught in nursing schools. These skills are acquired in orientation and via experience of lessons learned from travels where problems arose. The difference between a safe and dangerous travel regardless of stability of patient is usually the preparedness of the ICU nurse. A nurse who can anticipate the needed sedatives, analgesics, vasopressors, fluid, restraints, additional oxygen, and who communicates with the personnel responsible for the scan to minimize wait time in hallways is ideal to accomplish a successful patient transport. SEDATION CONTROL Sedation in the ICU occurs in 42-72% of admitted patients and is used in combination with analgesics [15-16]. Nurses adjust the amount and sometimes the type of sedation and analgesics to provide comfort and maximize safety in critically ill patients [17]. Common goals of sedation are to prevent ventilator dyssynchrony, self extubation, and maximize patient comfort. Regular assessment by the bedside nurse helps decrease the required dosing of sedatives and analgesics and can decrease ventilator time [17]. To help standardize how much sedation is necessary, each patient should have a sedation goal. It is not uncommon for the sedation goal to fluctuate based on the trajectory of the ICU course. Commonly used scales are the Richmond Agitation-Sedation Scale (RASS), Ramsay Scale, and the Riker Sedation Agitation Score (SAS), see Table 23-1 [17]. All three scales provide numeric ratings to objectively score patients from agitated to sedated in enough detail to guide the bedside nurse. Achieving the target level of sedation desired takes both expertise and experience. The three scales are listed below and demonstrate variability in the detail associated with each rating. The RASS is the most comprehensive and detailed. Ramsey is the vaguest and leaves less objective data to guide staff.

www.medicalebookpdf.com

Sedation protocols are frequently utilized in ICU settings and nurses are commonly involved in such protocols. Nurses are best suited to titrate sedation as they are the caregivers most often at the bedside Nurse driven sedation protocols have been shown to decrease ventilator acquired pneumonia, decreased time on the ventilator, and decreased extubation failure [17]. A decrease in both hospital length of stay and duration of mechanical ventilation has been demonstrated in protocolized sedation compared to non-protocolized sedation [18]. Nurse driven sedation protocols can also improve extubation success by increasing wakefulness during spontaneous breathing trials. Protocols can also result in better titration of sedatives [17]. Protocols are particularly helpful in guiding less experienced ICU nurses and standardizing nursing practice. Within individual ICUs, nursing management can fluctuate on how to sedate patients. Titration to a standardized scale with a target goal for each patient can help minimize over sedation and guide novice nurses. ICP MONITORING & MANAGEMENT Nursing tasks such as oral care, endotracheal suctioning, repositioning, chest physiotherapy (CPT), and auditory stimulation are routinely performed on

www.medicalebookpdf.com

critically ill patients. Practice patterns vary amongst nurses for performing these tasks in patients with ICP monitoring. Historically, nurses deliberately avoided such tasks in patients with ICP monitoring for concern for increasing ICP. In recent studies, oral care provided routinely by nursing staff either had no impact on ICP or a minimal 1-2 mm Hg, increase followed by a minimal 2-3 mmHg, decrease after oral care [20]. The type of oral care and mechanism for delivery varied. These preliminary studies suggest that oral care is associated with transient and minimal increases in ICP [20]. Endotracheal suctioning is necessary to clear secretions in ventilated patients. Suctioning has not been associated with immediate elevation of ICP. ICP may rise due to coughing, hypoxia, or tracheal stimulation rather than suctioning [20]. It is a common misconception that suctioning should be avoided. Suctioning is a necessary intervention, particularly in the intubated patient. Hyperoxygenation is recommended prior to suctioning. However hyperventilation prior to suctioning should be avoided. Safe suctioning is limited to one to two passes of the catheter and should be done over several seconds [20]. Repositioning every 2 hours is routine for critically ill patients. The routine practice of Chest Physiotherapy(CPT) varies widely among nurses. The goal of both is to thwart common ICU complications like skin breakdown and alveolar collapse [20]. In some institutions, both of these practices are limited in patients with a concern for ICP elevation. In a recent multicenter observational study, repositioning had no effect on ICP after one minute [20]. Patients can also be placed on rotation modules to continuously shift their position rather than turning every two hours. CPT should be performed in patients, even those with a risk for ICP elevation [20]. CPT can be performed manually or by modules placed into the bed to percuss and vibrate the patient. In general, both of these are safe in the patient with ICP monitoring. All of these tasks are common and left to the discretion of the nurse as to when to perform, how, and for what duration. Though studies are limited, most would agree to avoid these tasks in an ICP crisis, oral care, suctioning, repositioning, and CPT should be performed for critically ill patients with ICP monitors. Appropriately trained nurses know when and how to perform all of these tasks safely to help prevent common ICU complications and avoid potential ICP elevation. Auditory stimulation, from family members or staff talking to the patient is generally thought to be safe, does not increase ICP, and can be recommended to families of patients [20]. Management of ICP also varies widely both internationally and within individual ICUs. Common variances include type of ICP monitor, height of

www.medicalebookpdf.com

drain, target ICP, and how to lower ICP. Nurses are responsible for correct placement of height of drain, monitoring ICP, and lowering it if elevated. Most orders ask for ICP to be kept less than a set goal but leave discretion as to how to accomplish this task. Practice patterns fluctuate greatly particularly in draining CSF, both amount and duration, administering sedation or analgesics, hyperventilation, or giving osmotic therapy, like mannitol or hypertonic saline [21]. Perhaps some variability can be explained by nurses who may rely on additional data like, cerebral perfusion pressure, brain tissue oxygenation, oxygen saturation and a clinical exam, rather than a single ICP reading to initiate an intervention to prevent secondary brain injury [21]. Fluctuation also exists in how often ICP is checked and for how long. Most orders specify ICP checks every hour, but many nurses check ICP more frequently, especially in situations of elevated ICP. These additional checks are often based solely on the judgment of the nurse. Nurses are also responsible for frequently sending labs for patients on osmotic therapy and calculating osmolar gap. The timing of these lab draws in relation to the last dose of mannitol also affects the ability to safely give the next dose. In times of ICP crisis, practice variation is again noted from nurse to nurse. All of the following decisions can be made by the bedside nurse to reduce ICP: bolus sedation and/or analgesics, increase the rate of either, drain CSF from the patient's EVD, reposition the neck to midline, and/or elevate the head of bead. Often all of these interventions are performed before the responsible provider arrives on scene. Temperature Management It is widely established that fever and outcome are inversely related in the acute phase of neurocritical care. Hyperthermia is also associated with increased length of stay, higher mortality, and large infarct sizes [23-24]. Nurses are the primary medical professional to determine the presence of a fever during hourly checks. As a member of the multidisciplinary treatment team, nurses participate in the timing of intervention, the type of intervention and, and the degree of intervention needed [23]. An understanding of how and why nurses treat fever is fundamental to best neurocritical care practice. In some cases, there is a discrepancy between patient outcomes research and nursing practice [23]. Fever is a widespread problem with reports of incidences of 70-100% in TBI patients, 38% in ischemic stroke and more than 50% of hemorrhagic stroke patients [2426]. The high prevalence of fever translates to a daily concern for neurocritical care nurses. A lack of consistency is noted on the definition of a fever, from 37.2 °C to more than 39 °C and on how to treat a fever [24]. Options for hyperthermia management include: acetaminophen, ibuprofen, removal of blankets, ice packs, tepid bathing, circulating fans, water cooling blankets, higher tech surface

www.medicalebookpdf.com

cooling or intravascular cooling devices. In a recent survey of neuroscience nurses, only 19% of respondents had a hyperthermia management protocol for neuroscience patients [24]. The protocols varied nationwide for when to start treatment, from 37 °C to 40 °C. Of particular concern, was noted regional practice patterns with some areas of the country waiting for a temperature of 40 °C to initiate hyperthermia treatment [24]. Listed interventions in order of preference were: acetaminophen (80%), ice packs (43%), water cooling blankets (40%), and ibuprofen (30%) [24]. Medication dosages and route (oral or IV) varied among protocols. Acetaminophen was dosed between 650 mg to 1000 mg every four to six hours. Ibuprofen ranged from 600 mg to 800 mg every six to eight hours. Two additional non-steroidal anti-inflammatory medications, ketorolac and indomethacin, were also mentioned in some protocols [24]. Similarities in protocols included consistent use of acetaminophen, water cooling blankets, and ice packs [24]. Temperatures for water cooling blanket machines vary widely. Some nurses set the water cooling blankets to the lowest possible temperature in an attempt to more quickly reduce hyperthermia. Others, start at a warmer temperature and titrate down as the patient tolerates. However, in some patients, a lower temperature is not well tolerated and results in shivering. Hyperthermia protocols often do not address the temperature of the water cooling blanket, and it is left to the discretion of the nurse to regulate the temperature of both the patient and the device. Temperature checks also vary from every hour to every four hours in the ICU. A successfully hyperthermia protocol is necessary to manage such a common problem in neuroscience ICUs and requires participation from a multidisciplinary group. Education and understanding from all parties is key to empower the bedside nurse to identify, rapidly treat fevers, and transition seamlessly for more aggressive treatments [23]. Some institutions have nurse driven hyperthermia protocols for normothermia and hypothermia to provide a step wise approach for aggressive fever management. Such a protocol provides for administration of antipyretics, placement of ice packs, cooling blankets, or other devices and allows for rapid escalation. Shivering is another problem for nurses to detect, and treat. Some of the above mentioned hyperthermia protocols also have a shivering component to help the nurse detect subtle shivering and measures to prevent shivering. At times, titrating temperature of the water cooling machine is sufficient to stop shivering if the patient is no longer febrile. Other times, it is necessary to provide surface counter warming, layer a cooling blanket between sheets, increase sedation, or give anti-shivering agents like meperidine or buspirone. Meperidine can be given in the form of an IV push or a

www.medicalebookpdf.com

continuous infusion. With evidence based protocols, fever control can be adequately accomplished and patient outcomes can be achieved in a timely manner. DETECTION OF NEUROLOGIC CHANGES Neurocritical care nurses are often in the best position to detect a neurological change, whether it is a change in pupillary size, a new focal deficit, a decline in level of consciousness, worsening headache, or visual changes. Nurses can detect changes in the GCS or NIHSS or by comparison to previous exam. For sedated patients, it is vital for the nurse to safely lift sedation to obtain a high quality exam to verify the patient is at his or her neurologic baseline. Lifting sedation is done in balance between keeping the patient safe and obtaining an informative exam. Challenges can arise to differentiate if a patient has had a neurologic change or is sedated. This distinction is important to minimize unnecessary diagnostic and therapeutic interventions. The neurocritical care nurse is typically in the best position to identify patient who is has a subtle neurological change which may herald a significant neurologic deterioration such as cerebral vasospasm in patients with subarachnoid hemorrhage. The judgment and experience of the nurse who spends the most time with a patient and knows their fluctuations best is ideally suited to detect the these subtle of changes from the normal variations of sleep and medications. Then, timely assessment and action by the medical team can potentially lead to an earlier intervention in the hopes of limiting or preventing injury. In addition to hourly monitoring of the neurologic exam, nurses strive to minimize ICU delirium by continuously reorienting patients, help regulate a more consistent day-night schedules, and minimize sedation. Beyond neurologic assessments, neurocritical care nurses are responsible for a multiude of check intake and output, vitals, integrity of skin, continuous intravenous infusions, patency of peripheral and intravenous access, securement of the endotracheal tube, type of waveforms for all invasive monitors, and monitor for safety issues like agitation and risk for falls. Regular checks, toileting, and early mobilization can improve safety, minimize falls, and improve outcomes. Early mobilization, even in ventilated patients, can prevent pressure ulcers, decrease time on the ventilator, and decrease neuromuscular weakness [27]. Mobilization of patients can aid in detection of a neurologic change as it is much easier to assess a patient's neurologic exam, particularly strength, when they are out of bed. Mobilization of ICU patients is the responsibility of the ICU nurse and, at times,

www.medicalebookpdf.com

is supported by physical and occupational therapists. PREVENTION OF INFECTIONS Another vital role for ICU nurses is prevention of infection, both catheter associated urinary tract infections (CAUTIs) and blood stream infections (BSIs). Over 500,000 CAUTIs occur yearly in the United States [28]. Approximately 25% of patients in the hospital have a catheter and 10% develop a CAUTI [28]. Urinary catheters are commonly used in the ICU for accurate intake and output measurement. CAUTIs are the most common nosocomial infection in the ICU [29]. Though not as dangerous as BSI, CAUTIs are the most common source of bacteremia and commonly cause an increase in length of stay, additional antibiotic use, and increase in hospital costs. Nurses are able to address daily with the team the need for a catheter, promptly remove it once ordered, and maintain best practice for those patients who still require a catheter. Best practice includes keeping the urinary bag below the level of the bladder, regular emptying of the urinary drainage bag, and minimizing movement of the catheter [28]. In some units, the nurses remove urinary catheters placed in an outside hospital and re-insert a catheter upon admission. Barriers for nurses to remove a catheter include concern for skin integrity, lack of ability to measure output accurately, increased time needed for incontinence care particularly for obese and immobile patients [29]. BSIs, particularly central line-associated BSI (CLABSI) are most prevalent in the ICU setting as the ICU frequently uses central lines, lines are placed in emergent situations, accessed routinely, and required for prolonged periods [30]. Like with CAUTIs, CLABSIs lead to a longer length of stay, additional cost, and need for antibiotic use. Strategies to reduce CLABSIs include catheter insertion kits, education of physicians and nurses about hospital guidelines related to insertion, a catheter insertion checklist For any concern of contamination, the nurse should inform the provider inserting the line of a potential breach of aseptic technique and stop the procedure [30]. Falsely positive blood cultures contribute to prolongation of length of stay, use of antibiotics, and additional lab work [31]. Appropriate drawing of cultures is a nursing intervention and appropriate education of staff reduces the number of falsely positive blood cultures [31]. After education of staff, decreases in number of repeated unsuccessful attempts, improvement in sterilization, and reduction of single sets of blood cultures were found [31]. As a result, the number of false positive blood cultures also decreased. NURSING INVOLVEMENT ON A MULTI-DISCIPLINARY TEAM

www.medicalebookpdf.com

Bedside nurses must possess both technical and interpersonal skills to ensure safe care [21]. Fundamentals of neurocritical care nursing teach nurses to position the head of bed at thirty degrees, keep the head midline, and ensure the cervical collar is not too tight to not hinder jugular venous return. In addition, to the physiological interventions of lab work, serial neurologic checks, ventilator management, and monitoring of vitals, nurses are the main support for family members and update families most frequently. Adequate education and eloquent explanations alleviate much anxiety from families and help ensure nurses can focus on the patient. Consequently, the education and understanding of the nurse is vital to optimizing patient care. Nurses spend the majority of their day at the bedside providing direct patient care. In the process, nurses also develop a significant relationship with family members, particularly for patients with a longer length of ICU stay. It is fundamental for nurses to be involved in daily ICU rounds to offer insight into overnight events, accurately state the exam, understand the plan, coordinate any family meetings, and inquire about removal of lines or catheters and mobilization. Nurses should be involved in all family meetings; however, nursing involvement is not universal. Nurses usually have the closest relationship with the family members of an ICU patient [32]. Family members value nursing perspective and effort should be made to include the nurse in family meetings, even when they occur off the unit [32]. Nurses communicate with families routinely and can help coordinate a meeting, provide expectations, and a framework for the planned meeting so family members can arrive best prepared to discuss any concerns. Nurses also have a unique insight that can help a wide variety of caregivers provide consistent information [32]. Empowering the nurse to provide continuity of care improves family satisfaction and enhances decision making [32]. Physicians, especially those with less experience, will benefit from a team approach to a family meeting aided by the insight of the clinician who best knows the family. SUMMARY Expert neurocritical care nurses need both knowledge and practice skills to optimize the best possible patient outcomes in a neuroscience ICU. Providing frequent neurological exams, monitoring ICP, minimizing sedation, controlling fever, and safely transporting the patient are many of the practices neurocritical care nurses are challenged with on a daily basis. Neurocritical care nurses, together with the remainder of the multidisciplinary team, can optimize the best

www.medicalebookpdf.com

practices necessary to care for the neurocritical care patient and improve outcomes. REFERENCES 1. Carr BG, Addyson DK, Kahn JM. Variation in critical care beds per capita in the United States: Implications for pandemic and disaster planning. JAMA. 2010;303:1371-1372. 2. Halpern NA, Pastores SM. Critical care medicine in the United States 20002005: an analysis of bed numbers, occupancy rates, payer mix and costs. Crit Care Med. 2010;38:65-71. 3. Health Forum, LLC. American Hospital Association Hospital Statistics, 2011 (2009 survey data). Chicago, IL: American Hospital Association; 2011. 4. Odetola FO, Clark SJ, Freed GL, Bratton SL, Davis MM. A national survey of pediatric critical care resources in the United States. Pediatrics. 2005;115:e382-e386. 5. American Association of Critical-Care Nurses, American College of Chest Physicians, American Thoracic Society and Society of Critical Care Medicine. The aging of the U.S. population and increased need for critical care services. Critical Care Workforce Partnership Position Statement. November 2001. Available at: http://www.chestnet.org/downloads/practice/gr/HRSABack-grounder .pdf. Accessed July 12, 2013. 6. Joint Commission Resources. Improving Care in the ICU. 1st ed. Oakbrook Terrace, IL: Joint Commission Resources; 2004. 7. Society of Critical Care Medicine. Compensation of Critical Care Professionals. 2nd ed. Mount Prospect, IL: Society of Critical Care Medicine; 2008. 8. US Department of Health and Human Services, Health Resources and Services Administration Report to Congress. The Critical Care Workforce: A Study of the Supply and Demand for Critical Care Physicians. May 2006. Available at http://bhpr.hrsa.gov/healthworkforce/reports/studycriticalcarephys.pdf . Accessed July 12, 2013. 9. West E, Mays N, Rafferty AM, Rowan K, Sanderson C. Nursing resources

www.medicalebookpdf.com

and patient outcomes in intensive care: a systematic review of the literature. Int J Nurs Stud. 2009;46(7):993-1011. 10. Penoyer DA. Nurse staffing and patient outcomes in critical care: a concise review. Crit Care Med. 2010;38(7):1521-1528. 11. Kalisch BJ, Kalisch PA, Burns SM, Kocan MJ, Prendergast V. Intrahospital transport of neuro ICU patients. J Neurosci Nurs. 1995;27(2):69-77. 12. Voigt LP, Pastores SM, Raoof ND, Thaler HT, Halpern NA. Intrahospital transport of critically ill patients: outcomes, timing, and patterns. J Int Care Med. 2009;24(2):108-115. 13. Warren J, Fromm RE, Orr RA, Rotello LC, Horst HM. Guidelines for interand intrahospital transport of critically ill patients. Crit Care Med. 2004;32(1):256-262. 14. Beckmann U, Gillies DM, Berenholtz SM, Wu AW, Pronovost P. Incidents relating to intra-hospital transfer of critically ill patients. Int Care Med. 2004;30:1579-1585. 15. Salgado DR, Favor R, Goulart M, Brimioulle S, Vincent JL. Toward less sedation in the intensive care unit. A prospective observational study. J Crit Care. 2011;26:113-121. 16. Payen JF, Chanques G, Mantz J, Hercule C, Auriant I, Leguillou JL, et al. Current practices in sedation and analgesia for mechanically ventilated critically ill patients: a prospective multicenter patient-based study. Anesthesiology. 2007;106:687-695. 17. McGrane S, Pandharipande PP. Sedation in the intensive care unit. Minerva Anestesiol. 2012;78:369-380. 18. Brook AD, Ahrens TS, Schaiff R, Prentice D, Sherman G, Shannon W, et al. Effect of a nursing-implemented sedation protocol on the duration of mechanical ventilation. Crit Care Med. 1999;27:2609-2615. 19. Arias-Rivera S, Sanchez M, Santos-Diaz R, Gallardo-Murillo J, SanchezIzquierdo R, et al. Effect of a nursing-implemented sedation protocol on weaning outcome. Crit Care Med. 2008;36(7):2054-2060. 20. McNett MM, Olson DM. Evidence to guide nursing interventions for critically ill neurologically impaired patients with ICP monitoring. J Neurosci Nurs. 2013;45(3):120-123.

www.medicalebookpdf.com

21. Olson DM, Lewis LS, Bader MK, Bautista C, Malloy R, Riemen KE, McNett MM. Significant practice pattern variations associated with intracranial pressure monitoring. J Neurosci Nurs. 2013; 45(4):186-193. 22. McNett MM, Gianakis A. Nursing interventions for critically ill traumatic brain injury patients. J Neurosci Nurs. 2010;42(2):71-77. 23. Thompson HJ, Kirkness CJ, Mitcell PH. Fever management practices of neuroscience nurses, part II: nurse, patient, and barriers. J Neurosci Nurs. 2007;39(4):196-201. 24. Thomspson HJ, Kirkness CJ, Mitchell PH, Webb DJ. Fever management practices of neuroscience nurses: national and regional perspectives. J Neurosci Nurs. 2007;39(3):151-162. 25. McIlvoy LH. The effect of hypothermia and hyperthermia on acute brain injury. AACN Clinical Issues. 2005;16:488-500. 26. Kilpatrick MM, Lowery DW, Firlik AD, Yonas H, Marion DW. Hyperthermia in the neurosurgical intensive care unit. Neurosurgery. 2000;47:850-856. 27. Kress JP. Clinical trials of early mobilization of critically ill patients. Crit Care Med. 2009;37:442447. 28. Oman KS, Makic MB, Fink R, Schraeder N, Hulett, et al. Nurse-directed interventions to reduce catheter-associated urinary tract infections. Am J Infect Control. 2012;40(6):548-553. 29. Elpern EH, Killeen K, Ketchem A, Wiley A, Patel G, Lateef O. Reducing use of indwelling urinary catheters and associated urinary tract infections. Am J Crit Care. 2009;18(6):535-541. 30. Marshall J, Mermel LA, Classen D, et al. Strategies to prevent central lineassociated bloodstream infections in acute care hospitals. Infect Control Hosp Epidemiol. 2008;29:22-30. 31. Roth A. Wiklund AE, Palsson As, Melander, et al. Reducing blood culture contamination by a simple informational intervention. J Clin Micro. 2010;48(12):4552-4558. 32. Gay EB, Pronovost PJ, Bassett RD, Nelson JE. The intensive care unit family meeting: making it happen. J Crit Care. 2009;24(4):629-632.

www.medicalebookpdf.com

NEUROCRITICAL CARE NURSING QUESTIONS The following case study should be used for questions 1-3. Ms. B is a 63-year-old patient with a malignant left MCA stroke. She underwent a hemicraniectomy yesterday. Her vital signs are: HR 70 BP 110/70 RR 12 Sats 100% on AC 12-500-40-5 T 100.8 °F ICP 16. 1. What is the best nursing intervention based on these vitals? a. Administer 650 mg acetaminophen orally b. Put the patient on a cooling blanket c. Administer 800 mg of ibuprofen d. Wait for the temperature to reach 101 °F 2. The patient's temperature remains elevated despite round the clock acetaminophen, ice packs, and a cooling blanket. An intravascular cooling device is inserted for normothermia. What is the least helpful intervention? a. Placing a surface re-warmer on the patient b. Stopping sedation c. Continuing to give acetaminophen d. Monitoring the patient and EKG leads for any sign of shivering 3. The patient begins to shiver. What should the nurse do next? a. Start an infusion of meperidine b. Turn off the cooling device c. Give oral buspirone d. Administer prn IV meperidine 4. A patient with an unsecured aneurysm suddenly develops an ICP of 60 mm Hg and a right dilated, non-reactive pupil. The nurse calls for a member of the team to come evaluate immediately. What should the nurse do in the interim?

www.medicalebookpdf.com

a. Drain CSF by opening the EVD b. Grab mannitol or 23.4% NaCl c. Prepare to hyperventilate the patient d. All of the above 5. For a planned travel to head CT with a patient with ICP spikes, the nurse should do all of the following to safely obtain the head CT except? a. Time the travel after giving a dose of osmotic therapy b. Drain 5-10 ml of CSF prior to going to CT c. Disconnect the patient from as many lines as possible to expedite travel d. “Trial” the patient with their HOB flat for 10 minutes and closely monitor the ICP to test how well the patient will tolerated lying flat for head CT 6. In what situation should the nurse abort a head CT? a. SBP of 160 mm Hg (above goal of SBP
The Practice of Neurocritical Care

Related documents

765 Pages • 209,014 Words • PDF • 13.5 MB

12 Pages • 10,373 Words • PDF • 231 KB

3 Pages • 711 Words • PDF • 256.9 KB

11 Pages • 10,287 Words • PDF • 224.8 KB

1,148 Pages • 467,314 Words • PDF • 37.3 MB

8 Pages • 5,242 Words • PDF • 84.9 KB

6 Pages • 2,767 Words • PDF • 93.3 KB

18 Pages • 5,807 Words • PDF • 14 MB

224 Pages • 189,377 Words • PDF • 17.6 MB

591 Pages • 457,770 Words • PDF • 14.6 MB

17 Pages • 1,568 Words • PDF • 7.5 MB